Section C Part 2 MCQ

You might also like

Download as pdf or txt
Download as pdf or txt
You are on page 1of 344

Hock P2 2020

Section C - Decision Analysis.


Answers
Section C - Decision Analysis.

1. Question ID: ICMA 10.P2.215 (Topic: Assumptions and Basis of CVP Analysis)
Lazar Industries produces two products, Crates and Boxes. Per unit selling prices,
costs, and resource utilization for these products are as follows.

Crates Boxes
Selling price $20 $30

Direct material costs $ 5 $ 5


Direct labor costs 8 10
Variable overhead costs 3 5
Variable selling costs 1 2

Machine hours per unit 2 4


Production of Crates and Boxes involves joint processes and use of the same facilities.
The total fixed factory overhead cost is $2,000,000 and total fixed selling and
administrative costs are $840,000. Production and sales are scheduled for 500,000
units of Crates and 700,000 units of Boxes. Lazar maintains no direct materials, work-
in-process, or finished goods inventory.
Lazar can reduce direct material costs for Crates by 50% per unit, with no change in
direct labor costs. However, it would increase machine-hour production time by 1.5
hours per unit. For Crates, variable overhead costs are allocated based on machine
hours. What would be the effect on the total contribution margin if this change was
implemented?

 A. $250,000 decrease.
 B. $125,000 increase.correct
 C. $1,250,000 increase.
 D. $300,000 increase.
Question was not answered
Correct Answer Explanation:
The present contribution margin for crates is $3 per unit ($20 – 5 – 8 – 3 – 1).
If the change is made, variable overhead applied will change as follows:
Hock P2 2020
Section C - Decision Analysis.
Answers
Machine hours/unit currently required for Crates = 2 hours.
Variable overhead currently applied per unit = $3.
Therefore, the variable OH application rate is $1.50 per machine hour used ($3 ÷ 2).
If the machine hours/unit are increased by 1.5 hours per unit, each unit will require 3.5
hours of machine time (2 hours presently + 1.5 hour increase).
At an application rate of $1.50 per machine hour, 3.5 hours machine time per unit will
result in $5.25 VOH being applied per unit (3.5 × $1.50).
Now, we can calculate the contribution margin per unit for crates that would result from
this change:

Selling price $20.00


Direct materials (reduced by 50%) 2.50
Direct labor (unchanged) 8.00
Variable overhead applied 5.25
Variable selling costs (unchanged) 1.00
Contribution margin per unit $3.25
Since the current contribution margin per unit is $3.00, the contribution margin per unit
will increase by $0.25. 500,000 crates will be produced and sold. Therefore, the total
increase in the contribution margin as a result of this change will be $0.25 × 500,000, or
$125,000.
Explanation for Choice A:
This answer results from adjusting the variable selling cost per crate on the assumption
that variable selling costs are part of overhead and would thus be applied to production
at a higher rate due to more machine hours being required per unit.
Selling costs are not applied to production because they are period costs and are
expensed as they are incurred. Therefore, the variable selling cost per unit will not
change as a result of the increased number of machine hours required to produce each
crate.
Explanation for Choice C:
This answer results from incorporating the decrease in direct materials costs but not the
increase in the amount of variable overhead applied. The amount of variable overhead
applied will increase because variable overhead is allocated based on machine hours,
and the number of machine hours required per unit will increase.
Explanation for Choice D:
Hock P2 2020
Section C - Decision Analysis.
Answers
This is not the correct answer. Please see the correct answer for a complete
explanation.
We have been unable to determine how to calculate this incorrect answer choice. If you
have calculated it, please let us know how you did it so we can create a full explanation
of why this answer choice is incorrect. Please send us an email at
support@hockinternational.com. Include the full Question ID number and the actual
incorrect answer choice -- not its letter, because that can change with every study
session created. The Question ID number appears at the top of the question. Thank you
in advance for helping us to make your HOCK study materials better.
2. Question ID: CIA 1186 IV.16 (Topic: Assumptions and Basis of CVP Analysis)
The data available for the current year are given below.

Whole
company Division 1 Division 2
Variable manufacturing costs $400,000 $220,000 $180,000
Unallocated costs (e.g., president's salary) 100,000
Fixed costs controllable by division managers
(e.g., advertising, engineering supervision costs) 90,000 50,000 40,000
Net revenue 1,000,000 600,000 400,000
Variable selling and administrative costs 130,000 70,000 60,000
Fixed costs controllable by others
(e.g., depreciation, insurance) 120,000 70,000 50,000
Based upon the information presented above, the contribution margin for the company
was:

 A. $400,000
 B. $600,000
 C. $470,000correct
 D. $530,000
Question was not answered
Correct Answer Explanation:
Contribution margin is sales revenue minus variable costs. Net revenue was
$1,000,000, variable manufacturing costs were $400,000, and variable selling and
administrative costs were $130,000. $1,000,000 − $400,000 − $130,000 = $470,000.
Explanation for Choice A:
Hock P2 2020
Section C - Decision Analysis.
Answers
This is the variable manufacturing costs. The contribution margin is sales minus variable
costs.
Explanation for Choice B:
This is revenues minus variable manufacturing costs. The contribution margin is sales
minus all variable costs, not just variable manufacturing costs.
Explanation for Choice D:
This is the total variable costs. The contribution margin is sales minus variable costs.
3. Question ID: ICMA 1603.P2.071 (Topic: Assumptions and Basis of CVP
Analysis)
A company produces and sells 2,000 units of finished goods and incurs $60,000 of fixed
costs annually. The contribution margin is $60 per unit, and variable cost is $40 per unit.
If the company expects sales quantities to increase by 10% next year, the operating
profit will be

 A. $60,000.
 B. $120,000.
 C. $132,000.
 D. $72,000.correct
Question was not answered
Correct Answer Explanation:
If the company expects sales quantities to increase by 10% next year, the quantity sold
will become 2,200 units (2,000 units × 1.10). The contribution margin per unit is $60, so
the contribution margin will be 2,200 × $60, or $132,000. Fixed costs will remain the
same, at $60,000, so operating income will be $132,000 − $60,000, or $72,000.
Explanation for Choice A:
$60,000 is the current operating income at the current sales volume. It does not include
any adjustment for the expected 10% increase in sales volume.
Explanation for Choice B:
$120,000 is the contribution margin at the current sales volume (2,000 units × $60
contribution margin per unit = $120,000). However, this answer does not include any
increase in the contribution margin to reflect the expected 10% increase in sales
volume, nor does it include any deduction for fixed costs.
Explanation for Choice C:
$132,000 is the contribution margin if sales increase by 10% next year. Since the
contribution margin per unit is $60, the contribution margin at a sales level of 2,200 units
Hock P2 2020
Section C - Decision Analysis.
Answers
(2,000 units × 1.10) will be 2,200 × $60, or $132,000. However, this answer does not
include any deduction for fixed costs.
4. Question ID: CMA 690 5.12 (Topic: Assumptions and Basis of CVP Analysis)
Madengrad Company manufactures a single electronic product called Precisionmix.
This unit is a batch-density monitoring device attached to large industrial mixing
machines used in flour, rubber, petroleum, and chemical manufacturing. Precisionmix
sells for $900 per unit. The following variable costs are incurred to produce each
Precisionmix device:

Direct labor $180


Direct materials 240
Factory overhead 105
Total variable production costs $525
Marketing costs 75
Total variable costs $600
Madengrad's income tax rate is 40%, and annual fixed costs are $6,600,000. Except for
an operating loss incurred in the year of incorporation, the firm has been profitable over
the last 5 years.
If Madengrad Company achieves a sales and production volume of 8,000 units, the
annual before-tax income (loss) will be

 A. $(420,000)
 B. $(4,200,000)correct
 C. $(2,520,000)
 D. $1,780,000
Question was not answered
Correct Answer Explanation:
The answer is calculated the following way: Sales price (per unit) of $900 less variable
costs of $600 per unit = contribution margin of $300 per unit. If 8,000 units are sold, the
total contribution margin will be $2,400,000. $2,400,000 less fixed costs of $6,600,000 =
annual before-tax loss of $(4,200,000).
Explanation for Choice A:
This is not the correct answer. Please see the correct answer for an explanation.
We have been unable to determine how to calculate this incorrect answer choice. If you
have calculated it, please let us know how you did it so we can create a full explanation
of why this answer choice is incorrect. Please send us an email at
Hock P2 2020
Section C - Decision Analysis.
Answers
support@hockinternational.com. Include the full Question ID number and the actual
incorrect answer choice -- not its letter, because that can change with every study
session created. The Question ID number appears at the top of the question. Thank you
in advance for helping us to make your HOCK study materials better.
Explanation for Choice C:
This is the after-tax loss (the before-tax loss adjusted for the effect of the tax credit to be
realized in a future year due to the tax loss carryover). However, the question asks for
the before-tax income (loss).
Explanation for Choice D:
This is not the correct answer. Please see the correct answer for an explanation.
We have been unable to determine how to calculate this incorrect answer choice. If you
have calculated it, please let us know how you did it so we can create a full explanation
of why this answer choice is incorrect. Please send us an email at
support@hockinternational.com. Include the full Question ID number and the actual
incorrect answer choice -- not its letter, because that can change with every study
session created. The Question ID number appears at the top of the question. Thank you
in advance for helping us to make your HOCK study materials better.
5. Question ID: CMA 693 4.2 (Topic: Assumptions and Basis of CVP Analysis)
Delphi Company has developed a new project that will be marketed for the first time
during the next fiscal year. Although the Marketing Department estimates that 35,000
units could be sold at $36 per unit, Delphi's management has allocated only enough
manufacturing capacity to produce a maximum of 25,000 units of the new product
annually. The fixed costs associated with the new product are budgeted at $450,000 for
the year, which includes $60,000 for depreciation on new manufacturing equipment.
Data associated with each unit of product are presented as follows. Delphi is subject to
a 40% income tax rate.

Variable
Costs

Direct material $ 7.00


Direct labor 3.50
Manufacturing overhead 4.00
Total variable manufacturing cost $14.50
Selling expenses 1.50
Total variable cost $16.00
Hock P2 2020
Section C - Decision Analysis.
Answers
The maximum after-tax profit that can be earned by Delphi Company from sales of the
new product during the next fiscal year is

 A. $30,000.correct
 B. $50,000.
 C. $110,000.
 D. $66,000.
Question was not answered
Correct Answer Explanation:
Since only 25,000 units can be sold, the maximum contribution margin will be $500,000
(the contribution margin per unit of $20 multiplied by 25,000 units). This contribution
margin will be reduced by the fixed costs of $450,000 resulting in a maximum pre-tax
profit of $50,000. As taxes are 40%, the maximum after-tax profit is $30,000.
Explanation for Choice B:
This is the maximum profit before tax. The problem asks for the maximum after-tax
profit.
Explanation for Choice C:
This is the maximum contribution margin that can be earned for selling 25,000 units
minus total fixed costs reduced by depreciation. Depreciation is a fixed cost and should
be included in total fixed costs. Furthermore, the question asks for after-tax profit, and
taxes have not been deducted.
Explanation for Choice D:
This is the maximum contribution margin that can be earned for selling 25,000 units
minus total fixed costs reduced by depreciation. Depreciation is a fixed cost and should
be included in total fixed costs.
6. Question ID: CIA 1188 IV.17 (Topic: Assumptions and Basis of CVP Analysis)
Which of the following is a characteristic of a contribution income statement?

 A. Fixed expenses are listed separately from variable expenses.correct


 B. Fixed and variable operating expenses are combined as one line item, but fixed
manufacturing expenses are shown separately from variable manufacturing expenses.
 C. Fixed and variable expenses are combined as one line.
 D. Fixed and variable manufacturing costs are combined as one line item, but fixed
operating expenses are shown separately from variable operating expenses.
Question was not answered
Correct Answer Explanation:
Hock P2 2020
Section C - Decision Analysis.
Answers
The contribution income statement separates out fixed and variable cost on the financial
statement. This is necessary for CVP analysis.
Explanation for Choice B:
On the contribution income statement, all fixed and variable costs are separated, not
just fixed manufacturing costs.
Explanation for Choice C:
On the contribution income statement, fixed and variable costs are separated.
Explanation for Choice D:
On the contribution income statement, all fixed and variable costs are separated, not
just fixed operating costs.
7. Question ID: ICMA 10.P2.199 (Topic: Assumptions and Basis of CVP Analysis)
All of the following are assumptions of cost-volume-profit analysis except

 A. sales mix for multi-product situations do not vary with volume changes.
 B. variable costs per unit change proportionately with volume.correct
 C. total fixed costs do not change with a change in volume.
 D. revenues change proportionately with volume.
Question was not answered
Correct Answer Explanation:
This is not an assumption of cost-volume-profit analysis. On the contrary, it is an
assumption of cost-volume-profit analysis that the variable costs per unit will remain the
same as the volume changes, as long as the volume remains within the relevant range.
Variable costs in total change proportionately with volume, but cost-volume-profit
analysis assumes that variable costs per unit remain constant over the relevant range
as volume increases or decreases.
Explanation for Choice A:
This is an assumption of cost-volume-profit analysis.
Explanation for Choice C:
This is an assumption of cost-volume-profit analysis.
Explanation for Choice D:
This is an assumption of cost-volume-profit analysis.
8. Question ID: ICMA 10.P2.212 (Topic: Assumptions and Basis of CVP Analysis)
Hock P2 2020
Section C - Decision Analysis.
Answers
Wilkinson Company sells its single product for $30 per unit. The contribution margin
ratio is 45% and Wilkinson has fixed costs of $10,000 per month. If 3,000 units are sold
in the current month, Wilkinson's income would be

 A. $49,500.
 B. $90,000.
 C. $40,500.
 D. $30,500.correct
Question was not answered
Correct Answer Explanation:
Sales are $30 × 3,000 units, or $90,000. The contribution margin ratio is 45%, so the
contribution margin is $90,000 × 0.45, or $40,500. The contribution margin of 
$40,500
minus fixed costs of $10,000 equals income of $30,500.
Explanation for Choice A:
This is the sales revenue multiplied by 55%. Since the contribution margin ratio is 45%,
variable costs must be 55% of revenue. Thus, this is the total variable costs at a sales
volume of 3,000 units.
Explanation for Choice B:
This is the revenue at a sales volume of 3,000 units. Revenue minus variable costs
minus fixed costs equals operating income.
Explanation for Choice C:
This is the contribution margin at a sales volume of 3,000 units. The contribution margin
minus fixed costs equals operating income.
9. Question ID: CMA 691 4.11 (Topic: Assumptions and Basis of CVP Analysis)
If inventories are expected to change, the type of costing that provides the best
information for breakeven analysis is

 A. job order costing.


 B. absorption (full) costing.
 C. variable (direct) costing.correct
 D. joint costing.
Question was not answered
Correct Answer Explanation:
Variable costing provides the best information for breakeven analysis whether
inventories are expected to change or not, because variable costs are segregated from
Hock P2 2020
Section C - Decision Analysis.
Answers
fixed costs. To use the formula for the breakeven point in CVP analysis, fixed costs
need to be separated from variable costs.
Explanation for Choice A:
To use breakeven analysis, variable costs need to be segregated from fixed costs.
Variable costs are not segregated from fixed costs in job order costing.
Explanation for Choice B:
To use breakeven analysis, variable costs need to be segregated from fixed costs.
Variable costs are not segregated from fixed costs in absorption costing.
Explanation for Choice D:
To use breakeven analysis, variable costs need to be segregated from fixed costs.
Variable costs are not segregated from fixed costs in joint costing.
10. Question ID: CMA 690 5.15 (Topic: Assumptions and Basis of CVP Analysis)
Madengrad Company manufactures a single electronic product called Precisionmix.
This unit is a batch-density monitoring device attached to large industrial mixing
machines used in flour, rubber, petroleum, and chemical manufacturing. Precisionmix
sells for $900 per unit. The following variable costs are incurred to produce each
Precisionmix device:

Direct labor $180


Direct materials 240
Factory overhead 105
Total variable production costs $525
Marketing costs 75
Total variable costs $600
Madengrad's income tax rate is 40%, and annual fixed costs are $6,600,000. Except for
an operating loss incurred in the year of incorporation, the firm has been profitable over
the last 5 years.
Assume a 10% increase in annual fixed costs, a 20% unit cost increase for direct labor,
and a reduction in unit material costs of 25%, with no change in selling price. After
incorporating these changes, Madengrad Company's contribution margin ratio would be

 A. 69%
 B. 64%
 C. 34%
 D. 36%correct
Hock P2 2020
Section C - Decision Analysis.
Answers
Question was not answered
Correct Answer Explanation:
The contribution margin before the changes is $900 − $600, or $300. The 20% unit cost
increase in direct labor and a reduction of material cost of 25% will cause a decrease in
variable cost per unit of $24. The contribution margin after adjustments is $324 per unit
($300 contribution margin before changes + $24 reduction in variable cost). The
contribution margin percentage is found by dividing per unit contribution margin by the
per unit sales price ($324 ÷ $900).
Explanation for Choice A:
This is not the correct answer. Please see the correct answer for an explanation.
We have been unable to determine how to calculate this incorrect answer choice. If you
have calculated it, please let us know how you did it so we can create a full explanation
of why this answer choice is incorrect. Please send us an email at
support@hockinternational.com. Include the full Question ID number and the actual
incorrect answer choice -- not its letter, because that can change with every study
session created. The Question ID number appears at the top of the question. Thank you
in advance for helping us to make your HOCK study materials better.
Explanation for Choice B:
This answer results from dividing the adjusted variable cost per unit by the sales price
per unit. The contribution margin ratio for this problem is the adjusted contribution
margin per unit divided by the sales price per unit.
Explanation for Choice C:
This is not the correct answer. Please see the correct answer for an explanation.
We have been unable to determine how to calculate this incorrect answer choice. If you
have calculated it, please let us know how you did it so we can create a full explanation
of why this answer choice is incorrect. Please send us an email at
support@hockinternational.com. Include the full Question ID number and the actual
incorrect answer choice -- not its letter, because that can change with every study
session created. The Question ID number appears at the top of the question. Thank you
in advance for helping us to make your HOCK study materials better.
11. Question ID: CMA 1294 4.4 (Topic: Assumptions and Basis of CVP Analysis)
The following information relates to Clyde Corporation, which produced and sold 50,000
units during a recent accounting period.

Sales $850,000
Manufacturing costs:
Fixed 210,000
Hock P2 2020
Section C - Decision Analysis.
Answers
Variable 140,000
Selling and administrative costs:
Fixed 300,000
Variable 45,000
Income tax rate: 40%
For the next accounting period, if production and sales are expected to be 40,000 units,
the company should anticipate a contribution margin per unit of

 A. $13.30.correct
 B. $3.10.
 C. $7.30.
 D. $1.86.
Question was not answered
Correct Answer Explanation:
The contribution margin is the difference between sales and variable costs (sales −
variable costs). To find the contribution margin on a per unit basis, calculate the total
contribution margin at a level of 50,000 units and divide by the 50,000 units sold. Since
the contribution margin per unit does not change when the number of units changes, the
contribution margin per unit for 40,000 units will be the same as the contribution margin
per unit for 50,000 units.
Sales of $850,000 − Variable Manufacturing Costs of $140,000 − Variable Selling &
Administrative costs of $45,000 = Contribution Margin of $665,000. Dividing $665,000
by 50,000 units gives us a contribution margin per unit of $13.30.
Explanation for Choice B:
This answer results from subtracting all costs, both variable and fixed, from sales and
dividing the result by 50,000. The contribution margin is sales minus variable costs only.
Explanation for Choice C:
This answer results from subtracting all costs except fixed manufacturing costs from
sales and dividing the result by 50,000. The contribution margin is sales minus variable
costs only.
Explanation for Choice D:
This answer results from subtracting all costs, both variable and fixed, from sales, then
reducing the result by the income taxes due and dividing the final result by 50,000. The
contribution margin is sales minus variable costs only.
Hock P2 2020
Section C - Decision Analysis.
Answers
12. Question ID: CMA 1290 4.2 (Topic: Assumptions and Basis of CVP Analysis)
One of the major assumptions limiting the reliability of breakeven analysis is that

 A. Efficiency and productivity will continually increase.


 B. Total fixed costs will remain unchanged over the relevant range.correct
 C. Total variable costs will remain unchanged over the relevant range.
 D. The cost of production factors varies with changes in technology.
Question was not answered
Correct Answer Explanation:
In order for CVP to actually work mathematically, a number of assumptions need to be
made in order to simplify the real world and all of the variables that occur in it. The main
assumptions of CVP are: costs and revenues are predictable and linear; fixed costs will
remain constant; total variable costs change in proportion to the actual level of activity
while unit variable costs remain constant; inventory does not change; the time value of
money is ignored; unit selling prices remain fixed; and all costs are either variable or
fixed.
Explanation for Choice A:
For CVP analysis to work, efficiency and productivity must remain the same.
Explanation for Choice C:
Per unit variable cost will remain the same, but total variable costs will change in
proportion to the actual level of activity.
Explanation for Choice D:
It is assumed that cost of production is stable.
13. Question ID: ICMA 08.P3.126 (Topic: Assumptions and Basis of CVP Analysis)
Jeffries Company sells its single product for $30 per unit. The contribution margin ratio
is 45%, and fixed costs are $10,000 per month. Sales were 3,000 units in April and
4,000 units in May. How much greater is the May income than the April income?

 A. $13,500.correct
 B. $30,000.
 C. 10,000.
 D. $16,500.
Question was not answered
Correct Answer Explanation:
Since the contribution margin ratio is 45%, the unit contribution margin is $30 × 0.45, or
$13.50. Fixed costs do not change in total with changes in volume, as long as the
Hock P2 2020
Section C - Decision Analysis.
Answers
volume remains within the relevant range. Here, we assume that it does, since the
problem does not say differently. Therefore, the additional contribution margin earned
for the additional 1,000 units sold in May flowed straight to the operating income line,
with no reduction for fixed costs. May's income before taxes was thus greater than
April's income by $13.50 × 1,000 units, or $13,500.
Explanation for Choice B:
This is the difference in the revenue from April to May caused by the increase in sales
volume in May. However, increased revenue does not all flow to the operating income
line, as there will be increased variable costs associated with the additional units sold.
Explanation for Choice C:
This is the amount of fixed costs per month.
Explanation for Choice D:
This answer results from using the variable cost ratio of 1 − 0.45 as the contribution
margin ratio, instead of the contribution margin ratio of 0.45.
14. Question ID: ICMA 19.P2.001 (Topic: Assumptions and Basis of CVP Analysis)
Unit contribution margin is best defined as the difference between the

 A. sales price and variable costs.


 B. unit sales price and the unit variable costs.correct
 C. unit sales price and the unit variable costs divided by the sales price.
 D. unit sales price and variable costs multiplied by the number of units sold.
Question was not answered
Correct Answer Explanation:
Unit contribution margin is the difference between the unit sales price and the unit
variable costs.
Explanation for Choice A:
This is how total contribution is calculated, not unit contribution margin.
Explanation for Choice C:
This is how the unit contribution margin is calculated.
Explanation for Choice D:
This is how total revenue and variable costs are calculated.
15. Question ID: CMA 1273 4.4 (Topic: Assumptions and Basis of CVP Analysis)
Which of the following would decrease unit contribution margin the most?
Hock P2 2020
Section C - Decision Analysis.
Answers
 A. A 15% decrease in variable expenses.
 B. A 15% decrease in fixed expenses.
 C. A 15% decrease in selling price.correct
 D. A 15% increase in variable expenses.
Question was not answered
Correct Answer Explanation:
Assuming that the selling price is greater than variable costs, a 15% decrease in selling
price would have the greatest impact on the unit contribution margin.
Explanation for Choice A:
Decreasing variable expenses will increase the contribution margin.
Explanation for Choice B:
Changing fixed expenses will have no effect on the contribution margin.
Explanation for Choice D:
Assuming the selling price is greater than variable costs, a 15% increase in variable
costs would not have as great an impact on the contribution margin as a 15% decrease
in selling price.
16. Question ID: ICMA 08.P3.123 (Topic: Assumptions and Basis of CVP Analysis)
Allred Company sells its single product for $30 per unit. The contribution margin ratio is
45%, and fixed costs are $10,000 per month. Allred has an effective income tax rate of
40%. If Allread sells 1,000 units in the current month, Allred's variable expenses would
be

 A. $12,000.
 B. $16,500.correct
 C. $9,900.
 D. $13,500.
Question was not answered
Correct Answer Explanation:
Revenue for sales of 1,000 units would be $30 × 1,000, or $30,000. The contribution
margin is total revenue minus total variable costs. If the contribution margin ratio is 45%
of revenue, the variable costs must be 100% − 45%, or 55% of revenue. That is
$30,000 × 0.55, or $16,500.
Explanation for Choice A:
This is total revenue multiplied by the tax rate. That is not variable expenses.
Hock P2 2020
Section C - Decision Analysis.
Answers
Explanation for Choice C:
This is the variable costs multiplied by (1 − the tax rate). That is not variable expenses.
Explanation for Choice D:
This is the contribution margin, not the variable costs. Revenue minus variable costs
equals the contribution margin. Therefore, revenue minus the contribution margin would
equal variable costs.
17. Question ID: ICMA 10.P2.211 (Topic: Assumptions and Basis of CVP Analysis)
Breeze Company has a contribution margin of $4,000 and fixed costs of $1,000. If the
total contribution margin increases by $1,000, operating profit would

 A. increase by $1,000.correct
 B. remain unchanged.
 C. increase by more than $1,000.
 D. decrease by $1,000.
Question was not answered
Correct Answer Explanation:
Contribution margin minus fixed costs equals operating profit. If the contribution margin
is $4,000 and fixed costs are $1,000, operating profit is $3,000. If the contribution
margin increases by $1,000 to $5,000 and fixed costs remain at $1,000, operating profit
will become $4,000, a $1,000 increase.
Explanation for Choice B:
Contribution margin minus fixed costs equals operating profit. If the contribution margin
increases while the fixed costs remain the same, operating profit cannot remain
unchanged.
Explanation for Choice C:
Contribution margin minus fixed costs equals operating profit. If the contribution margin
increases while the fixed costs remain the same, operating profit cannot increase by
more than the contribution margin increases.
Explanation for Choice D:
Contribution margin minus fixed costs equals operating profit. If the contribution margin
increases while the fixed costs remain the same, operating profit will not decrease.
18. Question ID: CMA 691 4.8 (Topic: Assumptions and Basis of CVP Analysis)
In a manufacturing environment, the best short-term profit maximizing approach would
be to

 A. Maximize unit gross profit times the number of units sold.


Hock P2 2020
Section C - Decision Analysis.
Answers
 B. Maximize contribution per unit times the number of units sold.correct
 C. Minimize fixed overhead cost per unit by producing at full capacity.
 D. Minimize variable costs per unit times the number of units produced.
Question was not answered
Correct Answer Explanation:
By maximizing its total contribution margin (unit contribution margin times number of
units sold) a company would have the maximum contribution possible using the
capacity available to cover its fixed costs and contribute to profit. In the long-term,
adjustments can be made to capacity.
Explanation for Choice A:
This would be a long-term strategy, since gross profit would include any fixed cost.
However, the question asks for the best short-term profit maximizing approach.
Explanation for Choice C:
In the short-term, fixed costs are not relevant so therefore are not considered.
Explanation for Choice D:
Minimizing variable costs does not consider the price of the product so therefore might
not maximize short-term profit.
19. Question ID: CMA 694 4.30 (Topic: Breakeven Analysis)
Bruell Electronics Co. is developing a new product, surge protectors for high-voltage
electrical flows. The following cost information relates to the product:

Unit Costs
Direct materials $3.25
Direct labor 4.00
Distribution 0.75
The company will also be absorbing $120,000 of additional fixed costs associated with
this new product. A corporate fixed charge of $20,000 currently absorbed by other
products will be allocated to this new product.
How many surge protectors (rounded to the nearest hundred) must Bruell Electronics
sell at a selling price of $14 per unit to increase after-tax income by $30,000? Bruell
Electronics' effective income tax rate is 40%.

 A. 28,300 units.correct
 B. 10,700 units.
 C. 12,100 units.
Hock P2 2020
Section C - Decision Analysis.
Answers
 D. 20,000 units.
Question was not answered
Correct Answer Explanation:
The breakeven point is calculated by dividing the additional fixed costs plus the desired
pre-tax income by the contribution margin. It is important to note that the given
desired after-tax income is $30,000. To calculate the answer, you need to find the pre-
tax income. The pre-tax income is $50,000 ($30,000 / (1 − 0.40). Fixed costs + the
desired pre-tax net income = $120,000 + $50,000, or $170,000. $170,000 / [$14 −
($3.25 + $4.00 + $0.75)] = 28,333 units, which rounded to the nearest hundred is
28,300 units.
Note that it is possible to work this problem incorrectly but get the correct answer, by
making two offsetting mistakes: (1) by including the corporate fixed charge of $20,000
currently absorbed by other products that will be allocated to this new product as a fixed
cost of the new product. The $20,000 is already being paid by the company, and it will
not change as a result of this new product. Therefore, it is not relevant. And (2) by using
the desired after-tax net income of $30,000 instead of the pre-tax net income of $50,000
in the calculation.
Explanation for Choice B:
This is not the correct answer. Please see the correct answer for an explanation.
We have been unable to determine how to calculate this incorrect answer choice. If you
have calculated it, please let us know how you did it so we can create a full explanation
of why this answer choice is incorrect. Please send us an email at
support@hockinternational.com. Include the full Question ID number and the actual
incorrect answer choice -- not its letter, because that can change with every study
session created. The Question ID number appears at the top of the question. Thank you
in advance for helping us to make your HOCK study materials better.
Explanation for Choice C:
This is not the correct answer. Please see the correct answer for an explanation.
We have been unable to determine how to calculate this incorrect answer choice. If you
have calculated it, please let us know how you did it so we can create a full explanation
of why this answer choice is incorrect. Please send us an email at
support@hockinternational.com. Include the full Question ID number and the actual
incorrect answer choice -- not its letter, because that can change with every study
session created. The Question ID number appears at the top of the question. Thank you
in advance for helping us to make your HOCK study materials better.
Explanation for Choice D:
20,000 units is the breakeven volume. This volume does not include the desired
increase of $30,000 in after-tax income.
Hock P2 2020
Section C - Decision Analysis.
Answers
20. Question ID: ICMA 10.P2.198 (Topic: Breakeven Analysis)
Phillips & Company produces educational software. Its unit cost structure, based upon
an anticipated production volume of 150,000 units, is as follows.

Sales price $160


Variable costs 60
Fixed costs 55
The marketing department has estimated sales for the coming year at 175,000 units,
which is within the relevant range of Phillips' cost structure. Phillips' break-even volume
(in units) and anticipated operating income for the coming year would amount to

 A. 82,500 units and $7,875,000 of operating income.


 B. 96,250 units and $3,543,750 of operating income.
 C. 96,250 units and $7,875,000 of operating income.
 D. 82,500 units and $9,250,000 of operating income.correct
Question was not answered
Correct Answer Explanation:
Total fixed costs will be $8,250,000 ($55 × the budgeted production volume of 150,000
units). The unit contribution margin is $100 ($160 − $60). Therefore, the breakeven
volume is $8,250,000 ÷ $100, which is 82,500.
At a sales volume of 175,000, the total contribution margin would be 175,000 × $100, or
$17,500,000. Subtract budgeted fixed costs of $8,250,000, and the anticipated
operating income for the coming year would be $9,250,000.
Explanation for Choice A:
The operating income results from subtracting both variable costs per unit and fixed
costs per unit from the sales price and then multiplying the result by 175,000 units. This
treats fixed costs as though they were variable costs. However, 175,000 units is within
the relevant range, and therefore, fixed costs will be the same whether 150,000 units or
175,000 units are manufactured.
Explanation for Choice B:
The break-even volume in units results from using a total fixed cost figure that is the
budgeted per unit fixed cost amount of $55 multiplied by the anticipated volume of
175,000 ($9,625,000). This is incorrect, because fixed costs do not change with
changes in production volume, as long as the volume remains within the relevant range,
as this does. The total fixed costs should be calculated by multiplying the budgeted per
unit fixed cost amount of $55 by the budgeted anticipated volume of 150,000. As the
volume increases, fixed costs per unit decrease, as long as the volume remains within
the relevant range.
Hock P2 2020
Section C - Decision Analysis.
Answers
Fixed costs would have to calculated as $13,956,250 to arrive at an operating income
figure of $3,543,750, and fixed costs are not that high.
Explanation for Choice C:
The break-even volume in units results from using a total fixed cost figure that is the
budgeted per unit fixed cost amount of $55 multiplied by the anticipated volume of
175,000 ($9,625,000). This is incorrect, because fixed costs do not change with
changes in production volume, as long as the volume remains within the relevant range,
as this does. The total fixed costs should be calculated by multiplying the budgeted per
unit fixed cost amount of $55 by the budgeted anticipated volume of 150,000. As the
volume increases, fixed costs per unit decrease, as long as the volume remains within
the relevant range.
Operating income was also calculated using the total fixed cost figure of $55 × 175,000,
instead of $55 × 150,000.
21. Question ID: CMA 694 4.29 (Topic: Breakeven Analysis)
Bruell Electronics Co. is developing a new product, surge protectors for high-voltage
electrical flows. The following cost information relates to the product:

Unit Costs
Direct materials $3.25
Direct labor 4.00
Distribution 0.75
The company will also be absorbing $120,000 of additional fixed costs associated with
this new product. A corporate fixed charge of $20,000 currently absorbed by other
products will be allocated to this new product.
How many surge protectors (rounded to the nearest hundred) must Bruell Electronics
sell at a selling price of $14 per unit to gain $30,000 additional income before taxes?

 A. 10,700 units.
 B. 20,000 units.
 C. 12,100 units.
 D. 25,000 units.correct
Question was not answered
Correct Answer Explanation:
The volume needed to earn the desired pre-tax income is calculated by dividing
the additional fixed costs plus the desired pre-tax income by the contribution margin.
Fixed costs + desired pre-tax additional profit = $120,000 + $30,000, or $150,000.
$150,000 ÷ [$14 − ($3.25 + $4.00 + $0.75)] = 25,000 units.
Hock P2 2020
Section C - Decision Analysis.
Answers
The fixed charge of $20,000 currently absorbed by other products that will be allocated
to this new product is not an additional fixed cost, so it is not relevant to calculating
how many surge protectors Bruell must sell to gain $30,000 additional income before
taxes.
Explanation for Choice A:
This is not the correct answer. Please see the correct answer for an explanation.
We have been unable to determine how to calculate this incorrect answer choice. If you
have calculated it, please let us know how you did it so we can create a full explanation
of why this answer choice is incorrect. Please send us an email at
support@hockinternational.com. Include the full Question ID number and the actual
incorrect answer choice -- not its letter, because that can change with every study
session created. The Question ID number appears at the top of the question. Thank you
in advance for helping us to make your HOCK study materials better.
Explanation for Choice B:
This is not the correct answer. Please see the correct answer for an explanation.
We have been unable to determine how to calculate this incorrect answer choice. If you
have calculated it, please let us know how you did it so we can create a full explanation
of why this answer choice is incorrect. Please send us an email at
support@hockinternational.com. Include the full Question ID number and the actual
incorrect answer choice -- not its letter, because that can change with every study
session created. The Question ID number appears at the top of the question. Thank you
in advance for helping us to make your HOCK study materials better.
Explanation for Choice C:
This is not the correct answer. Please see the correct answer for an explanation.
We have been unable to determine how to calculate this incorrect answer choice. If you
have calculated it, please let us know how you did it so we can create a full explanation
of why this answer choice is incorrect. Please send us an email at
support@hockinternational.com. Include the full Question ID number and the actual
incorrect answer choice -- not its letter, because that can change with every study
session created. The Question ID number appears at the top of the question. Thank you
in advance for helping us to make your HOCK study materials better.
22. Question ID: ICMA 19.P2.002 (Topic: Breakeven Analysis)
A detergent company sells large containers of industrial cleaner at a selling price of $12
per container. Each container of cleaner requires $4.50 of direct materials, $2.50 direct
labor, and $1.00 of variable overhead. The company has total fixed costs of $2,000,000
and an income tax rate of 40%. Management has set a goal to achieve a targeted after-
tax net income of $2,400,000. What amount of dollar sales must the company achieve
in order to meet its goal?
Hock P2 2020
Section C - Decision Analysis.
Answers
 A. $18,000,000.correct
 B. $24,000,000.
 C. $14,400,000.
 D. $22,000,000.
Question was not answered
Correct Answer Explanation:
In order to calculate the amount of sales that the company must have to achieve a
certain level of after-tax profit, we will need to:
1) Calculate the level of pre-tax profit that the company is targeting.
2) Calculate the contribution margin per unit
3) Divide the fixed costs and the required pre-tax profit by the contribution margin per
unit to calculate the number of units that need to be sold to reach the profit target
4) Multiply the number of units by the sales price per unit to calculate the sales level at
which the company will achieve its profit target
Here is the work for each of these steps:
1) The after-tax profit is $2,400,000. The before-tax amount is calculated by dividing the
after-tax profit requirement by 1 minus the tax rate. $2,400,000 / (1 - .4) = $4,000,000.
This will be added to the fixed costs to calculate the number of units needed to be sold
to achieve this targeted profit level.
2) The contribution margin per unit is $4.00. It is calculated as the sales price - direct
materials - direct labor - variable overhead.
3) The total fixed costs and required before-tax profit is equal to $6,000,000 and dividing
this by the contribution per unit of $4, we get 1,500,000 as the number of units that need
to be sold to achieve that level of profit. However, the question asks for the amount of
sales, so we need to do one more step.
4) Multiplying the 1,500,000 units by the sales price of $12 per unit gives us the level of
sales needed to have an after-tax profit of $2,400,000.
Explanation for Choice B:
This answer choice incorrectly divided the after-tax profit requirement by the tax rate
instead of (1 - tax rate) in order to calculate the before-tax profit requirement. See the
correct answer for a complete explanation of the question.
Explanation for Choice C:
This answer choice did not include the variable overhead in the calculation of the
contribution margin. See the correct answer for a complete explanation of the question.
Explanation for Choice D:
Hock P2 2020
Section C - Decision Analysis.
Answers
This is not the correct answer. Please see the correct answer for an explanation.
We have been unable to determine how to calculate this incorrect answer choice. If you
have calculated it, please let us know how you did it so we can create a full explanation
of why this answer choice is incorrect. Please send us an email at
support@hockinternational.com. Include the full Question ID number and the actual
incorrect answer choice -- not its letter, because that can change with every study
session created. The Question ID number appears at the top of the question. Thank you
in advance for helping us to make your HOCK study materials better.
23. Question ID: ICMA 10.P2.206 (Topic: Breakeven Analysis)
Bargain Press is considering publishing a new textbook. The publisher has developed
the following cost data related to a production run of 6,000, the minimum possible
production run. Bargain Press will sell the textbook for $45 per copy.

Estimated
Cost
Development (reviews, class testing, editing) $ 35,000
Set-up costs 18,500
Depreciation on Equipment 9,320
General and Administrative 7,500
Miscellaneous Fixed Costs 4,400
Printing and Binding (variable cost) 30,000
Sales staff commissions (variable cost) 5,400
Bookstore commissions (variable cost) 67,500
Author's Royalties (variable cost) 27,000
Total costs at production of 6,000 copies $204,620
How many textbooks must Bargain Press sell in order to generate operating earnings
(earnings before interest and taxes) of 20% on sales? (Round your answer up to the
nearest whole textbook.)

 A. 5,412 copies.
 B. 2,076 copies.
 C. 6,199 copies.
 D. 5,207 copies.correct
Question was not answered
Correct Answer Explanation:
Hock P2 2020
Section C - Decision Analysis.
Answers
The fixed costs are: Development, $35,000; Set-up costs, $18,500; Depreciation,
$9,320; General and Administrative, $7,500; and Miscellaneous Fixed Costs, $4,400,
for a total of $74,720 in fixed costs.
The variable costs are: Printing and Binding, $5.00 per copy ($30,000 ÷ 6,000 copies);
Sales Commissions, $0.90 per copy ($5,400 ÷ 6,000 copies); Bookstore Commissions,
$11.25 per copy ($67,500 ÷ 6,000 copies); and Author's Royalties, $4.50 per copy
($27,000 ÷ 6,000 copies).
Since the desired operating earnings are 20% of revenue, that requirement is treated as
if it were another variable cost, or $9.00 per copy (20% of $45.00). Total variable costs
per unit, including the desired operating earnings, are $5.00 + $0.90 + $11.25 + $4.50 +
$9.00 = $30.65. The unit contribution margin, then, is $45.00 − $30.65, which is $14.35.
Dividing the total fixed costs of $74,720 by the unit contribution margin of $14.35 results
in a volume of 5,206.97, or 5,207 textbooks that need to be sold in order to generate
operating earnings before interest and taxes of 20% on sales. At this sales level, the
fixed costs are covered and an additional 20% of sales revenue is earned, over and
above the expenses.
Explanation for Choice A:
This answer results from incorrectly including Printing and Binding as a fixed expense
instead of a variable expense.
Explanation for Choice B:
This is not the correct answer. Please see the correct answer for an explanation.
We have been unable to determine how to calculate this incorrect answer choice. If you
have calculated it, please let us know how you did it so we can create a full explanation
of why this answer choice is incorrect. Please send us an email at
support@hockinternational.com. Include the full Question ID number and the actual
incorrect answer choice -- not its letter, because that can change with every study
session created. The Question ID number appears at the top of the question. Thank you
in advance for helping us to make your HOCK study materials better.
Explanation for Choice C:
This is not the correct answer. Please see the correct answer for an explanation.
We have been unable to determine how to calculate this incorrect answer choice. If you
have calculated it, please let us know how you did it so we can create a full explanation
of why this answer choice is incorrect. Please send us an email at
support@hockinternational.com. Include the full Question ID number and the actual
incorrect answer choice -- not its letter, because that can change with every study
session created. The Question ID number appears at the top of the question. Thank you
in advance for helping us to make your HOCK study materials better.
24. Question ID: CMA 687 5.24 (Topic: Breakeven Analysis)
Hock P2 2020
Section C - Decision Analysis.
Answers
Gleason Co. has two products, a frozen dessert and ready-to-bake breakfast rolls,
ready for introduction. However, plant capacity is limited, and only one product can be
introduced at present. Therefore, Gleason has conducted a market study, at a cost of
$26,000, to determine which product will be more profitable. The results of the study
follow.

Sales of Desserts Sales of Rolls


at $1.80/unit at $1.20/unit
Volume Probability Volume Probability
250,000 0.30 200,000 0.20
300,000 0.40 250,000 0.50
350,000 0.20 300,000 0.20
400,000 0.10 350,000 0.10
The costs associated with the two products have been estimated by Gleason's cost
accounting department and are shown as follows.

Dessert Rolls
Ingredients per unit $ 0.40 $ 0.25
Direct labor per unit 0.35 0.30
Variable overhead per unit 0.40 0.20
Production tooling* 48,000 25,000
Advertising 30,000 20,000
*Gleason treats production tooling as a current operating expense rather than
capitalizing it as a fixed asset.
In order to recover the costs of production tooling and advertising for the breakfast rolls,
Gleason's sales of the breakfast rolls would have to be

 A. 60,000 units.
 B. 37,500 units.
 C. 100,000 units.correct
 D. Some amount other than those given.
Question was not answered
Correct Answer Explanation:
The formula for the breakeven volume is Total Fixed Costs / Unit Contribution Margin.
The total fixed cost is $45,000 ($25,000 + $20,000). The unit contribution margin is
Hock P2 2020
Section C - Decision Analysis.
Answers
$0.45 ($1.20 − $0.75). $45,000 / $0.45 = 100,000 units that will need to be sold to break
even.
Explanation for Choice A:
This answer results from dividing fixed costs (advertising and production tooling cost) by
the variable cost per unit [$45,000 / ($0.25 + $0.30 + $0.20)]. The formula for the
breakeven volume is Total Fixed Costs / Unit Contribution Margin.
Explanation for Choice B:
This answer results from dividing the total fixed cost of $45,000 by the sale price of
$1.20. The formula for the breakeven volume is Total Fixed Costs / Unit Contribution
Margin.
Explanation for Choice D:
The correct answer is one of the answer choices given.
25. Question ID: CIA 1186 IV.10 (Topic: Breakeven Analysis)
A company has just completed the final development of its only product, general
recombinant bacteria, which can be programmed to kill most insects before dying
themselves. The product has taken 3 years and $6,000,000 to develop. The following
costs are expected to be incurred on a monthly basis for the normal production level of
1,000,000 pounds of the new product:

Direct materials $ 300,000


Direct labor 1,250,000
Variable factory overhead 450,000
Fixed factory overhead 2,000,000
Variable selling, general, and adm. expenses 900,000
Fixed selling, general, and adm. expenses 1,500,000
Total: $6,400,000
At a sales price of $5.90 per pound, the sales in pounds necessary to ensure a
$3,000,000 profit the first year would be (to the nearest thousand pounds):

 A. 14,000,000 pounds.
 B. 13,017,000 pounds.
 C. 25,600,000 pounds.
 D. 15,000,000 pounds.correct
Question was not answered
Correct Answer Explanation:
Hock P2 2020
Section C - Decision Analysis.
Answers
The cost information given is for one month's production. The unit contribution margin is
$3 per pound, calculated as follows:

Selling price per pound $5.90


Less: Variable costs per lb, based on 1,000,000 lbs:
Direct materials: $300,000 ÷ 1,000,000 lbs 0.30
Direct labor: $1,250,000 ÷ 1,000,000 lbs 1.25
Variable factory overhead: $450,000 ÷ 1,000,000 lbs 0.45
Variable selling, general, and adm. expense: $900,000 ÷ 1,000,000 lbs 0.90
Contribution margin per pound $3.00
Fixed factory overhead for one month is $2,000,000, and fixed selling, general and
administrative expense for one month is $1,500,000.
Since the company's income tax rate is not given, we can assume the question is
asking for before-tax profit.
The required profit given is for a year, whereas the costs given are for one month.
Therefore, we must adjust either the fixed costs or the required profit amount so they
are both for the same duration. Since the question asks for an annual sales volume, we
will adjust the costs by multiplying them by 12. Fixed factory overhead for one year is
$24,000,000 and fixed selling, general and administrative expense is $18,000,000.
The formula to find the required number of units to sell to earn a specific profit is:
(Fixed Costs + Required Profit) / Contribution Margin Per Unit
($24,000,000 + $18,000,000 + $3,000,000) / $3 = 15,000,000 pounds
Explanation for Choice A:
This is the breakeven volume, not the volume required to earn a $3,000,000 profit. The
required profit should be included as another fixed cost in the calculation.
Explanation for Choice B:
This is not the correct answer. Please see the correct answer for an explanation.
We have been unable to determine how to calculate this incorrect answer choice. If you
have calculated it, please let us know how you did it so we can create a full explanation
of why this answer choice is incorrect. Please send us an email at
support@hockinternational.com. Include the full Question ID number and the actual
incorrect answer choice -- not its letter, because that can change with every study
session created. The Question ID number appears at the top of the question. Thank you
in advance for helping us to make your HOCK study materials better.
Hock P2 2020
Section C - Decision Analysis.
Answers
Explanation for Choice C:
This is not the correct answer. Please see the correct answer for an explanation.
We have been unable to determine how to calculate this incorrect answer choice. If you
have calculated it, please let us know how you did it so we can create a full explanation
of why this answer choice is incorrect. Please send us an email at
support@hockinternational.com. Include the full Question ID number and the actual
incorrect answer choice -- not its letter, because that can change with every study
session created. The Question ID number appears at the top of the question. Thank you
in advance for helping us to make your HOCK study materials better.
26. Question ID: ICMA 19.P2.003 (Topic: Breakeven Analysis)
Projected sales for a tent manufacturer are $510,000. Each tent sells for $850 and
requires $350 of variable costs to produce. The tent manufacturer’s total fixed costs are
$145,000. The tent manufacturer’s margin of safety is

 A. 710 units.
 B. 730 units.
 C. 1,310 units.
 D. 310 units.correct
Question was not answered
Correct Answer Explanation:
The margin of safety is the difference between the company's actual level of sales
(which we know are $510,000, or 600 units) and the breakeven level of sales. To
calculate the breakeven level of sales we need to divide the fixed costs by the
contribution per unit. The contribution per unit is $500.
The breakeven level of sales is calculated as $145,000 / $500. This gives a breakeven
level of sales of 290 units.
Subtracting breakeven sales from the actual sales gives the margin of safety of 310
units.
Explanation for Choice A:
This is more than the actual units sold by the company. See the correct answer for a
complete explanation of this question.
Explanation for Choice B:
This is more than the actual units sold by the company. See the correct answer for a
complete explanation of this question.
Explanation for Choice C:
Hock P2 2020
Section C - Decision Analysis.
Answers
This is more than the actual units sold by the company. See the correct answer for a
complete explanation of this question.
27. Question ID: CIA 1196 III.91 (Topic: Breakeven Analysis)
A company manufactures a single product. Estimated cost data regarding this product
and other information for the product and the company are as follows:

Sales price per unit $40


Total variable production cost per unit $22
Sales commission (on sales) 5%
Fixed costs and expenses:
Manufacturing overhead $5,598,720
General and administrative $3,732,480
Effective income tax rate 40%
The number of units the company must sell in the coming year in order to reach its
breakeven point is:

 A. 518,400 units.
 B. 972,000 units.
 C. 388,800 units.
 D. 583,200 units.correct
Question was not answered
Correct Answer Explanation:
The answer for this problem is found by dividing Fixed Cost by the unit contribution
margin. The sales price per unit of $40, less the variable production costs of $22, less
sales commission (5% of sales), which are $2 per unit ($40 × 5%), equals a contribution
margin per unit of $16. Total fixed costs are $5,598,720 + $3,732,480, which equals
$9,331,200. $9,331,200 ÷ $16 = 583,200 units.
Taxes are not relevant when calculating the breakeven point, because at that point, net
income will be zero and the tax liability will also be zero.
Explanation for Choice A:
This answer did not include sales commissions in the calculation of variable costs.
Explanation for Choice B:
This answer included taxes in the calculation of variable costs. Taxes are not relevant
when calculating the breakeven point, because at that point, net income will be zero and
the tax liability will also be zero.
Hock P2 2020
Section C - Decision Analysis.
Answers
Explanation for Choice C:
This answer was found by dividing total fixed costs ($9,331,200) by the variable costs
($24).
28. Question ID: CIA 1196 III.90 (Topic: Breakeven Analysis)
Two companies produce and sell the same product in a competitive industry. Thus, the
selling price of the product for each company is the same. Company 1 has a
contribution margin ratio of 40% and fixed costs of $25 million. Company 2 is more
automated, making its fixed costs 40% higher than those of Company 1. Company 2
also has a contribution margin ratio that is 30% greater than that of Company 1. By
comparison, Company 1 will have the _____ breakeven point in terms of dollar sales
volume and will have the _____ dollar profit potential once the indifference point in
dollar sales volume is exceeded.

 A. Lower, Greater
 B. Higher, Lesser
 C. Higher, Greater
 D. Lower, Lessercorrect
Question was not answered
Correct Answer Explanation:
Based on the given information, Company 1 will have a lower breakeven point because
its fixed costs are lower. Also, once the indifference point in dollar sales volume is
exceeded, Company 2 will have higher dollar profit because its contribution margin is
30% greater than Company 1. Company 2's contribution margin ratio is 52% (Company
1's contribution margin ratio of 40% × 1.3), whereas Company 1's contribution margin
ratio is only 40%.
Explanation for Choice A:
Since Company 1 has a lower contribution margin, it will have lower profit once the
indifference point is exceeded.
Explanation for Choice B:
Company 1 has lower fixed costs, so it will have a lower breakeven point.
Explanation for Choice C:
Company 1 has lower fixed costs, so it will have a lower breakeven point. Also,
Company 1 has a lower contribution margin, so once the indifference point is exceeded
it will also have lower profit.
29. Question ID: CIA 591 IV.17 (Topic: Breakeven Analysis)
Hock P2 2020
Section C - Decision Analysis.
Answers
A company makes a product that sells for $30. During the coming year, fixed costs are
expected to be $180,000, and variable costs are estimated at $26 per unit. How many
units must the company sell to break even?

 A. 6,924
 B. None of the answers are correct.
 C. 45,000correct
 D. 6,000
Question was not answered
Correct Answer Explanation:
The breakeven point in units is found by dividing fixed costs by the unit contribution
margin. The unit contribution margin is $4 ($30 − $26). $180,000 Fixed Costs ÷ $4 Unit
Contribution Margin = 45,000 units.
Explanation for Choice A:
This answer results from dividing fixed costs by the variable costs. The breakeven point
in units is found by dividing fixed costs by the unit contribution margin.
Explanation for Choice B:
The correct answer is one of the answer choices. The breakeven point in units is found
by dividing fixed costs by the unit contribution margin.
Explanation for Choice D:
This answer results from dividing fixed cost by the sales price. 6,000 units of sales
would only be enough to cover fixed costs, not the variable costs ($180,000 Fixed Cost
÷ $30 sales price).
30. Question ID: CMA 1286 5.12 (Topic: Breakeven Analysis)
Cost-volume-profit (CVP) analysis is a key factor in many decisions, including choice of
product lines, pricing of products, marketing strategy, and use of productive facilities. A
calculation used in a CVP analysis is the breakeven point. Once the breakeven point
has been reached, operating income will increase by the

 A. Gross margin per unit for each additional unit sold.


 B. Variable costs per unit for each additional unit sold.
 C. Fixed costs per unit for each additional unit sold.
 D. Contribution margin per unit for each additional unit sold.correct
Question was not answered
Correct Answer Explanation:
Hock P2 2020
Section C - Decision Analysis.
Answers
At the breakeven point, total fixed costs are being covered. Any units sold above and
beyond the breakeven point will contribute to profit by the amount of additional units
sold times the unit contribution margin.
Explanation for Choice A:
Gross margin will include some fixed costs.
Explanation for Choice B:
Operating income will increase by the amount of unit contribution margin times the
number of units sold above the breakeven point.
Explanation for Choice C:
At breakeven, fixed costs have already been covered.
31. Question ID: CIA 596 III.84 (Topic: Breakeven Analysis)
A company that sells its single product for $40 per unit uses cost-volume-profit analysis
(CVP) in its planning. The company's after-tax net income for the past year was
$1,188,000 after applying an effective tax rate of 40%. The projected costs for
manufacturing and selling its single product in the coming year are shown below.

Variable cost per unit:


Direct material $ 5.00
Direct labor 4.00
Manufacturing overhead 6.00
Selling and administrative costs 3.00
Total variable cost per unit $18.00
Annual fixed operating costs:
Manufacturing overhead $6,200,000
Selling and administrative costs 3,700,000
Total annual fixed cost $9,900,000
The dollar sales volume required in the coming year to earn the same after-tax net
income as the past year is:

 A. $20,160,000
 B. $26,400,000
 C. $21,600,000correct
 D. $23,400,000
Hock P2 2020
Section C - Decision Analysis.
Answers
Question was not answered
Correct Answer Explanation:
The company desires to maintain the same profit level as the year before. If the after-
tax profit was $1,188,000, then the pre-tax profit would be $1,980,000 ($1,188,000 / (1 -
40%). To find the dollar sales volume required to earn a pre-tax profit of $1,980,000 and
an after-tax profit of $1,188,000, divide total fixed cost plus the desired pre-tax profit by
the unit contribution margin ratio.
Sales price per unit of $40 minus variable cost per unit of $18 = contribution margin per
unit of $22. The contribution margin ratio is thus $22 / $40, which is 55%.
(Fixed costs of $9,900,000 + the required pre-tax profit of $1,980,000) / contribution
margin ratio of 0.55 = $21,600,000.
Explanation for Choice A:
This answer results from using the after-tax profit requirement as a fixed cost in the
calculation. The after-tax profit needs to be converted to its equivalent before-tax profit
before being used.
Explanation for Choice B:
This answer results from using the variable cost per unit instead of the unit contribution
margin in the calculation.
Explanation for Choice D:
This is not the correct answer. Please see the correct answer for an explanation.
We have been unable to determine how to calculate this incorrect answer choice. If you
have calculated it, please let us know how you did it so we can create a full explanation
of why this answer choice is incorrect. Please send us an email at
support@hockinternational.com. Include the full Question ID number and the actual
incorrect answer choice -- not its letter, because that can change with every study
session created. The Question ID number appears at the top of the question. Thank you
in advance for helping us to make your HOCK study materials better.
32. Question ID: CMA 690 5.13 (Topic: Breakeven Analysis)
Madengrad Company manufactures a single electronic product called Precisionmix.
This unit is a batch-density monitoring device attached to large industrial mixing
machines used in flour, rubber, petroleum, and chemical manufacturing. Precisionmix
sells for $900 per unit. The following variable costs are incurred to produce each
Precisionmix device:

Direct labor $180


Direct materials 240
Hock P2 2020
Section C - Decision Analysis.
Answers
Factory overhead 105
Total variable production costs $525
Marketing costs 75
Total variable costs $600
Madengrad's income tax rate is 40%, and annual fixed costs are $6,600,000. Except for
an operating loss incurred in the year of incorporation, the firm has been profitable over
the last 5 years.
The annual sales volume required for Madengrad Company to break even is

 A. 8,400 units.
 B. 11,000 units.
 C. 13,888 units.
 D. 22,000 units.correct
Question was not answered
Correct Answer Explanation:
The formula to calculate the breakeven point in units is: Fixed Costs / Contribution
Margin Per Unit. Fixed costs are $6,600,000 and the contribution margin per unit is
$300 ($900 sale price per unit − $600 total variable costs per unit). Therefore, the
breakeven point is $6,600,000 / $300, which is 22,000 units.
The income tax rate given is not relevant, because if a company just breaks even, it will
have no income tax liability because its profit will be zero.
Explanation for Choice A:
This is not the correct answer. Please see the correct answer for an explanation.
We have been unable to determine how to calculate this incorrect answer choice. If you
have calculated it, please let us know how you did it so we can create a full explanation
of why this answer choice is incorrect. Please send us an email at
support@hockinternational.com. Include the full Question ID number and the actual
incorrect answer choice -- not its letter, because that can change with every study
session created. The Question ID number appears at the top of the question. Thank you
in advance for helping us to make your HOCK study materials better.
Explanation for Choice B:
This answer results from dividing fixed costs by the total variable costs per unit.
However, fixed costs should be divided by the contribution margin per unit.
Explanation for Choice C:
This is not the correct answer. Please see the correct answer for an explanation.
Hock P2 2020
Section C - Decision Analysis.
Answers
We have been unable to determine how to calculate this incorrect answer choice. If you
have calculated it, please let us know how you did it so we can create a full explanation
of why this answer choice is incorrect. Please send us an email at
support@hockinternational.com. Include the full Question ID number and the actual
incorrect answer choice -- not its letter, because that can change with every study
session created. The Question ID number appears at the top of the question. Thank you
in advance for helping us to make your HOCK study materials better.
33. Question ID: ICMA 10.P2.202 (Topic: Breakeven Analysis)
Carson Inc. manufactures only one product and is preparing its budget for next year
based on the following information.

Selling price per unit $ 100


Variable costs per unit 75
Fixed costs 250,000
Effective tax rate 35%
If Carson wants to achieve a net income of $1.3 million next year, its sales must be

 A. 80,000 units.
 B. 70,200 units.
 C. 62,000 units.
 D. 90,000 units.correct
Question was not answered
Correct Answer Explanation:
The first step is to find what before-tax Income needs to be in order to achieve Carson’s
goal of net after-tax income of $1.3 million. Before-tax income = After-tax income ÷ (1 –
the tax rate). Therefore, before-tax Income = $1,300,000 ÷ 0.65, which is $2,000,000.
Volume in units required to earn a specified profit is (Fixed Costs + Before-Tax Profit
Desired) ÷ Contribution Margin Per Unit.
The contribution margin per unit is the $100 selling price minus the $75 variable costs,
or $25.
Fixed cost is given as $250,000.
The volume in units required to earn the desired profit is:
($250,000 + $2,000,000) ÷ $25 = 90,000 units.
Explanation for Choice A:
Hock P2 2020
Section C - Decision Analysis.
Answers
This answer results from dividing the desired before-tax net income by the contribution
margin per unit. This omits consideration of fixed costs. The numerator needs to be the
desired before-tax net income plus the fixed costs.
Explanation for Choice B:
This is not the correct answer. Please see the correct answer for a complete
explanation.
We have been unable to determine how to calculate this incorrect answer choice. If you
have calculated it, please let us know how you did it so we can create a full explanation
of why this answer choice is incorrect. Please send us an email at
support@hockinternational.com. Include the full Question ID number and the actual
incorrect answer choice -- not its letter, because that can change with every study
session created. The Question ID number appears at the top of the question. Thank you
in advance for helping us to make your HOCK study materials better.
Explanation for Choice C:
This answer results from using the after-tax desired net income of $1,300,000 instead of
its before-tax equivalent to calculate the needed volume. The after-tax net income must
be converted to before-tax net income by dividing it by (1 − the tax rate).
34. Question ID: ICMA 13.P2.038 (Topic: Breakeven Analysis)
A company attempts to achieve an annual after-tax operating profit of $2,400,000 by
selling a good for $3,000 per unit. Production of the good involves fixed costs of
$15,000,000 and variable cost per unit of $2,000. Assuming an average income tax rate
of 40%, the volume (in units) required to produce the target amount of profit would be

 A. 16,440.
 B. 19,000.correct
 C. 15,000.
 D. 21,000.
Question was not answered
Correct Answer Explanation:
When the target profit is a fixed dollar amount, the pre-tax profit requirement is treated
as an additional fixed cost in the breakeven calculation. With a tax rate of 40%, the pre-
tax profit that will be needed to have $2,400,000 of after-tax profit is $4,000,000
($2,400,000 divided by 1 − the tax rate). This is added to the $15,000,000 in fixed costs
to give a total "fixed" cost that must be covered of $19,000,000. The contribution per
unit is $1,000, calculated as the selling price minus variable costs per unit ($3,000
minus $2,000). The number of units needed to have $2,400,000 of after-tax profit is
19,000, calculated as $19,000,000 ÷ $1,000.
Explanation for Choice A:
Hock P2 2020
Section C - Decision Analysis.
Answers
This answer choice did not calculate the pre-tax profit requirement properly. It results
from multiplying the after-tax profit required by 1 minus the tax rate instead of
by dividing the after-tax profit required by 1 minus the tax rate.
Explanation for Choice C:
This is the number of units that the company must sell in order to breakeven. At this
level of sales, there will be no profit.
Explanation for Choice D:
The pre-tax profit required is calculated as the after-tax profit required divided by 1
minus the tax rate. This answer results from calculating the pre-tax profit required by
dividing by the tax rate, not by 1 minus the tax rate.
35. Question ID: CMA 690 5.14 (Topic: Breakeven Analysis)
Madengrad Company manufactures a single electronic product called Precisionmix.
This unit is a batch-density monitoring device attached to large industrial mixing
machines used in flour, rubber, petroleum, and chemical manufacturing. Precisionmix
sells for $900 per unit. The following variable costs are incurred to produce each
Precisionmix device:

Direct labor $180


Direct materials 240
Factory overhead 105
Total variable production costs $525
Marketing costs 75
Total variable costs $600
Madengrad's income tax rate is 40%, and annual fixed costs are $6,600,000. Except for
an operating loss incurred in the year of incorporation, the firm has been profitable over
the last 5 years.
For Madengrad Company to achieve an after-tax net income of $540,000, annual sales
revenue must be

 A. $2,700,000
 B. $22,500,000correct
 C. $23,850,000
 D. $21,420,000
Question was not answered
Correct Answer Explanation:
Hock P2 2020
Section C - Decision Analysis.
Answers
The formula to calculate the target profit volume in units is: Fixed Costs + Target Pre-
Tax Income / Contribution Margin Per Unit. To determine the required annual sales
volume to achieve the target net income, we first need to convert the target after-tax net
income to target pre-tax net income, then calculate the number of units of sales required
to meet that goal, and then multiply that number of sales by the sales price.
The desired after-tax profit is $540,000. Therefore, the desired before-tax profit is
$540,000 / (1 − tax rate), which is $540,000 / 0.60, or $900,000.
The target profit volume in units is ($6,600,000 + $900,000) / ($900 − $600), which is
$7,500,000 / $300, or 25,000 units.
The sales revenue required to reach the target after-tax profit is therefore 25,000 ×
$900, or $22,500,000.
Explanation for Choice A:
The formula to calculate the target profit volume in units is: Fixed Costs + Target Pre-
Tax Income / Contribution Margin Per Unit. This answer results from omitting fixed costs
from the numerator.
Explanation for Choice C:
This answer results from dividing the after-tax net income desired by the tax rate to
calculate the equivalent before-tax net income. However, the after-tax net income needs
to be divided by (1 − the tax rate) to convert it to a before-tax net income amount.
Explanation for Choice D:
This answer results from using the target after-tax net income to calculate the number of
units of sales required to meet the desired profit goal. However, the after-tax net income
needs to be converted to before-tax net income first.
36. Question ID: ICMA 19.P2.004 (Topic: Breakeven Analysis)
A company sells two products with the following results for the year just ended.

Product 1 Product 2
Sales $12,000,000 $3,000,000
Variable costs 4,800,000 1,500,000
Fixed costs 5,400,000 400,000
Assuming the product mix and the sales mix remain the same, the company’s
breakeven point in sales dollars is

 A. $13,810,000.
 B. $12,100,000.
 C. $10,000,000.correct
Hock P2 2020
Section C - Decision Analysis.
Answers
 D. $9,800,000.
Question was not answered
Correct Answer Explanation:
Since we are not told how many units have been sold of each product, we will need to
use the contribution margin ratio to calculate the breakeven point instead of contribution
margin per unit.
For Product 1 the contribution margin ratio is 60% ($4,800,000 VC / $12,000,000 sales)
and for Product 2 it is 50% ($1,500,000 VC / $3,000,000 sales). To calculate the
weighted average contribution margin ratio we need to recognize that sales of Product 1
are 4 times higher than Product 2. Another way of expressing that is that the sales of
Product 1 are 80% of total sales and the sales of Product 2 are 20% of total sales. The
calculation of weighted average contribution margin is: (.60 * .8) + (.5 * .2), and this is
equal to .58.
Dividing the total fixed costs of $5,800,000 by the weighted average contribution margin
gives us the breakeven revenue of $10,000,000.
Explanation for Choice A:
This answer incorrectly has the contribution margin ratio of Product 1 as 40% instead of
60%. See the correct answer for a full explanation of this question.
Explanation for Choice B:
This is not the correct answer. Please see the correct answer for an explanation.
We have been unable to determine how to calculate this incorrect answer choice. If you
have calculated it, please let us know how you did it so we can create a full explanation
of why this answer choice is incorrect. Please send us an email at
support@hockinternational.com. Include the full Question ID number and the actual
incorrect answer choice -- not its letter, because that can change with every study
session created. The Question ID number appears at the top of the question. Thank you
in advance for helping us to make your HOCK study materials better.
Explanation for Choice D:
This is not the correct answer. Please see the correct answer for an explanation.
We have been unable to determine how to calculate this incorrect answer choice. If you
have calculated it, please let us know how you did it so we can create a full explanation
of why this answer choice is incorrect. Please send us an email at
support@hockinternational.com. Include the full Question ID number and the actual
incorrect answer choice -- not its letter, because that can change with every study
session created. The Question ID number appears at the top of the question. Thank you
in advance for helping us to make your HOCK study materials better.
37. Question ID: ICMA 10.P2.205 (Topic: Breakeven Analysis)
Hock P2 2020
Section C - Decision Analysis.
Answers
Robin Company wants to earn a 6% return on sales after taxes. The company's
effective income tax rate is 40%, and its contribution margin is 30%. If Robin has fixed
costs of $240,000, the amount of sales required to earn the desired return is

 A. $375,000.
 B. $400,000.
 C. $1,200,000.correct
 D. $1,000,000.
Question was not answered
Correct Answer Explanation:
The company wants to earn a 6% return on sales after taxes. Since the amount of sales
is the unknown, we will let X equal the sales revenue. Therefore, the desired after-tax
profit is 0.06X.
Target pre-tax net income = Target after-tax income ÷ (1 – tax rate)
Target after-tax net income = 0.06X
The tax rate is 0.40, so 1 – the tax rate = 0.60
So the formula for the target pre-tax income is:
Target pre-tax income = 0.06X ÷ 0.60
Target pre-tax income = 0.10X
Now, we can use the profit formula to find X:
Sales – Variable Costs – Fixed Costs = Pre-tax Profit
Sales = X.
Since the contribution margin ratio is 0.30, variable costs are 0.70 of sales, or 0.70X.
Fixed costs are given as $240,000.
The target pre-tax income, per our calculations above, is 0.10X.
Therefore:
X – 0.70X – 240,000 = 0.10X
Solving for X:
0.30X – 240,000 = 0.10X
0.20X = $240,000
X = $1,200,000
Here is an alternate equation that also results in the correct answer, provided by a
student:
240,000 + 0.10X = 0.30X
Solving for X:
240,000 = 0.20X
1,200,000 = X
Hock P2 2020
Section C - Decision Analysis.
Answers
Explanation for Choice A:
This answer results from two mistakes:
(1) Using 30% as the amount of variable cost percentage instead of 70%. Since the
contribution margin is 30%, the variable cost percentage must be 70% of sales (100% −
30%), not 30%.
(2) Not converting the 6% desired after-tax profit margin to its equivalent before-tax
profit margin. The before-tax profit margin is equal to the after-tax profit margin divided
by (1 − the tax rate).
Explanation for Choice B:
This answer results from using 30% as the amount of variable cost percentage instead
of 70%. Since the contribution margin is 30%, the variable cost percentage must be
70% of sales (100% − 30%), not 30%.
Explanation for Choice D:
This answer results from using the desired after-tax net income to calculate the needed
revenue. The desired after-tax net income needs to be converted to the desired before-
tax net income, since revenue is a before-tax amount.
The before-tax profit margin is equal to the after-tax profit margin divided by (1 − the tax
rate).
38. Question ID: ICMA 10.P2.207 (Topic: Breakeven Analysis)
Zipper Company invested $300,000 in a new machine to produce cones for the textile
industry. Zipper's variable costs are 30% of the selling price, and its fixed costs are
$600,000. Zipper has an effective income tax rate of 40%. The amount of sales required
to earn an 8% after-tax return on its investment would be

 A. $2,133,333.
 B. $914,286.correct
 C. $891,429.
 D. $2,080,000.
Question was not answered
Correct Answer Explanation:
The question asks for the amount of sales required to earn an 8% after-tax return on the
investment of $300,000. The first step is to calculate what the desired after-tax amount
is. That is 8% of $300,000, which is $24,000.
The next step is to convert this after-tax amount to its before-tax equivalent. We do that
by dividing it by (1 – the tax rate). Since the tax rate is 40%, (1 – the tax rate) is 60%, or
Hock P2 2020
Section C - Decision Analysis.
Answers
0.60. Dividing $24,000 by 0.60, we get $40,000 of before-tax net income that is
required.
We now set up an equation, letting X = the dollars of sales. If X = the dollars of sales,
then 0.3X = the variable costs.
X − 0.3X – 600,000 = 40,000
0.7X = 640,000
X = $914,286
Explanation for Choice A:
This answer results from using 70% of the selling price as the variable costs and 30% of
the selling price as the contribution margin ratio per unit instead of 30% of the selling
price for the variable costs and 70% of the selling price for the contribution margin ratio
per unit.
Explanation for Choice C:
This answer results from using the desired after-tax return on investment in the formula
to calculate the required sales. The after-tax return needs to be converted to its before-
tax equivalent before it is used in the formula, because the sales revenue required is a
before-tax amount.
Explanation for Choice D:
This is not the correct answer. Please see the correct answer for an explanation.
We have been unable to determine how to calculate this incorrect answer choice. If you
have calculated it, please let us know how you did it so we can create a full explanation
of why this answer choice is incorrect. Please send us an email at
support@hockinternational.com. Include the full Question ID number and the actual
incorrect answer choice -- not its letter, because that can change with every study
session created. The Question ID number appears at the top of the question. Thank you
in advance for helping us to make your HOCK study materials better.
39. Question ID: ICMA 10.P2.201.02 204 (Topic: Breakeven Analysis)
Starlight Theater stages a number of summer musicals at its theater in northern Ohio.
Preliminary planning has just begun for the upcoming season, and Starlight has
developed the following estimated data.

Average
Number of Attendance per Ticket Variable Fixed
Production Performances Performance Price Costs1 Costs2
Mr. Wonderful 12 3,500 $18 $3 $165,000
That's Life 20 3,000 15 1 249,000
All That Jazz 12 4,000 20 0 316,000
Hock P2 2020
Section C - Decision Analysis.
Answers
1
Represent payments to production companies and are based on tickets sold.
2
Costs directly associated with the entire run of each production for costumes, sets, and
artist fees.
Starlight will also incur $565,000 of common fixed operating charges (administrative
overhead, facility costs, and advertising) for the entire season, and is subject to a 30%
income tax rate.
If management desires Mr. Wonderful to produce an after-tax contribution of $210,000
toward the firm's overall operating income for the year, total attendance for the
production would have to be

 A. 20,800.
 B. 25,833.
 C. 25,000.
 D. 31,000.correct
Question was not answered
Correct Answer Explanation:
An after-tax contribution of $210,000 must be converted to its before-tax equivalent by
multiplying the after-tax amount by (1 − tax rate). Thus, the desired before-tax
contribution is $210,000 ÷ 0.70, which is $300,000. This desired before-tax contribution
is treated as a fixed cost and break-even analysis is done.
Total fixed cost, including the desired before-tax contribution, for Mr. Wonderful is
$165,000 + $300,000, or $465,000.
The unit contribution margin for Mr. Wonderful is $15 ($18 − $3). Thus, the total
attendance required for the performances of Mr. Wonderful will be $465,000 ÷ $15, or
31,000.
Explanation for Choice A:
This answer results from converting the after-tax desired contribution from Mr.
Wonderful to a before-tax contribution by multiplying the $210,000 after-tax amount by
(1 − the tax rate). Instead, to convert the after-tax amount to a before-tax amount, it
should be divided by (1 − the tax rate).
Explanation for Choice B:
This answer results from dividing the total fixed costs plus the after-tax desired
contribution for Mr. Wonderful by the sales price of a ticket instead of by the contribution
margin. The contribution margin is the ticket price minus variable costs.
Explanation for Choice C:
Hock P2 2020
Section C - Decision Analysis.
Answers
This answer results from using the desired after-tax contribution from Mr. Wonderful as
a fixed cost. The after-tax desired contribution of $210,000 must be converted to a
before-tax contribution before it can be used as a fixed cost. That is done by dividing it
by (1 − the tax rate).
40. Question ID: ICMA 10.P2.208 (Topic: Breakeven Analysis)
Breakeven quantity is defined as the volume of output at which revenues are equal to

 A. marginal costs.
 B. total costs.correct
 C. variable costs.
 D. fixed costs.
Question was not answered
Correct Answer Explanation:
The breakeven quantity is the number of units that must be sold in order to cover all
costs, both fixed and variable, so that the company does not have a loss.
Explanation for Choice A:
The marginal cost is the cost of the next unit produced or sold (whichever is
appropriate). The breakeven quantity is not the volume where revenues are equal to
marginal cost.
Explanation for Choice C:
Variable costs are costs that are incurred only when a product is made (or sold), such
as material or labor or sales commissions. The total variable cost increases as
production or sales increases, but the per unit variable cost remains unchanged as
production or sales volume increases or decreases. the breakeven quantity is not the
volume where revenues are equal to variable costs. Fixed costs also need to be
covered, or a loss equal to the amount of the fixed costs will result.
Explanation for Choice D:
Fixed costs are costs that do not change, regardless of the level of production or sales,
as long as the volume remains within the relevant range. The breakeven quantity is not
the volume of ourput at which revenues are equal to fixed costs. Variable costs also
need to be covered, or a loss equal to the amount of the variable costs will result.
41. Question ID: ICMA 1603.P2.012 (Topic: Breakeven Analysis)
A company uses cost-volume-profit analysis to evaluate a new product. The total fixed
costs of production per year are $160,000. The unit variable cost is $50. Which one of
the following combinations of unit selling price and breakeven number of units sold per
year is correct?

 A. $100 selling price and 1,600 breakeven number of units.


Hock P2 2020
Section C - Decision Analysis.
Answers
 B. $50 selling price and 3,200 breakeven number of units.
 C. $70 selling price and 8,000 breakeven number of units.correct
 D. $25 selling price and 6,400 breakeven number of units.
Question was not answered
Correct Answer Explanation:
This is a true statement.
With a selling price of $70, the contribution margin would be $70 − $50 = $20. With fixed
production costs of $160,000, the breakeven point would be $160,000 ÷ $20 = 8,000
units.
Explanation for Choice A:
This answer results from mistaking the contribution margin for the selling price. If
the contribution margin were $100, the breakeven number of units given $160,000 in
fixed production costs would be $160,000 ÷ $100 = 1,600 units.
However, the question is asking for the selling price and the breakeven number of
units, not the contribution margin and the breakeven number of units. The selling price
is the conribution margin plus the unit variable cost of $50; and the contribution margin
is the selling price minus the unit variable cost of $50.
The breakeven number of units at a $100 selling price would be $160,000 divided by
the contribution margin of ($100 − $50), which equals 3,200 units. So this is not a true
statement.
Explanation for Choice B:
This answer results from mistaking the contribution margin for the selling price. If
the contribution margin were $50, the breakeven number of units given $160,000 in
fixed production costs would be $160,000 ÷ $50 = 3,200 units.
However, the question is asking for the selling price and the breakeven number of
units, not the contribution margin and the breakeven number of units. The selling price
is the conribution margin plus the unit variable cost of $50; and the contribution margin
is the selling price minus the unit variable cost of $50.
The breakeven number of units at a $50 selling price does not exist. The unit
contribution margin would be $50 − $50, or zero. There would be no contribution margin
available to cover the fixed production costs and no volume at which the company could
break even.
Explanation for Choice D:
This answer results from mistaking the contribution margin for the selling price. If
the contribution margin were $25, the breakeven number of units given $160,000 in
fixed production costs would be $160,000 ÷ $25 = 6,400 units.
Hock P2 2020
Section C - Decision Analysis.
Answers
However, the question is asking for the selling price and the breakeven number of
units, not the contribution margin and the breakeven number of units. The selling price
is the conribution margin plus the unit variable cost of $50; and the contribution margin
is the selling price minus the unit variable cost of $50.
The breakeven number of units at a $25 selling price does not exist. The unit
contribution margin would be $25 − $50, or a $25 loss on each unit sold. There would
be no contribution margin available to cover the fixed production costs and no volume at
which the company could break even.
42. Question ID: CMA 694 4.28 (Topic: Breakeven Analysis)
Bruell Electronics Co. is developing a new product, surge protectors for high-voltage
electrical flows. The following cost information relates to the product:

Unit Costs
Direct materials $3.25
Direct labor 4.00
Distribution 0.75
The company will also be absorbing $120,000 of additional fixed costs associated with
this new product. A corporate fixed charge of $20,000 currently absorbed by other
products will be allocated to this new product.
If the selling price is $14 per unit, the breakeven point in units (rounded to the nearest
hundred) for surge protectors is

 A. 15,000 units.
 B. 8,500 units.
 C. 10,000 units.
 D. 20,000 units.correct
Question was not answered
Correct Answer Explanation:
The breakeven point is calculated by dividing the additional fixed costs by the
contribution margin. Additional fixed costs are $120,000, and the unit contribution
margin is $6 [$14 − ($3.25 + $4.00 + $0.75)]. $120,000 ÷ $6 = 20,000 units.
The fixed charges of $20,000 that will be allocated to this product are costs that will be
there whether or not this new product is produced. If they were not allocated to this
product, they would be allocated to another product or products. Producing or not
producing this product will make no difference in those costs. Therefore, they are not
included in the calculation of the breakeven point for this product.
Hock P2 2020
Section C - Decision Analysis.
Answers
The breakeven point is the number of sales of this product necessary to cover the fixed
costs that will result from the production of this product (not the fixed costs that would
be there whether this product is produced or not). The fixed costs that will be allocated
to this product but which do not result from its production are irrelevant for this analysis.
Explanation for Choice A:
This is the additional fixed cost divided by the variable cost per unit. The added fixed
cost should be divided by the added contribution margin instead.
Explanation for Choice B:
This is not the correct answer. Please see the correct answer for an explanation.
We have been unable to determine how to calculate this incorrect answer choice. If you
have calculated it, please let us know how you did it so we can create a full explanation
of why this answer choice is incorrect. Please send us an email at
support@hockinternational.com. Include the full Question ID number and the actual
incorrect answer choice -- not its letter, because that can change with every study
session created. The Question ID number appears at the top of the question. Thank you
in advance for helping us to make your HOCK study materials better.
Explanation for Choice C:
This answer results from two errors:
1) Reducing the $120,000 in fixed costs by the $20,000 fixed costs that will be allocated
to the new product. The $20,000 in fixed costs to be allocated to the new product are
not relevant to this decision because they are costs that will be there whether or not this
new product is produced. If they were not allocated to this product, they would be
allocated to another product or products. Producing or not producing this product will
make no difference in those costs. Therefore, they are not included in the calculation of
the breakeven point for this product.
2) Dividing that (incorrect) fixed cost amount by a unit contribution margin calculated as
the price reduced only by the direct labor cost per unit. The unit contribution margin
needs to include a reduction for all variable costs.
43. Question ID: CIA 594 III.42 (Topic: Breakeven Analysis)
The following data pertains to XYZ Company for the current year of operations.

Total Per Unit


Sales (40,000 units) $1,000,000 $25
Raw materials 160,000 4
Direct labor 280,000 7
Factory overhead:
Hock P2 2020
Section C - Decision Analysis.
Answers
Variable 80,000 2
Fixed 360,000
Selling and general expenses:
Variable 120,000 3
Fixed 225,000
How many units does XYZ Company need to produce and sell to make a before-tax
profit of 10% of sales?

 A. 36,562 units.
 B. 90,000 units.correct
 C. 25,000 units.
 D. 65,000 units.
Question was not answered
Correct Answer Explanation:
Variable costs per unit total $16 ($4 + $7 + $2 + $3). The required before-tax profit of
10% of the sales price needs to be considered an additional variable cost for this
purpose and the contribution margin needs to be adjusted accordingly. 10% of the $25
sales price is $2.50. Thus the contribution margin for this purpose is $25 − $16 − $2.50
= $6.50.
Fixed costs include both factory overhead ($360,000) and selling & administrative costs
($225,000), or $585,000.
Therefore, the number of units XYZ Company needs to produce and sell to make a
before-tax profit of 10% of sales is $585,000 ÷ $6.50 = 90,000.
Explanation for Choice A:
This is total fixed costs divided by variable costs per unit. The correct answer is total
fixed costs divided by an adjusted contribution margin where the required profit per unit
is included as a variable cost.
Explanation for Choice C:
This is the fixed selling and administrative costs divided by the contribution margin per
unit. Total fixed costs include both factory overhead and selling & administrative costs;
and the contribution margin per unit needs to be adjusted to include the amount of
required profit per unit as a variable cost.
Explanation for Choice D:
65,000 units is the breakeven volume, where operating profit is zero.
Hock P2 2020
Section C - Decision Analysis.
Answers
44. Question ID: CMA 692 4.29 (Topic: Breakeven Analysis)
Barnes Corporation manufactures skateboards and is in the process of preparing next
year's budget. The pro forma income statement for the current year is presented as
follows.

Sales $1,500,000
Cost of sales:
Direct materials 250,000
Direct labor 150,000
Variable overhead 75,000
Fixed overhead 100,000
Gross profit $ 925,000
Selling and G&A Variable 200,000
Selling and G&A Fixed 250,000
Operating income $ 475,000
The breakeven point (rounded to the nearest dollar) for Barnes Corporation for the
current year is

 A. $146,341.
 B. $181,818.
 C. $729,730.
 D. $636,364.correct
Question was not answered
Correct Answer Explanation:
The breakeven revenue is found by dividing total fixed cost by the contribution margin
ratio. Sales of $1,500,000 less variable cost of $675,000 equals contribution margin of
$825,000. The contribution margin ratio is $825,000 ÷ $1,500,000, which is 55%. Total
fixed costs are $100,000 + $250,000, for a total of $350,000 fixed costs. $350,000 ÷
0.55 = breakeven point in sales revenue of $636,364.
Explanation for Choice A:
The breakeven revenue is found by dividing total fixed cost by the contribution margin
ratio. This answer results from using only fixed manufacturing overhead as the fixed
cost and using only variable manufacturing costs in calculating the contribution margin
ratio. Fixed cost should include non-manufacturing fixed costs, and the calculation of
the contribution margin ratio should include non-manufacturing variable costs.
Hock P2 2020
Section C - Decision Analysis.
Answers
Explanation for Choice B:
The breakeven revenue is found by dividing total fixed cost by the contribution margin
ratio. This answer results from using fixed costs totaling $100,000. Total fixed costs are
more than $100,000.
Explanation for Choice C:
The breakeven revenue is found by dividing total fixed cost by the contribution margin
ratio. This is total selling, general and administrative costs (both fixed and variable)
divided by the gross profit ratio.
45. Question ID: ICMA 10.P2.201.01.201 (Topic: Breakeven Analysis)
Starlight Theater stages a number of summer musicals at its theater in northern Ohio.
Preliminary planning has just begun for the upcoming season, and Starlight has
developed the following estimated data.

Average
Number of Attendance per Ticket Variable Fixed
Production Performances Performance Price Costs1 Costs2
Mr. Wonderful 12 3,500 $18 $3 $165,000
That's Life 20 3,000 15 1 249,000
All That Jazz 12 4,000 20 0 316,000
1
Represent payments to production companies and are based on tickets sold.
2
Costs directly associated with the entire run of each production for costumes, sets, and
artist fees.
Starlight will also incur $565,000 of common fixed operating charges (administrative
overhead, facility costs, and advertising) for the entire season, and is subject to a 30%
income tax rate.
If Starlight's schedule of musicals is held as planned, how many patrons would have to
attend for Starlight to break even during the summer season?

 A. 81,390.
 B. 79,938.correct
 C. 77,918.
 D. 79,302.
Question was not answered
Correct Answer Explanation:
This is a break-even analysis when more than one product is sold.
Hock P2 2020
Section C - Decision Analysis.
Answers
The weighted average unit contribution margin is based on total attendance, as follows:

# Avg. Att. Total % of


Production UCM
Perf. Per Perf. Attendance Total
Mr. Wonderful 12 × 3,500 = 42,000 0.28 $15
That's Life 20 × 3,000 = 60,000 0.40 14
All That Jazz 12 × 4,000 = 48,000 0.32 20
150,000
The weighted average unit contribution margin is: (0.28 × $15) + (0.40 × $14) + (0.32 ×
20) = $16.20
Total fixed costs are $730,000 + $565,000, or $1,295,000.
Therefore, the break-even point in total units for this 3-product firm is: $1,295,000 ÷
$16.20, which is 79,938.
Explanation for Choice A:
This answer results from using the number of performances of each production to
calculate the weights for the weighted average unit contribution margin. The weighting
should be based on total attendance at the performances, which incorporates both the
number of performances of each musical and the average attendance per performance.
Explanation for Choice C:
This answer results from using the average attendance per performance to calculate the
weights for the weighted average unit contribution margin. The weighting should be
based on total attendance at the performances, which incorporates both the number of
performances of each musical and the average attendance per performance.
Explanation for Choice D:
This answer results from using a simple average (unweighted) of the unit contribution
margins for the three performances to calculate the breakeven volume. The average
unit contribution margin used should be a weighted unit contribution margin. The
weighting should be based on total attendance at the performances, which incorporates
both the number of performances of each musical and the average attendance per
performance.
46. Question ID: CIA 594 III.43 (Topic: Using CVP Analysis in Decision-Making)
The following data pertains to XYZ Company for the current year of operations.

Total Per Unit


Sales (40,000 units) $1,000,000 $25
Hock P2 2020
Section C - Decision Analysis.
Answers
Raw materials 160,000 4
Direct labor 280,000 7
Factory overhead:
Variable 80,000 2
Fixed 360,000
Selling and general expenses:
Variable 120,000 3
Fixed 225,000
Assuming that XYZ Company sells 80,000 units, what is the maximum that can be paid
for an advertising campaign while still breaking even?

 A. $135,000correct
 B. $1,015,000
 C. $695,000
 D. $535,000
Question was not answered
Correct Answer Explanation:
This problem can be solved in more than one way. Here is one way:
The contribution margin per unit is $9 ($25 − $16 total variable costs). The total
contribution margin at a volume of 80,000 units is 80,000 × $9, or $720,000.
Total fixed costs are $585,000 ($360,000 + $225,000). Whenever the total contribution
margin and total fixed costs are equal, the company is breaking even. Therefore, letting
X equal the maximum amount that can be paid for an advertising campaign (a fixed
cost) while not exceeding the total contribution margin, the formula to find X is:
$585,000 + X = $720,000
X = $135,000
Explanation for Choice B:
This is not the correct answer. Please see the correct answer for an explanation.
We have been unable to determine how to calculate this incorrect answer choice. If you
have calculated it, please let us know how you did it so we can create a full explanation
of why this answer choice is incorrect. Please send us an email at
support@hockinternational.com. Include the full Question ID number and the actual
incorrect answer choice -- not its letter, because that can change with every study
Hock P2 2020
Section C - Decision Analysis.
Answers
session created. The Question ID number appears at the top of the question. Thank you
in advance for helping us to make your HOCK study materials better.
Explanation for Choice C:
This is not the correct answer. Please see the correct answer for an explanation.
We have been unable to determine how to calculate this incorrect answer choice. If you
have calculated it, please let us know how you did it so we can create a full explanation
of why this answer choice is incorrect. Please send us an email at
support@hockinternational.com. Include the full Question ID number and the actual
incorrect answer choice -- not its letter, because that can change with every study
session created. The Question ID number appears at the top of the question. Thank you
in advance for helping us to make your HOCK study materials better.
Explanation for Choice D:
This is not the correct answer. Please see the correct answer for an explanation.
We have been unable to determine how to calculate this incorrect answer choice. If you
have calculated it, please let us know how you did it so we can create a full explanation
of why this answer choice is incorrect. Please send us an email at
support@hockinternational.com. Include the full Question ID number and the actual
incorrect answer choice -- not its letter, because that can change with every study
session created. The Question ID number appears at the top of the question. Thank you
in advance for helping us to make your HOCK study materials better.
47. Question ID: ICMA 10.P2.214 (Topic: Using CVP Analysis in Decision-Making)
Specialty Cakes Inc. produces two types of cakes, a 2 lbs. round cake and a 3 lbs.
heart-shaped cake. Total fixed costs for the firm are $94,000. Variable costs and sales
data for the two types of cakes are presented below.

2 lbs. 3 lbs.
Round Cake Heart-shaped Cake
Selling price per unit $12 $20
Variable cost per unit $ 8 $15
Current sales (units) 10,000 15,000
If the product sales mix were to change to three heart-shaped cakes for each round
cake, the breakeven volume for each of these products would be

 A. 15,326 round cakes, 8,109 heart-shaped cakes.


 B. 12,261 round cakes, 8,174 heart-shaped cakes.
 C. 8,174 round cakes, 12,261 heart-shaped cakes.
 D. 4,948 round cakes, 14,843 heart-shaped cakes.correct
Hock P2 2020
Section C - Decision Analysis.
Answers
Question was not answered
Correct Answer Explanation:
The contribution margin for round cakes is $4 ($12 − $8). The contribution margin for
heart-shaped cakes is $5 ($20 − $15).
If three heart-shaped cakes are sold for each 1 round cake, the heart-shaped cakes will
represent 75% of sales and round cakes will represent 25% of sales.
The unit contribution margin is a weighted average of the contribution margins of the
two different cakes, weighted according to their percentages of sales. That is: (0.25 ×
$4) + (0.75 × $5) = $4.75.
The break-even volume in total number of units is the fixed cost of $94,000 divided by
the weighted average unit contribution margin of $4.75, which is 19,789.47 units. This
19,789.47 units is split between heart-shaped cakes and round cakes at the ratio of
75% heart-shaped cakes and 25% round cakes. Therefore, the number of each type of
cake to break even at this product mix is:

 Round cakes: 19,789.47 × 0.25 = 4,947.34 or 4,948 (since a partial cake cannot be
sold); and
 
Heart-shaped cakes: 19,789.47 × 0.75 = 14,842.10, or 14,843.
Explanation for Choice A:
This is not the correct answer. Please see the correct answer for a complete
explanation.
We have been unable to determine how to calculate this incorrect answer choice. If you
have calculated it, please let us know how you did it so we can create a full explanation
of why this answer choice is incorrect. Please send us an email at
support@hockinternational.com. Include the full Question ID number and the actual
incorrect answer choice -- not its letter, because that can change with every study
session created. The Question ID number appears at the top of the question. Thank you
in advance for helping us to make your HOCK study materials better.
Explanation for Choice B:
This is the break-even volume at the product sales mix, although the volumes for the
two products are reversed. But even if the reversal were corrected, this would not be the
correct answer because this is for the current product sales mix, not the expected sales
mix.
Explanation for Choice C:
This is the break-even volume at the current product sales mix of round and heart-
shaped cakes. If the product sales mix changes to three heart-shaped cakes for each
round cake, the breakeven volume for the two cakes will be different.
Hock P2 2020
Section C - Decision Analysis.
Answers
48. Question ID: CMA 692 4.30 (Topic: Using CVP Analysis in Decision-Making)
Barnes Corporation manufactures skateboards and is in the process of preparing next
year's budget. The pro forma income statement for the current year is presented as
follows.

Sales $1,500,000
Cost of sales:
Direct materials 250,000
Direct labor 150,000
Variable overhead 75,000
Fixed overhead 100,000
Gross profit $ 925,000
Selling and G&A Variable 200,000
Selling and G&A Fixed 250,000
Operating income $ 475,000
For the coming year, the management of Barnes Corporation anticipates a 10%
increase in sales, a 12% increase in variable costs, and a $45,000 increase in fixed
expenses. The breakeven point for next year will be

 A. $729,027.correct
 B. $474,000.
 C. $214,018.
 D. $862,103.
Question was not answered
Correct Answer Explanation:
The breakeven point in revenue is total fixed costs divided by the contribution margin
ratio percentage. The answer for this problem is found by calculating the next year's
budgeted sales, next year's budgeted variable costs and next year's budgeted fixed
costs, calculating the budgeted contribution margin ratio percentage, which is the
budgeted contribution margin divided by budgeted sales, and then dividing the total
budgeted fixed costs by the budgeted contribution margin ratio percentage.
Current year sales are $1,500,000, so a 10% increase would be $1,650,000
($1,500,000 × 110%). Current year variable costs are $250,000 + $150,000 + $75,000 +
$200,000, or $675,000. So a 12% increase would be $675,000 × 112%, or $756,000.
Thus, the budgeted contribution margin would be $1,650,000 − $756,000, or $894,000.
Hock P2 2020
Section C - Decision Analysis.
Answers
The budgeted contribution margin ratio is therefore $894,000 ÷ $1,650,000, or
0.54181818.
Total fixed cost for the current year is $100,000 + $250,000, or $350,000. The budgeted
fixed cost is $350,000 + $45,000, or $395,000.
Therefore, the breakeven point in revenue for next year's budget would be $395,000 ÷
0.54181818 = $729,027.
Explanation for Choice B:
The breakeven point in revenue is total fixed costs divided by the contribution margin
ratio percentage. The answer for this problem is found by calculating the next year's
budgeted sales, next year's budgeted variable costs and next year's budgeted fixed
costs, calculating the budgeted contribution margin ratio percentage, which is the
budgeted contribution margin divided by budgeted sales, and then dividing the total
budgeted fixed costs by the budgeted contribution margin ratio percentage.
This is not the correct answer. We have been unable to determine how to calculate this
incorrect answer choice. If you have calculated it, please let us know how you did it so
we can create a full explanation of why this answer choice is incorrect. Please send
us an email at support@hockinternational.com. Include the full Question ID number and
the actual incorrect answer choice -- not its letter, because that can change with every
study session created. The Question ID number appears at the top of the question.
Thank you in advance for helping us to make your HOCK study materials better.
Explanation for Choice C:
The breakeven point in revenue is total fixed costs divided by the contribution margin
ratio percentage. The answer for this problem is found by calculating the next year's
budgeted sales, next year's budgeted variable costs and next year's budgeted fixed
costs, calculating the budgeted contribution margin ratio percentage, which is the
budgeted contribution margin divided by budgeted sales, and then dividing the total
budgeted fixed costs by the budgeted contribution margin ratio percentage.
This incorrect answer results from multiplying the budgeted fixed costs by the
budgeted contribution margin ratio percentage instead of dividing the budgeted fixed
costs by the budgeted contribution margin ratio percentage.
Explanation for Choice D:
The breakeven point in revenue is total fixed costs divided by the contribution margin
ratio percentage. The answer for this problem is found by calculating the next year's
budgeted sales, next year's budgeted variable costs and next year's budgeted fixed
costs, calculating the budgeted contribution margin ratio percentage, which is the
budgeted contribution margin divided by budgeted sales, and then dividing the total
budgeted fixed costs by the budgeted contribution margin ratio percentage.
Hock P2 2020
Section C - Decision Analysis.
Answers
This incorrect answer results from calculating the budgeted contribution margin ratio
percentage incorrectly, by dividing next year's budgeted variable costs by next year's
budgeted sales, instead of dividing next year's budgeted contribution margin by next
year's budgeted sales.
49. Question ID: CMA 1294 4.2 (Topic: Using CVP Analysis in Decision-Making)
United Industries manufactures three products at its highly automated factory. The
products are very popular, with demand far exceeding the company's ability to supply
the marketplace. To maximize profit, management should focus on each product's

 A. Segment margin.
 B. Contribution margin ratio.
 C. Gross margin.
 D. Contribution margin per machine hour.correct
Question was not answered
Correct Answer Explanation:
The contribution margin per machine hour will indicate which product generates the
most profit per unit of the scarce resource (machine hours). The company will maximize
its profitability by producing those products that have the greatest contribution margin
per scarce resource.
Explanation for Choice A:
Since the company can sell all the product it can produce, the limiting factor is the use
of scarce resources (machine hours). The segment margin does not tell us anything
about the profitability of products on the basis of how much of the scarce resource each
requires.
Explanation for Choice B:
The contribution margin ratio is useful only if it is put into sales dollars. It is possible that
a higher ratio, lower volume product could be unprofitable.
Explanation for Choice C:
Since the company can sell all the product it can produce, the limiting factor is the use
of scarce resources (machine hours). The gross margin does not tell us anything about
the profitability of products on the basis of how much of the scarce resource each
requires.
50. Question ID: CMA 1291 4.15 (Topic: Using CVP Analysis in Decision-Making)
Siberian Ski Company recently expanded its manufacturing capacity, which will allow it
to produce up to 15,000 pairs of cross-country skis of the mountaineering model or the
touring model. The Sales Department assures management that it can sell between
9,000 pairs and 13,000 pairs of either product this year. Because the models are very
similar, Siberian Ski will produce only one of the two models.
Hock P2 2020
Section C - Decision Analysis.
Answers
The following information was compiled by the Accounting Department.

Per Unit (Pair) Data


Mountaineering Touring
Selling price $88.00 $80.00
Variable costs 52.80 52.80
Fixed costs will total $369,600 if the mountaineering model is produced but will be only
$316,800 if the touring model is produced. Siberian Ski is subject to a 40% income tax
rate.
If the Siberian Ski Company Sales Department could guarantee the annual sale of
12,000 pairs of either model, Siberian Ski would

 A. Produce 12,000 pairs of mountaineering skis because they are more profitable.correct
 B. Produce 12,000 pairs of touring skis because they have a lower fixed cost.
 C. Be indifferent as to which model is sold because each model has the same variable
cost per unit.
 D. Produce 12,000 pairs of mountaineering skis because they have a lower breakeven
point.
Question was not answered
Correct Answer Explanation:
The easiest way to find the solution is to prepare simple income statements and do a
comparison based on sales of 12,000 units.

Mountaineering Touring
Sales $1,056,000 $960,000
Variable costs 633,600 633,600
Fixed costs 369,600 316,800
Net operating income $ 52,800 $ 9,600
Based on the information given, the Mountaineering ski would be more profitable, even
though fixed costs are greater.
Explanation for Choice B:
Fixed cost is not the only factor that determines which product is more profitable.
Explanation for Choice C:
Hock P2 2020
Section C - Decision Analysis.
Answers
The amount of variable cost per unit is not the only factor that determines which product
is more profitable.
Explanation for Choice D:
Having a lower breakeven point does not necessarily mean that the Mountaineering skis
will be more profitable since the sales level of 12,000 units is above the breakeven point
for both skis.
51. Question ID: ICMA 13.P2.041 (Topic: Using CVP Analysis in Decision-Making)
TwelCo produces and sells two main products, with contribution margins per unit as
follows.
Product A: $10.00 per unit
Product B: $ 8.00 per unit
Fixed costs for the year are budgeted at $264,480, and TwelCo calculated its break-
even point at 28,500 units. What percentage of units sold are expected to be Product
A?

 A. 44%.
 B. 64%.correct
 C. 56%.
 D. 36%.
Question was not answered
Correct Answer Explanation:
The first step in solving this problem is to calculate the weighted average contribution
margin per unit. We know that fixed cost divided by the weighted average contribution
margin per unit equals the breakeven point in units. Therefore:

$264,480
= 28,500
WACM per unit
Solving for WACM per unit, we get WACM per unit = $9.28.
Product A represents an unknown percentage in the basket of goods, and that is what
we need to find. Therefore, let A equal the unknown percentage of Product A. The
unknown percentage of Product B in the basket will be 1 − A, since the percentages of
A and B must total to 1:
$10A + $8(1 − A) = $9.28
Solving for A:
10A + 8 − 8A = 9.28
2A + 8 = 9.28
Hock P2 2020
Section C - Decision Analysis.
Answers
2A = 1.28
A = 0.64 or 64%
Explanation for Choice A:
This is the contribution margin of Product B divided by the total of the contribution
margins of both Product A and Product B ($10 divided by $18). This is not the correct
way to calculate the percentage of units sold expected to be Product A.
The first step in solving this problem is to calculate the weighted average contribution
margin per unit. Fixed cost divided by the weighted average contribution margin per unit
equals the breakeven point in units.

Fixed Cost
= Breakeven Point in Units
WACM per unit
Using that formula, plug in the Fixed Cost and the Breakeven Point in Units and solve
for WACM per unit.
Product A represents an unknown percentage in the basket of goods, and that is what
we need to find. Therefore, let A equal the unknown percentage of Product A. The
unknown percentage of Product B in the basket will be 1 - A, since the percentages of A
and B must total to 1. The equation to solve is:
$10A + $8(1 − A) = Weighted Average Contribution Margin Per Unit
Explanation for Choice C:
This is the contribution margin of Product A divided by the total of the contribution
margins of both Product A and Product B ($10 divided by $18). This is not the correct
way to calculate the percentage of units sold expected to be Product A.
The first step in solving this problem is to calculate the weighted average contribution
margin per unit. Fixed cost divided by the weighted average contribution margin per unit
equals the breakeven point in units.

Fixed Cost
= Breakeven Point in Units
WACM per unit
Using that formula, plug in the Fixed Cost and the Breakeven Point in Units and solve
for WACM per unit.
Product A represents an unknown percentage in the basket of goods, and that is what
we need to find. Therefore, let A equal the unknown percentage of Product A. The
unknown percentage of Product B in the basket will be 1 - A, since the percentages of A
and B must total to 1. The equation to solve is:
$10A + $8(1 − A) = Weighted Average Contribution Margin Per Unit
Explanation for Choice D:
Hock P2 2020
Section C - Decision Analysis.
Answers
This is the percentage represented by Product B.
52. Question ID: ICMA 10.P2.354 (Topic: Using CVP Analysis in Decision-Making)
Susan Hines has developed an estimate of the earnings per share for her firm, a
reseller of lawn equipment, for the next year using the following parameters.

Sales $20 million


Cost of goods sold 70% of sales
General & administrative expenses $300,000
Selling expense $100,000 plus 10% of sales
Debt outstanding $5 million @ 8% interest rate
Effective tax rate 35%
Common shares outstanding 2 million
She is now interested in the sensitivity of earnings per share to sales forecast changes.
A 10% sales increase would increase earnings per share by

 A. 7.0 cents per share.


 B. 13.0 cents per share.correct
 C. 20.0 cents per share.
 D. 10.4 cents per share.
Question was not answered
Correct Answer Explanation:
Since the firm is a reseller, all of its costs in cost of goods sold are variable costs
because they represent the cost to purchase the product that was resold. The current
gross profit margin is $6,000,000 ($20,000,000 × 0.30). If sales increase by 10%, the
gross profit margin will increase by 10% of $6,000,000, or $600,000.
Selling expense has a fixed component and a variable component. The variable
component will increase by 10% of the increased sales. Sales will increase by
$2,000,000 (10% of $20,000,000), so selling expense will increase by $2,000,000 ×
0.10, or $200,000.
Net income before tax will therefore increase by $600,000 − $200,000, or $400,000.
The increase in net income after tax will be $400,000 × (1 − 0.35), which is $260,000.
There are 2,000,000 shares outstanding, so the increased net income after tax per
share will be $260,000 ÷ 2,000,000, which is $0.13 per share.
Explanation for Choice A:
This is not the correct answer. Please see the correct answer for an explanation.
Hock P2 2020
Section C - Decision Analysis.
Answers
We have been unable to determine how to calculate this incorrect answer choice. If you
have calculated it, please let us know how you did it so we can create a full explanation
of why this answer choice is incorrect. Please send us an email at
support@hockinternational.com. Include the full Question ID number and the actual
incorrect answer choice -- not its letter, because that can change with every study
session created. The Question ID number appears at the top of the question. Thank you
in advance for helping us to make your HOCK study materials better.
Explanation for Choice C:
This answer results from using the amount of increase in net income before tax. The
increase in net income after tax should be used instead.
Explanation for Choice D:
This answer results from increasing earnings per share by 10% to reflect the 10%
increase in sales. Only revenue and variable expenses will increase as a result of the
increase in sales. Fixed expenses will be unchanged. For that reason, the percentage
increase in earnings per share will not be the same as the percentage of increase in
sales.
53. Question ID: CMA 687 4.14 (Topic: Using CVP Analysis in Decision-Making)
For a profitable company, the amount by which sales can decline before losses occur is
known as the

 A. Marginal income rate.


 B. Sales volume variance.
 C. Hurdle rate.
 D. Margin of safety.correct
Question was not answered
Correct Answer Explanation:
The margin of safety is the amount by which sales can decrease before losses can
occur (budgeted/actual sales − sales level at breakeven).
Explanation for Choice A:
The marginal income rate is the rate of return that is gained from making one more sale
or one more investment.
Explanation for Choice B:
The Sales volume variance measures the impact of differences in sales volume. The
calculation is [(Actual Sales volume − Budgeted Sales volume) × Standard contribution
per unit].
Explanation for Choice C:
Hock P2 2020
Section C - Decision Analysis.
Answers
The hurdle rate is a capital budgeting term. It is the rate of return that must received on
a specific project under consideration before it will be acceptable to management.
54. Question ID: ICMA 10.P2.258 (Topic: Using CVP Analysis in Decision-Making)
Milton Manufacturing occasionally has capacity problems in its metal shaping division,
where the chief cost driver is machine hours. In evaluating the attractiveness of its
individual products for decision-making purposes, which measurement tool should the
firm select?

 A. If machine hours do not constrain the number of units produced, contribution margin
per machine hour. If machine hours constrain the number of units produced, contribution
margin.
 B. If machine hours do not constrain the number of units produced, gross profit. If
machine hours constrain the number of units produced, contribution margin.
 C. If machine hours do not constrain the number of units produced, contribution margin. If
machine hours constrain the number of units produced, contribution margin per machine
hour.correct
 D. If machine hours do not constrain the number of units produced, contribution margin. If
machine hours constrain the number of units produced, contribution margin ratio.
Question was not answered
Correct Answer Explanation:
If machine hours are not limited, the company would give priority to producing the
product with the highest contribution margin, assuming there is a market for all of the
units the company would choose to produce. If machine hours are limited, by comparing
the contribution margins per machine hour required to produce one unit of product for
all product options, the company can select the product that will provide the greatest
contribution to fixed costs and the bottom line per limited hour, again assuming there is
a market for all of the units the company would be able to produce. If unused machine
hours remain after the company produces all it can sell of the product with the highest
contribution margin per machine hour, an additional product could be considered.
Explanation for Choice A:
When there is unlimited capacity, it is appropriate to evaluate products based on their
contribution margin. However, when a resource is limited, you need to determine the
contribution margin per unit of that limited resource. Therefore, these are reversed.
Explanation for Choice B:
Machine hours are limited and are frequently a problem for this division. The company
needs to determine how to allocate these hours in order to maximize their value. To do
this, the company needs to calculate the contribution margin per unit of the limited
resource, which in this case is machine hours.
Explanation for Choice D:
Hock P2 2020
Section C - Decision Analysis.
Answers
Contribution margin is a great way to evaluate products when there is unlimited
capacity. However, when you have limited availability of a resource such as machine
hours, you need to determine which product will provide the highest contribution
margin per unit of the limited resource and give priority to producing that product,
assuming of course that there is a market for all you can produce.
55. Question ID: CMA 679 5.13 (Topic: Using CVP Analysis in Decision-Making)
BE&H Manufacturing is considering dropping a product line. It currently produces a
multi-purpose woodworking clamp in a simple manufacturing process that uses special
equipment. Variable costs amount to $6.00 per unit. Fixed overhead costs, exclusive of
depreciation, have been allocated to this product at a rate of $3.50 a unit and will
continue whether or not production ceases. Depreciation on the special equipment
amounts to $20,000 a year. If production of the clamp is stopped, the special equipment
can be sold for $18,000; if production continues, however, the equipment will be useless
for further production at the end of 1 year and will have no salvage value. The clamp
has a selling price of $10 a unit. Ignoring tax effects, the minimum number of units that
would have to be sold in the current year to break even on a cash flow basis is

 A. 5,000
 B. 20,000
 C. 36,000
 D. 4,500correct
Question was not answered
Correct Answer Explanation:
To calculate the breakeven point in this situation, we need to use the amount that the
equipment could be sold for this year as the fixed costs. The reason for using the
salvage value today is that in a year's time, the salvage value will be zero; and the
question asks for the breakeven point on a cash flow basis. So $18,000 is the
(opportunity) cost of continuing to produce the part for another year.
The contribution margin is $10 − $6, or $4. Thus, the breakeven point on a cash flow
basis for the current year is $18,000 ÷ $4, or 4,500 units.
Explanation for Choice A:
This answer results from using the amount of the depreciation as the fixed cost. The
question asks for the breakeven point on a cash flow basis, and depreciation is a
noncash expense.
Explanation for Choice B:
This is not the correct answer. Please see the correct answer for an explanation.
We have been unable to determine how to calculate this incorrect answer choice. If you
have calculated it, please let us know how you did it so we can create a full explanation
Hock P2 2020
Section C - Decision Analysis.
Answers
of why this answer choice is incorrect. Please send us an email at
support@hockinternational.com. Include the full Question ID number and the actual
incorrect answer choice -- not its letter, because that can change with every study
session created. The Question ID number appears at the top of the question. Thank you
in advance for helping us to make your HOCK study materials better.
Explanation for Choice C:
This is the amount the equipment could be sold for divided by $0.50 as the contribution
margin per unit. $0.50 is the selling price per unit minus the variable cost per unit minus
the allocated fixed cost per unit. Allocated fixed cost is not included in the calculation of
the contribution margin per unit for purposes of CVP analysis.
56. Question ID: ICMA 1603.P2.028 (Topic: Using CVP Analysis in Decision-
Making)
A company currently sells 46,000 units of its product annually at a sales price of $38 per
unit. Variable costs per unit total $21 and the total fixed costs each year are $749,000.
Fixed costs include the annual salary of three sales staff, which is $55,000 each.
Management is considering changing the sales staff’s compensation. Under this
proposal, sales staff salaries would decrease to $25,000, but sales staff would also
receive a commission of $2 per unit for each unit sold. Management estimates this
option will increase sales 10%. Should management change to the commission-based
plan, and why?

 A. No, because it will decrease operating income by $23,000.


 B. Yes, because it will increase operating income by $67,000.correct
 C. No, because it will decrease net income by $23,000.
 D. Yes, because it will increase net income by $67,000.
Question was not answered
Correct Answer Explanation:
Currently, operating income is:
46,000 × ($38 − $21) − $749,000 = $33,000
Under the proposed change, sales will increase to 50,600 units (46,000 × 1.10), variable
cost will increase by $2 to $23, and fixed cost will decrease by $90,000 ($30,000
decrease in annual salary for each of three sales staff) to $659,000. Operating income
will become:
50,600 × ($38 − $23) − $659,000 = $100,000
Management should change to the commission-based plan because it will increase
operating income by $67,000 ($100,000 − $33,000).
Hock P2 2020
Section C - Decision Analysis.
Answers
Explanation for Choice A:
This answer results from failing to change fixed cost to reflect the fact that the fixed
sales salaries would decrease from $55,000 to $25,000 for each of three sales staff
under the proposed change.
Explanation for Choice C:
Net income includes an adjustment for interest and taxes. With a change in before-tax
income, income tax expense will change. We are not given any information on tax rates,
so it is not possible to determine the amount of change in net income.
Explanation for Choice D:
Net income includes an adjustment for interest and taxes. With a change in before-tax
income, income tax expense will change. We are not given any information on tax rates,
so it is not possible to determine the amount of change in net income.
57. Question ID: CMA 1290 4.1 (Topic: Using CVP Analysis in Decision-Making)
When used in cost-volume-profit analysis, sensitivity analysis

 A. Determines the most profitable mix of products to be sold.


 B. Allows the decision maker to introduce probabilities in the evaluation of decision
alternatives.
 C. Is done through various possible scenarios and computes the impact on profit of
various predictions of future events.correct
 D. Is limited because in cost-volume-profit analysis, costs are not separated into fixed and
variable components.
Question was not answered
Correct Answer Explanation:
Sensitivity analysis allows the decision maker to change the input data and then
measure the impact on profit. In respect to CVP analysis, the decision maker, for
example, has the opportunity to determine the effect that a change in fixed costs would
have on the breakeven point.
Explanation for Choice A:
A major assumption of CVP analysis is the constant mix of products.
Explanation for Choice B:
Expected value analysis, not sensitivity analysis, allows the decision maker to introduce
probabilities in the evaluation of decision alternatives.
Explanation for Choice D:
Under CVP analysis, fixed and variable costs are separated.
Hock P2 2020
Section C - Decision Analysis.
Answers
58. Question ID: CIA 596 III.85 (Topic: Using CVP Analysis in Decision-Making)
A company that sells its single product for $40 per unit uses cost-volume-profit analysis
in its planning. The company's after-tax net income for the past year was $1,188,000
after applying an effective tax rate of 40%. The projected costs for manufacturing and
selling its single product in the coming year are shown below.

Variable cost per unit:


Direct material $ 5.00
Direct labor 4.00
Manufacturing overhead 6.00
Selling and administrative costs 3.00
Total variable cost per unit $18.00
Annual fixed operating costs:
Manufacturing overhead $6,200,000
Selling and administrative costs 3,700,000
Total annual fixed cost $9,900,000
The company has learned that a new direct material is available that will increase the
quality of its product. The new material will increase the direct material costs by $3 per
unit. The company will increase the selling price of the product to $50 per unit and
increase its marketing costs by $1,575,000 to advertise the higher-quality product. The
number of units the company has to sell in order to earn a 10% before-tax return on
sales would be:

 A. 337,500 units.
 B. 412,500 units.
 C. 346,875 units.
 D. 478,125 units.correct
Question was not answered
Correct Answer Explanation:
An increase in direct material costs of $3 will increase per unit variable costs from $18
per unit to $21 per unit ($18 + $3). Total fixed costs will increase from $9,900,000 to
$11,475,000 because of the increase in marketing costs ($9,900,000 + $1,575,000).
The company will increase the sales price to $50 per unit. Therefore, the per unit CM
will be $29 ($50 − $21). The required profit is 10% of sales, so with a sales price of $50
per unit, the required profit per unit is 10% of $50, which is $5.
Hock P2 2020
Section C - Decision Analysis.
Answers
To calculate the required sales in units to achieve the desired profit, divide the revised
fixed costs of $11,475,000 by ($50 − $21 − $5), which is $24. $11,475,000 ÷ $24 =
478,125.
Explanation for Choice A:
This answer results from adding the profit requirement to the denominator of the
calculation, instead of subtracting it.
Explanation for Choice B:
This answer results from using the current fixed costs in the numerator of the
calculation. The fixed costs will increase because of the increased marketing expenses.
Explanation for Choice C:
This answer results from subtracting the advertising costs from the fixed costs in the
numerator of the calculation, instead of adding it.
59. Question ID: ICMA 10.P2.209 (Topic: Using CVP Analysis in Decision-Making)
Eagle Brand Inc. produces two products. Data regarding these products are presented
below.

Product X Product Y
Selling price per unit $100 $130
Variable costs per unit $ 80 $100
Raw materials used per unit 4 lbs. 10 lbs.
Eagle Brand has 1,000 lbs. of raw materials which can be used to produce Products X
and Y.
Which one of the alternatives below should Eagle Brand accept in order to maximize
contribution margin?

 A. 200 units of product X and 50 units of product Y.


 B. 250 units of product X.correct
 C. 100 units of product Y.
 D. 200 units of product X and 20 units of product Y.
Question was not answered
Correct Answer Explanation:
To answer this question, we need to determine which product should have priority.
Since the same raw materials can be used for either product, the product that is given
priority should be the product with the higher contribution margin per pound of raw
Hock P2 2020
Section C - Decision Analysis.
Answers
materials used, assuming there is enough demand to sell all of the product that will be
produced.

Product X Product Y
Selling price per unit $100 $130
Variable costs per unit 80 100
Contribution margin per unit $ 20 $ 30
Raw materials used per unit (pounds) 4 10
Contribution margin per pound of raw materials used $ 5 $ 3
The contribution margin per pound of raw materials used is higher for Product X than for
Product Y. Therefore, priority should be given to manufacturing Product X. With 1,000
lbs. of raw material, 250 units of Product X can be manufactured (1,000 ÷ 4 lbs. used
per unit). Assuming the company can sell 250 units of Product X (which we are not told
they can), the company should use the raw materials it has to produce 250 units of
Product X.
Explanation for Choice A:
This production volume is impossible, because it requires more raw materials than the
1,000 pounds that are available.
Explanation for Choice C:
This answer results from calculating the contribution margin for each product, choosing
the product with the highest contribution margin, and then calculating how many of that
product can be produced using the 1,000 lbs. of raw materials available.
The way to maximize total contribution margin is to produce the product that has the
highest contribution margin per unit of the constrained resource. In this case, the
constrained resource is raw materials. So the product that should be produced is the
product with the highest contribution margin per pound of raw materials used.
Explanation for Choice D:
The way to maximize total contribution margin is to produce the product that has the
highest contribution margin per unit of the constrained resource. In this case, the
constrained resource is raw materials. So the product that should be produced is the
product with the highest contribution margin per pound of raw materials used. Assuming
that demand is great enough to sell all of the production, production should be
concentrated on just the one product with the highest contribution margin per pound of
raw materials used.
60. Question ID: CIA 586 IV.9 (Topic: Using CVP Analysis in Decision-Making)
Hock P2 2020
Section C - Decision Analysis.
Answers
A company sells two products, X and Y. The sales mix consists of a composite unit of 2
units of X for every 5 units of Y (2:5). Fixed costs are $49,500. The unit contribution
margins for X and Y are $2.50 and $1.20, respectively.
If the company had a profit of $22,000, the unit sales must have been:

 A. Product X: 13,000, Product Y: 32,500correct


 B. Product X: 5,000, Product Y: 12,500
 C. Product X: 23,800, Product Y: 59,500
 D. Product X: 32,500, Product Y: 13,000
Question was not answered
Correct Answer Explanation:
The formula for this answer is: (Total fixed cost + Profit) ÷ Composite Contribution
Margin. The composite contribution margin in this problem is $11 [(2 × $2.50) + (5 ×
$1.20)]
($49,500 + $22,000) ÷ $11 composite contribution margin = 6,500 composite units.
Each unit, or basket of goods, contains 2 units of X and 5 units of Y.
Therefore, the number of units sold of product X is 13,000 (2 × 6,500 composite units,
or basket of goods). The number of units sold of product Y is 32,500 (5 × 6,500
composite units or baskets).
Explanation for Choice B:
This is not the correct answer. Please see the correct answer for an explanation.
We have been unable to determine how to calculate this incorrect answer choice. If you
have calculated it, please let us know how you did it so we can create a full explanation
of why this answer choice is incorrect. Please send us an email at
support@hockinternational.com. Include the full Question ID number and the actual
incorrect answer choice -- not its letter, because that can change with every study
session created. The Question ID number appears at the top of the question. Thank you
in advance for helping us to make your HOCK study materials better.
Explanation for Choice C:
This is not the correct answer. Please see the correct answer for an explanation.
We have been unable to determine how to calculate this incorrect answer choice. If you
have calculated it, please let us know how you did it so we can create a full explanation
of why this answer choice is incorrect. Please send us an email at
support@hockinternational.com. Include the full Question ID number and the actual
incorrect answer choice -- not its letter, because that can change with every study
session created. The Question ID number appears at the top of the question. Thank you
in advance for helping us to make your HOCK study materials better.
Hock P2 2020
Section C - Decision Analysis.
Answers
Explanation for Choice D:
This answer results from using 5 units of X and 2 units of Y in each basket. Each basket
contains 2 units of X and 5 units of Y.
61. Question ID: ICMA 10.P2.213.01 213 (Topic: Using CVP Analysis in Decision-
Making)
Cervine Corporation makes two types of motors for use in various products. Operating
data and unit cost information for its products are presented below.

Product A Product B
Annual unit capacity 10,000 20,000
Annual unit demand 10,000 20,000

Selling price $100 $ 80


Variable manufacturing cost 53 45
Fixed manufacturing cost 10 10
Variable selling & administrative 10 11
Fixed selling & administrative 5 4
Fixed other administrative 2 0
Unit operating profit $ 20 $ 10

Machine hours per unit 2.0 1.5


Cervine has 40,000 productive machine hours available. What is the maximum total
contribution margin that Cervine can generate in the coming year?

 A. $689,992.correct
 B. $850,000.
 C. $980,000.
 D. $665,000.
Question was not answered
Correct Answer Explanation:
The first step is to determine which product generates the highest contribution per unit
of the constrained resource, which is machine hours, because that will determine which
product should receive priority in manufacturing. Product A's unit contribution margin is
Hock P2 2020
Section C - Decision Analysis.
Answers
$37 ($100 − $53 − $10), and it requires 2.0 machine hours per unit. Therefore, its unit
contribution margin per machine hour is $37 ÷ 2, or $18.50. Product B's unit contribution
margin is $24 ($80 − $45 − $11), and it requires 1.5 machine hours per unit. Therefore,
its unit contribution margin per machine hour is $24 ÷ 1.5, or $16.
Since Product A's contribution margin per machine hour is higher than Product B's,
priority in manufacturing should be given to Product A. All 10,000 units demanded of
Product A will be manufactured. That will require 10,000 × 2 machine hours, or 20,000
machine hours. Since 40,000 productive machine hours are available, 20,000 (40,000 −
20,000) remaining machine hours will be available for Product B. Since Product B
requires 1.5 machine hours per unit, 13,333 (20,000 ÷ 1.5) units of Product B can be
manufactured.
From the units that can be manufactured with the existing machine hours available, the
maximum contribution margin that Cervine can generate in the coming year is (10,000 ×
$37) + (13,333 × $24), which equals $689,992.
Explanation for Choice B:
This is the maximum contribution margin that could be generated from the quantity of
each product that is demanded. However, the quantity that Cervine can produce is
limited by the number of productive machine hours that the company has available.
Therefore, not all of the units demanded can be manufactured.
Explanation for Choice C:
This is the sum of the total demand for Product A multiplied by Product B's unit
contribution margin, and the total demand for Product B multiplied by Product A's unit
contribution margin. In addition to the fact that the unit contribution margins are
reversed, the quantity that Cervine can produce is limited by the number of productive
machine hours that the company has available. Therefore, not all of the units demanded
can be manufactured.
Explanation for Choice D:
This answer results from giving priority in manufacturing to Product B. However,
Product B's unit contribution per machine hour is only $16, whereas Product A's unit
contribution per machine hour is $18.50. Therefore, all of the units demanded of
Product A should be manufactured, and any remaining machine hours should be used
to manufacture as much of Product B as possible.
62. Question ID: CIA 593 IV.17 (Topic: Using CVP Analysis in Decision-Making)
A company with $280,000 of fixed costs has the following data:

Product A Product B
Sales price per unit $5 $6
Variable costs per unit $3 $5
Hock P2 2020
Section C - Decision Analysis.
Answers
Assume three units of A are sold for each unit of B sold. How much will sales be in
dollars of product B at the breakeven point?

 A. $280,000
 B. $200,000
 C. $840,000
 D. $240,000correct
Question was not answered
Correct Answer Explanation:
The first step is to calculate the composite units contribution margin for products A and
B. The formula is: [3 units of A × ($5 − $3)] + [1 unit of B × ($6 − $5)] = (3 × $2) + (1 ×
$1) = $6 + $1 = $7 composite unit contribution margin. This is the contribution margin
for a basket of goods that includes 3 units of A and 1 unit of B.
The second step is to calculate the composite breakeven point in number of baskets.
Fixed cost $280,000 ÷ Composite CM $7 = 40,000 composite baskets of goods
containing 3 units of A and 1 unit of B.
The third step is to calculate the breakeven point in sales for Product B. If the
breakeven number of baskets is 40,000 and each basket contains 1 unit of Product B,
then the breakeven number of units of Product B is 40,000 × 1, or 40,000. 40,000 units
of Product B × $6 sales price of Product B = $240,000 sales revenue for Product B at
the breakeven point.
Explanation for Choice A:
$280,000 is the fixed cost. The incorrect answer of $280,000 could be calculated in
various ways:

1. by multiplying the breakeven quantity of 40,000 baskets by the $7 composite unit


contribution margin
2. by multiplying the total breakeven units of 160,000 by the weighted average
contribution margin of $1.75
3. by dividing the fixed cost by the $1.00 contribution margin for Product B
The correct process is to either:

1. divide the fixed cost by the composite contribution margin, then multiply the result
by the number of units of B in each basket, then multiply that result by the sales
price of B, or
2. divide the fixed cost by the weighted average contribution margin per unit, then
multiply the result by the proportion of B in total sales, then multiply the result by
the sales price per unit for B.
Hock P2 2020
Section C - Decision Analysis.
Answers
Explanation for Choice B:
The incorrect answer of $200,000 is the breakeven number of units of Product B
multiplied by the sales price per unit of A.
The correct breakeven revenue for Product B is the breakeven number of units of B
multiplied by the sales price per unit of B.
Explanation for Choice C:
The incorrect answer of $840,000 could be calculated in more than one way:

1. by dividing the $280,000 fixed cost by the $2 contribution margin for A, then
multiplying the result by the $6 sales price for B, or
2. by calculating the breakeven revenue for both A and B.
The correct process is to either:

1. divide the fixed cost by the composite contribution margin, then multiply the result
by the number of units of B in each basket, then multiply that result by the sales
price of B, or
2. divide the fixed cost by the weighted average contribution margin, then multiply the
result by the proportion of B in total sales, then multiply the result by the sales price
per unit for B.
63. Question ID: ICMA 13.P2.039 (Topic: Using CVP Analysis in Decision-Making)
Blount Inc. is considering discontinuing a certain product line if it does not have a
margin of safety higher than 15%. The breakeven sales are $76,800 and the margin of
safety is $13,200. Based on this information, the controller has recommended that
Blount keep this product line. Did the controller make the appropriate decision?

 A. No, because the margin of safety ratio of 17.2% is not better than 15%.
 B. No, because the margin of safety ratio of 14.7% is not better than 15%.correct
 C. Yes, because the margin of safety ratio of 17.2% is better than 15%.
 D. Yes, because the margin of safety ratio of 14.7% is better than 15%.
Question was not answered
Correct Answer Explanation:
The margin of safety is the amount of the excess of budgeted sales levels over
breakeven sales levels. In other words, it measures the amount by which sales can fall
and the company can still remain profitable, or at worst, break even.
The budgeted sales are the breakeven sales amount of $76,800 plus the margin of
safety of $13,200, or $90,000. The margin of safety in terms of a percentage is the
$13,200 margin of safety divided by the $90,000 budgeted sales, which equals 0.1467
Hock P2 2020
Section C - Decision Analysis.
Answers
or 14.7%. The company requires a margin of safety higher than 15%, and 14.7% is
below that level, so the controller did not make the appropriate recommendation.
Explanation for Choice A:
The margin of safety is the amount of the excess of budgeted sales levels over
breakeven sales levels. In other words, it measures the amount by which sales can fall
and the company can still remain profitable, or at worst, break even.
A margin of safety of 17.2% would be better than the 15% required. However, 17.2% is
not the margin of safety. 17.2% results from dividing the margin of safety by the
breakeven sales. This is not the correct way to calculate the margin of safety in terms of
a percentage. The margin of safety as a percentage is the margin of safety divided by
the total budgeted sales. Total budgeted sales are the breakeven sales amount plus the
margin of safety.
Explanation for Choice C:
The margin of safety is the amount of the excess of budgeted sales levels over
breakeven sales levels. In other words, it measures the amount by which sales can fall
and the company can still remain profitable, or at worst, break even.
A margin of safety of 17.2% would be better than the 15% required. However, 17.2% is
not the margin of safety. 17.2% results from dividing the margin of safety by the
breakeven sales. This is not the correct way to calculate the margin of safety in terms of
a percentage. The margin of safety as a percentage is the margin of safety divided by
the total budgeted sales. Total budgeted sales are the breakeven sales amount plus the
margin of safety.
Explanation for Choice D:
The margin of safety is the amount of the excess of budgeted sales levels over
breakeven sales levels. In other words, it measures the amount by which sales can fall
and the company can still remain profitable, or at worst, break even.
A margin of safety ratio of 14.7% is not better than the minimum 15% required.
64. Question ID: CIA 1192 IV.17 (Topic: Using CVP Analysis in Decision-Making)
Data regarding four different products manufactured by an organization are presented
as follows. Direct material and direct labor are readily available from the respective
resource markets. However, the manufacturer is limited to a maximum of 3,000
machine hours per month.

Products A B C D
Unit price $15 $18 $20 $25
Variable cost $7 $11 $10 $16
Units Produced per Machine Hour
Hock P2 2020
Section C - Decision Analysis.
Answers
A B C D
3 4 2 3
The product that is the most profitable for the manufacturer in this situation is:

 A. Product C.
 B. Product D.
 C. Product A.
 D. Product B.correct
Question was not answered
Correct Answer Explanation:
In this question we deal with an operation with a constraint. To solve this question we
have to identify which product has the highest contribution per machine hour (the limited
resource). That will be the most profitable product.
First, we need calculate the unit contribution margin (sales price minus variable cost per
unit) for each product: for product A it is $8 ($15 − $7); for product B it is $7 ($18 − $11);
for product C it is $10 ($20 − $10); and for product D it is $9 ($25 − $16).
Knowing how many units of each product can be produced per machine hour, we can
now calculate contribution per machine hour for each product. For product A, the
contribution margin per hour is $24 ($8 per unit × 3 units per machine hour), for product
B it is $28 ($7 × 4), for product C it is $20 ($10 × 2), for product D it is $27 ($9 × 3).
Comparing the calculated contribution per machine hour we select the highest, which is
for product B.
Note: If the information for this problem had been given as the number of machine
hours required to manufacture one unit, we would have divided the contribution
margin per unit by the number of machine hours required to manufacture one unit to
calculate the contribution margin per machine hour. For example, 4 units of Product B
can be produced per machine hour. That means one unit requires 1/4 hour. If we had
been told that one unit requires 1/4 hour to manufacture, we would have calculated the
contribution margin per unit by dividing $7 by 0.25. The result would have been exactly
the same as when we multiply $7 by 4 units produced per machine hour: $28.
Explanation for Choice A:
Product C is the least profitable product. See the correct answer for a complete
explanation.
Explanation for Choice B:
Product D is the second most profitable product. See the correct answer for a complete
explanation.
Hock P2 2020
Section C - Decision Analysis.
Answers
Explanation for Choice C:
Product A is the third most profitable product. See the correct answer for a complete
explanation.
65. Question ID: ICMA 10.P2.216 (Topic: Using CVP Analysis in Decision-Making)
Ticker Company sells two products. Product A provides a contribution margin of $3 per
unit, and Product B provides a contribution margin of $4 per unit. If Ticker's sales mix
shifts toward Product A, which one of the following statements is correct?

 A. The contribution margin ratios for Products A and B will change.


 B. The overall contribution margin ratio will increase.
 C. Operating income will decrease if the total number of units sold remains
constant.correct
 D. The total number of units necessary to break even will decrease.
Question was not answered
Correct Answer Explanation:
If Ticker's sales mix shifts toward Product A while the total number of units sold remains
the same, that means Ticker is selling more of Product A and less of Product B. Product
A has a lower contribution margin than Product B. If the company is selling more of the
lower contribution margin product and less of the higher contribution margin product, the
total contribution margin will decrease.
Because the total contribution margin decreases while the total fixed costs remain the
same, operating income will also decrease.
Explanation for Choice A:
The individual contribution margin ratios for Products A and B will not change.
Explanation for Choice B:
If Ticker's sales mix shifts toward Product A while the total number of units sold remains
the same, that means Ticker is selling more of Product A and less of Product B. Product
A has a lower contribution margin than Product B. If the company is selling more of the
lower contribution margin product and less of the higher contribution margin product, the
total contribution margin will decrease, not increase.
Explanation for Choice D:
If Ticker's sales mix shifts toward Product A while the total number of units sold remains
the same, that means Ticker is selling more of Product A and less of Product B. Product
A has a lower contribution margin than Product B. This change in the product mix will
cause the weighted average contribution margin to decrease, because more weight will
be put on the product with the lower contribution margin. The decrease in the weighted
Hock P2 2020
Section C - Decision Analysis.
Answers
average contribution margin will cause the total number of units necessary to break
even to increase, not decrease.
66. Question ID: CMA 1291 4.13 (Topic: Using CVP Analysis in Decision-Making)
Siberian Ski Company recently expanded its manufacturing capacity, which will allow it
to produce up to 15,000 pairs of cross-country skis of the mountaineering model or the
touring model. The Sales Department assures management that it can sell between
9,000 pairs and 13,000 pairs of either product this year. Because the models are very
similar, Siberian Ski will produce only one of the two models.
The following information was compiled by the Accounting Department.

Per Unit (Pair) Data


Mountaineering Touring
Selling price $88.00 $80.00
Variable costs 52.80 52.80
Fixed costs will total $369,600 if the mountaineering model is produced but will be only
$316,800 if the touring model is produced. Siberian Ski is subject to a 40% income tax
rate.
If Siberian Ski Company desires an after-tax net income of $24,000, how many pairs of
touring model skis will the company have to sell?

 A. 4,460 pairs.
 B. 13,118 pairs.correct
 C. 12,529 pairs.
 D. 13,853 pairs.
Question was not answered
Correct Answer Explanation:
The company desires an after-tax income of $24,000. The formula to calculate the sales
breakeven even volume is: Total fixed cost + Before-Tax Profit / Unit Contribution
Margin.
$316,800 (fixed costs) + $40,000 before-tax profit ($24,000 ÷ (1-40% tax rate))/ $27.20
contribution margin per unit ($80 sales price − $52.80 variable cost) = 13,118 units
Explanation for Choice A:
This is not the correct answer. Please see the correct answer for an explanation.
We have been unable to determine how to calculate this incorrect answer choice. If you
have calculated it, please let us know how you did it so we can create a full explanation
of why this answer choice is incorrect. Please send us an email at
Hock P2 2020
Section C - Decision Analysis.
Answers
support@hockinternational.com. Include the full Question ID number and the actual
incorrect answer choice -- not its letter, because that can change with every study
session created. The Question ID number appears at the top of the question. Thank you
in advance for helping us to make your HOCK study materials better.
Explanation for Choice C:
This answer results from using the after-tax desired net income in the calculation. The
desired after-tax net income needs to be converted to before-tax income for use in this
calculation.
Explanation for Choice D:
This is not the correct answer. Please see the correct answer for an explanation.
We have been unable to determine how to calculate this incorrect answer choice. If you
have calculated it, please let us know how you did it so we can create a full explanation
of why this answer choice is incorrect. Please send us an email at
support@hockinternational.com. Include the full Question ID number and the actual
incorrect answer choice -- not its letter, because that can change with every study
session created. The Question ID number appears at the top of the question. Thank you
in advance for helping us to make your HOCK study materials better.
67. Question ID: CMA 687 5.22 (Topic: Using CVP Analysis in Decision-Making)
Gleason Co. has two products, a frozen dessert and ready-to-bake breakfast rolls,
ready for introduction. However, plant capacity is limited, and only one product can be
introduced at present. Therefore, Gleason has conducted a market study, at a cost of
$26,000, to determine which product will be more profitable. The results of the study
follow.

Sales of Desserts Sales of Rolls


at $1.80/unit at $1.20/unit
Volume Probability Volume Probability
250,000 0.30 200,000 0.20
300,000 0.40 250,000 0.50
350,000 0.20 300,000 0.20
400,000 0.10 350,000 0.10
The costs associated with the two products have been estimated by Gleason's cost
accounting department and are shown as follows.

Dessert Rolls
Ingredients per unit $ 0.40 $ 0.25
Hock P2 2020
Section C - Decision Analysis.
Answers
Direct labor per unit 0.35 0.30
Variable overhead per unit 0.40 0.20
Production tooling* 48,000 25,000
Advertising 30,000 20,000
*Gleason treats production tooling as a current operating expense rather than
capitalizing it as a fixed asset.
Applying a deterministic approach, Gleason's revenue from sales of frozen desserts
would be

 A. $549,000.
 B. $216,000.
 C. $540,000.correct
 D. Some amount other than those given.
Question was not answered
Correct Answer Explanation:
The deterministic approach selects the most probable value and uses it. In this problem,
we use the most probable volume to calculate revenue. The most probable volume of
frozen desserts is 300,000 with a 40% probability, so expected revenue would be
$540,000 (300,000 units × $1.80/unit).
Explanation for Choice A:
This would be the answer if using the probabilistic approach, which results in the
expected value. However, the question asked for the deterministic approach.
Explanation for Choice B:
The answer incorrectly multiplied the most likely sales volume of frozen desserts
(300,000) by its probability (0.40) and then multiplied it again by the price per unit to
calculate the sales revenue. This is not the correct way to use the deterministic
approach. The deterministic approach selects the most probable value and uses it; but it
would not be multiplied by its probability before using it to calculate the sales revenue.
Explanation for Choice D:
The correct answer is one of the answer choices given.
68. Question ID: CIA 593 IV.11 (Topic: Using CVP Analysis in Decision-Making)
A company has sales of $500,000, variable costs of $300,000, and pretax profit of
$150,000. If the company increased the sales price per unit by 10%, reduced fixed
costs by 20%, and left variable cost per unit unchanged, what would be the new
breakeven point in sales dollars?
Hock P2 2020
Section C - Decision Analysis.
Answers
 A. $100,000
 B. $88,000correct
 C. $125,000
 D. $110,000
Question was not answered
Correct Answer Explanation:
To find the breakeven point, first calculate what fixed costs are. Based on the
information given, fixed costs must be $50,000 ($500,000 revenue − $300,000 variable
costs − $150,000 pretax profit). If the company reduces fixed costs by 20%, fixed costs
will be $40,000 ($50,000 × 80%). The contribution margin given the stated changes will
be $250,000 [($500,000 × 110%) − $300,000] The contribution margin ratio is 45.45%
($250,000 ÷ $550,000). The formula for breakeven sales revenue is: Fixed Costs ÷
Contribution Margin Ratio. $40,000 ÷ 45.45% = $88,000 breakeven sales revenue.
Explanation for Choice A:
This answer results from assuming that variable costs will increase by the same 10% as
sales increase. The sales increase comes from increased prices, not increased volume.
The question says that variable cost per unit is left unchanged, and since volume does
not increase, the total variable cost will not increase.
Explanation for Choice C:
This answer results from two errors:

1. The first error is assuming that variable costs will increase by the same 10% as
sales increase. The sales increase comes from increased prices, not increased
volume. The question says that variable cost per unit is left unchanged, and since
volume does not increase, the total variable cost will not increase.
2. The second error is failing to reduce fixed costs. The question says that fixed costs
will be reduced by 20%.
Explanation for Choice D:
This answer results from failing to reduce fixed costs. The question says that fixed costs
will be reduced by 20%.
69. Question ID: CMA 1291 4.14 (Topic: Using CVP Analysis in Decision-Making)
Siberian Ski Company recently expanded its manufacturing capacity, which will allow it
to produce up to 15,000 pairs of cross-country skis of the mountaineering model or the
touring model. The Sales Department assures management that it can sell between
9,000 pairs and 13,000 pairs of either product this year. Because the models are very
similar, Siberian Ski will produce only one of the two models.
The following information was compiled by the Accounting Department.
Hock P2 2020
Section C - Decision Analysis.
Answers
Per Unit (Pair) Data
Mountaineering Touring
Selling price $88.00 $80.00
Variable costs 52.80 52.80
Fixed costs will total $369,600 if the mountaineering model is produced but will be only
$316,800 if the touring model is produced. Siberian Ski is subject to a 40% income tax
rate.
The total sales revenue at which Siberian Ski Company would make the same profit or
loss regardless of the ski model it decided to produce is

 A. $880,000.correct
 B. $422,400.
 C. $686,400.
 D. $924,000.
Question was not answered
Correct Answer Explanation:
We need to know the level of revenue that would be the same for both models at which
the profit would also be the same for both models.
Let M = the quantity of the Mountaineering model and T = the quantity of the Touring
model. Since we have two unknowns, we will need two different equations, both using
both unknowns. The question asks for a level of total revenue which will be the same for
both models at which the profit will also be the same for both models. So one of our
equations will need to be a revenue function and the other will need to be a profit
function.
Since the profit must be the same for both models, we will create an expression of the
profit for each model, using the contribution margins for each model multiplied by the
quantity for each and from that subtracting the fixed cost for each. Since the profit must
be the same, we set the two expressions equal to one another:
35.2M - 369,600 = 27.2T - 316,800
Since the revenue must also be the same for both models, next we create an
expression of the revenue for each model and then set those two expressions equal to
one another:
88M = 80T
We now have two equations in two unknowns. There is more than one way to solve
these two equations. Here is one of the ways.
Hock P2 2020
Section C - Decision Analysis.
Answers
Simplify the the revenue equation so that M is expressed in terms of T by dividing both
sides of the revenue equation by 88. The result is M = 0.90909T
Take this value for M expressed in terms of T and plug it into the profit equation in place
of M. We now have an equation in only one variable, T, and we can simplify it and solve
for T:
(35.2 × 0.90909T) − 369,600 = 27.2T − 316,800
32T −369,600 = 27.2T − 316,800
4.8T = 52,800
T = 11,000
Take this value for T and plug it into the revenue equation in place of T. This will
actually give you the answer, because it will result in total revenue:
88M = (80 ×11,000)
88M = 880,000
However, let's go further and solve for both quantities. Solving for M, we get:
M = 10,000
So the quantity of the Touring model is 11,000; and the quantity of the Mountaineering
model is 10,000.
Total revenue for the Touring model will be $80 × 11,000, or $880,000. Total revenue
for the Mountaineering model will be $88 ×10,000, or $880,000.
Profit for the Touring model will be ($35.20 × 10,000) − $369,600, or $(17,600). Profit
for the Mountaineering model will be ($27.20 × 11,000) − $316,800, or $(17,600).
Explanation for Choice B:
This is not the correct answer. Please see the correct answer for an explanation.
We have been unable to determine how to calculate this incorrect answer choice. If you
have calculated it, please let us know how you did it so we can create a full explanation
of why this answer choice is incorrect. Please send us an email at
support@hockinternational.com. Include the full Question ID number and the actual
incorrect answer choice -- not its letter, because that can change with every study
session created. The Question ID number appears at the top of the question. Thank you
in advance for helping us to make your HOCK study materials better.
Explanation for Choice C:
This is not the correct answer. Please see the correct answer for an explanation.
We have been unable to determine how to calculate this incorrect answer choice. If you
have calculated it, please let us know how you did it so we can create a full explanation
Hock P2 2020
Section C - Decision Analysis.
Answers
of why this answer choice is incorrect. Please send us an email at
support@hockinternational.com. Include the full Question ID number and the actual
incorrect answer choice -- not its letter, because that can change with every study
session created. The Question ID number appears at the top of the question. Thank you
in advance for helping us to make your HOCK study materials better.
Explanation for Choice D:
This is not the correct answer. Please see the correct answer for an explanation.
We have been unable to determine how to calculate this incorrect answer choice. If you
have calculated it, please let us know how you did it so we can create a full explanation
of why this answer choice is incorrect. Please send us an email at
support@hockinternational.com. Include the full Question ID number and the actual
incorrect answer choice -- not its letter, because that can change with every study
session created. The Question ID number appears at the top of the question. Thank you
in advance for helping us to make your HOCK study materials better.
70. Question ID: ICMA 10.P2.217 (Topic: Using CVP Analysis in Decision-Making)
Lazar Industries produces two products, Crates and Trunks. Per unit selling prices,
costs, and resource utilization for these products are as follows.

Crates Trunks
Selling price $20 $30

Direct material costs $ 5 $ 5


Direct labor costs 8 10
Variable overhead costs 3 5
Variable selling costs 1 2

Machine hours per unit 2 4


Production of Crates and Trunks involves joint processes and use of the same facilities.
The total fixed factory overhead cost is $2,000,000 and total fixed selling and
administrative costs are $840,000. Production and sales are scheduled for 500,000
Crates and 700,000 Trunks. Lazar has a normal capacity to produce a total of
2,000,000 units in any combination of Crates and Trunks, and maintains no direct
materials, work-in-process, or finished goods inventory.
Due to plant renovations Lazar Industries will be limited to 1,000,000 machine hours.
What is the maximum amount of contribution margin Lazar can generate during the
renovation period?
Hock P2 2020
Section C - Decision Analysis.
Answers
 A. $7,000,000.
 B. $1,500,000.
 C. $2,000,000.correct
 D. $3,000,000.
Question was not answered
Correct Answer Explanation:
Since we have a constrained resource – machine hours – the first thing to do is
calculate the contribution margin of each product per machine hour, in order to
determine which product should be given priority in manufacturing.

Crates Trunks
Selling price $20 $30
Variable costs (DM, DL, VOH, VS) 17 22
Contribution margin per unit $ 3 $ 8
Machine hours required per unit 2 4
Contribution margin per machine hour
(CM per unit / MH per unit) $1.50 $2.00
Priority in manufacturing should be given to the Trunks, since they have the higher
contribution margin per machine hour required to manufacture them. The Trunks require
4 machine hours each. If 1,000,000 machine hours are available, then the maximum
number of Trunks that can be manufactured is 1,000,000 ÷ 4, or 250,000. No Crates will
be able to be manufactured.
The unit contribution margin for Trunks is $8. If 250,000 Trunks are manufactured and
sold, the total contribution margin will be $8 × 250,000, or $2,000,000. This is the
maximum amount of contribution margin Lazar can generate during the renovation
period.
Explanation for Choice A:
This is not the correct answer. Please see the correct answer for a complete
explanation.
We have been unable to determine how to calculate this incorrect answer choice. If you
have calculated it, please let us know how you did it so we can create a full explanation
of why this answer choice is incorrect. Please send us an email at
support@hockinternational.com. Include the full Question ID number and the actual
incorrect answer choice -- not its letter, because that can change with every study
session created. The Question ID number appears at the top of the question. Thank you
in advance for helping us to make your HOCK study materials better.
Hock P2 2020
Section C - Decision Analysis.
Answers
Explanation for Choice B:
This is the total contribution margin Lazar can earn if it produces 500,000 Crates, which
it could do in 1,000,000 machine hours available. However, this is not the maximum
amount of contribution margin Lazar can generate during the renovation period.
Explanation for Choice D:
This is not the correct answer. Please see the correct answer for a complete
explanation.
We have been unable to determine how to calculate this incorrect answer choice. If you
have calculated it, please let us know how you did it so we can create a full explanation
of why this answer choice is incorrect. Please send us an email at
support@hockinternational.com. Include the full Question ID number and the actual
incorrect answer choice -- not its letter, because that can change with every study
session created. The Question ID number appears at the top of the question. Thank you
in advance for helping us to make your HOCK study materials better.
71. Question ID: CIA 586 IV.8 (Topic: Using CVP Analysis in Decision-Making)
A company sells two products, X and Y. The sales mix consists of a composite unit of 2
units of X for every 5 units of Y (2:5). Fixed costs are $49,500. The unit contribution
margins for X and Y are $2.50 and $1.20, respectively.
Considering the company as a whole, the number of composite units to break even is:

 A. 1,650
 B. 4,500correct
 C. 8,250
 D. 22,500
Question was not answered
Correct Answer Explanation:
Breakeven analysis is more difficult for a multi-product company. The breakeven point is
found by dividing fixed cost by the composite contribution margin. The composite
contribution margin in this problem is $11 [(2 × $2.50) + (5 × $1.20)]. The breakeven
point = $49,500 ÷ $11 = 4,500 units. Each composite unit, or basket of goods, contains
2 units of X and 5 units of Y.
Explanation for Choice A:
This is not the correct answer. Please see the correct answer for an explanation.
We have been unable to determine how to calculate this incorrect answer choice. If you
have calculated it, please let us know how you did it so we can create a full explanation
of why this answer choice is incorrect. Please send us an email at
support@hockinternational.com. Include the full Question ID number and the actual
Hock P2 2020
Section C - Decision Analysis.
Answers
incorrect answer choice -- not its letter, because that can change with every study
session created. The Question ID number appears at the top of the question. Thank you
in advance for helping us to make your HOCK study materials better.
Explanation for Choice C:
This is the fixed costs divided by the contribution margin for the five units of Y in each
composite basket. To calculate the number of composite baskets required to break
even, the divisor should be the composite contribution margin, which is the total
contribution margin for all the units in one basket.
Explanation for Choice D:
This is not the correct answer. Please see the correct answer for an explanation.
We have been unable to determine how to calculate this incorrect answer choice. If you
have calculated it, please let us know how you did it so we can create a full explanation
of why this answer choice is incorrect. Please send us an email at
support@hockinternational.com. Include the full Question ID number and the actual
incorrect answer choice -- not its letter, because that can change with every study
session created. The Question ID number appears at the top of the question. Thank you
in advance for helping us to make your HOCK study materials better.
72. Question ID: CIA 1193 IV.12 (Topic: Using CVP Analysis in Decision-Making)
Total production costs of prior periods for a company are listed as follows. Assume that
the same cost behavior patterns can be extended linearly over the range of 3,000 to
35,000 units and that the cost driver for each cost is the number of units produced.

Production (units/month): 3,000 9,000 16,000 35,000


Cost X: $23,700 $52,680 $86,490 $178,260
Cost Y: 47,280 141,840 252,160 551,600
The company is concerned about its current operating performance that is summarized
as follows:

Sales ($12.50 per unit) $300,000


Variable costs 180,000
Net operating loss (40,000)
How many additional units should have been sold in order for the company to break
even?

 A. 16,000
 B. 8,000correct
Hock P2 2020
Section C - Decision Analysis.
Answers
 C. 12,800
 D. 32,000
Question was not answered
Correct Answer Explanation:
At the current loss position of $40,000, 24,000 units are sold ($300,000 of revenue ÷
$12.50 selling price). This means that the variable costs per unit are $7.50 ($180,000 ÷
24,000), and the contribution is $5 per unit ($12.50 − $7.50). To cover the $40,000 loss
the company needs to sell 8,000 additional units ($40,000 ÷ $5 per unit).
Explanation for Choice A:
This is $80,000 in fixed costs divided by the contribution margin per unit of $5. Fixed
costs are not $80,000; they are $160,000. And dividing the fixed costs by the
contribution margin results in the breakeven volume, not the additional units needed to
be sold in order for the company to break even.
Explanation for Choice C:
This is not the correct answer. Please see the correct answer for an explanation.
We have been unable to determine how to calculate this incorrect answer choice. If you
have calculated it, please let us know how you did it so we can create a full explanation
of why this answer choice is incorrect. Please send us an email at
support@hockinternational.com. Include the full Question ID number and the actual
incorrect answer choice -- not its letter, because that can change with every study
session created. The Question ID number appears at the top of the question. Thank you
in advance for helping us to make your HOCK study materials better.
Explanation for Choice D:
32,000 is the total units needed in order for the company to break even. However, the
question asks for the additional units needed in order for the company to break even.
So the correct answer is the difference between the number of units that actually were
sold and the breakeven volume.
73. Question ID: CMA 687 5.23 (Topic: Using CVP Analysis in Decision-Making)
Gleason Co. has two products, a frozen dessert and ready-to-bake breakfast rolls,
ready for introduction. However, plant capacity is limited, and only one product can be
introduced at present. Therefore, Gleason has conducted a market study, at a cost of
$26,000, to determine which product will be more profitable. The results of the study
follow.

Sales of Desserts Sales of Rolls


at $1.80/unit at $1.20/unit
Volume Probability Volume Probability
Hock P2 2020
Section C - Decision Analysis.
Answers
250,000 0.30 200,000 0.20
300,000 0.40 250,000 0.50
350,000 0.20 300,000 0.20
400,000 0.10 350,000 0.10
The costs associated with the two products have been estimated by Gleason's cost
accounting department and are shown as follows.

Dessert Rolls
Ingredients per unit $ 0.40 $ 0.25
Direct labor per unit 0.35 0.30
Variable overhead per unit 0.40 0.20
Production tooling* 48,000 25,000
Advertising 30,000 20,000
*Gleason treats production tooling as a current operating expense rather than
capitalizing it as a fixed asset.
The expected value of Gleason's operating profit directly traceable to the sale of frozen
desserts is

 A. $198,250.
 B. $150,250.
 C. $120,250.correct
 D. Some amount other than those given.
Question was not answered
Correct Answer Explanation:
The expected value of the sales volume is 305,000 units, and the unit contribution
margin is $0.65 ($1.80 − $1.15). Thus, the total expected value of the contribution
margin is 305,000 × $0.65, or $198,250.
Production tooling ($48,000) and advertising ($30,000) are also traceable to the frozen
desserts. Even though they are fixed costs, they are relevant costs because they would
not be incurred if the frozen desserts were not produced and marketed. Therefore, the
total of these fixed costs, $78,000, needs to be deducted from the contribution margin.
Operating profit traceable to the frozen desserts is $120,250 ($198,250 − $78,000).
Explanation for Choice A:
Hock P2 2020
Section C - Decision Analysis.
Answers
This is the contribution margin for the frozen desserts, calculated using the expected
value of sales volume (the weighted average of the possible volumes), multiplied by the
contribution margin per unit ($1.80 − $1.15). However, the operating profit directly
traceable to the sale of frozen desserts must also include fixed costs that would not be
incurred if the frozen desserts were not produced and sold. Because they are incurred
only if the frozen desserts are produced and marketed, those are relevant costs. Those
fixed costs include the production tooling for the production of the frozen desserts and
the advertising expense for the frozen desserts.
Explanation for Choice B:
This answer is almost correct, but does not include subtracting advertising. Advertising
is a relevant expense, because it would not be incurred if the frozen desserts were not
produced and marketed.
Explanation for Choice D:
The correct answer is one of the answer choices given.
74. Question ID: CIA 577 IV.11 (Topic: Using CVP Analysis in Decision-Making)
Which of the following will result in raising the breakeven point?

 A. An increase in the contribution margin per unit.


 B. An increase in the semi-variable cost per unit.correct
 C. A decrease in income tax rates.
 D. A decrease in the variable cost per unit.
Question was not answered
Correct Answer Explanation:
Semi-variable costs have a fixed component and a variable component. A basic fixed
amount must be paid even if there is no activity; and added to that fixed amount is a
variable amount that varies with activity.
An increase in either a fixed or variable expense will increase the breakeven point. The
formula to calculate the breakeven point in units is Fixed Costs / Unit Contribution
Margin. If variable costs increase, the Unit Contribution Margin in the denominator will
decrease, which will cause the breakeven point to increase. If fixed costs increase, the
numerator will increase, which will also cause the breakeven point to increase.
Therefore, an increase in a semi-variable cost would increase the breakeven point as
well.
Explanation for Choice A:
An increase in the contribution margin per unit would decrease the breakeven point.
Explanation for Choice C:
Hock P2 2020
Section C - Decision Analysis.
Answers
Income taxes are considered to be a variable expense, and as such a decrease in the
income tax rate would increase the contribution margin, thereby lowering the breakeven
point.
Explanation for Choice D:
A decrease in variable cost per unit would increase the contribution margin, thereby
decreasing the breakeven point.
75. Question ID: CMA 1286 5.13 (Topic: Using CVP Analysis in Decision-Making)
The margin of safety is a key concept of CVP analysis. The margin of safety is

 A. the difference between budgeted contribution margin and breakeven contribution


margin.
 B. the contribution margin rate.
 C. the difference between budgeted sales and breakeven sales.correct
 D. the difference between the breakeven point in sales and cash flow breakeven.
Question was not answered
Correct Answer Explanation:
The margin of safety is the amount by which sales can decrease before losses will
occur (budgeted or actual sales minus sales level at breakeven). This is major concept
of CVP analysis.
Explanation for Choice A:
The contribution margin is sales less variable costs.
Explanation for Choice B:
The contribution margin rate is unit contribution margin divided by unit sales price.
Explanation for Choice D:
The difference between breakeven point in sales and cash flow breakeven is irrelevant
to financial analysis.
76. Question ID: CMA 694 4.26 (Topic: Using CVP Analysis in Decision-Making)
Condensed monthly operating income data for Korbin Inc. for May follows:

Urban Suburban
Store Store Total
Sales $80,000 $120,000 $200,000
Variable costs 32,000 84,000 116,000
Contribution margin $48,000 $36,000 $84,000
Hock P2 2020
Section C - Decision Analysis.
Answers
Direct fixed costs 20,000 40,000 60,000
Store segment margin $28,000 $(4,000) $24,000
Common fixed cost 4,000 6,000 10,000
Operating income $24,000 $(10,000) $14,000
Additional information regarding Korbin's operations follows:

 One-fourth of each store's direct fixed costs would continue if either store were
closed.
 Korbin allocates common fixed costs to each store on the basis of sales dollars.
 Management estimates that closing the Suburban Store would result in a 10%
decrease in the Urban Store's sales, while closing the Urban Store would not affect
the Suburban Store's sales.
 The operating results for May are representative of all months.
Korbin is considering a promotional campaign at the Suburban Store that would not
affect the Urban Store. Increasing annual promotional expense at the Suburban Store
by $60,000 in order to increase this store's sales by 10% would result in a monthly
increase (decrease) in Korbin's operating income during the year (rounded) of

 A. $7,000
 B. $487
 C. $(5,000)
 D. $(1,400)correct
Question was not answered
Correct Answer Explanation:
The monthly cost for the advertising is $5,000 ($60,000 ÷ 12). The advertising will
increase sales by 10%, so the contribution margin will increase by $3,600 ($36,000
contribution margin × 1.10). Therefore, the advertising promotion will cost the company
an additional $1,400 ($3,600 increase in contribution margin minus $5,000 advertising
cost).
Explanation for Choice A:
The answer of a $7,000 increase to monthly operating income results from correctly
considering the increase in sales revenue (a $12,000 increase to monthly operating
income) and the increase in advertising expense (a $5,000 decrease to monthly
operating income). However, the increased sales will result in a 10% increase in
variable costs, as well, which will reduce operating income. That reduction is not
considered in this answer.
Hock P2 2020
Section C - Decision Analysis.
Answers
Explanation for Choice B:
This is not the correct answer. Please see the correct answer for an explanation.
We have been unable to determine how to calculate this incorrect answer choice. If you
have calculated it, please let us know how you did it so we can create a full explanation
of why this answer choice is incorrect. Please send us an email at
support@hockinternational.com. Include the full Question ID number and the actual
incorrect answer choice -- not its letter, because that can change with every study
session created. The Question ID number appears at the top of the question. Thank you
in advance for helping us to make your HOCK study materials better.
Explanation for Choice C:
The answer of a $5,000 decrease to monthly operating income does not include the
increase in the contribution margin from the increased sales resulting from the
promotion.
77. Question ID: ICMA 10.P2.213.02 263 (Topic: Using CVP Analysis in Decision-
Making)
Cervine Corporation makes two types of motors for use in various products. Operating
data and unit cost information for its products are presented below.

Product A Product B
Annual unit capacity 10,000 20,000
Annual unit demand 10,000 20,000

Selling price $100 $ 80


Variable manufacturing cost 53 45
Fixed manufacturing cost 10 10
Variable selling & administrative 10 11
Fixed selling & administrative 5 4
Fixed other administrative 2 0
Unit operating profit $ 20 $ 10

Machine hours per unit 2.0 1.5


Cervine has 40,000 productive machine hours available. The relevant contributions per
machine hour for each product to be utilized in making a decision on product priorities
for the coming year, are
Hock P2 2020
Section C - Decision Analysis.
Answers
 A. Product A $18.50, Product B $16.00.correct
 B. Product A $17.00, Product B $14.00.
 C. Product A $37.00, Product B $24.00.
 D. Product A $20.00, Product B $10.00.
Question was not answered
Correct Answer Explanation:
The unit contribution margin per machine hour is the unit contribution margin for each
product divided by the number of machine hours required for the production of each unit
of product. Product A's unit contribution margin is $37 ($100 selling price − $53 variable
manufacturing cost − $10 variable selling & administrative), and it requires 2 machine
hours per unit. Therefore, its UCM per machine hour is $18.50. Product B's unit
contribution margin is $24 ($80 selling price − $45 variable manufacturing cost − $11
variable selling & administrative), and it requires 1.5 machine hours per unit. Therefore,
its UCM per machine hour is $16.
Explanation for Choice B:
These are the selling prices minus the variable manufacturing cost, fixed manufacturing
cost, and variable selling & administrative cost for each product. What is needed are the
unit contribution margins per machine hour. The unit contribution margins per machine
hour are the unit selling prices minus the unit variable costs (only variable costs) for
each product, and the difference divided by the number of machine hours required for
each unit.
Explanation for Choice C:
These are the unit contribution margins for each product. However, the unit contribution
margins per machine hour are needed. The unit contribution margins per machine hour
are the unit contribution margins for each product divided by the number of machine
hours required for each unit.
Explanation for Choice D:
These are the unit operating profits, not the unit contribution margins per machine hour.
The unit contribution margins per machine hour are the unit contribution margins for
each product divided by the number of machine hours required for each unit.
78. Question ID: ICMA 1603.P2.001 (Topic: Using CVP Analysis in Decision-
Making)
A company sells two products: Sparta and Volta. Volta is manufactured by a third party
supplier, which charges the company a contractual price for each unit of Volta
manufactured. A summary of revenue and costs assumptions for each product is as
follows.
Hock P2 2020
Section C - Decision Analysis.
Answers
Sparta Volta
Planned sales units
prior to promotion 100,000 20,000
Unit selling price $10 $20
Unit variable cost $3 $10
Fixed costs $500,000 $ 0

The company has the opportunity to spend an additional $10,000 in promotional


expenditures on either Sparta or Volta, anticipating a 10% increase in unit sales volume
as a result. Both product lines have idle capacity and can support the increase in unit
volume. The company should spend the additional promotional expenditure on

 A. Sparta, because it would generate an additional $10,000 in operating profit.


 B. Volta, because it would generate an additional $10,000 in operating profit.
 C. Volta, because it would generate an additional $20,000 in operating profit.
 D. Sparta, because it would generate an additional $60,000 in operating profit.correct
Question was not answered
Correct Answer Explanation:
Prior to the promotion, the operating income for each product is:
Sparta: (100,000 × $7) − $500,000 = $200,000.
Volta: (20,000 × $10) − $0 = $200,000
After the promotion, the operating income for each product, if it is the one selected for
the promotion, will be:
Sparta: (110,000 × $7) − $510,000 = $260,000.
Volta: (22,000 × $10) − $10,000 = $210,000
The company should spend the additional promotional expenditure on Sparta, because
operating income for Sparta will increase from $200,000 to $260,000, an increase of
$60,000. That is greater than the $10,000 increase in Volta's operating income if the
additional expenditure were made on Volta.
Explanation for Choice A:
This answer results from increasing the fixed cost by 10% to reflect the 10% increase in
unit sales volume expected to result from the promotion. Fixed costs do not change in
total with changes in activity as long as the activity remains within the relevant range.
Unless an exam question states otherwise, we assume that activity that increases or
decreases remains within the relevant range and thus fixed costs do not change as a
result of the change in activity.
Hock P2 2020
Section C - Decision Analysis.
Answers
Explanation for Choice B:
It is true that if the additional promotional expenditure were made for Volta, it would
generate an additional $10,000 in operating income. However, that is not the best
choice.
Explanation for Choice C:
An increase of $20,000 in operating profit for Volta results from failing to increase fixed
costs by the extra promotional cost of $10,000. Furthermore, spending the additional
promotional expense on Volta is not the best choice.
79. Question ID: ICMA 10.P2.200 (Topic: Using CVP Analysis in Decision-Making)
Ace Manufacturing plans to produce two products, Product C and Product F, during the
next year, with the following characteristics.

Product C Product F
Selling price per unit $10 $15
Variable cost per unit 8 10
Expected sales (units) 20,000 5,000
Total projected fixed costs for the company are $30,000. Assume that the product mix
would be the same at the breakeven point as at the expected level of sales of both
products. What is the projected number of units (rounded) of Product C to be sold at the
breakeven point?

 A. 15,000 units.
 B. 2,308 units.
 C. 11,538 units.
 D. 9,231 units.correct
Question was not answered
Correct Answer Explanation:
The unit contribution margin for Product C is $2, and Product C represents 80% of the
total sales in units (20,000 ÷ 25,000). The unit contribution margin for Product F is $5,
and Product F represents 20% of the total sales in units (5,000 ÷ 25,000). Therefore,
the weighted average unit contribution margin is ($2 × 0.80) + ($5 × 0.20) = $2.60.
The breakeven point for sales in total units is calculated as Fixed Costs divided by Unit
Contribution Margin. Fixed Costs are $30,000, so the breakeven total number of units is
$30,000 ÷ $2.60, or 11,538 units.
Since Product C represents 80% of the total number of units sold, the breakeven
number of units for Product C is 11,528 × 0.80, or 9,230.4 units. Since we cannot
produce 0.4 of a unit, we round it up to 9,231 units.
Hock P2 2020
Section C - Decision Analysis.
Answers
Explanation for Choice A:
This would be the breakeven point for Product C if it were the only product being sold by
Ace Manufacturing. However, more than one product is being sold. Since the
contribution margin from Product C is not the only source of contribution margin
available to cover fixed costs, its breakeven point is different.
Explanation for Choice B:
This is the number of units of Product F to be sold at the breakeven point. See correct
answer for full calculation.
Explanation for Choice C:
This is the total breakeven number of units, including both Product C and Product F.
80. Question ID: CMA 692 4.22 (Topic: Using CVP Analysis in Decision-Making)
Mason Enterprises has prepared the following budget for the month of July.

Selling Price Variable Cost Sales


Per Unit Per Unit Per Unit
Product A $10.00 $4.00 15,000
Product B 15.00 8.00 20,000
Product C 18.00 9.00 5,000
Assuming that total fixed costs will be $150,000 and the mix remains constant, the
breakeven point (rounded to the next higher whole unit) will be

 A. 21,819 units.correct
 B. 20,455 units.
 C. 6,818 units.
 D. 21,429 units.
Question was not answered
Correct Answer Explanation:
To calculate the breakeven point for a basket of goods, we divide the fixed cost by the
contribution margin for the basket of goods. The contribution margin for a basket of
goods is a weighted average of the contribution margins of the products in the basket,
using each product's percentage of the total units sold as its weight.
Product A makes up 37.5% of the total sales units (15,000 ÷ 40,000); Product B makes
up 50% of the total (20,000 ÷ 40,000); and Product C makes up 12.5% of the total
(5,000 ÷ 40,000).
The contribution margins for A, B and C are $6, $7, and $9, respectively.
Hock P2 2020
Section C - Decision Analysis.
Answers
The weighted average contribution margin for the basket of goods is as follows:
(0.375 × $6) + (0.5 × $7) + (0.125 × $9) = $6.875
The breakeven point is the fixed costs of $150,000 divided by the weighted average
contribution margin. $150,000 ÷ $6.875 = 21,818.18 total units, or 21,819 rounded.
Explanation for Choice B:
This answer results from dividing the fixed cost by the average of the three contribution
margins. However, the fixed cost should be divided by a weighted average of the three
contribution margins.
Explanation for Choice C:
This answer results from summing the contribution margins of the three products and
dividing the fixed costs by the sum. The fixed cost should be divided by a weighted
average of the contribution margins for all three products, not the total of them.
Explanation for Choice D:
This answer results from dividing the fixed costs by $7, which is the contribution margin
of Product B and is the median of the three contribution margins. The fixed cost should
be divided by a weighted average of the contribution margins for all three products.
81. Question ID: CMA 687 4.13 (Topic: Using CVP Analysis in Decision-Making)
Donnelly Corporation manufactures and sells T-shirts imprinted with college names and
slogans. Last year, the shirts sold for $7.50 each, and the variable cost to manufacture
them was $2.25 per unit. The company needed to sell 20,000 shirts to break even. The
net income last year was $5,040. Donnelly's expectations for the coming year include
the following:

 The sales price of the T-shirts will be $9


 Variable cost to manufacture will increase by one-third
 Fixed costs will increase by 10%
 The income tax rate of 40% will be unchanged
If Donnelly Corporation wishes to earn $22,500 in net income for the coming year, the
company's sales volume in dollars must be

 A. Some amount other than those given.


 B. $229,500correct
 C. $257,625
 D. $213,750
Question was not answered
Correct Answer Explanation:
Hock P2 2020
Section C - Decision Analysis.
Answers
This question is asking for the sales revenue given a requirement for an after-tax net
income of $22,500. To solve it, we need to use the version of the breakeven formula for
determining the sales revenue required to result in a specific dollar amount of profit.
Target Sales Revenue = (FC + Target Pretax Income) / Contribution Margin Ratio
In order to use this formula, we need to calculate three things: (1) the amount of fixed
cost in the coming year, (2) the amount of desired net income before tax for the coming
year, and (3) the contribution margin ratio for the coming year.
(1) We are told that fixed cost for the coming year will be 10% higher than the previous
year's fixed cost. So we need to find what the fixed cost was for last year. We know the
break-even point in units for last year (20,000) and we know the unit contribution margin
for last year ($7.50 − $2.25 = $5.25). So we can find the fixed cost for last year by using
the Break-Even Point in Units formula and solving for FC: FC / Unit Contribution Margin
= BEP in Units.
FC / 5.25 = 20,000
Solving for FC, we get FC = $105,000
Since fixed cost for the coming year will be 10% higher than last year, fixed cost for the
coming year will be $105,000 × 1.10, which is $115,500.
(2) The formula to find before-tax net income when we know the after-tax net income is
After-Tax NI / (1 − tax rate). Therefore, the desired before tax net income is $22,500 / (1
− 0.40), which is $37,500.
(3) We are told that variable cost to manufacture will increase by one-third. Variable
cost last year was $2.25 per unit. Therefore, variable cost in the coming year will
increase by 1/3 of $2.25, which is $0.75, so variable cost will be $3 per unit. We are told
that the sales price will be $9. Therefore, the Contribution Margin Ratio will be $6 / $9,
which is 2/3 or 0.666667.
Now, we can calculate the Target Sales Revenue, because we have the fixed cost, the
target pretax income, and the contribution margin ratio for the coming year.
Target Sales Revenue = ($115,500 + $37,500) / 0.666667 = $229,500.
Explanation for Choice A:
The correct answer is given.
Explanation for Choice C:
This answer results from calculating the desired before-tax net income by dividing the
after-tax net income by the tax rate. To calculate the desired before-tax net income, the
after-tax net income should be divided by (1 − the tax rate).
Explanation for Choice D:
Hock P2 2020
Section C - Decision Analysis.
Answers
This answer results from using fixed costs of $105,000 to calculate the target sales in
dollars. Fixed costs are expected to increase by 10% in the coming year.
82. Question ID: CMA 679 5.26 (Topic: Using CVP Analysis in Decision-Making)
Moorehead Manufacturing Company produces two products for which the following data
have been tabulated. Fixed manufacturing cost is applied at a rate of $1.00 per machine
hour.

Per Unit XY-7 BD-4


Selling price $4.00 $3.00
Variable manufacturing cost $2.00 $1.50
Fixed manufacturing cost $0.75 $0.20
Variable selling cost $1.00 $1.00
The sales manager has had a $160,000 increase in the budget allotment for advertising
and wants to apply the money to the most profitable product. The products are not
substitutes for one another in the eyes of the company's customers.
Suppose the sales manager chooses to devote the entire $160,000 to increased
advertising for BD-4. The minimum increase in sales revenue for BD-4 required to offset
the increased advertising would be:

 A. $320,000
 B. $1,600,000
 C. $160,000
 D. $960,000correct
Question was not answered
Correct Answer Explanation:
To find the minimum increase in revenue required for BD-4, divide the increase in
advertising expense by the unit contribution margin ratio for BD-4. The sales price of
$3.00 for BD-4 minus total variable costs of $2.50 = unit contribution margin of $0.50.
The unit contribution margin ratio is the unit contribution margin of $0.50 divided by the
sale price of $3.00 for BD-4. Therefore, the unit contribution margin ratio is
16.666667%. The $160,000 increase in advertising cost ÷ 0.16666667 = $960,000.
Explanation for Choice A:
This is the increase in advertising expense divided by 0.50. The 0.50 could be the
contribution margin for BD-4 ($3.00 − $2.50 total variable costs); or it could be the
contribution margin ratio for BD-4 if the variable selling cost of $1.00 is omitted in error
([$3.00 − $1.50]) ÷ $3.00).
Hock P2 2020
Section C - Decision Analysis.
Answers
To find the minimum increase in sales revenue required, divide the increase in
advertising expense by the unit contribution margin ratio, and the variable costs used in
calculating the unit contribution margin ratio need to include the variable selling cost.
The unit contribution margin is always used as the divisor when you are looking for the
required volume. When you need the required revenue, you must use the unit
contribution margin ratio.
Explanation for Choice B:
This answer results from including fixed manufacturing costs in the calculation of the
contribution margin ratio for BD-4. Only variable costs per unit should be subtracted
from the price per unit in calculating the unit contribution ratio.
Explanation for Choice C:
This is the increase in advertising expense divided by the variable selling cost per unit
for BD-4. To find the minimum increase in revenue required for BD-4, divide the
increase in advertising expense by the unit contribution margin ratio for BD-4.
83. Question ID: CIA 1184 IV.3 (Topic: Using CVP Analysis in Decision-Making)
Two companies are expected to have annual sales of 1,000,000 decks of playing cards
next year. Estimates for next year are presented below:

Company 1 Company 2
Selling price per deck $ 3.00 $ 3.00
Cost of paper per deck 0.62 0.65
Printing ink per deck 0.13 0.15
Labor per deck 0.75 1.25
Variable overhead per deck 0.30 0.35
Fixed costs $960,000 $252,000
Given these data, which of the following responses is correct?
(1) Breakeven Point in Units for Company 1
(2) Breakeven Point in Units for Company 2
(3) Volume in Units at Which Profits of Company 1 and Company 2 are equal

 A. (1) 800,000 : (2) 420,000 : (3) 1,180,000correct


 B. (1) 533,334 : (2) 105,000 : (3) 1,180,000
 C. (1) 800,000 : (2) 420,000 : (3) 1,000,000
 D. (1) 533,334 : (2) 105,000 : (3) 1,000,000
Question was not answered
Hock P2 2020
Section C - Decision Analysis.
Answers
Correct Answer Explanation:

Company Company
1 2
Selling price $3.00 $3.00
Variable cost 1.80 2.40
Contribution margin $1.20 $0.60
1) The breakeven point in units for Company 1 is $960,000 ÷ $1.20 = 800,000.
2) The breakeven point in units for Company 2 is $252,000 ÷ $0.60 = $420,000.
3) To determine the single quantity of units that will result in the same profits for both
companies, develop profit formulas for both companies, set them equal to one another
and solve for the unknown, which is the number of units sold. The profit formula to use
for both companies is:
Profit = (Unit Contribution Margin × Number of Units Sold) − Fixed Costs
where the Number of Units Sold is the unknown.
1.20X − 960,000 = Company 1's Profit
0.60X − 252,000 = Company 2's Profit
Setting the two equations equal to one another:
1.20X − 960,000 = 0.60X − 252,000
Solving for X:
Subtract 0.60X from both sides of the equation:
0.60X − 960,000 = − 252,000
Add 960,000 to both sides of the equation:
0.60X = 708,000
Divide both sides of the equation by 0.60:
X = 1,180,000
Explanation for Choice B:
This is not the correct answer. Please see the correct answer for an explanation.
We have been unable to determine how to calculate this incorrect answer choice. If you
have calculated it, please let us know how you did it so we can create a full explanation
of why this answer choice is incorrect. Please send us an email at
support@hockinternational.com. Include the full Question ID number and the actual
incorrect answer choice -- not its letter, because that can change with every study
session created. The Question ID number appears at the top of the question. Thank you
in advance for helping us to make your HOCK study materials better.
Hock P2 2020
Section C - Decision Analysis.
Answers
Explanation for Choice C:
This is not the correct answer. Please see the correct answer for an explanation.
We have been unable to determine how to calculate this incorrect answer choice. If you
have calculated it, please let us know how you did it so we can create a full explanation
of why this answer choice is incorrect. Please send us an email at
support@hockinternational.com. Include the full Question ID number and the actual
incorrect answer choice -- not its letter, because that can change with every study
session created. The Question ID number appears at the top of the question. Thank you
in advance for helping us to make your HOCK study materials better.
Explanation for Choice D:
This is not the correct answer. Please see the correct answer for an explanation.
We have been unable to determine how to calculate this incorrect answer choice. If you
have calculated it, please let us know how you did it so we can create a full explanation
of why this answer choice is incorrect. Please send us an email at
support@hockinternational.com. Include the full Question ID number and the actual
incorrect answer choice -- not its letter, because that can change with every study
session created. The Question ID number appears at the top of the question. Thank you
in advance for helping us to make your HOCK study materials better.
84. Question ID: ICMA 13.P2.040 (Topic: Using CVP Analysis in Decision-Making)
Specialty Inc. has a limited supply of 1,200 lbs of raw materials which can be used to
produce either product X or Y, details of which are given below.

Product X Product Y
Selling price per unit $200 $250
Variable costs per unit 176 200
Raw materials used per unit 8 lbs. 10 lbs.
Which one of the following should Specialty produce in order to maximize contribution
margin?

 A. 100 units of product X and 80 units of product Y.


 B. 120 units of product Y.correct
 C. 150 units of product X.
 D. 100 units of product X and 40 units of product Y.
Question was not answered
Correct Answer Explanation:
Hock P2 2020
Section C - Decision Analysis.
Answers
When operating at capacity, operating income is maximized by maximizing contribution
margin per unit of the resource that is limiting either the production or the sale of
products.
The contribution margin per unit of Product X is $24. Product X requires 8 lbs. of raw
materials per unit. Therefore, the contribution margin per unit of raw materials for
Product X is $24 ÷ 8, or $3.
The contribution margin per unit of Product Y is $50. Product Y requires 10 lbs. of raw
materials per unit. Therefore, the contribution margin per unit of raw materials for
Product Y is $50 ÷ 10, or $5.
Since the contribution margin per unit of raw materials used is higher for Product Y than
for Product X, the company should use all of its raw materials to produce Product Y,
assuming demand is sufficient to enable the company to sell all of the Product Y it
produces. Since 1,200 pounds of raw material are available and each unit of Product Y
requires 10 pounds of material, Specialty Inc. will be able to produce 120 units of
Product Y (1,200 ÷ 10).
Explanation for Choice A:
This combination of production for Products X and Y is not possible given the amount of
raw material that is available.
Explanation for Choice C:
When operating at capacity, operating income is maximized by maximizing contribution
margin per unit of the resource that is limiting either the production or the sale of
products. While it is possible for the company to produce 150 units of X with 1,200
pounds of raw materials, this would not maximize contribution margin.
Explanation for Choice D:
When operating at capacity, operating income is maximized by maximizing contribution
margin per unit of the resource that is limiting either the production or the sale of
products. While it is possible for the company to produce 100 units of Product X and 40
units of Product Y, this would not maximize contribution margin.
85. Question ID: CMA 693 4.1 (Topic: Using CVP Analysis in Decision-Making)
Delphi Company has developed a new project that will be marketed for the first time
during the next fiscal year. Although the Marketing Department estimates that 35,000
units could be sold at $36 per unit, Delphi's management has allocated only enough
manufacturing capacity to produce a maximum of 25,000 units of the new product
annually. The fixed costs associated with the new product are budgeted at $450,000 for
the year, which includes $60,000 for depreciation on new manufacturing equipment.
Data associated with each unit of product are presented as follows. Delphi is subject to
a 40% income tax rate.
Hock P2 2020
Section C - Decision Analysis.
Answers
Variable
Costs
Direct material $ 7.00
Direct labor 3.50
Manufacturing overhead 4.00
Total variable manufacturing cost $14.50
Selling expenses 1.50
Total variable cost $16.00
The number of units of the new product that Delphi Company must sell during the next
fiscal year in order to break even is

 A. 22,500.correct
 B. 18,140.
 C. 20,930.
 D. 25,500.
Question was not answered
Correct Answer Explanation:
Given that the sales price is $36 and that the variable costs are $16, the contribution per
unit is $20. With fixed costs of $450,000, Delphi must sell 22,500 units to break even
($450,000 ÷ $20 per unit).
Explanation for Choice B:
This answer results from two errors:
(1) Using only the variable manufacturing cost per unit to calculate the contribution per
unit. All variable costs should be used, including variable selling and administrative
costs.
(2) Subtracting the depreciation from the fixed costs before dividing the fixed costs by
the contribution per unit. The depreciation is part of the fixed costs.
Explanation for Choice C:
This answer results from using only the variable manufacturing cost per unit to calculate
the contribution per unit. All variable costs should be used, including variable selling and
administrative costs.
Explanation for Choice D:
Hock P2 2020
Section C - Decision Analysis.
Answers
This answer results from adding the $60,000 in depreciation to the fixed costs of
$450,000 and dividing $510,000 by the contribution per unit. The depreciation is
included in the $450,000 fixed costs and no adjustment should be done.
86. Question ID: CIA 582 IV.23 (Topic: Using CVP Analysis in Decision-Making)
A company's breakeven point in sales volume may be affected by equal percentage
increases in both selling price and variable cost per unit (assume all other factors are
constant within the relevant range). The equal percentage changes in selling price and
variable cost per unit will cause the breakeven point in sales revenue to

 A. Remain unchanged.correct
 B. Decrease by more than the percentage increase in the selling price.
 C. Decrease by less than the percentage increase in selling price.
 D. Increase by the percentage change in variable cost per unit.
Question was not answered
Correct Answer Explanation:
The breakeven point in sales revenue is equal to fixed cost divided by the contribution
margin ratio. If the sales price and the variable costs increase by the same percentage,
then the contribution margin ratio will remain unchanged and the breakeven point in
sales revenue will also remain unchanged.
Example: A company with $10,000 in fixed costs has a product that sells for $10 per
unit with a variable cost of $8 per unit. The contribution margin is $2, and the
contribution margin ratio is $2 ÷ $10, or 0.20. The breakeven point in revenue is
$10,000 ÷0.20, or $50,000. If the sales price increases by 5% and the variable costs
increase by 5%, the new selling price will be $10.50 ($10 × 1.05) and the new variable
costs will be $8.40 ($8 × 1.05). The new contribution margin will be $2.10, and the new
contribution margin ratio will be $2.10 ÷ $10.50, or 0.20. The breakeven point in
revenue will remain $10,000 ÷ 0.20, or $50,000.
Explanation for Choice B:
Equal percentage increases in both selling price and variable cost per unit will not cause
the breakeven sales revenue to decrease by more than the percentage increase in the
selling price. Because the variable costs increase by the same percentage as the sales
revenue increases, the contribution margin ratio remains the same after the increases.
Explanation for Choice C:
Equal percentage increases in both selling price and variable cost per unit will cause the
breakeven sales volume to decrease by a lesser percentage than the percentage
increase in the selling price. However, this question asks for the effect on the breakeven
point in sales revenue, not the effect on the breakeven point in sales volume. Because
the variable costs increase by the same percentage as the sales revenue increases, the
contribution margin ratio remains the same after the increases.
Hock P2 2020
Section C - Decision Analysis.
Answers
Explanation for Choice D:
Equal percentage increases in both selling price and variable cost per unit will not cause
the breakeven sales revenue to increase by the percentage change in variable cost per
unit. Because the sales revenue increases by the same percentage as the variable
costs increase, the contribution margin ratio remains the same after the increases.
87. Question ID: CMA 690 5.16 (Topic: Using CVP Analysis in Decision-Making)
Madengrad Company manufactures a single electronic product called Precisionmix.
This unit is a batch-density monitoring device attached to large industrial mixing
machines used in flour, rubber, petroleum, and chemical manufacturing. Precisionmix
sells for $900 per unit. The following variable costs are incurred to produce each
Precisionmix device:

Direct labor $180


Direct materials 240
Factory overhead 105
Total variable production costs $525
Marketing costs 75
Total variable costs $600
Madengrad's income tax rate is 40%, and annual fixed costs are $6,600,000. Except for
an operating loss incurred in the year of incorporation, the firm has been profitable over
the last 5 years.
Assume a 10% increase in annual fixed costs, a 20% unit cost increase for direct labor,
and a reduction in unit material costs of 25%, with no change in selling price.
Madengrad Company's breakeven point would increase (decrease) (rounded to the
nearest whole unit) by

 A. (1,620) units.
 B. 407 units.correct
 C. 1,604 units.
 D. 3,960 units.
Question was not answered
Correct Answer Explanation:
The breakeven point currently is fixed costs of $6,600,000 divided by unit contribution
margin of $300, which is 22,000 units.
With the changes, fixed costs will become $6,600,000 × 1.10, which is $7,260,000.
Direct labor per unit will become $180 × 1.20, or $216 per unit. Direct materials per unit
Hock P2 2020
Section C - Decision Analysis.
Answers
will become $240 × 0.75, or $180. Total variable costs per unit will be $216 + $180 +
$105 + $75, which is $576. The contribution margin per unit will be $900 − $576, or
$324. Therefore, the revised breakeven point will be $7,260,000 ÷ $324, which is
22,407 units.
22,407 units − the current breakeven point of 22,000 units = an increase of 407 units.
Explanation for Choice A:
This is not the correct answer. Please see the correct answer for an explanation.
We have been unable to determine how to calculate this incorrect answer choice. If you
have calculated it, please let us know how you did it so we can create a full explanation
of why this answer choice is incorrect. Please send us an email at
support@hockinternational.com. Include the full Question ID number and the actual
incorrect answer choice -- not its letter, because that can change with every study
session created. The Question ID number appears at the top of the question. Thank you
in advance for helping us to make your HOCK study materials better.
Explanation for Choice C:
This answer results from using the total variable costs per unit to calculate the
breakeven points (both before the changes and after the changes) instead of the
contribution margin per unit. The fixed costs should be divided by the contribution
margin per unit to calculate the breakeven point. The contribution margin per unit is the
selling price minus all variable costs.
Explanation for Choice D:
This is not the correct answer. Please see the correct answer for an explanation.
We have been unable to determine how to calculate this incorrect answer choice. If you
have calculated it, please let us know how you did it so we can create a full explanation
of why this answer choice is incorrect. Please send us an email at
support@hockinternational.com. Include the full Question ID number and the actual
incorrect answer choice -- not its letter, because that can change with every study
session created. The Question ID number appears at the top of the question. Thank you
in advance for helping us to make your HOCK study materials better.
88. Question ID: CMA 1294 4.5 (Topic: Using CVP Analysis in Decision-Making)
Austin Manufacturing, which is subject to a 40% income tax rate, had the following
operating data for the period just ended.

Selling price per unit $60


Variable cost per unit $22
Fixed costs $504,000
Hock P2 2020
Section C - Decision Analysis.
Answers
Management plans to improve the quality of its sole product by (1) replacing a
component that costs $3.50 with a higher-grade unit that costs $5.50, and (2) acquiring
a $180,000 packing machine. Austin will depreciate the machine over a 10-year life with
no estimated salvage value by the straight-line method of depreciation. If the company
wants to earn after-tax income of $172,800 in the upcoming period, it must sell

 A. 23,800 units.
 B. 19,300 units.
 C. 21,316 units.
 D. 22,500 units.correct
Question was not answered
Correct Answer Explanation:
Replacing the component with a higher priced component will add $2 to variable costs.
The new variable cost will be $24 per unit and the new contribution margin will be $60 −
$24, or $36. Acquiring the packing machine will add $18,000 in new depreciation
expense to fixed costs ($180,000 ÷ 10 years life), so the new fixed cost will be
$522,000. In order to earn after-tax income of $172,800, pre-tax income needs to be
$288,000 ($172,800 ÷ (1 - 40%).
Fixed costs of $522,000 + desired pre-tax income of $288,000 divided by the unit
contribution margin of $36 = 22,500 units the company needs to sell to earn the desired
after-tax net income of $172,800.
Explanation for Choice A:
This answer results from adding the full cost of the new machine and the desired after-
tax net income to the present fixed costs to calculate the numerator of the calculation of
the number of units to be sold. The full cost of the new machine should not be used,
only one year of depreciation expense on it should be used. And the after-tax net
income needs to be converted to before-tax net income before using it in the
calculation.
Explanation for Choice B:
This answer results from using the after-tax desired profit as part of the numerator in the
calculation of the number of units of sales required. The after-tax desired profit needs to
be converted to before-tax profit before using it in the calculation.
Explanation for Choice C:
This answer results from dividing the new fixed cost plus the required pre-tax net
income by the current unit contribution margin. The unit contribution margin will change
because of replacing the current component that costs $3.50 with a higher-grade unit
that costs $5.50.
89. Question ID: CMA 687 5.21 (Topic: Using CVP Analysis in Decision-Making)
Hock P2 2020
Section C - Decision Analysis.
Answers
Gleason Co. has two products, a frozen dessert and ready-to-bake breakfast rolls,
ready for introduction. However, plant capacity is limited, and only one product can be
introduced at present. Therefore, Gleason has conducted a market study, at a cost of
$26,000, to determine which product will be more profitable. The results of the study
follow.

Sales of Desserts Sales of Rolls


at $1.80/unit at $1.20/unit
Volume Probability Volume Probability
250,000 0.30 200,000 0.20
300,000 0.40 250,000 0.50
350,000 0.20 300,000 0.20
400,000 0.10 350,000 0.10
The costs associated with the two products have been estimated by Gleason's cost
accounting department and are shown as follows.

Dessert Rolls
Ingredients per unit $ 0.40 $ 0.25
Direct labor per unit 0.35 0.30
Variable overhead per unit 0.40 0.20
Production tooling* 48,000 25,000
Advertising 30,000 20,000
*Gleason treats production tooling as a current operating expense rather than
capitalizing it as a fixed asset.
According to Gleason's market study, the expected value of the sales volume of the
breakfast rolls is

 A. 275,000 units.
 B. Some amount other than those given.
 C. 125,000 units.
 D. 260,000 units.correct
Question was not answered
Correct Answer Explanation:
Hock P2 2020
Section C - Decision Analysis.
Answers
Expected sales volume is a weighted average of the possible sales volume, weighted
according to their probabilities. To find the expected sales volume, multiply each
estimated sales volume by its probability of occurring and sum the results.
(200,000 × 0.2) + (250,000 × 0.5) + (300,000 × 0.2) + (350,000 × 0.1) = 260,000.
Expected sales volume is 260,000 units.
Explanation for Choice A:
This calculation represents the average of the possible sales volumes of the rolls. The
expected value is a weighted average with the various probabilities of each sales
volume as the weights.
Explanation for Choice B:
The correct answer is one of the answer choices given.
Explanation for Choice C:
The answer of 125,000 was calculated by using the estimated sales volume of 250,000
times its probability of 0.5. This is not the expected value of the sales volume of rolls.
90. Question ID: ICMA 10.P2.197 (Topic: Using CVP Analysis in Decision-Making)
Bolger and Co. manufactures large gaskets for the turbine industry. Bolger's per unit
sales price and variable costs for the current year are as follows.

Sales price per unit $300


Variable costs per unit 210
Bolger's total fixed costs aggregate $360,000. As Bolger's labor agreement is expiring
at the end of the year, management is concerned about the effect a new agreement will
have on its unit breakeven point. The controller performed a sensitivity analysis to
ascertain the estimated effect of a $10 per unit direct labor increase and a $10,000
reduction in fixed costs. Based on these data, it was determined that the breakeven
point would

 A. decrease by 125 units.


 B. decrease by 1,000 units.
 C. increase by 500 units.
 D. increase by 375 units.correct
Question was not answered
Correct Answer Explanation:
First, calculate the current breakeven point; then, calculate the breakeven point after the
new labor agreement takes effect; and finally, determine the difference.
Breakeven point in units = Fixed cost ÷ Contribution Margin per Unit
Hock P2 2020
Section C - Decision Analysis.
Answers
The current contribution margin per unit is $300 − $210, or $90. Thus the current
breakeven point is $360,000 ÷ $90 = 4,000 units.
With the changes in costs, the contribution margin per unit will change to $300 − $220,
or $80 and fixed costs will decrease to $350,000. The new breakeven point will be
$350,000 ÷ $80, or 4,375 units.
The increase in the breakeven point is 4,375 – 4,000, or 375 units.
Explanation for Choice A:
This is the estimated $10,000 reduction in fixed costs divided by the $80 unit
contribution margin after the direct labor increase ($300 − [$210 + $10]). To answer this
question correctly, it is necessary to first calculate the current breakeven point; then,
calculate the breakeven point after the new labor agreement takes effect; and finally,
determine the difference.
Explanation for Choice B:
This is the estimated $10,000 reduction in fixed costs divided by the $10 per unit direct
labor increase (which leads to a $10 per unit decrease in the contribution margin). To
answer this question correctly, it is necessary to first, calculate the current breakeven
point; then, calculate the breakeven point after the new labor agreement takes effect;
and finally, determine the difference.
Explanation for Choice C:
The way to solve this problem is to first, calculate the current breakeven point; then,
calculate the breakeven point after the new labor agreement takes effect; and finally,
determine the difference.
This answer results from calculating the breakeven point after the new labor contract
takes effect without decreasing fixed costs for the $10,000 cost savings expected.
91. Question ID: CMA 1296 4.3 (Topic: Using CVP Analysis in Decision-Making)
Kator Co. is a manufacturer of industrial components. One of their products that is used
as a subcomponent in auto manufacturing is KB-96. This product has the following
financial structure per unit:

Selling price $150


Direct materials $ 20
Direct labor 15
Variable manufacturing overhead 12
Fixed manufacturing overhead 30
Shipping and handling 3
Hock P2 2020
Section C - Decision Analysis.
Answers
Fixed selling and administrative 10
Total costs $ 90
During the next year, KB-96 sales are expected to be 10,000 units. All of the costs will
remain the same except that fixed manufacturing overhead will increase by 20% and
direct materials will increase by 10%. The selling price per unit for next year will be
$160. Based on this data, the contribution margin from KB-96 for next year will be

 A. $1,080,000correct
 B. $1,110,000
 C. $750,000
 D. $620,000
Question was not answered
Correct Answer Explanation:
The unit contribution margin formula is sales price per unit − all variable costs per unit.
The sales price next year will be $160. Variable cost include direct material, direct labor,
variable overhead and shipping & handling. Direct material next year will be ($20 × 10%
increase in cost) = $22. Direct labor = $15. Variable overhead = $12. Shipping and
Handling = $3. Total variable cost = $52. The unit contribution margin = $160 − $52 =
$108. The company expects to sell 10,000 units during the year. Therefore, the total
contribution margin for the next year is expected to be $108 × 10,000, or $1,080,000.
Explanation for Choice B:
This answer results from omitting the shipping and handling cost from the calculation of
the contribution margin. Shipping and handling costs are variable costs and should be
deducted in the calculation of the contribution margin.
Explanation for Choice C:
This answer results from two errors:

1. The shipping and handling cost was omitted from the calculation of the contribution
margin. The contribution margin is sales less all variable costs, and shipping and
handling cost is a variable cost that should be deducted in calculating the
contribution margin for the next year.
2. A fixed cost value of $360,000 was deducted, calculated by multiplying
the current year's fixed manufacturing cost of $30 per unit by the next year's
forecasted sales of 10,000 units, and then increasing the resulting amount by 20%.
The question asks for the expected contribution margin for the next year. The
contribution margin is sales less all variable costs. It does not include fixed costs,
so fixed manufacturing costs should not be deducted.
Hock P2 2020
Section C - Decision Analysis.
Answers
It should be noted also that it is not possible to calculate a forecasted amount for fixed
manufacturing costs from the information given. The current year's fixed manufacturing
costs per unit are derived from the current year's total fixed manufacturing costs
divided by the current year's production. It would be necessary to know the current
year's production level in order to determine the current year's fixed manufacturing
costs. The current year's fixed manufacturing costs could be "backed into" by
multiplying the current year's per unit fixed manufacturing cost by the current year's
production volume, if the current year's production volume were known. Since the
current year's production volume is not given, it is not possible to calculate the current
year's fixed manufacturing costs. Therefore, it is also not possible to determine
forecasted fixed manufacturing cost for next year of 20% more than this year's fixed
manufacturing costs.
Explanation for Choice D:
This answer results from two errors:

1. The direct materials cost per unit was not increased to reflect the 10% increase
expected for the next year.
2. A fixed cost value of $480,000 was deducted, calculated by multiplying
the current year's fixed manufacturing cost per unit and fixed selling and
administrative costs per unit ($40 in total) by the next year's forecasted sales of
10,000 units, and then increasing the resulting amount by 20%. The question asks
for the expected contribution margin for the next year. The contribution margin is
sales less all variable costs. It does not include fixed costs, so fixed costs should
not be deducted.
It should be noted also that there are three problems with trying to calculate next year's
forecasted fixed costs from the information given.

1. it is not possible to calculate a forecasted amount for fixed costs from the
information given. The current year's fixed costs per unit are derived from
the current year's total fixed costs divided by the current year's activity. For
example, to calculate the current year's fixed manufacturing costs, it would be
necessary to know the current year's production level. The current year's fixed
manufacturing costs could be "backed into" by multiplying the current year's per
unit fixed manufacturing cost by the current year's production volume, if the current
year's production volume were known. Since the current year's production volume
is not given, it is not possible to calculate the current year's fixed manufacturing
costs. Therefore, it is also not possible to determine forecasted fixed manufacturing
costs for next year of 20% more than this year's fixed manufacturing costs.
2. The fixed selling and administrative cost per unit is also a derived amount. It is
derived from the total fixed selling and administrative costs divided by the total
sales in units for the current year. Since the total sales in units for the current year
is not given, it is not possible to calculate total fixed selling and administrative costs
Hock P2 2020
Section C - Decision Analysis.
Answers
for the current year, so those costs cannot be used as a forecast for fixed selling
and administrative costs for the next year.
3. Furthermore, the fixed selling and administrative costs are not expected to increase
next year, so including them in the amount that was increased by 20% for next year
is incorrect.
92. Question ID: ICMA 10.P2.203 (Topic: Using CVP Analysis in Decision-Making)
MetalCraft produces three inexpensive socket wrench sets that are popular with do-it-
yourselfers. Budgeted information for the upcoming year is as follows.

Estimated
Model Selling Price Variable Cost Sales Volume
No. 109 $10.00 $ 5.50 30,000 sets
No. 145 $15.00 $ 8.00 75,000 sets
No. 153 $20.00 $14.00 45,000 sets
Total fixed cost for the socket wrench product line is $961,000. If the company's actual
experience remains consistent with the estimated sales volume percentage distribution,
and the firm desires to generate total operating income of $161,200, how many Model
No. 153 socket sets will MetalCraft have to sell?

 A. 181,000.
 B. 155,000.
 C. 54,300.correct
 D. 26,000.
Question was not answered
Correct Answer Explanation:
This is a breakeven analysis when more than one product is sold combined with the
requirement for a certain amount of operating income.

109 145 153


Estimated Sales Volume 30,000 75,000 45,000
Percentage of Total (150,000) 20% 50% 30%

Selling price 10.00 15.00 20.00


Variable cost 5.50 8.00 14.00
Contribution margin 4.50 7.00 6.00
Hock P2 2020
Section C - Decision Analysis.
Answers
The weighted average contribution margin for the mix is:
(0.20 × 4.50) + (0.50 × 7.00) + (0.30 × 6.00) = $6.20
Required number of total units for a $161,200 operating profit with $961,000 in fixed
costs:
(961,000 + 161,200) / 6.20 = 181,000 total units of all products
Product No.153 is 30% of that. So the number of units of No. 153 that need to be sold to
generate the required amount of operating income is 0.30 × 181,000, or 54,300.
Explanation for Choice A:
This is the total number of units that need to be sold, but this number includes all three
products. The question asks for the number of units of Product No. 153 that would need
to be sold.
Explanation for Choice B:
This is the total fixed costs of $961,000 divided by the weighted average contribution
margin of $6.20. This is the total number of units of all the products that would need to
be sold in order to break even. This calculation does not include consideration for the
requirement that operating income should be $161,200, plus the resulting number of
units is the total number of units that need to be sold of all products, not the number of
units needed for Product No. 153.
Explanation for Choice D:
This is the required amount of operating income of $161,200 divided by the weighted
average contribution margin of $6.20. The numerator of this calculation needs to be the
total fixed costs plus the required amount of operating income. Furthermore, the
resulting number of units will be the total number of units that will need to be sold of all
products, not the number of unit sold required of Product No. 153.
93. Question ID: CMA 1283 5.19 (Topic: Using CVP Analysis in Decision-Making)
The process of evaluating the effect of changes in variables such as sales price or wage
rates on operating income is called

 A. sensitivity analysis.correct
 B. iterative analysis.
 C. regression analysis.
 D. matrix analysis.
Question was not answered
Correct Answer Explanation:
Hock P2 2020
Section C - Decision Analysis.
Answers
Sensitivity analysis is used to determine the effect that a change in a variable will have
on the final result. It enables management to identify which variables are more critical to
the ultimate result of a project.
Explanation for Choice B:
Iterative analysis is incorrect.
Explanation for Choice C:
Regression analysis is used to find a linear relationship between two or more variables.
Explanation for Choice D:
Matrix analysis is incorrect.
94. Question ID: ICMA 13.P2.048 (Topic: Using CVP Analysis in Decision-Making)
Leslie Corporation manufactures classroom desk chairs and tables. In the present
market, the company can sell as many units of product as it can manufacture, but it is
constrained by its availability of machine-hour capacity. Sales price and cost information
for each unit of product are shown below.

Desk
Chairs Tables
Sales price $75 $180
Variable costs 60 155
Contribution margin $15 $ 25
Producing a desk chair requires 1½ machine hours; producing a table requires 2½
machine hours. Which product, if any, is most profitable given the machine-hour
constraints?

 A. Desk chairs.
 B. There is not enough data to identify the most profitable product.
 C. Tables.
 D. Both products are equally profitable.correct
Question was not answered
Correct Answer Explanation:
The product with the higher contribution margin per unit of the constrained resource is
the most profitable. The contribution margin per unit for desk chairs is $15 and desk
chairs require 1.5 machine hours each, so the contribution margin per machine hour for
desk chairs is $15 ÷ 1.5, or $10. The contribution margin per unit for tables is $25 and
tables require 2.5 machine hours each, so the contribution margin per machine hour for
Hock P2 2020
Section C - Decision Analysis.
Answers
tables is $25 ÷ 2.5, or $10. Therefore, desk chairs and tables are equally profitable,
given the machine-hour constraint.
Explanation for Choice A:
The product with the higher contribution margin per unit of the constrained resource is
the most profitable. The contribution margin per machine hour for desks is not higher
than it is for tables.
Explanation for Choice B:
There is enough data to identify the product that is most profitable.
Explanation for Choice C:
The product with the higher contribution margin per unit of the constrained resource is
the most profitable. The contribution margin per machine hour for tables is not higher
than it is for desks.
95. Question ID: CMA 687 4.11 (Topic: Using CVP Analysis in Decision-Making)
Donnelly Corporation manufactures and sells T-shirts imprinted with college names and
slogans. Last year, the shirts sold for $7.50 each, and the variable cost to manufacture
them was $2.25 per unit. The company needed to sell 20,000 shirts to break even. The
net income last year was $5,040. Donnelly's expectations for the coming year include
the following:

 The sales price of the T-shirts will be $9


 Variable cost to manufacture will increase by one-third
 Fixed costs will increase by 10%
 The income tax rate of 40% will be unchanged
The number of T-shirts Donnelly Corporation must sell to break even in the coming year
is

 A. Some amount other than those given.


 B. 17,500
 C. 19,250correct
 D. 20,000
Question was not answered
Correct Answer Explanation:
In this problem, we first need to calculate fixed cost for last year. Given that the
breakeven volume last year was 20,000 T-shirts and the contribution margin was $5.25
($7.50 − $2.25 variable cost), we can use the breakeven formula of total fixed cost / unit
contribution margin to calculate what fixed costs were last year:
Hock P2 2020
Section C - Decision Analysis.
Answers
X / $5.25 = 20,000
X = $105,000
Now, we can find what fixed costs are expected to be in the coming year. Fixed costs
are expected to increase by 10%, so fixed costs in the coming year are expected to be
$115,500 ($105,000 × 1.10).
The unit contribution margin next year will be $9 − $3, or $6. Using the breakeven
formula again, the breakeven point next year will be $115,500 / $6, or 19,250 units.
Explanation for Choice A:
The correct answer is among the answer choices given.
Explanation for Choice B:
This answer results from using the previous year's fixed costs to calculate the
breakeven point for the coming year. The question says the fixed costs will increase by
10%..
Explanation for Choice D:
This is last year's breakeven point.
96. Question ID: ICMA 19.P2.013 (Topic: Using CVP Analysis in Decision-Making)
A circuit board company conducts a joint manufacturing process to produce 10,000
units of Board A and 10,000 units of Board B. The total joint variable manufacturing cost
to produce these two products is $2,000,000. The company can sell all 10,000 units of
Board B at the split-off point for $300 per unit, or process Board B further and sell all
10,000 units at $375 per unit. The total additional cost to process Board B further would
be $500,000, and all additional costs would be variable. If the company decides to
process Board B further, what effect would the decision have on operating income?

 A. $3,250,000 increase in operating income.


 B. $2,250,000 increase in operating income.
 C. $750,000 decrease in operating income.
 D. $250,000 increase in operating income.correct
Question was not answered
Correct Answer Explanation:
There is a lot of information in this question that is not relevant to what we need to do.
We need to calculate the incremental cost and the incremental revenue of the
company processing Product B after the split-off point.
At the split-off point, the selling price is $300 per unit and if they process it further, the
selling price will be $375 per unit. This is an additional $75 of revenue per unit and since
there are 10,000 units, it is an increase in total revenue of $750,000.
Hock P2 2020
Section C - Decision Analysis.
Answers
The incremental costs of processing further are $500,000.
If the company produces Product B further, they will have a $250,000 increase in
operating income.
Explanation for Choice A:
This is the amount of operating income from Product B is no joint costs are allocated to
Product B. This is not the question that is being asked.
Explanation for Choice B:
This is the amount of operating income from Product B if one-half of the joint production
costs are allocated to Product B. This is not the question that is being asked.
Explanation for Choice C:
This answer incorrectly includes half of the joint production costs in the calculation.
These costs are not part of the decision to process Product B further, or not.
97. Question ID: ICMA 10.P2.210 (Topic: Using CVP Analysis in Decision-Making)
For the year just ended, Silverstone Company's sales revenue was $450,000.
Silverstone’s fixed costs were $120,000 and its variable costs amounted to $270,000.
For the current year sales are forecasted at $500,000. If the fixed costs do not change,
Silverstone’s profits this year will be

 A. $60,000.
 B. $110,000.
 C. $200,000.
 D. $80,000.correct
Question was not answered
Correct Answer Explanation:
The contribution margin ratio for the past year was 0.40, calculated as ($450,000 −
$270,000) ÷ $450,000. Assuming the contribution margin does not change, for the
current year the contribution margin will be the current year’s forecasted sales of
$500,000 × 0.40, or $200,000. Fixed costs last year were $120,000, and if they do not
change, Silverstone’s profits this year will be the contribution margin of $200,000 – fixed
costs of $120,000, or $80,000.
Explanation for Choice A:
This is the previous year's profits.
Explanation for Choice B:
Hock P2 2020
Section C - Decision Analysis.
Answers
This answer results from using the previous year's variable costs as the current year's
variable costs. The current year's variable costs will be higher than the previous year's
variable costs, because the forecasted sales for the current year are higher.
Explanation for Choice C:
This is the forecasted contribution margin for the current year. The contribution margin
minus fixed costs equals profits.
98. Question ID: CMA 692 4.26 (Topic: Using CVP Analysis in Decision-Making)
J J Motors, Inc. employs 45 sales personnel to market its line of luxury automobiles.
The average car sells for $23,000, and a 6% commission is paid to the salesperson. J J
Motors is considering a change to a commission arrangement that would pay each
salesperson a salary of $2,000 per month plus a commission of 2% of the sales made
by that salesperson. The amount of total monthly car sales at which J J Motors would
be indifferent as to which plan to select is

 A. $1,250,000.
 B. $3,000.000.
 C. $2,250,000.correct
 D. $1,500,000.
Question was not answered
Correct Answer Explanation:
There is more than one correct way to calculate the answer to this problem. One
solution is to calculate the sales volume where J J Motors will indifferent as to which
plan is selected. Remember that the total sales volume will be the sales made per
salesperson multiplied by the number of sales personnel.
The solution can be calculated algebraically as follows:
Let X be the monthly sales per salesperson. Create two equations, each containing one
of the possible commission payment methods. The two equations are:
C = 0.06X and C = $2,000 × 0.06X, where X = the monthly car sales.
Next, set the right sides of the two commission payment methods equal to one another,
since we are looking for the total monthly sales at which the same commission amounts
would be paid under both methods. Solve for X, and then multiply the value you get for
X by 45 to get the total sales for all salespeople per month.
0.06X = $2,000 + 0.02X
Simplify by subtracting 0.02X from both sides of the equation:
0.04X = $2,000
Solve for X by dividing both sides of the equation by 0.04:
Hock P2 2020
Section C - Decision Analysis.
Answers
X = $50,000, which is the sales per salesperson at which the payment under both plans
will be equal.
Multiply $50,000 by 45 salespeople to get the total sales per month at which both
payment plans will result in the same payments to salespeople:
$50,000 × 45 = $2,250,000
To check:
At a sales volume of $2,250,000, a straight commission of 6% will result in a total
payment of $2,250,000 × 0.06, or $135,000.
At a sales volume of $2,250,000, a salary of $2,000 per month paid to each salesperson
plus a commission of 2% will result in a total payment of:
($2,000 × 45) + (0.02 × $2,250,000) = $135,000
If you calculated the answer a different way but got the same answer, then your method
is also correct.
Explanation for Choice A:
A monthly total sales level of $1,250,000 would result in total commissions paid of
$75,000 if J J Motors continues paying commissions at the rate of 6% of sales
(1,250,000 × 0.06). If J J Motors adopts the new commission formula of $2,000 per
month per salesperson plus 2% of sales, the total commissions paid would be $115,000
($2,000 × 45) + (0.02 × $1,250,000). At the sales level of $1,250,000, J J Motors would
prefer the existing commission formula because the commission payout would be lower.
Explanation for Choice B:
A monthly total sales level of $3,000,000 would result in total commissions paid of
$180,000 if J J Motors continues paying commissions at the rate of 6% of sales
($3,000,000 × 0.06). If J J Motors adopts the new commission formula of $2,000 per
month per salesperson plus 2% of sales, the total commissions paid would be $150,000
($2,000 × 45) + (0.02 × $3,000,000). At a sales level of $3,000,000, J J Motors would
prefer the new commission formula because the commission payout would be lower.
Explanation for Choice D:
A monthly total sales level of $1,500,000 would result in total commissions paid of
$90,000 if J J Motors continues paying commissions at the rate of 6% of sales
(1,500,000 × 0.06). If J J Motors adopts the new commission formula of $2,000 per
month per salesperson plus 2% of sales, the total commissions paid would be $120,000
($2,000 × 45) + (0.02 × $1,500,000). At a sales level of $1,500,000, J J Motors would
prefer the existing commission formula because the commission payout would be lower.
99. Question ID: CMA 1294 4.3 (Topic: Using CVP Analysis in Decision-Making)
Hock P2 2020
Section C - Decision Analysis.
Answers
Marston Enterprises sells three chemicals: petrol, septine, and tridol. Petrol is the
company's most profitable product; tridol is the least profitable. Which one of the
following events will definitely decrease the firm's overall breakeven point for the
upcoming accounting period?

 A. The installation of new computer-controlled machinery and subsequent layoff of


assembly-line workers.
 B. A decrease in tridol's selling price.
 C. An increase in anticipated sales of petrol relative to sales of septine and tridol.correct
 D. An increase in the overall market for septine.
Question was not answered
Correct Answer Explanation:
Petrol is the most profitable product for the firm, so an increase in the sales of petrol
would increase the weighted average unit contribution margin of the three products. The
increase in the weighted average unit contribution margin would decrease the
breakeven point (Fixed costs/average unit contribution margin = breakeven point in
units).
Explanation for Choice A:
The installation of new computer-controlled machinery and the layoff of assembly-line
workers would increase fixed costs. However, the unit contribution margin would
increase as a result of the layoffs. Whether the breakeven point would increase or
decrease would depend upon the magnitude of these two changes.
Explanation for Choice B:
Decreasing the selling price of tridol would decrease its unit contribution margin. The
decrease in the unit contribution margin would increase the firm's overall breakeven
point.
Explanation for Choice D:
An increase in the overall market for septine would not, by itself, affect the firm's overall
breakeven point because it would not change either the firm's total fixed costs or
septine's unit contribution margin.
100. Question ID: CMA 679 5.25 (Topic: Using CVP Analysis in Decision-Making)
Moorehead Manufacturing Company produces two products for which the following data
have been tabulated. Fixed manufacturing cost is applied at a rate of $1.00 per machine
hour.

Per Unit XY-7 BD-4


Selling price $4.00 $3.00
Hock P2 2020
Section C - Decision Analysis.
Answers
Variable manufacturing cost $2.00 $1.50
Fixed manufacturing cost $0.75 $0.20
Variable selling cost $1.00 $1.00
The sales manager has had a $160,000 increase in the budget allotment for advertising
and wants to apply the money to the most profitable product. The products are not
substitutes for one another in the eyes of the company's customers.
Suppose the sales manager chooses to devote the entire $160,000 to increased
advertising for XY-7. The minimum increase in sales units of XY-7 required to offset the
increased advertising is:

 A. 128,000 units.
 B. 160,000 units.correct
 C. 640,000 units.
 D. 80,000 units.
Question was not answered
Correct Answer Explanation:
The unit contribution margin for XY-7 is $1 [$4.00 − ($2.00 variable manufacturing cost
+ $1.00 variable selling cost)]. The increase of $160,000 in advertising fixed cost divided
by the $1 contribution margin = 160,000 increase in units sold that would be required to
offset the increased advertising.
Explanation for Choice A:
This is the $160,000 increase in advertising expense divided by ($4.00 selling price −
$2.00 variable manufacturing costs − $0.75 fixed manufacturing cost) for XY-7. Fixed
manufacturing cost should not be included in the calculation of the unit contribution
margin, and all variable costs—not just variable manufacturing costs—should be
included in the calculation of the contribution margin.
Explanation for Choice C:
This is the $160,000 increase in advertising expense divided by ($4.00 selling price −
$2.00 variable manufacturing costs − $0.75 fixed manufacturing cost − $1.00 variable
selling cost) for XY-7. The calculation of $0.25 for XY-7's unit contribution margin
includes fixed manufacturing cost. Only variable costs are included in the calculation of
contribution margin.
Explanation for Choice D:
This is the $160,000 increase in advertising expense divided by ($4.00 selling price −
$2.00 variable manufacturing costs) for XY-7. All variable costs—not just variable
Hock P2 2020
Section C - Decision Analysis.
Answers
manufacturing costs—should be included in the calculation of the contribution margin.
The use of $2.00 for XY-7's variable costs omits the variable selling cost.
101. Question ID: CMA 679 5.27 (Topic: Using CVP Analysis in Decision-Making)
Moorehead Manufacturing Company produces two products for which the following data
have been tabulated. Fixed manufacturing cost is applied at a rate of $1.00 per machine
hour.

Per Unit XY-7 BD-4


Selling price $4.00 $3.00
Variable manufacturing cost $2.00 $1.50
Fixed manufacturing cost $0.75 $0.20
Variable selling cost $1.00 $1.00
The sales manager has had a $160,000 increase in the budget allotment for advertising
and wants to apply the money to the most profitable product. The products are not
substitutes for one another in the eyes of the company's customers.
Suppose Moorehead has only 100,000 machine hours that can be made available to
produce additional units of XY-7 and BD-4. If the potential increase in sales units for
either product resulting from advertising is far in excess of this production capacity,
which product should be advertised and what is the estimated increase in contribution
margin earned?

 A. Product XY-7 should be produced, yielding a contribution margin of $75,000.


 B. Product BD-4 should be produced, yielding a contribution margin of $187,500.
 C. Product XY-7 should be produced, yielding a contribution margin of $133,333.
 D. Product BD-4 should be produced, yielding a contribution margin of $250,000.correct
Question was not answered
Correct Answer Explanation:
In this problem, it is necessary to optimize the contribution margin of a scarce resource.
The scarce resource in this problem is machine hours.

XY-7 BD-4
Sales price $4.00 $3.00
Variable costs $3.00 $2.50
Contribution margin $1.00 $0.50
Fixed manufacturing costs are applied based on machine hours. Therefore, the
machine hours required to produce one unit of product XY-7 are 0.75 ($0.75 ÷ $1.00)
Hock P2 2020
Section C - Decision Analysis.
Answers
and 0.20 for product BD-4 ($0.20 ÷ $1.00). This means that the CM per machine hour
for XY-7 is $1.33 ($1.00 ÷ 0.75), and the CM per machine hour for BD-4 is $2.50 ($0.50
÷ 0.20). Since BD-4 has the higher contribution margin per machine hours used and
since the potential increase in sales units resulting from the advertising is far in excess
of this production capacity, we know that all that Moorhead can produce can be sold.
Therefore, only BD-4 should be produced. Since 100,000 machine hours are available
and each unit of BD-4 requires 0.20 hour to produce, the company can produce
500,000 units (100,000 ÷ 0.20). The total contribution margin will be be $250,000 ($0.50
UCM × 500,000 units sold). It is always best to optimize the contribution margin of the
scarce resource.
Explanation for Choice A:
This is not the correct answer. Please see the correct answer for an explanation.
We have been unable to determine how to calculate this incorrect answer choice. If you
have calculated it, please let us know how you did it so we can create a full explanation
of why this answer choice is incorrect. Please send us an email at
support@hockinternational.com. Include the full Question ID number and the actual
incorrect answer choice -- not its letter, because that can change with every study
session created. The Question ID number appears at the top of the question. Thank you
in advance for helping us to make your HOCK study materials better.
Explanation for Choice B:
This is not the correct answer. Please see the correct answer for an explanation.
We have been unable to determine how to calculate this incorrect answer choice. If you
have calculated it, please let us know how you did it so we can create a full explanation
of why this answer choice is incorrect. Please send us an email at
support@hockinternational.com. Include the full Question ID number and the actual
incorrect answer choice -- not its letter, because that can change with every study
session created. The Question ID number appears at the top of the question. Thank you
in advance for helping us to make your HOCK study materials better.
Explanation for Choice C:
This is the contribution margin that would be earned if Moorhead were to use its
facilities to produce only XY-7. 0.75 machine hours are required to produce one unit of
XY-7. 100,000 machine hours available divided by 0.75 machine hours required per unit
of XY-7 equals 133,333 units of XY-7 that could be produced using the available
machine hours. Since the contribution margin per unit of XY-7 is $1.00 ($4.00 selling
price minus $2.00 variable manufacturing cost minus $1.00 variable selling cost), the
contribution margin received for producing 133,333 units of XY-7 would be $133,333.
However, that contribution margin is not the maximum contribution margin possible from
the use of the available resources.
Hock P2 2020
Section C - Decision Analysis.
Answers
102. Question ID: ICMA 13.P2.042 (Topic: Relevant Information for Decision-
Making, Economic Costs)
Sudden economic changes have forced the AutoFacsimilie Co. to alter its business
strategy. The company is considering eliminating product lines, laying off production
workers, reducing advertising, and closing one of its factories. In taking these actions,
which one of the following costs should be considered sunk costs?

 A. The costs of selling or demolishing the factory.


 B. Production workers' wages, severance, and advertising.
 C. Utility costs at the closed factory and real estate taxes.
 D. Research and development costs of eliminated product lines.correct
Question was not answered
Correct Answer Explanation:
Sunk costs are costs that have already been incurred and cannot be changed no matter
which decision is made today. Research and development costs of eliminated product
lines have already been spent, and no decision made now can change them. Therefore,
those costs are sunk costs.
Explanation for Choice A:
Sunk costs are costs that have already been spent and cannot be changed no matter
which decision is made today. The cost of selling the factory or demolishing it have not
yet been incurred and could therefore be avoided in the future by deciding to continue to
operate the factory. Therefore, they are not sunk costs.
Explanation for Choice B:
Sunk costs are costs that have already been spent and cannot be changed no matter
which decision is made today. As none of the production worker’s wages, severance
and advertising costs have been incurred yet, all of them could be avoided depending
on the decision that is made today. Therefore, they are not sunk costs.
Explanation for Choice C:
Sunk costs are costs that have already been spent and cannot be changed no matter
which decision is made today.
As long as the company continues to own the factory, it will continue to have expenses
related to its ownership, even if the factory is closed. Real estate taxes will continue to
be due and payable, because they are based on the value of the property, whether or
not the property is occupied. It will be necessary to keep some utilities connected in
order to preserve the value of the factory, even if it is not being used. For example, if the
factory is located in a cold climate, they will need to keep it heated at least somewhat so
that frost damage does not occur in the building. If the factory is located in a hot climate,
Hock P2 2020
Section C - Decision Analysis.
Answers
they will need to keep it cooled at least somewhat so that mold does not grow in the
building. So those expenses will continue in the future, until they sell the building.
Therefore, utility costs and real estate taxes for the closed factory are not sunk costs
because they are not costs that have already been spent. They are future costs.
103. Question ID: CMA 694 4.19 (Topic: Relevant Information for Decision-Making,
Economic Costs)
Management accountants are frequently asked to analyze various decision situations,
including the following:
I. The cost of a special device that is necessary if a special order is accepted.
II. The cost proposed annually for the plant service for the grounds at corporate
headquarters.
III. Joint production costs incurred, to be considered in a sell-at-split versus a process-
further decision.
IV. The costs associated with alternative uses of plant space, to be considered in a
make/buy decision.
V. The cost of obsolete inventory acquired several years ago, to be considered in a
keep-versus-disposal decision.
The costs described in situations I and IV are:

 A. Discretionary costs.
 B. Prime costs.
 C. Relevant costs.correct
 D. Sunk costs.
Question was not answered
Correct Answer Explanation:
Situations I and IV are incremental (differential) costs that are relevant to the decision.
Explanation for Choice A:
Discretionary costs are costs that can be deferred to future periods without creating a
significant impact on the current period's results.
Explanation for Choice B:
Prime costs include Direct Material and Direct Labor.
Explanation for Choice D:
Sunk cost are past costs and are not relevant to the decision process.
104. Question ID: ICMA 10.P2.221 (Topic: Relevant Information for Decision-
Making, Economic Costs)
Hock P2 2020
Section C - Decision Analysis.
Answers
In a joint manufacturing process, joint costs incurred prior to a decision as to whether to
process the products after the split-off point should be viewed as

 A. standard costs.
 B. sunk costs.correct
 C. relevant costs.
 D. differential costs.
Question was not answered
Correct Answer Explanation:
Sunk costs are costs that have already been incurred and cannot be changed. Joint
costs incurred prior to a decision as to whether to process the products after the split-off
point are sunk costs, because they have already been incurred at the point when the
decision is made.
Explanation for Choice A:
Standard costs are estimated costs for direct materials, direct labor and manufacturing
overhead that are predetermined or estimated as they would apply under specified
conditions. Standard costs are a fundamental element of the budgeting process. Joint
costs incurred are actual costs, so they cannot be standard costs.
Explanation for Choice C:
The only relevant factors are the incremental revenues and incremental costs that
would be incurred if the decision is made to process the products further. Joint costs
incurred prior to the decision are not relevant costs because they would not differ
between the alternatives.
Explanation for Choice D:
Differential costs are costs that differ between options. The joint costs have already
been incurred prior to the decision about whether to process the products further after
the split-off point. The joint costs will be the same whether the products are processed
further or not. Therefore, they are not differential costs.
105. Question ID: CMA 691 4.10 (Topic: Relevant Information for Decision-Making,
Economic Costs)
Sunk costs

 A. Are fixed costs.


 B. Are relevant to decision making.
 C. In and of themselves are not relevant to decision making.correct
 D. Are substitutes for opportunity costs.
Question was not answered
Hock P2 2020
Section C - Decision Analysis.
Answers
Correct Answer Explanation:
Sunk costs are past incurred costs that are irrelevant in the decision making process
because they have already been incurred and no current decision can change them.
Explanation for Choice A:
Fixed costs can be sunk costs, or they may not be sunk costs.
Explanation for Choice B:
Sunk costs are irrelevant to the decision making process because they have already
been incurred and no current decision can change them.
Explanation for Choice D:
Opportunity costs and sunk costs are two different things.
106. Question ID: CMA 687 5.27 (Topic: Relevant Information for Decision-Making,
Economic Costs)
Gleason Co. has two products, a frozen dessert and ready-to-bake breakfast rolls,
ready for introduction. However, plant capacity is limited, and only one product can be
introduced at present. Therefore, Gleason has conducted a market study, at a cost of
$26,000, to determine which product will be more profitable. The results of the study
follow.

Sales of Desserts Sales of Rolls


at $1.80/unit at $1.20/unit
Volume Probability Volume Probability
250,000 0.30 200,000 0.20
300,000 0.40 250,000 0.50
350,000 0.20 300,000 0.20
400,000 0.10 350,000 0.10
The costs associated with the two products have been estimated by Gleason's cost
accounting department and are shown as follows.

Dessert Rolls
Ingredients per unit $ 0.40 $ 0.25
Direct labor per unit 0.35 0.30
Variable overhead per unit 0.40 0.20
Production tooling* 48,000 25,000
Hock P2 2020
Section C - Decision Analysis.
Answers
Advertising 30,000 20,000
*Gleason treats production tooling as a current operating expense rather than
capitalizing it as a fixed asset.
Assuming that Gleason elects to produce the frozen dessert, the profit that would have
been earned on the breakfast rolls is a(n)

 A. Opportunity cost.correct
 B. Avoidable cost.
 C. Deferrable cost.
 D. Sunk cost.
Question was not answered
Correct Answer Explanation:
An opportunity cost is the contribution to income that is lost by not using a limited
resource in its next best alternative use. Therefore, if Gleason produces the frozen
dessert, the profit that could have been earned on the breakfast rolls if they had been
produced instead is an opportunity cost.
Explanation for Choice B:
Avoidable costs are those costs that can be eliminated if a specific activity is
discontinued.
Explanation for Choice C:
Deferrable costs are those costs that can be deferred to some future date without the
deferral creating a significant impact on current operations.
Explanation for Choice D:
Sunk costs are costs that have already been incurred and thus have no relevance in
management's decisions for the future.
107. Question ID: CMA 696 4.16 (Topic: Relevant Information for Decision-Making,
Economic Costs)
In a decision analysis situation, which one of the following costs is
generally not relevant to the decision?

 A. Differential cost.
 B. Historical cost.correct
 C. Avoidable cost.
 D. Incremental cost.
Question was not answered
Hock P2 2020
Section C - Decision Analysis.
Answers
Correct Answer Explanation:
Historical costs are sunk costs that are not relevant to the decision making process.
They are costs that have already been incurred, and nothing can be done to change
them no matter what decision is made. In the decision making process you want to look
only at costs that differ among the related options.
Explanation for Choice A:
Differential costs are costs that differ between two alternatives. These costs are relevant
to the decision making process.
Explanation for Choice C:
Avoidable costs are relevant to decision making process because they are costs that
can be avoided depending upon the decision that is made. In the decision making
process we need to look at all avoidable (variable and fixed) costs that will vary among
the related options.
Explanation for Choice D:
Incremental costs are additional costs that result when production, or some other factor
is increased. These cost are relevant to the decision making process.
108. Question ID: ICMA 10.P2.223 (Topic: Relevant Information for Decision-
Making, Economic Costs)
Profits that are lost by moving an input from one use to another are referred to as

 A. out-of-pocket costs.
 B. cannibalization charges.
 C. opportunity costs.correct
 D. replacement costs.
Question was not answered
Correct Answer Explanation:
An opportunity cost is the contribution to income that is lost by not using a limited
resource in its best alternative use. Profits lost by moving an input from one use to
another is an example of an opportunity cost.
Explanation for Choice A:
Out-of-pocket costs are actual, explicit costs that are paid directly. Profits lost by moving
an input from one use to another are implicit costs because there is no real expense
paid out. The cost is the loss of an opportunity to earn a profit.
Explanation for Choice B:
Hock P2 2020
Section C - Decision Analysis.
Answers
Cannibalization charges are related to competing products produced by the same
company. When one product "cannibalizes" another product, it takes sales away from
the other product. Usually the product that cannibalizes the sales is a new product that
takes sales away from an existing product produced by the same company. However,
this question relates to profits sacrificed by not producing a product, not profits lost
because sales are taken away by another, competing product produced by the same
company.
Explanation for Choice D:
Replacement costs are explicit future costs required to replace something used today.
Profits lost by moving an input from one use to another are implicit costs.
109. Question ID: CMA 1290 4.11 (Topic: Relevant Information for Decision-
Making, Economic Costs)
The relevance of a particular cost to a decision is determined by the

 A. Potential effect on the decision.correct


 B. Riskiness of the decision.
 C. Accuracy and verifiability of the cost.
 D. Size of the cost.
Question was not answered
Correct Answer Explanation:
The relevance of a cost depends on whether it varies among the related options.
Explanation for Choice B:
The riskiness of a project is not relevant if it does not vary among the related options.
Explanation for Choice C:
Accuracy and verifiability of a cost do not determine whether the cost is relevant to a
decision. Costs may be relevant even if they are estimates.
Explanation for Choice D:
The size of the cost is not relevant if the cost does not vary among the related options.
110. Question ID: ICMA 19.P2.006 (Topic: Relevant Information for Decision-
Making, Economic Costs)
A company plans to add a new product that would affect its indirect labor costs in two
ways. First, the production manager from an existing product would serve as manager
of the new product. Her current assistant manager would be promoted and assume her
previous position. Second, the existing maintenance staff would provide facility and
machine maintenance that would require 30 hours of labor each month, but no increase
in their total weekly hours worked. The company’s production managers earn $60,000
Hock P2 2020
Section C - Decision Analysis.
Answers
annually, assistant production managers are paid $50,000 each year, and maintenance
employees earn $20 per hour. No additional hiring is planned. The annual relevant
indirect labor costs for adding the new product would total

 A. $7,200.
 B. $60,600.
 C. $10,000.correct
 D. $67,200.
Question was not answered
Correct Answer Explanation:
The only relevant costs are the costs that are incurred just because the company is
adding the new product line.
The amount paid to the maintenance staff is not relevant because it states that they will
not work any additional hours as a result of adding the new product line. So, while some
costs would be allocated to the new product line, the company will not pay any more to
the maintenance staff as a result of adding the product line.
For the manager, the only new cost that the company would have as a result of adding
the new product line is the $10,000 increase that the assistant manager would receive
when they become a manager.
The relevant indirect labor costs for the addition of the new product line is only $10,000.
Explanation for Choice A:
This answer incorrectly treats the $7,200 annual amount paid to the maintenance staff
as a relevant cost.
See the correct answer for a full explanation.
Explanation for Choice B:
This answer incorrectly treats the entire amount of the new manager's salary as a
relevant cost, Only the increase of $10,000 is relevant to this decision.
Also, this answer incorrectly treats the $600 a month paid to the maintenance staff as a
relevant cost.
See the correct answer for a full explanation.
Explanation for Choice D:
This answer incorrectly treats the entire amount of the new manager's salary as a
relevant cost, Only the increase of $10,000 is relevant to this decision.
Also, this answer incorrectly treats the $7,200 annual amount paid to the maintenance
staff as a relevant cost.
Hock P2 2020
Section C - Decision Analysis.
Answers
See the correct answer for a full explanation.
111. Question ID: CIA 1186 IV.17 (Topic: Relevant Information for Decision-
Making, Economic Costs)
The data available for the current year is given below.

Whole
company Division 1 Division 2
Variable manufacturing cost of goods sold $ 400,000 $220,000 $180,000
Unallocated costs (e.g., president's salary) 100,000
Fixed costs controllable by division managers
(e.g., advertising, engineering supervision costs) 90,000 50,000 40,000
Net revenue 1,000,000 600,000 400,000
Variable selling and administrative costs 130,000 70,000 60,000
Fixed costs controllable by others
(e.g., depreciation, insurance) 120,000 70,000 50,000
Using the information presented above, the net operating income contributed to the
company by Division 1 was:

 A. $190,000correct
 B. $380,000
 C. $310,000
 D. $260,000
Question was not answered
Correct Answer Explanation:
This question is asking for the net operating income contributed by Division 1, so we
need to subtract all direct costs of the division and costs allocated to it from its net
revenue. Revenue of $600,000 minus total variable costs (manufacturing and S&A) of
$290,000 minus controllable fixed costs of $50,000 minus fixed cost controllable by
others of $70,000 equals $190,000.
Explanation for Choice B:
This is Division 1's net revenue minus its variable manufacturing costs. However, all
direct and allocated divisional costs need to be subtracted.
Explanation for Choice C:
This is the contribution margin, but the question is asking for net operating income
contributed by the division, so we need to subtract out all costs allocated to division.
Hock P2 2020
Section C - Decision Analysis.
Answers
Explanation for Choice D:
This answer is the contribution controllable by the divisional manager, but net operating
income contributed is found by subtracting out the fixed allocated costs (controllable by
others) as well.
112. Question ID: ICMA 10.P2.218 (Topic: Relevant Information for Decision-
Making, Economic Costs)
Johnson waits two hours in line to buy a ticket to an NCAA Final Four Tournament. The
opportunity cost of buying the $200 ticket is

 A. the value of the $200 to the ticket agent.


 B. Johnson's best alternative use of the $200.
 C. Johnson's best alternative use of both the $200 and the two hours spent in line.correct
 D. Johnson's best alternative use of the two hours it took to wait in line.
Question was not answered
Correct Answer Explanation:
An opportunity cost is the contribution to income or benefit that is forgone by not using a
limited resource in its best alternative use. In this case, there are two limited resources:
Johnson's time and money. If Johnson had not spent two hours waiting in line, he might
have been able to earn some money by working. And if Johnson had not spent $200 on
the ticket, he would have been able to use the $200 to buy something else.
Explanation for Choice A:
An opportunity cost is the contribution to income or benefit that is forgone by not using a
limited resource in its best alternative use. The value of the $200 to the ticket agent is
not an opportunity cost of Johnson's.
Explanation for Choice B:
An opportunity cost is the contribution to income or benefit that is forgone by not using a
limited resource in its best alternative use. Johnson's best alternative use of the $200 is
a part of the opportunity cost but it is not the full opportunity cost.
Explanation for Choice D:
An opportunity cost is the contribution to income or benefit that is forgone by not using a
limited resource in its best alternative use. Johnson's best alternative use of the two
hours it took to wait in line is a part of the opportunity cost but it is not the full
opportunity cost.
113. Question ID: ICMA 10.P2.220 (Topic: Relevant Information for Decision-
Making, Economic Costs)
In order to avoid pitfalls in relevant-cost analysis, management should focus on
Hock P2 2020
Section C - Decision Analysis.
Answers
 A. anticipated fixed costs and variable costs of all alternatives.
 B. long-run fixed costs of each alternative.
 C. variable cost items that differ for each alternative.
 D. anticipated revenues and costs that differ for each alternative.correct
Question was not answered
Correct Answer Explanation:
In order to avoid pitfalls in relevant-cost analysis, management should focus on
anticipated revenues and costs that differ for each alternative.
A "pitfall" is a trap or a danger that can catch a person who is not being careful. Picture
someone walking along, not watching where he is walking, and he falls into a pit. That is
what a pitfall is. It is something that goes wrong because you have not been paying
attention to what you were doing. (For example, answering an exam question incorrectly
because you missed reading a fact that was important!)
So when this question asks "In order to avoid pitfalls in relevant-cost analysis,
management should focus on . . ." it means in order to avoid arriving at the wrong
conclusion when doing relevant-cost analysis, management should focus on . . .
The definition of relevant revenues and relevant costs is expected future revenues and
costs that differ among alternatives. If a revenue or an expense will be the same no
matter which alternative is selected, it is irrelevant to the decision. The only relevant
revenues and costs are those that will be affected by the decision that is to be made.
Furthermore, it is important to focus on the future since nothing can be done to change
past costs that have already been incurred. Those are called sunk costs, and sunk
costs are irrelevant to the decision because they cannot be affected by the decision that
is to be made.
Although the question asks only about relevant cost analysis and this answer includes
revenues, this is the best answer from among the answer choices given.
Explanation for Choice A:
"Anticipated fixed costs and variable costs of all alternatives" is not correct because
some of these costs might be exactly the same for each alternative. The costs that are
exactly the same for each alternative are not relevant, because they will not be changed
by whatever alternative is chosen. The only relevant revenues and costs are those
future revenues and costs that will be affected by the decision that is to be made.
Explanation for Choice B:
In the long run, all costs are variable. Therefore, there really is no such thing as a long-
run fixed cost.
Hock P2 2020
Section C - Decision Analysis.
Answers
However, "fixed costs of each alternative" is not correct because because some or all of
the fixed costs of each alternative might be exactly the same. If costs are the same for
each alternative, they are not relevant costs. The only relevant revenues and costs are
those future revenues and costs that will be affected by the decision that is to be made.
Explanation for Choice C:
"Variable cost items that differ for each alternative" is too limited. Relevant costs are all
costs that differ for each alternative, not only variable costs.
114. Question ID: CMA 687 5.25 (Topic: Relevant Information for Decision-Making,
Economic Costs)
Gleason Co. has two products, a frozen dessert and ready-to-bake breakfast rolls,
ready for introduction. However, plant capacity is limited, and only one product can be
introduced at present. Therefore, Gleason has conducted a market study, at a cost of
$26,000, to determine which product will be more profitable. The results of the study
follow.

Sales of Desserts Sales of Rolls


at $1.80/unit at $1.20/unit
Volume Probability Volume Probability
250,000 0.30 200,000 0.20
300,000 0.40 250,000 0.50
350,000 0.20 300,000 0.20
400,000 0.10 350,000 0.10
The costs associated with the two products have been estimated by Gleason's cost
accounting department and are shown as follows.

Dessert Rolls
Ingredients per unit $ 0.40 $ 0.25
Direct labor per unit 0.35 0.30
Variable overhead per unit 0.40 0.20
Production tooling* 48,000 25,000
Advertising 30,000 20,000
*Gleason treats production tooling as a current operating expense rather than
capitalizing it as a fixed asset.
The cost incurred by Gleason for the market study is a(n)
Hock P2 2020
Section C - Decision Analysis.
Answers
 A. Sunk cost.correct
 B. Opportunity cost.
 C. Prime cost.
 D. Incremental cost.
Question was not answered
Correct Answer Explanation:
Since the market study cost has been already incurred, it is a sunk cost and will not
have any relevance to future management decisions.
Explanation for Choice B:
An opportunity cost is the cost of not selecting the best alternative. This does not relate
to the cost of the market study.
Explanation for Choice C:
Prime costs include direct labor and direct material. The market study is not a cost for
either direct labor or direct material.
Explanation for Choice D:
Incremental costs are additional cost that result when production or some other factor is
increased. The market study is not an incremental cost.
115. Question ID: CMA 1292 4.1 (Topic: Relevant Information for Decision-Making,
Economic Costs)
The opportunity cost of making a component part in a factory with no excess capacity is
the

 A. Cost of the production given up in order to manufacture the component.


 B. Variable manufacturing cost of the component.
 C. Net benefit given up from the best alternative use of the capacity.correct
 D. Fixed manufacturing cost of the component.
Question was not answered
Correct Answer Explanation:
Opportunity cost is the benefit to net income that is lost by not using a limited resource
for its bet alternative use. When there is no excess capacity in a factory, the net benefit
forgone from making a component is the contribution margin given up by not being able
to make something else instear, or the infome that could be earned from using the
factory for an alternative use such as renting it to another manufacturer.
If there is an alternative use of the capacity then opportunity cost would be the
contribution lost because of not selecting that alternative use of the capacity.
Hock P2 2020
Section C - Decision Analysis.
Answers
Explanation for Choice A:
Opportunity cost is a forgone benefit, not a cash outflow, so it is not a cost of
production. Opportunity cost is the lost contribution from not using a limited resource for
its best alternative.
Explanation for Choice B:
Opportunity cost is a forgone benefit, not a cash outflow, so it is not variable
manufacturing cost. Opportunity cost is the lost contribution from not using a limited
resource for its best alternative.
Explanation for Choice D:
Opportunity cost is a forgone benefit, not a cash outflow, so it is not fixed manufacturing
cost. Opportunity cost is the lost contribution from not using a limited resource for its
best alternative.
116. Question ID: ICMA 10.P2.219 (Topic: Relevant Information for Decision-
Making, Economic Costs)
In a management decision process, the cost measurement of the benefits sacrificed due
to selecting an alternative use of resources is most often referred to as a(n)

 A. differential cost.
 B. opportunity cost.correct
 C. sunk cost.
 D. relevant cost.
Question was not answered
Correct Answer Explanation:
An opportunity cost is the contribution to income that is lost by not using a limited
resource in its best alternative use. Benefits sacrificed due to selecting an alternative
use of resources are most often referred to as opportunity costs.
Explanation for Choice A:
Differential costs are costs that differ between or among alternatives. Benefits sacrificed
due to selecting an alternative use of resources can function as differential costs in
decision-making, if they differ between or among the alternatives being considered.
However, the cost measurement of the benefits sacrificed due to selecting an
alternative use of resources is not referred to most often as a differential cost.
Explanation for Choice C:
Sunk costs are costs that have been incurred and cannot be changed. The cost
measurement of the benefits sacrificed due to selecting an alternative use of resources
is never referred to as a sunk cost.
Hock P2 2020
Section C - Decision Analysis.
Answers
Explanation for Choice D:
Relevant costs are expected future costs that differ between or among alternatives.
Only relevant costs should be considered in the decision-making process. Benefits
sacrificed due to selecting an alternative use of resources can function as relevant costs
in decision-making, if they differ between or among the alternatives being considered.
However, the cost measurement of the benefits sacrificed due to selecting an
alternative use of resources is not referred to most often as a relevant cost.
117. Question ID: CMA 690 5.23 (Topic: Relevant Information for Decision-Making,
Economic Costs)
When a decision is made in an organization, it is selected from a group of alternative
courses of action. The loss associated with choosing the alternative that does not
maximize the benefit is the

 A. Incremental cost.
 B. Expected value.
 C. Opportunity cost.correct
 D. Net realizable value.
Question was not answered
Correct Answer Explanation:
By definition opportunity cost is the benefit to income (inflow) that is lost by not using a
limited resource for its best alternative use.
Explanation for Choice A:
Incremental costs are additional costs that result when production, or some other factor
is increased.
Explanation for Choice B:
The expected value of a product is the weighted average of potential outcomes. It is
calculated by taking the expected volume of sales and multiplying by the estimated
probabilities for each potential outcome and adding together.
Explanation for Choice D:
Net realizable value is value of an asset less any disposal cost.
118. Question ID: ICMA 10.P2.224 (Topic: Relevant Information for Decision-
Making, Economic Costs)
In differential cost analysis, which one of the following best fits the description of a sunk
cost?

 A. Cost of the forklift driver to move the material to the manufacturing floor.
 B. Cost of a large crane used to move materials.correct
Hock P2 2020
Section C - Decision Analysis.
Answers
 C. Direct materials required in the manufacture of a table.
 D. Purchasing department costs incurred in acquiring material.
Question was not answered
Correct Answer Explanation:
This is an historical cost that has already been incurred and cannot be changed in the
future. That is the description of a sunk cost.
Explanation for Choice A:
This is a current cost that will continue in the future unless it is changed. Sunk costs are
past costs that have already been incurred and cannot be changed.
Explanation for Choice C:
This is a current variable cost that will continue in the future unless it is changed. Sunk
costs are past costs that have already been incurred and cannot be changed.
Explanation for Choice D:
This is a current cost that will be continue in the future unless it is changed. Sunk costs
are past costs that have already been incurred and cannot be changed.
119. Question ID: CMA 689 1.24 (Topic: Relevant Information for Decision-Making,
Economic Costs)
When making a decision to increase the robotic automation equipment in an existing
facility, a firm takes all of the following into consideration except

 A. Technological efficiency.
 B. Economies of scale.
 C. Opportunity cost.
 D. The initial cost of the current facility.correct
Question was not answered
Correct Answer Explanation:
The initial cost of the current facility will play no part in the decision to increase robotic
automation equipment in an existing facility because the initial cost of the current facility
is a sunk cost. That is, it is a cost that has already been paid and can no longer be
avoided. As a result, sunk costs are not used as a consideration in future endeavors of
the company.
Explanation for Choice A:
Technological efficiency, otherwise known as productive efficiency, states that the
allocation of resources is efficient so long as there can be no further reallocation of
these resources that would permit more of one good to be produced without reducing
Hock P2 2020
Section C - Decision Analysis.
Answers
the output of some other good. Thus, allocation of inputs would be inefficient if there
were another, better, way to reallocate the inputs to get more of one good and no less
of the other good. Thus, this decision of increasing robotic automation equipment is
onedealing with inputs and therefore these inputs need to be technologically efficient.
Explanation for Choice B:
By definition, economies of scale refers to the situation where long run average costs
decline as the firm expands its use for all its inputs. Therefore, the company should take
into consideration the economies of scale of increasing robotic automation equipment in
an existing facility.
Explanation for Choice C:
An opportunity cost is the measure of benefits lost due to using resources for a given
purpose. All decisions look at opportunity costs to measure the best use of resources.
120. Question ID: CMA 690 5.24 (Topic: Relevant Information for Decision-Making,
Economic Costs)
Relevant costs refer to

 A. Past costs that are expected to be different in the future.


 B. Costs that would be incurred within the relevant range of production.
 C. Anticipated future costs that will differ among various alternatives.correct
 D. All fixed costs.
Question was not answered
Correct Answer Explanation:
Relevant costs are expected future costs that differ among alternatives. For example, if
there are two options available to a manager, the manager would consider in making
the decision only those costs that differ between the two options. Any costs that would
be the same under both options would not be considered because whichever choice is
made will not make a difference in those costs. They will be the same no matter which
option is chosen, hence they are not relevant to the decision.
Explanation for Choice A:
Past costs are sunk costs and are not relevant to the decision at hand because they
have already been incurred and nothing can be done to change them.
Explanation for Choice B:
Costs incurred within the relevant range of production may or may not be relevant in a
decision-making process. Relevant costs are expected future costs that differ among
alternatives. Thus some costs that are incurred within the relevant range of production
may be relevant to a decision if they would be different depending on what decision was
made. On the other hand, other costs that are incurred within the relevant range of
Hock P2 2020
Section C - Decision Analysis.
Answers
production would not be relevant, if they would be the same no matter which decision
was made.
Explanation for Choice D:
In decision-making, relevant costs refer to those costs that vary among the options.
Some fixed costs may vary among the alternatives and would be relevant, but not all
fixed costs are relevant to a particular decision.
121. Question ID: CMA 694 4.20 (Topic: Relevant Information for Decision-Making,
Economic Costs)
Management accountants are frequently asked to analyze various decision situations,
including the following:
I. The cost of a special device that is necessary if a special order is accepted.
II. The cost proposed annually for the plant service for the grounds at corporate
headquarters.
III. Joint production costs incurred, to be considered in a sell-at-splitoff versus a
process-further decision.
IV. The costs associated with alternative uses of plant space, to be considered in a
make/buy decision.
V. The cost of obsolete inventory acquired several years ago, to be considered in a
keep versus disposal decision.
The costs described in situations III and V are:

 A. Prime costs.
 B. Sunk costs.correct
 C. Discretionary costs.
 D. Imputed costs.
Question was not answered
Correct Answer Explanation:
Situation III and V are describing sunk costs. Sunk costs are costs that have already
been incurred and thus are not relevant to current decisions.
Explanation for Choice A:
Prime cost include only Direct Material and Direct Labor.
Explanation for Choice C:
Discretionary costs are costs that can be deferred to future periods without creating a
significant impact on the current period's results.
Explanation for Choice D:
Hock P2 2020
Section C - Decision Analysis.
Answers
An imputed cost is an opportunity cost. It is a benefit that is given up as a result of using
the company's resources elsewhere. It is a cost that is not stated and must be
calculated in some way. A common example of an imputed cost is the interest cost of
equity capital.
122. Question ID: ICMA 10.P2.247.01 247 (Topic: Relevant Information for
Decision-Making, Economic Costs)
Verla Industries is trying to decide which one of the following two options to pursue.
Either option will take effect on January 1st of the next year.
Option One - Acquire a New Finishing Machine.
The cost of the machine is $1,000,000 and will have a useful life of five years. Net pre-
tax cash flows arising from savings in labor costs will amount to $100,000 per year for
five years. Depreciation expense will be calculated using the straight-line method for
both financial and tax reporting purposes. As an incentive to purchase, Verla will receive
a trade-in allowance of $50,000 on their current fully depreciated finishing machine.
Option Two - Outsource the Finishing Work.
Verla can outsource the work to LM Inc. at a cost of $200,000 per year for five years. If
they outsource, Verla will scrap their current fully depreciated finishing machine.
Verla's effective income tax rate is 40%. The weighted-average cost of capital is 10%.
When comparing the two options, the $50,000 trade-in allowance would be considered

 A. relevant because it is a decrease in cash outflow.correct


 B. irrelevant because it does not affect cash.
 C. irrelevant because it does not affect taxes.
 D. relevant because it is an increase in cash outflows.
Question was not answered
Correct Answer Explanation:
Relevant cash flows are those expected future cash flows that differ between
alternatives. The trade-in allowance will reduce the cash outflow under Option One,
since it offsets the purchase price. However, it will not be available under Option Two.
Therefore, it differs between alternatives and is a relevant cash flow.
Explanation for Choice B:
The trade-in allowance is relevant because it differs between alternatives and it does
have an impact on the cash flows. The trade-in allowance will reduce the cash outflow
under Option One, since it offsets the purchase price. However, it will not be available
under Option Two.
Explanation for Choice C:
Hock P2 2020
Section C - Decision Analysis.
Answers
Relevant cash flows are expected future cash flows that differ between alternatives.
Since the trade-in allowance applies to only one of the options, it differs between
alternatives and is a relevant cash flow.
Explanation for Choice D:
The trade-in allowance is relevant because it differs between alternatives and it does
have an impact on the cash flows. However, the trade-in allowance will reduce the cash
outflow under Option One, not increase it, because it offsets the purchase price.
123. Question ID: ICMA 10.P2.252 (Topic: Relevant Information for Decision-
Making, Economic Costs)
Capital Company has decided to discontinue a product produced on a machine
purchased four years ago at a cost of $70,000. The machine has a current book value
of $30,000. Due to technologically improved machinery now available in the
marketplace, the existing machine has no current salvage value. The company is
reviewing the various aspects involved in the production of a new product. The
engineering staff advised that the existing machine can be used to produce the new
product. Other costs involved in the production of the new product will be materials of
$20,000 and labor priced at $5,000.
Ignoring income taxes, the costs relevant to the decision to produce or not to produce
the new product would be

 A. $55,000.
 B. $25,000.correct
 C. $30,000.
 D. $95,000.
Question was not answered
Correct Answer Explanation:
Relevant costs are expected future costs that differ among alternatives. The costs
involved in the production of the new product — materials of $20,000 and labor at
$5,000 — are the costs that are relevant to the decision to produce or not produce the
new product. The purchase of the machine occurred in the past, and that cost as well as
the current book value of the machine is not relevant to this decision.
Explanation for Choice A:
This includes the variable costs of materials and labor as well as the book value of the
machine. The $30,000 book value of the machine is not relevant to this decision. It is a
sunk cost, i.e., an historical cost that cannot be changed. Therefore, it would not differ
between the options and so it is not a relevant cost.
Explanation for Choice C:
Hock P2 2020
Section C - Decision Analysis.
Answers
$30,000 is the book value of the existing machine. The book value of the machine is not
relevant to the decision, as the cost of the machine was incurred in the past and cannot
be changed. It is a sunk cost. The historical costs will be there whether the new product
is produced or not.
Explanation for Choice D:
This includes the variable costs of materials and labor as well as the original cost of the
machine. The $70,000 original cost of the machine is not relevant to this decision. The
machine was purchased four years ago and will be part of the company whether the
new product is produced or not. It is a sunk cost that cannot be changed.
124. Question ID: ICMA 19.P2.005 (Topic: Relevant Information for Decision-
Making, Economic Costs)
Sunk costs are not relevant in the decision-making process because they are

 A. historical costs.correct
 B. indirect costs.
 C. period costs.
 D. fixed costs.
Question was not answered
Correct Answer Explanation:
Sunk costs are costs that have already been incurred and, no matter what decision is
made now, are not able to be recovered. Because of this, sunk costs are irrelevant in
the decision-making process.
Explanation for Choice B:
Indirect costs may be relevant for the decision-making process.
Explanation for Choice C:
Period costs may be relevant for the decision-making process.
Explanation for Choice D:
Fixed costs may be relevant for the decision-making process.
125. Question ID: CMA 1288 5.12 (Topic: Relevant Information for Decision-
Making, Economic Costs)
Management accountants are frequently asked to analyze various decision situations
including the following.

1. Alternative uses of plant space, to be considered in a make/buy decision.


2. Joint production costs incurred, to be considered in a sell-at-split versus a process-
further decision.
Hock P2 2020
Section C - Decision Analysis.
Answers
3. Research and development costs incurred in prior months, to be considered in a
product-introduction decision.
4. The cost of a special device that is necessary if a special order is accepted.
5. The cost of obsolete inventory acquired several years ago, to be considered in a
keep-versus-disposal decision.
The costs described in situations 1 and 4 are

 A. Prime costs.
 B. Sunk costs.
 C. Discretionary costs.
 D. Relevant costs.correct
Question was not answered
Correct Answer Explanation:
The costs that are best described in situations 1 and 4 are relevant costs. These types
of costs would be part of the decision to make or buy, or whether to accept the special
order.
Explanation for Choice A:
Prime costs include only direct materials and direct labor.
Explanation for Choice B:
Sunk costs are those costs that have already been incurred and will not be relevant to
management's future decisions. The costs described in situations 1 and 4 are not sunk
costs.
Explanation for Choice C:
Discretionary costs are costs that can be deferred to future periods without creating a
significant impact in the current period. The costs described in situations 1 and 4 are not
discretionary costs.
126. Question ID: ICMA 10.P2.255 (Topic: Relevant Information for Decision-
Making, Economic Costs)
Grapevine Corporation produces two joint products, JP-1 and JP-2, and a single by-
product, BP-1, in Department 2 of its manufacturing plant. JP-1 is subsequently
transferred to Department 3 where it is refined into a more expensive, higher-priced
product, JP-1R, and a by-product known as BP-2. Recently, Santa Fe Company
introduced a product that would compete directly with JP-1R and, as a result, Grapevine
must reevaluate its decision to process JP-1 further. The market for JP-1 will not be
affected by Santa Fe's product, and Grapevine plans to continue production of JP-1,
even if further processing is terminated. Should this latter action be necessary,
Department 3 will be dismantled.
Hock P2 2020
Section C - Decision Analysis.
Answers
Which of the following items should Grapevine consider in its decision to continue or
terminate Department 3 operations?

1. The selling price per pound of JP-1.


2. The total hourly direct labor cost in Department 3.
3. Unit marketing and packaging costs for BP-2.
4. Supervisory salaries of Department 3 personnel who will be transferred elsewhere
in the plant, if processing is terminated.
5. Department 2 joint cost allocated to JP-1 and transferred to Department 3.
6. The cost of existing JP-1R inventory.

 A. 1, 2, 3, 4, 5.
 B. 2, 3, 5, 6.
 C. 1, 2, 3.correct
 D. 2, 3, 4.
Question was not answered
Correct Answer Explanation:

1. The selling price per pound of JP-1 is relevant to the decision, because this is the
revenue that the company would be receiving if they discontinue processing JP-1
further.
2. The total hourly direct labor cost in Department 3 is relevant to the decision,
because these are costs that the company will no longer have if they discontinue
processing JP-1 further.
3. Unit marketing and packaging costs for BP-2, the byproduct, are relevant to the
decision, because these costs will go away if they discontinue processing JP-1
further.
4. Supervisory salaries of Department 3 personnel who will be transferred elsewhere
in the plant, if processing is terminated are not relevant to the decision, because
they will not change regardless of what decision is made.
5. Department 2 joint cost allocated to JP-1 and transferred to Department 3 is not
relevant to the decision, because it will not change regardless of what decision is
made.
6. The cost of existing JP-1R inventory is not relevant to the decision, because that is
a sunk cost that has already been incurred, and it will not be changed by the
decision.
Explanation for Choice A:
Hock P2 2020
Section C - Decision Analysis.
Answers
1. The selling price per pound of JP-1 is relevant to the decision, because this is the
revenue that the company would be receiving if they discontinue processing JP-1
further.
2. The total hourly direct labor cost in Department 3 is relevant to the decision,
because these are costs that the company will no longer have if they discontinue
processing JP-1 further.
3. Unit marketing and packaging costs for BP-2, the byproduct, are relevant to the
decision, because these costs will go away if they discontinue processing JP-1
further.
4. Supervisory salaries of Department 3 personnel who will be transferred elsewhere
in the plant, if processing is terminated are not relevant to the decision, because
they will not change regardless of what decision is made.
5. Department 2 joint cost allocated to JP-1 and transferred to Department 3 is not
relevant to the decision, because it will not change regardless of what decision is
made.
6. The cost of existing JP-1R inventory is not relevant to the decision, because that is
a sunk cost that has already been incurred, and it will not be changed by the
decision.
Explanation for Choice B:

1. The selling price per pound of JP-1 is relevant to the decision, because this is the
revenue that the company would be receiving if they discontinue processing JP-1
further.
2. The total hourly direct labor cost in Department 3 is relevant to the decision,
because these are costs that the company will no longer have if they discontinue
processing JP-1 further.
3. Unit marketing and packaging costs for BP-2, the byproduct, are relevant to the
decision, because these costs will go away if they discontinue processing JP-1
further.
4. Supervisory salaries of Department 3 personnel who will be transferred elsewhere
in the plant, if processing is terminated are not relevant to the decision, because
they will not change regardless of what decision is made.
5. Department 2 joint cost allocated to JP-1 and transferred to Department 3 is not
relevant to the decision, because it will not change regardless of what decision is
made.
6. The cost of existing JP-1R inventory is not relevant to the decision, because that is
a sunk cost that has already been incurred, and it will not be changed by the
decision.
Explanation for Choice D:
Hock P2 2020
Section C - Decision Analysis.
Answers
1. The selling price per pound of JP-1 is relevant to the decision, because this is the
revenue that the company would be receiving if they discontinue processing JP-1
further.
2. The total hourly direct labor cost in Department 3 is relevant to the decision,
because these are costs that the company will no longer have if they discontinue
processing JP-1 further.
3. Unit marketing and packaging costs for BP-2, the byproduct, are relevant to the
decision, because these costs will go away if they discontinue processing JP-1
further.
4. Supervisory salaries of Department 3 personnel who will be transferred elsewhere
in the plant, if processing is terminated are not relevant to the decision, because
they will not change regardless of what decision is made.
5. Department 2 joint cost allocated to JP-1 and transferred to Department 3 is not
relevant to the decision, because it will not change regardless of what decision is
made.
6. The cost of existing JP-1R inventory is not relevant to the decision, because that is
a sunk cost that has already been incurred, and it will not be changed by the
decision.
127. Question ID: CMA 1285 5.30 (Topic: Relevant Information for Decision-
Making, Economic Costs)
The opportunity cost of making a component part in a factory with excess capacity for
which there is no alternative use is

 A. The fixed manufacturing cost of the component.


 B. The total manufacturing cost of the component.
 C. Zero.correct
 D. The total variable cost of the component.
Question was not answered
Correct Answer Explanation:
The key term is "no alternative use." Opportunity cost is the benefit to income that is lost
by not using a resource for its best alternative use. In this question there is no
alternative use for the plant capacity, so the opportunity cost of using the idle capacity to
manufacture a component is zero.
Explanation for Choice A:
Opportunity cost has to do with lost benefit (inflow). Fixed manufacturing cost is an
outflow.
Explanation for Choice B:
Opportunity cost has to do with lost benefit (inflow). Manufacturing cost is an outflow.
Hock P2 2020
Section C - Decision Analysis.
Answers
Explanation for Choice D:
Opportunity cost has to do with lost benefit (inflow). Total variable cost is an outflow.
128. Question ID: CIA 1193 IV.15 (Topic: Relevant Information for Decision-
Making, Economic Costs)
A printing company is considering replacing an old printing press. The old printing press
has a book value of $24,000 and a trade-in value of $14,000. A new printing press
would cost $85,000 after trade-in of the old press. It is estimated that the new printing
press would reduce operating costs by $20,000 per year. If the company decides not to
purchase the new press, the $85,000 could instead be used to retire debt that is
currently costing $9,000 per year in interest. Which of the given amounts is an example
of a sunk cost?

 A. The interest on the existing debt.


 B. The trade-in value of the old printing press.
 C. The estimated reduction in operating costs.
 D. The book value of the old printing press.correct
Question was not answered
Correct Answer Explanation:
A sunk cost is a cost that has already been incurred and is not relevant to the decision
process. The amount paid for the old printing press (less accumulated depreciation) is
an example of a sunk cost.
Explanation for Choice A:
The interest on debt is an avoidable cost that is relevant to the decision process.
Explanation for Choice B:
A sunk cost is a cost that has already been incurred and is not relevant to the decision
process. The trade-in value of the old printing press will affect the decision process, so
therefore, is relevant and is not a sunk cost.
Explanation for Choice C:
A sunk cost is a cost that has already been incurred and is not relevant to the decision
process. The estimated reduction in operating expenses is the difference between
keeping the old printing press and purchasing the new one. It is relevant to the decision
process and is not even a cost.
129. Question ID: CMA 1295 4.21 (Topic: Relevant Information for Decision-
Making, Economic Costs)
A decision-making concept, described as "the contribution to income that is forgone by
not using a limited resource for its best alternative use," is called
Hock P2 2020
Section C - Decision Analysis.
Answers
 A. Potential cost.
 B. Opportunity cost.correct
 C. Incremental cost.
 D. Marginal cost.
Question was not answered
Correct Answer Explanation:
Opportunity cost is the contribution to income that is forgone by not using a limited
resource for its best alternative use.
Explanation for Choice A:
Potential cost is a cost that may be incurred at a future date.
Explanation for Choice C:
Incremental cost is the cost difference between two alternative options.
Explanation for Choice D:
Marginal cost is the cost of producing one more unit.
130. Question ID: CMA 1290 4.3 (Topic: Relevant Information for Decision-Making,
Economic Costs)
Relevant or differential cost analysis

 A. Considers all variable and fixed costs as they change with each decision
alternative.correct
 B. Allows the decision maker to group all types of costs together to facilitate decision
making.
 C. Takes all variable and fixed costs into account to analyze decision alternatives.
 D. Considers only variable costs as they change with each decision alternative.
Question was not answered
Correct Answer Explanation:
Relevant and differential cost analysis looks at those costs that vary with the selected
options.
Explanation for Choice B:
Those costs that would not vary among the selected options would be ignored.
Explanation for Choice C:
Not all costs would be considered in relevant or differential cost analysis.
Explanation for Choice D:
Hock P2 2020
Section C - Decision Analysis.
Answers
It is possible that some fixed costs would be included in the analysis if they differ
between/among the options.
131. Question ID: CMA 1290 4.12 (Topic: Relevant Information for Decision-
Making, Economic Costs)
Opportunity costs are

 A. The same as variable costs.


 B. Relevant to decision making.correct
 C. Equal to historical costs.
 D. Not used for decision making.
Question was not answered
Correct Answer Explanation:
Opportunity cost is relevant to decision making. Opportunity cost is the benefit to
income (inflow) that is lost by not using a limited resource for its best alternative use.
Explanation for Choice A:
Opportunity cost is the benefit to income (inflow) that is lost by not using a limited
resource for its best alternative use. Variable costs are those costs that vary with the
level of production (outflow).
Explanation for Choice C:
Historical costs are those cost that have been incurred in the past.
Explanation for Choice D:
Opportunity cost are relevant to the decision making process.
132. Question ID: CMA 1288 5.13 (Topic: Relevant Information for Decision-
Making, Economic Costs)
Management accountants are frequently asked to analyze various decision situations
including the following.

1. Alternative uses of plant space, to be considered in a make/buy decision.


2. Joint production costs incurred, to be considered in a sell-at-split versus a process-
further decision.
3. Research and development costs incurred in prior months, to be considered in a
product-introduction decision.
4. The cost of a special device that is necessary if a special order is accepted.
5. The cost of obsolete inventory acquired several years ago, to be considered in a
keep-versus-disposal decision.
The costs described in situations 2, 3, and 5 are
Hock P2 2020
Section C - Decision Analysis.
Answers
 A. Relevant costs.
 B. Sunk costs.correct
 C. Discretionary costs.
 D. Differential costs.
Question was not answered
Correct Answer Explanation:
The costs described in 2, 3 and 5, are sunk costs. Sunk costs are those costs that have
already been incurred and are not relevant to management's future decisions. Joint
costs are not relevant as are R&D, or obsolete inventory.
Explanation for Choice A:
Relevant costs are expected future costs that are differ among alternatives. Relevant
costs are the costs that will be different as a result of making one decision versus
another decision. The costs described in situations 2, 3, and 5 are costs that will be the
same no matter what is done in the future, because they have already been incurred
and cannot be changed by any current decision about the future. Therefore, they are
not relevant to any current decision.
Explanation for Choice C:
Discretionary costs are costs that can be deferred to future periods without creating a
significant impact in the current period. The costs described in situations 2, 3, and 5 are
costs that will be the same no matter what is done in the future, because they have
already been incurred and cannot be changed by any current decision about the future.
They are not discretionary costs.
Explanation for Choice D:
Differential (incremental) costs are costs that differ between two alternatives that may
be chosen in the future. The costs described in situations 2, 3, and 5 are costs that will
be the same no matter what is done in the future, because they have already been
incurred and cannot be changed by any current decision about the future.
133. Question ID: CMA 691 4.7 (Topic: Relevant Information for Decision-Making,
Economic Costs)
Total unit costs are

 A. Relevant for cost-volume-profit analysis.


 B. Needed for determining sunk costs.
 C. Irrelevant in marginal analysis.correct
 D. Independent of the cost system used to generate them.
Question was not answered
Correct Answer Explanation:
Hock P2 2020
Section C - Decision Analysis.
Answers
Marginal analysis (incremental or differential) looks only at those costs which differ
among the possible choices. Total unit costs include all costs (fixed and variable), some
of which may be irrelevant to the decision process.
Explanation for Choice A:
Total unit costs include both fixed and variable costs on a per unit basis. CVP analysis
separates out these costs.
Explanation for Choice B:
Total unit costs are not needed for determining sunk costs. Sunk costs are costs that
have been incurred and no future decision can change them, so sunk costs are
irrelevant to the decision process.
Explanation for Choice D:
Total unit cost is not independent of the cost system that was used to generate them.
134. Question ID: CMA 1296 4.1 (Topic: Relevant Information for Decision-Making,
Economic Costs)
The term that best refers to past costs that have been incurred and are not relevant to
any future decisions is

 A. incurred marginal costs.


 B. sunk costs.correct
 C. discretionary costs.
 D. full absorption costs.
Question was not answered
Correct Answer Explanation:
Sunk costs are past costs that have been incurred and are not relevant to any future
decisions.
Explanation for Choice A:
Marginal costs are the addition to total cost that results from a one-unit change in
production or the addition to total cost that would result from a project that is under
consideration. Marginal costs are relevant to future decisions.
Explanation for Choice C:
Discretionary costs are costs that may be deferred to a future period without creating a
significant impact on operations in the current period. Discretionary costs may or may
not be relevant to future decisions.
Explanation for Choice D:
Hock P2 2020
Section C - Decision Analysis.
Answers
Under absorption costing, fixed factory overhead costs are allocated to the units
produced during the period according to some predetermined ratio and are therefore a
product cost. Absorption costs may or may not be relevant to future decisions.
135. Question ID: CMA 694 4.21 (Topic: Relevant Information for Decision-Making,
Economic Costs)
Management accountants are frequently asked to analyze various decision situations,
including the following:
I. The cost of a special device that is necessary if a special order is accepted.
II. The cost proposed annually for the plant service for the grounds at corporate
headquarters.
III. Joint production costs incurred, to be considered in a sell-at-split versus a process-
further decision.
IV. The costs associated with alternative uses of plant space, to be considered in a
make/buy decision.
V. The cost of obsolete inventory acquired several years ago, to be considered in a
keep-versus-disposal decision.
The cost described in situation II is a

 A. prime cost.
 B. imputed cost.
 C. relevant cost.
 D. discretionary cost.correct
Question was not answered
Correct Answer Explanation:
Discretionary costs are costs that can be deferred to future periods without creating a
significant impact in the current period. They have no relationship to input and output.
This is best described in situation II since the annual cost of service for the grounds at
corporate headquarters would have no effect on plant production or sales volume.
Explanation for Choice A:
Prime costs include direct labor, and direct materials. This is not described in situation
II.
Explanation for Choice B:
An imputed cost is an opportunity cost. It is a benefit that is given up as a result of using
the company's resources elsewhere. It is a cost that is not stated and must be
calculated in some way. Imputed costs are those costs that are not paid in cash, but are
implied. The best example of an imputed cost is the interest charge on equity capital.
Explanation for Choice C:
Hock P2 2020
Section C - Decision Analysis.
Answers
Relevant costs are those costs that will differ among the decision options. Situation II
would not be considered a relevant cost.
136. Question ID: CMA 690 5.26 (Topic: Relevant Information for Decision-Making,
Economic Costs)
Which one of the following costs would be relevant in short-term decision making?

 A. Total variable costs that are the same in the considered alternatives.
 B. Opportunity costs that are the same in the considered alternatives.
 C. All costs of inventory.
 D. Incremental fixed costs.correct
Question was not answered
Correct Answer Explanation:
Relevant costs are expected future costs that differ among alternatives. Incremental
costs are additional costs that result when production or some other factor is increased.
Incremental costs can be either fixed or variable, so in this case incremental fixed costs
would be relevant in the short-term.
Explanation for Choice A:
Relevant costs are expected future costs that differ among alternatives. If variable
costs are the same in the considered alternatives then these costs are not relevant.
Explanation for Choice B:
Relevant costs are expected future costs that differ among alternatives. If opportunity
costs are the same in the considered alternatives then they would not be relevant.
Explanation for Choice C:
Relevant costs are expected future costs that differ among alternatives. All costs of
inventory are not relevant costs, though in a decision among suppliers for purchase of a
specific item, the costs charged by each supplier would be relevant if they were
different.
137. Question ID: CMA 691 4.12 (Topic: Relevant Information for Decision-Making,
Economic Costs)
The term relevant cost applies to all the following decision situations except the

 A. Determination of a product price.correct


 B. Acceptance of a special order.
 C. Manufacture or purchase of component parts.
 D. Replacement of equipment.
Question was not answered
Hock P2 2020
Section C - Decision Analysis.
Answers
Correct Answer Explanation:
Relevant costs are incremental or differential costs that vary among the possible
choices. The costs will be the same regardless of what product price is chosen.
Explanation for Choice B:
Relevant costs are important to the decision whether to accept a special order or not.
Explanation for Choice C:
Relevant costs are incremental or differential costs that vary among the possible
choices. Relevant costs will be important to the decision whether to manufacture or
purchase component parts because the relevant costs will be different for each of the
options.
Explanation for Choice D:
Relevant costs are incremental or differential costs that vary among the possible
choices. Relevant costs will be important in the decision of whether or not to replace
equipment because the relevant costs will be different for each of the options.
138. Question ID: CMA 696 4.19 (Topic: Relevant Information for Decision-Making,
Economic Costs)
When an organization decides on a course of action that is selected from a group of
alternative courses of action, the benefit lost by not choosing the best alternative course
of action is the

 A. net realizable value.


 B. expected value.
 C. opportunity cost.correct
 D. incremental cost.
Question was not answered
Correct Answer Explanation:
By definition, opportunity cost is the benefit to income (inflow) that is lost by not using a
limited resource for its best alternative use.
Explanation for Choice A:
Net realizable value is the value of an asset less any disposal cost.
Explanation for Choice B:
The expected value of a product is the weighted average of the potential outcomes. It is
calculated by taking the expected volume of sales and multiplying by the estimated
probabilities for each potential outcome and adding together.
Explanation for Choice D:
Hock P2 2020
Section C - Decision Analysis.
Answers
Incremental costs are additional costs that are incurred when production, or some other
factor is increased.
139. Question ID: ICMA 10.P2.259 (Topic: Using Economics Concepts in
Production Decisions)
Elgers' Company produces valves for the plumbing industry. Elgers' per unit sales price
and variable costs are as follows.

Sales price $12


Variable costs 8
Elgers' practical plant capacity is 40,000 units. Elgers' total fixed costs aggregate
$48,000 and it has a 40% effective tax rate.
The maximum net profit that Elgers can earn is

 A. $48,000.
 B. $112,000.
 C. $96,000.
 D. $67,200.correct
Question was not answered
Correct Answer Explanation:
Elgers' unit contribution margin is $4 ($12 − $8). The plant capacity is 40,000 units, so
the company's maximum total contribution margin is 40,000 × $4, or $160,000. Fixed
costs are $48,000, so the maximum net income before tax is $112,000. Taxes are 40%,
or $44,800, and net profit is $112,000 − $44,800, which is $67,200. This is the
maximum net profit Elgers can earn.
Explanation for Choice A:
This is the amount of Elgers' fixed costs.
Explanation for Choice B:
This is the net income before income tax.
Explanation for Choice C:
This is the total contribution margin reduced by 40% for taxes. It does not include any
deduction for fixed costs.
140. Question ID: CMA 1277 5.1 (Topic: Using Economics Concepts in Production
Decisions)
The term incremental cost refers to

 A. A cost that continues to be incurred in the absence of activity.


Hock P2 2020
Section C - Decision Analysis.
Answers
 B. A cost common to all choices in question and not clearly or feasibly allocable to any of
them.
 C. The difference in total costs that results from selecting one choice instead of
another.correct
 D. The profit forgone by selecting one choice instead of another.
Question was not answered
Correct Answer Explanation:
Incremental costs are additional costs that result when one choice is made instead of
another, for example increasing production or some other factor.
Explanation for Choice A:
This is the definition of fixed cost.
Explanation for Choice B:
A cost that is common to all choices in question is an irrelevant cost, because it will not
be any different regardless of what choice is made.
Explanation for Choice D:
This is the definition of opportunity cost.
141. Question ID: CMA 1295 1.19 (Topic: Using Economics Concepts in
Production Decisions)

Number of Workers Total Product Units Average Selling Price


10 20 $50.00
11 25 49.00
12 28 47.50
The marginal revenue product when one worker is added to a team of 11 workers is

 A. $225.00
 B. $105.00correct
 C. $47.50
 D. $42.00
Question was not answered
Correct Answer Explanation:
Marginal revenue product is the amount of change in total revenue that arises from
adding one more unit of a resource, such as labor.
Hock P2 2020
Section C - Decision Analysis.
Answers
In order to find the marginal revenue product when one worker is added to a team of 11
workers, look at the last two rows of the table. Notice that the Price is in Average terms.
Therefore, you must multiply that column by the units produced to find total revenue at
each worker level. Then, subtract the revenue from using a team of 11 from the revenue
from using a team of 12 members:

12 workers: $47.50 × 28 = $1,330


11 workers: $49.00 × 24 = (1,225)
Marginal revenue product $ 105

Explanation for Choice A:


$225 is the marginal revenue product when one worker is added to a team of 10.
Explanation for Choice C:
Marginal revenue product is the amount of change in total revenue that arises from
adding one more unit of a resource, such as labor. $47.50 is the average selling price
when 12 workers are hired.
Explanation for Choice D:
This is not the correct answer. Marginal revenue product is the amount of change in
total revenue that arises from adding one more unit of a resource, such as labor. Please
see the correct answer for a complete explanation.
We have been unable to determine how to calculate this incorrect answer choice. If you
have calculated it, please let us know how you did it so we can create a full explanation
of why this answer choice is incorrect. Please send us an email at
support@hockinternational.com. Include the full Question ID number and the actual
incorrect answer choice -- not its letter, because that can change with every study
session created. The Question ID number appears at the top of the question. Thank you
in advance for helping us to make your HOCK study materials better.
142. Question ID: ICMA 10.P2.228 (Topic: Using Economics Concepts in
Production Decisions)
Daily costs for Kelso Manufacturing include $1,000 of fixed costs and total variable
costs are shown below.

Unit Output 10 11 12 13 14 15
Cost $125 $250 $400 $525 $700 $825
Hock P2 2020
Section C - Decision Analysis.
Answers
The average total cost at an output level of 11 units is

 A. $250.00.
 B. $125.00.
 C. $215.91.
 D. $113.64.correct
Question was not answered
Correct Answer Explanation:
Total cost at an output level of 11 units is $1,000 fixed costs plus $250 variable costs,
for a total cost of $1,250. The average total cost per unit at that output level is $1,250
divided by 11, or $113.64.
Explanation for Choice A:
This is the total variable cost to produce 11 units. The question asks for the average
total cost per unit at the output level of 11 units. The average total cost at an output
level of 11 units is the sum of the total variable cost for 11 units and the total fixed cost
for 11 units divided by 11 units.
Explanation for Choice B:
$125.00 is the incremental variable cost of the 11th unit. The question asks for the
average total cost per unit at the output level of 11 units. The average total cost at an
output level of 11 units is the sum of the total variable cost for 11 units and the total
fixed cost for 11 units divided by 11 units.
Explanation for Choice C:
This is the incremental variable cost of the 11th unit plus the average fixed cost per unit
for 11 units. The average total cost at an output level of 11 units is the sum of
the total variable cost for 11 units and the total fixed cost for 11 units divided by 11
units.
143. Question ID: CIA 585 IV.8 (Topic: Using Economics Concepts in Production
Decisions)
Orange Company's controller developed the following contribution income statement
for year 1:

Per Unit
Sales (150,000 units at $30) $4,500,000 $30
Variable costs:
Direct materials $1,050,000 $ 7
Hock P2 2020
Section C - Decision Analysis.
Answers
Direct labor 1,500,000 10
Mfg. overhead 300,000 2
Selling & marketing 300,000 2
3,150,000 $21
Contribution margin $1,350,000 $ 9
Fixed costs:
Mfg. overhead $600,000 $ 4
Selling & marketing 300,000 2
$ 900,000 6
Net income $ 450,000 $ 3
Orange Co. based its next year's budget on the assumption that fixed costs, unit sales,
and the sales price would remain as they were in year 1, but with net income being
reduced to $300,000. By July of year 2, the controller was able to predict that unit sales
would increase over year 1 levels by 10%. Based on the year 2 budget and the new
information, the predicted year 2 net income would be:

 A. $330,000
 B. $585,000
 C. $420,000correct
 D. $300,000
Question was not answered
Correct Answer Explanation:
In the original Year 2 budget, net income will be reduced to $300,000 from $450,000 in
Year 1, a $150,000 decrease. The question says that fixed costs will not change for
Year 2, and in the original Year 2 budget, that sales revenue will not change. Therefore,
if net income is to decrease by $150,000, the decrease must come from an increase of
$150,000 in variable costs. Actual variable costs in Year 1 totaled $3,150,000. If they
increase by $150,000, they will increase to a total of $3,300,000. So the original Year 2
budget is as follows:

Sales (150,000 units @ $30) $4,500,000


Variable costs 3,300,000
Contribution margin $1,200,000
Fixed costs:
Hock P2 2020
Section C - Decision Analysis.
Answers
Manufacturing overhead 600,000
Selling & marketing exp. 300,000
Net income 300,000
Now, we adjust the revenue to the revised level of 110% of $4,500,000. However, when
we do that, we must also increase the variable costs, because this revenue increase will
come from increased unit sales, not from price increases. So we must also increase
variable costs to 110% of $3,300,000. Now, we have an income statement that looks
like this:

Sales ($4,500,000 × 1.10) $4,950,000


Variable costs ($3,300,000 × 1.10) 3,630,000
Contribution margin $1,320,000
Fixed costs:
Manufacturing overhead 600,000
Selling & marketing expense 300,000
Net income $ 420,000
Explanation for Choice A:
This answer is incorrect because it incorporates a 10% increase in fixed costs, whereas
the question says that fixed costs will remain the same as they were in year 1.
Explanation for Choice B:
This answer results from increasing the actual total variable costs in year 1 by 10% for
the revised year 2 budget to reflect the increased unit sales. However, the original year
2 budget incorporated an increase in variable costs per unit, and the revised year 2
budget should begin with the original year 2 budgeted variable costs and increase that
amount by 10% to reflect the increased unit sales instead of increasing the year 1 actual
variable costs by 10%.
Explanation for Choice D:
This is the budgeted net income in the original year 2 budget that uses year 1 revenue
as the budgeted revenue for year 2. The new information is that unit sales will increase
by 10% in year 2 over year 1 levels.
144. Question ID: HOCK DA 2 (Topic: Using Economics Concepts in Production
Decisions)
Stark Rehabilitation Hospital is an inpatient institution for severely injured patients who
need intense therapy to recover normal functioning. The hospital has a maximum
Hock P2 2020
Section C - Decision Analysis.
Answers
capacity of 130 patients. The hospital employs nurses, doctors and physical therapists.
Expenses are as follows:
Medical staff:

 1 nurse for every 20 patients, average cost $66,000 per year per nurse
 1 doctor for every 60 patients, average cost $150,000 per year per doctor
 1 physical therapist for every 10 patients, average cost $80,000 per year per
therapist
The cost per patient per day for meals, medicine and supplies averages $60 per day per
patient.
Utilities cost a base amount of $2,500 per month plus an average of $200 per patient
per month.
Medical and exercise equipment, occupancy expense and administrative expense total
$7,700,000 per year.
The hospital patient census (i.e., number of patients at any one time) varies from a
minimum of 100 patients to fully occupied at 130 patients.
Stark charges $300 per day per patient, and this includes the room, meals, medicine,
supplies and all services.
What items are variable costs for Stark?

 A. Utilities
 B. Medical and exercise equipment, occupancy expense and administative expense
 C. Medical staff
 D. Meals, medicine and suppliescorrect
Question was not answered
Correct Answer Explanation:
Variable costs change in total in proportion to changes in the level of activity. Meals,
medicine and supplies fit this description.
Explanation for Choice A:
Utilities are actually a semi-variable (mixed) cost because they cost a base amount of
$2,500 per month plus an average of $200 per patient per month.
Explanation for Choice B:
Variable costs change in total in proportion to changes in the level of activity. Medical
and exercise equipment, occupancy expense and administrative expenses do not
change in total in proportion to changes in the level of activity.
Hock P2 2020
Section C - Decision Analysis.
Answers
Explanation for Choice C:
Medical staff costs are actually semi-fixed (step) costs because they are fixed over a
given, small range of activity and above that level of activity, the cost suddenly jumps
(here because additional medical staff need to be hired).
145. Question ID: ICMA 10.P2.262 (Topic: Using Economics Concepts in
Production Decisions)
Phillips and Company produces educational software. Its current unit cost, based upon
an anticipated volume of 150,000 units, is as follows.

Selling price $150


Variable costs 60
Contribution margin 90
Fixed costs 60
Operating income 30
Sales for the coming year are estimated at 175,000 units, which is within the relevant
range of Phillip's cost structure. Cost management initiatives are expected to yield a
20% reduction in variable costs and a reduction of $750,000 in fixed costs. Phillip's cost
structure for the coming year will include a

 A. per unit contribution margin of $72 and fixed costs of $55.


 B. total contribution margin of $15,300,000 and fixed costs of $8,250,000.
 C. variable cost ratio of 32% and operating income of $9,600,000.correct
 D. contribution margin ratio of 68% and operating income of $7,050,000.
Question was not answered
Correct Answer Explanation:
The best way to solve this is to prepare an income statement that incorporates the sales
for the coming year and the cost management initiatives, and then calculate the items in
each answer choice.

Sales 175,000 units @ $150 $26,250,000


Variable Costs 175,000 units @ ($60 × 0.8) 8,400,000
Contribution Margin $17,850,000
Fixed Costs ($60 × 150,000) − $750,000 8,250,000
Operating Income $ 9,600,000
Hock P2 2020
Section C - Decision Analysis.
Answers
The variable cost ratio is 8,400,000 ÷ 26,250,000 = 0.32; and operating income is
$9,600,000.
Explanation for Choice A:
$72 is the current per unit contribution margin of $90 ($150 − $60) reduced by 20% ($90
× 0.80). However, it is variable costs that are expected to be reduced by 20%, not the
contribution margin. A per unit fixed cost of $55 would result from dividing the expected
cost reduction of $750,000 by the current year’s anticipated volume of 150,000 units.
However, fixed costs do not change with levels of sales or production as long as the
volume remains within the relevant range; and so they should not be evaluated on a per
unit basis but should be evaluated in total.
Explanation for Choice B:
The contribution margin here is calculated by recalculating the variable costs with the
expected 20% reduction and multiplying the resulting per unit contribution margin by the
current year’s expected volume of 150,000 units. The total contribution margin for the
coming year should be based on the next year’s expected volume.
Explanation for Choice D:
Operating income of $7,050,000 results from increasing fixed costs by 20%, not
reducing them by 20%. The expected total fixed cost for the current year is $60 ×
150,000, or $9,000,000. Fixed cost should decrease by 20% for the coming year.
146. Question ID: CMA 1295 1.18 (Topic: Using Economics Concepts in
Production Decisions)

Number of Workers Total Product Units Average Selling Price


10 20 $50.00
11 25 49.00
12 28 47.50
The marginal revenue per unit when one worker is added to a team of 11 workers is

 A. $35.00correct
 B. $105.00
 C. $225.00
 D. $47.50
Question was not answered
Correct Answer Explanation:
Marginal revenue is the addition to revenue that arises from increasing production and
sales by one unit. Marginal revenue per unit when one worker is added is the addition to
Hock P2 2020
Section C - Decision Analysis.
Answers
revenue from that one more worker (or the marginal revenue product of that one more
worker) divided by the number of additional units that can be produced with that one
more worker.
The marginal revenue of adding one more worker to a team of 11 equals the change in
the total revenue from 25 units × $49 = $1,225 to 28 units × $47.50 = $1,330. To find
the marginal revenue per unit, divide that amount of change by the number of additional
units produced with the 12th worker. Since 3 additional units are produced, the marginal
revenue per unit when one worker is added to a team of 11 workers is ($1,330 −
$1,225) ÷ 3 = $35.
Explanation for Choice B:
Marginal revenue is the addition to revenue that arises from increasing production and
sales by one unit. Marginal revenue per unit when one worker is added is the addition to
revenue from that one more worker (or the marginal revenue product of that one more
worker) divided by the number of additional units that can be produced with that one
more worker. This is the increase in revenue, or marginal revenue product, when the
number of workers is increased from 11 to 12.
Explanation for Choice C:
Marginal revenue is the addition to revenue that arises from increasing production and
sales by one unit. Marginal revenue per unit when one worker is added is the addition to
revenue from that one more worker (or the marginal revenue product of that one more
worker) divided by the number of additional units that can be produced with that one
more worker. This is the increase in revenue, or marginal revenue product, when the
number of workers is increased from 10 to 11.
Explanation for Choice D:
Marginal revenue is the addition to revenue that arises from increasing production and
sales by one unit. Marginal revenue per unit when one worker is added is the addition to
revenue from that one more worker (or the marginal revenue product of that one more
worker) divided by the number of additional units that can be produced with that one
more worker. This is the average selling price of the units at the level of sales that
results when 12 workers are hired, not the marginal revenue per unit when the number
of workers is increased from 11 to 12.
147. Question ID: ICMA 13.P2.052 (Topic: Using Economics Concepts in
Production Decisions)
Which one of the following is the most important difference between a monopoly and a
firm facing perfect competition, assuming both are unconstrained profit-maximizers?

 A. The monopolist will set the output so that marginal cost equals the average cost.
 B. The monopoly equilibrium price is the optimal level for society as a whole.
Hock P2 2020
Section C - Decision Analysis.
Answers
 C. The competitive firm sets its price to maximize total revenue, while the monopolist
maximizes the price.
 D. The monopolist's marginal revenue is less than its price, while the competitive firm's
marginal revenue equals its price.correct
Question was not answered
Correct Answer Explanation:
A monopolist's marginal revenue curve is below its demand curve because as
production increases, a monopolist will have to lower its price for all the units it sells in
order to get consumers to buy more of its output. Therefore, the additional (marginal)
revenue received from producing an additional unit will be less than the price received
for that unit. Here is an example:

Marginal
Quantity Total
Revenue
Price Sold Revenue
$20 0 $0
18 1 18 $18
16 2 32 14
14 3 42 10
12 4 48 6
10 5 50 2
8 6 48 (2)
In a perfectly competitive market, there are many buyers and sellers and customers are
indifferent as to which seller they buy from. The product is standardized, so the same
product is available from every seller. In a perfectly competitive market, the demand
curve for any particular firm is perfectly elastic (horizontal). This is because the firm is a
price taker and is unable to influence the price by either increasing or decreasing
production. So whatever number of units it would like to sell, the firm must sell at the
market price.
If an individual firm tries to charge more than the market price, though, it will sell
nothing. If it drops its price below the market price, it can still sell as much of its product
as it wants to. But if it drops its price below the market price, its total revenue will be
lower than it could have been, because it could have sold the exact same amount at the
market price and earned more total revenue. In a perfectly competitive market, the
marginal revenue from the sale of one more unit is equal to the market price.
Explanation for Choice A:
Hock P2 2020
Section C - Decision Analysis.
Answers
As is true for any firm, the monopolist's maximum profit is at the point where its marginal
revenue equals its marginal cost. Therefore, to maximize its revenue, the monopolist
will set its output at the point where its marginal revenue equals its marginal cost.
Explanation for Choice B:
In a monopoly, the firm produces less than the ideal output level. Compared with a
perfectly competitive market, prices will be higher and output levels lower in a
monopolized market. Additionally, options are limited to consumers as there is only one
supplier of the product in the market.
Explanation for Choice C:
The competitive firm does not set its price to maximize total revenue. The competitive
firm is a price taker and is unable to influence the market price by either increasing or
decreasing production. So whatever number of units it would like to sell, the competitive
firm must sell all of them at the market price.
Even though it has a monopoly on the market, the monopolist is still subject to the law
of supply and demand. A monopolist's marginal revenue curve is below its demand
curve because as production increases, a monopolist will have to lower its price for all
the units it sells in order to get consumers to buy more of its output.
148. Question ID: CIA 578 IV.18 (Topic: Using Economics Concepts in Production
Decisions)
An organization's sales revenue is expected to be $72,600, a 10% increase over last
year. For the same period, total fixed costs of $22,000 are expected to be the same as
last year. If the number of units sold is expected to increase by 1,100, the marginal
revenue per unit will be:

 A. $4
 B. $46
 C. $6correct
 D. $20
Question was not answered
Correct Answer Explanation:
Given that expected sales are to be $72,600 and that is a 10% increase over last year's
sales, then last year's sales must have been $66,000 ($72,600 ÷ 1.10). This means that
sales increased $6,600 over last year ($72,600 − $66,000). Units sold is expected to
increase by 1,100 units. The marginal revenue per unit is the amount of increase in
sales revenue divided by the amount of increase in the number of units sold. Dividing
$6,600 by 1,100 units gives us a marginal revenue per unit of $6.
Explanation for Choice A:
Hock P2 2020
Section C - Decision Analysis.
Answers
The correct way to solve this is to calculate the amount of expected increase in sales
revenue and then divide that amount of expected increase in sales revenue by the
amount of expected increase in units sold. See correct answer for a full explanation.
We have been unable to determine how to calculate this incorrect answer choice. If you
have calculated it, please let us know how you did it so we can create a full explanation
of why this answer choice is incorrect. Please send us an email at
support@hockinternational.com. Include the full Question ID number and the actual
incorrect answer choice -- not its letter, because that can change with every study
session created. The Question ID number appears at the top of the question. Thank you
in advance for helping us to make your HOCK study materials better.
Explanation for Choice B:
This is the expected sales of $72,600 minus expected fixed costs of $22,000, the
remainder divided by the expected increased in units sold of 1,100. This is not the way
to calculate the marginal revenue per unit of increase in sales. The correct way to solve
this is to calculate the amount of expected increase in sales revenue and then divide
that amount of expected increase in sales revenue by the amount of expected increase
in units sold. See correct answer for a full explanation.
Explanation for Choice D:
The correct way to solve this is to calculate the amount of expected increase in sales
revenue and then divide that amount of expected increase in sales revenue by the
amount of expected increase in units sold. See correct answer for a full explanation.
We have been unable to determine how to calculate this incorrect answer choice. If you
have calculated it, please let us know how you did it so we can create a full explanation
of why this answer choice is incorrect. Please send us an email at
support@hockinternational.com. Include the full Question ID number and the actual
incorrect answer choice -- not its letter, because that can change with every study
session created. The Question ID number appears at the top of the question. Thank you
in advance for helping us to make your HOCK study materials better.
149. Question ID: ICMA 10.P2.227 (Topic: Using Economics Concepts in
Production Decisions)
Auburn Products Inc. has compiled the following daily cost information for its
manufacturing operation.

Output (units) Fixed Cost Variable Cost


0 $2,000 $ 0
1 2,000 200
2 2,000 380
3 2,000 550
Hock P2 2020
Section C - Decision Analysis.
Answers
4 2,000 700
5 2,000 860
6 2,000 1,040
7 2,000 1,250
8 2,000 1,500
Auburn's average total cost at an output level of 3 units is

 A. $2,550.
 B. $850.correct
 C. $667.
 D. $1,217.
Question was not answered
Correct Answer Explanation:
The total cost at an output level of 3 units is $2,000 fixed costs plus $550 variable costs,
for a total of $2,550. The average total cost per unit is $2,550 divided by 3 units, or
$850.
Explanation for Choice A:
This is the total cost at an output level of 3 units. The question asks for
the average total cost, which means the average total cost per unit.
Explanation for Choice C:
This is the average fixed cost per unit at an output level of 3. The question asks for the
average total cost per unit at that output level.
Explanation for Choice D:
This answer results from interpreting the amounts in the variable cost column to be the
variable cost per unit at each production level. The amounts in the variable cost column
are the total variable costs at each production level, not the variable cost per unit at
each production level.
150. Question ID: CIA 585 IV.6 (Topic: Using Economics Concepts in Production
Decisions)
Green Company produces Product A and sells it for $18.00. The following cost data
apply:

Type of cost Per Unit


Direct materials (3 lb. x $1.50) $ 4.50
Hock P2 2020
Section C - Decision Analysis.
Answers
Direct labor 6.45
Variable overhead 1.35
Fixed overhead 1.50
Variable selling expense 1.10
Fixed selling expense 2.20
$17.10
Green has thought of marketing a new Product B with the same cost structure as
Product A except that the price will be $15.60. Green Company currently has the plant
capacity necessary for this expansion. Because of the cost structure, Green Company
will find the production and sale of Product B in the short run to be

 A. not profitable unless the price can be raised to $17.10.


 B. not profitable at $15.60 because the fixed selling expense and fixed manufacturing
overhead will not be covered by the price.
 C. not profitable at any price.
 D. profitable to produce and sell Product B in the short run at the price of $15.60.correct
Question was not answered
Correct Answer Explanation:
Assuming the company has excess capacity, the company will be better off if it is able
to sell Product B at any price above its incremental costs of $13.40 ($4.50 DM + $6.45
DL + $1.35 variable overhead + $1.10 variable selling cost). At the price of $15.60 per
unit, Product B would contribute $2.20 per unit toward profit.
Explanation for Choice A:
It would be profitable in the short-term to produce and sell Product B at a price greater
than its variable costs $13.40.
Explanation for Choice B:
Fixed selling and fixed manufacturing overhead do not need to be covered by the price
of Product B because they are already being covered by Product A.
Explanation for Choice C:
As long as the price for Product B is greater than its variable costs it would be profitable
in the short-term to produce and sell Product B.
151. Question ID: CMA 697 1.5 (Topic: Using Economics Concepts in Production
Decisions)
Hock P2 2020
Section C - Decision Analysis.
Answers
Average
Average Average
Total Cost
Units of Product Fixed Cost Variable Cost
6 $15.00 $25.00 $40.00
7 12.86 24.00 36.86
8 11.25 23.50 34.75
9 10.00 23.75 33.75
The marginal cost of producing the ninth unit is

 A. $23.75
 B. $25.75correct
 C. $23.50
 D. $33.75
Question was not answered
Correct Answer Explanation:
Marginal cost cost refers to the additional cost of production from producing one
additional unit. To find the marginal cost of producing the ninth unit, one must find the
total cost for producing both 8 units and 9 units of the good. The difference in total cost
is the marginal cost of producing the ninth unit.
This chart gives average total costs, not total costs. When doing this calculation, the
marginal cost of producing the 9th unit is $25.75. The marginal cost of the 9th unit =
Total cost of 9 units − Total cost of 8 units = ($33.75 × 9) − ($34.75 × 8) = $303.75 −
$278.00, and the marginal cost of producing the 9th unit = $25.75.
Explanation for Choice A:
This is the average variable cost of producing 9 units. Marginal cost cost refers to the
additional cost of production from producing one additional unit. To find the marginal
cost of producing the ninth unit, one must find the total cost for producing both 8 units
and 9 units of the good. The difference in total cost is the marginal cost of producing the
ninth unit.
Explanation for Choice C:
This is the average variable cost of producing 8 units. Marginal cost cost refers to the
additional cost of production from producing one additional unit. To find the marginal
cost of producing the ninth unit, one must find the total cost for producing both 8 units
and 9 units of the good. The difference in total cost is the marginal cost of producing the
ninth unit.
Explanation for Choice D:
Hock P2 2020
Section C - Decision Analysis.
Answers
This is the average cost per unit of producing 9 units. Marginal cost cost refers to the
additional cost of production from producing one additional unit. To find the marginal
cost of producing the ninth unit, one must find the total cost for producing both 8 units
and 9 units of the good. The difference in total cost is the marginal cost of producing the
ninth unit.
152. Question ID: ICMA 10.P2.230 (Topic: Using Economics Concepts in
Production Decisions)
Auburn Products Inc. has compiled the following daily cost information for its
manufacturing operation.

Output (units) Fixed Cost Variable Cost


0 $2,000 $ 0
1 2,000 200
2 2,000 380
3 2,000 550
4 2,000 700
5 2,000 860
6 2,000 1,040
7 2,000 1,250
8 2,000 1,500
Auburn's marginal cost for the 7th unit is

 A. $210.correct
 B. $179.
 C. $286.
 D. $464.
Question was not answered
Correct Answer Explanation:
Marginal cost is the addition to total cost by increasing production by one unit. At the
output level of 7 units, total cost increases from $3,040 to $3,250, an increase of $210.
Explanation for Choice B:
This is the total variable cost at the output level of 7 units divided by 7 units. It could
also be calculated as the amount of increase in total cost from the output level of 0 units
to the output level of 7 units divided by 7 units. Marginal cost is the addition to total cost
Hock P2 2020
Section C - Decision Analysis.
Answers
by increasing production by one unit. This answer is the addition to total cost by
increasing production by 7 units.
Explanation for Choice C:
This is not the correct answer. Please see the correct answer for a complete
explanation.
We have been unable to determine how to calculate this incorrect answer choice. If you
have calculated it, please let us know how you did it so we can create a full explanation
of why this answer choice is incorrect. Please send us an email at
support@hockinternational.com. Include the full Question ID number and the actual
incorrect answer choice -- not its letter, because that can change with every study
session created. The Question ID number appears at the top of the question. Thank you
in advance for helping us to make your HOCK study materials better.
Explanation for Choice D:
This is the total cost of $3,250 at the output level of 7 units divided by 7 units. This is the
average total cost per unit at that level. Marginal cost is the addition to total cost by
increasing production by one unit.
153. Question ID: ICMA 13.P2.043 (Topic: Using Economics Concepts in
Production Decisions)
Oak Fine Furnishings manufactures a wide range of home furnishings. One of their
products is an oak headboard. The company currently sells 4,000 headboards at an
average price of $100 per unit. To manufacture the headboards, the variable costs are
$55 per unit and the total fixed costs assigned to the oak headboard are $150,000. If
the sale of headboards increases by 50% and all else remains constant, this would
result in

 A. fixed costs of $225,000.


 B. earnings before interest and taxes of $120,000.correct
 C. a 50% increase in earnings before interest and taxes.
 D. a gross margin of $380,000.
Question was not answered
Correct Answer Explanation:
The contribution margin per unit is $45 ($100 − $55). With a 50% increase in sales,
sales will become 6,000 units and earnings before interest and taxes will become (6,000
× $45) − $150,000 = $120,000.
Explanation for Choice A:
By definition, even if production increases, the fixed costs will remain the same, or
$150,000.
Hock P2 2020
Section C - Decision Analysis.
Answers
Explanation for Choice C:
Earnings will not increase by the same percentage as revenues increase because of the
presence of fixed costs within the company's cost structure. Fixed costs do not change
with changes in volume as long as the volume remains within the relevant range.
Because fixed costs assigned to headboard will not increase when sales increase, the
increase in earnings attributable to headboards should be more than 50%.
Explanation for Choice D:
This answer results from increasing revenue by 50% without also increasing variable
costs by 50%. Variable costs must increase as well.
154. Question ID: ICMA 10.P2.229 (Topic: Using Economics Concepts in
Production Decisions)
Harper Products' cost information for the normal range of output in a month is shown
below.

Output in units Total Cost


20,000 $3,000,000
22,500 3,325,000
25,000 3,650,000
What is Harper's short-run marginal cost?

 A. $130.correct
 B. $150.
 C. $146.
 D. $26.
Question was not answered
Correct Answer Explanation:
The amount of increase in total cost at each level is $325,000. The amount of increase
in output at each level (second and third levels) is 2,500 units. Therefore, in the short
run, the marginal cost per unit is $325,000 divided by 2,500, which is $130.
Explanation for Choice B:
Marginal cost is the addition to total cost by increasing production by one unit. This is
the total cost at the output level of 20,000 divided by 20,000 units. Thus, this is the
average total cost per unit at the level of 20,000 units. It is not the marginal cost.
Explanation for Choice C:
Hock P2 2020
Section C - Decision Analysis.
Answers
Marginal cost is the addition to total cost by increasing production by one unit. This is
the total cost at the output level of 25,000 divided by 25,000 units. Thus, this is the
average total cost per unit at the level of 25,000 units. It is not the marginal cost.
Explanation for Choice D:
This is the amount of increase in the total cost at each level of production divided by
12,500. The divisor should be the amount of increase in output at either the second or
third output level.
155. Question ID: ICMA 10.P2.231 (Topic: Using Economics Concepts in
Production Decisions)
Daily costs for Kelso Manufacturing include $1,000 of fixed costs and total variable
costs are shown below.

Unit Output 10 11 12 13 14 15
Cost $150 $300 $480 $620 $750 $900
The marginal cost of the 12th unit is

 A. $104.16.
 B. $180.00.correct
 C. $140.00.
 D. $40.00.
Question was not answered
Correct Answer Explanation:
Marginal cost is the addition to total cost by increasing production by one unit. When
production is increased from 11 units to 12 units, total cost increases from $300 to
$480. So the marginal cost of the 12th unit is $480 − $300, or $180.
Explanation for Choice A:
This is not the correct answer. Please see the correct answer for a complete
explanation.
We have been unable to determine how to calculate this incorrect answer choice. If you
have calculated it, please let us know how you did it so we can create a full explanation
of why this answer choice is incorrect. Please send us an email at
support@hockinternational.com. Include the full Question ID number and the actual
incorrect answer choice -- not its letter, because that can change with every study
session created. The Question ID number appears at the top of the question. Thank you
in advance for helping us to make your HOCK study materials better.
Explanation for Choice C:
Hock P2 2020
Section C - Decision Analysis.
Answers
Marginal cost is the addition to total cost by increasing production by one unit. This is
the difference between the total cost to manufacture 13 units and the total cost to
manufacture 12 units. This is the marginal cost of the 13th unit, not the marginal cost of
the 12th unit.
Explanation for Choice D:
Marginal cost is the addition to total cost by increasing production by one unit. This is
the total cost of 12 units ($480) divided by 12 units. Thus, it is the average total cost per
unit at the production level of 12 units, not the marginal cost of the 12th unit.
156. Question ID: ICMA 10.P2.234 (Topic: Using Economics Concepts in
Production Decisions)
Parker Manufacturing is analyzing the market potential for its specialty turbines. Parker
developed its pricing and cost structures for their specialty turbines over various
relevant ranges. The pricing and cost data for each relevant range are presented below.

Units produced and sold: 1-5 6 - 10 11 - 15 16 - 20


Total fixed costs $200,000 $400,000 $600,000 $800,000
Unit variable cost 50,000 50,000 45,000 45,000
Unit selling price 100,000 100,000 100,000 100,000
Which one of the following production/sales levels would produce the highest operating
income for Parker?

 A. 10 units.
 B. 17 units.
 C. 8 units.
 D. 14 units.correct
Question was not answered
Correct Answer Explanation:
The best way to determine this is to prepare an operating income statement for each of
the production and sales options given in the answer choices. Sales are based on the
units sold multiplied by the sales price per unit, variable costs are based on the number
of units sold multiplied by the variable cost per unit for the applicable range, and total
fixed costs for each relevant range need to be included as given. Operating income is
sales minus variable costs minus fixed costs.

Choices: 8 10 14 17
Sales $800,000 $1,000,000 $1,400,000 $1,700,000
Variable Cost 400,000 500,000 630,000 765,000
Hock P2 2020
Section C - Decision Analysis.
Answers
Fixed Cost 400,000 400,000 600,000 800,000
Operating Income $ 0 $ 100,000 $ 170,000 $ 135,000
The highest operating income level occurs at sales of 14 units.
Explanation for Choice A:
The best way to determine this is to prepare an operating income statement for each of
the production and sales options given in the answer choices. This answer could result
from any error in calculations or in evaluating the results of the calculations. See correct
answer for details.
Explanation for Choice B:
The best way to determine this is to prepare an operating income statement for each of
the production and sales options given in the answer choices. This answer could result
from any error in calculations or in evaluating the results of the calculations. See correct
answer for details.
Explanation for Choice C:
The best way to determine this is to prepare an operating income statement for each of
the production and sales options given in the answer choices. This answer could result
from any error in calculations or in evaluating the results of the calculations. See correct
answer for details.
157. Question ID: ICMA 10.P2.233 (Topic: Using Economics Concepts in
Production Decisions)
Daily sales and cost data for Crawford Industries are shown below.

Sales
Total
Units $ Costs
20 $2,000 $1,200
21 2,090 1,250
22 2,170 1,290
23 2,240 1,330
24 2,300 1,380
25 2,350 1,440
The marginal cost of the 23rd unit is

 A. $30.00.
 B. $57.83.
Hock P2 2020
Section C - Decision Analysis.
Answers
 C. $50.00.
 D. $40.00.correct
Question was not answered
Correct Answer Explanation:
Marginal cost is the addition to total cost by increasing production by one unit. The total
cost of 23 units is $1,330, and the total cost of 22 units is $1,290. $1,330 − $1,290 =
$40. This is the marginal cost, or the addition to total cost by increasing production from
22 units to 23 units.
Explanation for Choice A:
This is not the correct answer. Please see the correct answer for an explanation.
We have been unable to determine how to calculate this incorrect answer choice. If you
have calculated it, please let us know how you did it so we can create a full explanation
of why this answer choice is incorrect. Please send us an email at
support@hockinternational.com. Include the full Question ID number and the actual
incorrect answer choice -- not its letter, because that can change with every study
session created. The Question ID number appears at the top of the question. Thank you
in advance for helping us to make your HOCK study materials better.
Explanation for Choice B:
This is the total cost of producing 23 units ($1,330) divided by 23 units. Thus it is the
average total cost per unit to produce 23 units. It is not the marginal cost of the 23rd
unit. Marginal cost is the addition to total cost by increasing production by one unit.
Explanation for Choice C:
Marginal cost is the addition to total cost by increasing production by one unit. This is
the difference between total costs at the production level of 24 units and the total costs
at the production level of 23 units. Thus it is the marginal cost of the 24th unit, not the
marginal cost of the 23rd unit.
158. Question ID: HOCK DA 1 (Topic: Using Economics Concepts in Production
Decisions)
Stark Rehabilitation Hospital is an inpatient institution for severely injured patients who
need intense therapy to recover normal functioning. The hospital has a maximum
capacity of 130 patients. The hospital employs nurses, doctors and physical therapists.
Expenses are as follows:
Medical staff:

 1 nurse for every 20 patients, average cost $66,000 per year per nurse
 1 doctor for every 60 patients, average cost $150,000 per year per doctor
Hock P2 2020
Section C - Decision Analysis.
Answers
 1 physical therapist for every 10 patients, average cost $80,000 per year per
therapist
The cost per patient per day for meals, medicine and supplies averages $60 per day per
patient.
Utilities cost a base amount of $2,500 per month plus an average of $200 per patient
per month.
Medical and exercise equipment, occupancy expense and administrative expense total
$7,700,000 per year.
The hospital patient census (i.e., number of patients at any one time) varies from a
minimum of 100 patients to fully occupied at 130 patients.
Stark charges $300 per day per patient, and this includes the room, meals, medicine,
supplies and all services.
What items are fixed costs for Stark?

 A. Medical and exercise equipment, occupancy expense and administrative


expensecorrect
 B. Meals, medicine and supplies
 C. Utilities
 D. Medical staff
Question was not answered
Correct Answer Explanation:
Fixed costs are costs which do not change in total as long as the volume remains within
the relevant range. Medical and exercise equipment, occupancy expense and
administrative expenses do not change in total as long as the volume remains within the
relevant range.
Explanation for Choice B:
Fixed costs are costs which do not vary with activity. Meals, medicine and supplies vary
with activity.
Explanation for Choice C:
Fixed costs are costs which do not vary with activity. Utilities vary with activity.
Explanation for Choice D:
Medical staff costs are actually semi-fixed (step) costs because they are fixed over a
given, small range of activity and above that level of activity, the cost suddenly jumps
(here because additional medical staff need to be hired).
Hock P2 2020
Section C - Decision Analysis.
Answers
159. Question ID: CMA 1290 1.15 (Topic: Using Economics Concepts in
Production Decisions)
Marginal revenue is

 A. The change in total revenue associated with producing and selling one more
unit.correct
 B. The change in total revenue associated with increasing prices.
 C. Greater than price in pure competition.
 D. Equal to price in monopolistic competition.
Question was not answered
Correct Answer Explanation:
Marginal revenue, by definition, is the change in total revenue associated with
producing and selling one more unit.
Explanation for Choice B:
Marginal revenue is equal to the change in total revenue associated with producing and
selling one more unit, not with increasing prices.
Explanation for Choice C:
Marginal revenue is not greater than price in pure competition. Marginal revenue is
equal to price.
Explanation for Choice D:
Marginal revenue is equal to marginal cost in monopolistic competition.
160. Question ID: CIA 1183 IV.24 (Topic: Using Economics Concepts in
Production Decisions)
A company produced the following data (rounded) on its product:

Unit Cost Marginal


Units Produced Fixed Variable Total Cost Revenue
1 $100 $85 $185 $85 $90
2 50 70 120 55 90
3 33 65 98 55 90
4 25 67 92 73 90
5 20 75 95 107 90
If two units of product were produced and sold, the total contribution margin would be:
Hock P2 2020
Section C - Decision Analysis.
Answers
 A. $25
 B. $40correct
 C. $50
 D. $70
Question was not answered
Correct Answer Explanation:
The unit contribution margin is unit sales price − unit variable costs. The sales price of
$90 less variable costs per unit of $70 equals a unit contribution margin of $20. If 2 units
are produced and sold, the total contribution margin would be $20 × 2 units = $40 total
contribution margin.
Explanation for Choice A:
The total contribution margin equals total revenue minus total variable costs. The total
variable cost is not $85 + $70. The information on each line in the variable unit cost
column applies to all units produced at that given production level, not just the
incremental unit produced. In other words, if one unit is produced, the variable cost to
produce that one unit will be $85 in total. If two units are produced, the variable cost to
produce those two units will be $70 × 2 in total. If three units are produced, the variable
cost to produce those three units will be $65 × 3 in total, and so on.
Explanation for Choice C:
This is fixed costs on a per unit basis. The total contribution margin equals total revenue
minus total variable costs.
Explanation for Choice D:
This is the unit variable cost at a production level of 2 units. The total contribution
margin equals total revenue minus total variable costs.
161. Question ID: CMA 1295 1.17 (Topic: Using Economics Concepts in
Production Decisions)

Number of Workers Total Product Units Average Selling Price


10 20 $50.00
11 25 49.00
12 28 47.50
The marginal physical product when one worker is added to a team of 10 workers is

 A. 1 unit.
 B. 5 units.correct
 C. 25 units.
Hock P2 2020
Section C - Decision Analysis.
Answers
 D. 8 units.
Question was not answered
Correct Answer Explanation:
The marginal physical product of a worker is the additional amount of production that is
achieved when one more unit of input such as labor is employed. Therefore, in
deciphering the marginal physical product when one worker is added to a team of 10
workers, one must look at the total product units achieved with 10 workers and subtract
that number from the total product units that are achieved when 11 workers are
employed. From the table, the calculation shows that the marginal physical product is 5
units. (25 total product units − 20 total product units).
Explanation for Choice A:
The marginal physical product of a worker is the additional amount of production that is
achieved when one more unit of input such as labor is employed. This is the number of
additional workers.
Explanation for Choice C:
The marginal physical product of a worker is the additional amount of production that is
achieved when one more unit of input such as labor is employed. This is the total
product units achieved when 11 workers are employed.
Explanation for Choice D:
The marginal physical product of a worker is the additional amount of production that is
achieved when one more unit of input such as labor is employed. This is the increase in
output from adding 2 more workers to a team of 10 workers.
162. Question ID: ICMA 10.P2.260 (Topic: Using Economics Concepts in
Production Decisions)
Dayton Corporation manufactures pipe elbows for the plumbing industry. Dayton's per
unit sales price and variable costs are as follows.

Sales price $10


Variable costs 7
Dayton's practical plant capacity is 35,000 units. Dayton's total fixed costs amount to
$42,000, and the company has a 50% effective tax rate.
If Dayton produced and sold 30,000 units, net income would be

 A. $90,000.
 B. $48,000.
 C. $45,000.
Hock P2 2020
Section C - Decision Analysis.
Answers
 D. $24,000.correct
Question was not answered
Correct Answer Explanation:
Dayton's unit contribution margin is $3 ($10 − $7). Although the plant capacity is 35,000
units, if Dayton were to produce and sell only 30,000 units, the company's total
contribution margin would be 30,000 × $3, or $90,000. Fixed costs are $42,000, so the
net income before tax would be $48,000. Taxes are 50%, or $24,000, and net income
after tax is $48,000 − $24,000, which is $24,000.
Explanation for Choice A:
This is the contribution margin.
Explanation for Choice B:
This is net income before tax.
Explanation for Choice C:
This would be net income after tax if fixed costs were not included as a reduction of
income.
163. Question ID: HOCK DA 4 (Topic: Using Economics Concepts in Production
Decisions)
Stark Rehabilitation Hospital is an inpatient institution for severely injured patients who
need intense therapy to recover normal functioning. The hospital has a maximum
capacity of 130 patients. The hospital employs nurses, doctors and physical therapists.
Expenses are as follows:
Medical staff:

 1 nurse for every 20 patients, average cost $66,000 per year per nurse
 1 doctor for every 60 patients, average cost $150,000 per year per doctor
 1 physical therapist for every 10 patients, average cost $80,000 per year per
therapist
The cost per patient per day for meals, medicine and supplies averages $60 per day per
patient.
Utilities cost a base amount of $2,500 per month plus an average of $200 per patient
per month.
Medical and exercise equipment, occupancy expense and administrative expense total
$7,700,000 per year.
The hospital patient census (i.e., number of patients at any one time) varies from a
minimum of 100 patients to fully occupied at 130 patients.
Hock P2 2020
Section C - Decision Analysis.
Answers
Stark charges $300 per day per patient, and this includes the room, meals, medicine,
supplies and all services.
What are Stark's semi-variable costs?

 A. Utilitiescorrect
 B. Medical staff
 C. Meals, medicine and supplies
 D. Medical and exercise equipment, occupancy expense and administrative expense
Question was not answered
Correct Answer Explanation:
A semi-variable cost has both a fixed component and a variable component. There is a
basic fixed amount that must be paid regardless of activity; and added to that fixed
amount is an amount which varies with activity. An example might be charges made by
a credit card processor, where the processor charges a certain basic monthly fee
whether or not there is any activity, plus a per-transaction charge. Utilities would fit this
description better than any of the other answer choices. For example, some utility
expenses are required just to maintain the building and the needs of the administrative
staff, even if there were no patients. Here, that basic amount is $2,500 per month. With
each additional patient, the cost increases by $200 per month because each additional
patient requires more utilities.
Explanation for Choice B:
A semi-variable cost has both a fixed component and a variable component. There is a
basic fixed amount that must be paid regardless of activity; and added to that fixed
amount is an amount which varies with activity. An example might be charges made by
a credit card processor, where the processor charges a certain basic monthly fee
whether or not there is any activity, plus a per-transaction charge. Medical staff
expenses do not fit this description.
Explanation for Choice C:
A semi-variable cost has both a fixed component and a variable component. There is a
basic fixed amount that must be paid regardless of activity; and added to that fixed
amount is an amount which varies with activity. An example might be charges made by
a credit card processor, where the processor charges a certain basic monthly fee
whether or not there is any activity, plus a per-transaction charge. Meals, medicine and
supplies do not fit this description.
Explanation for Choice D:
A semi-variable cost has both a fixed component and a variable component. There is a
basic fixed amount that must be paid regardless of activity; and added to that fixed
amount is an amount which varies with activity. An example might be charges made by
Hock P2 2020
Section C - Decision Analysis.
Answers
a credit card processor, where the processor charges a certain basic monthly fee
whether or not there is any activity, plus a per-transaction charge. Medical and exercise
equipment, occupancy expense and administrative expense do not fit this description.
164. Question ID: CMA 696 4.15 (Topic: Using Economics Concepts in Production
Decisions)
In a decision analysis situation, which one of the following costs is not likely to contain a
variable cost component?

 A. Depreciation.correct
 B. Overhead.
 C. Labor.
 D. Selling.
Question was not answered
Correct Answer Explanation:
Depreciation is a fixed cost that will not depend on the production level.
Explanation for Choice B:
Overhead can be variable, fixed or mixed (variable and fixed).
Explanation for Choice C:
Manufacturing labor will depend on the production level, so it is variable. Labor can also
contain a fixed component, such as management salaries.
Explanation for Choice D:
Selling can be variable, fixed or mixed (variable and fixed).
165. Question ID: CMA 697 1.4 (Topic: Using Economics Concepts in Production
Decisions)

Average
Average Average
Total Cost
Units of Product Fixed Cost Variable Cost
6 $15.00 $25.00 $40.00
7 12.86 24.00 36.86
8 11.25 23.50 34.75
9 10.00 23.75 33.75
The total cost of producing seven units is

 A. $258.02correct
Hock P2 2020
Section C - Decision Analysis.
Answers
 B. $280.00
 C. $90.02
 D. $168.00
Question was not answered
Correct Answer Explanation:
Total cost = variable cost + fixed cost. This table presents numbers for the average
variable, fixed and total costs. Therefore, to find the total cost of producing seven units,
one must look at the average total cost of producing seven units and multiply the total
cost, in this case $36.86, by the number produced, in this case 7. This multiplication is
necessary since the $36.83 is a per unit average cost, and we want to know how much
it would cost to produce 7 units. When doing so, the result is $258.02. ($36.86 × 7).
Explanation for Choice B:
This is the average total cost of producing 6 units multiplied by 6. The result is the total
cost of producing 6 units. The question asks for the total cost of producing 7 units, not 6
units.
Explanation for Choice C:
This is the total fixed cost for producing 7 units. The total cost of production is the cost
of the variable costs plus the fixed costs.
Explanation for Choice D:
This is the total variable cost for producing 7 units. The total cost of production is the
cost of the variable costs plus the fixed costs.
166. Question ID: HOCK DA 3 (Topic: Using Economics Concepts in Production
Decisions)
Stark Rehabilitation Hospital is an inpatient institution for severely injured patients who
need intense therapy to recover normal functioning. The hospital has a maximum
capacity of 130 patients. The hospital employs nurses, doctors and physical therapists.
Expenses are as follows:
Medical staff:

 1 nurse for every 20 patients, average cost $66,000 per year per nurse
 1 doctor for every 60 patients, average cost $150,000 per year per doctor
 1 physical therapist for every 10 patients, average cost $80,000 per year per
therapist
The cost per patient per day for meals, medicine and supplies averages $60 per day per
patient.
Hock P2 2020
Section C - Decision Analysis.
Answers
Utilities cost a base amount of $2,500 per month plus an average of $200 per patient
per month.
Medical and exercise equipment, occupancy expense and administrative expense total
$7,700,000 per year.
The hospital patient census (i.e., number of patients at any one time) varies from a
minimum of 100 patients to fully occupied at 130 patients.
Stark charges $300 per day per patient, and this includes the room, meals, medicine,
supplies and all services.
What costs are semi-fixed costs for Stark?

 A. Meals, medicine and supplies


 B. Medical staffcorrect
 C. Utilities
 D. Medical and exercise equipment, occupancy expense and administrative expense
Question was not answered
Correct Answer Explanation:
Semi-fixed costs are fixed over a given range of activity and vary at certain volume
increments. A semi-fixed cost moves upward in a "step" fashion, staying at a certain
level over a small range and then moving to the next level quickly. This description
matches the behavior of the medical staff costs.
This is a different concept from that of fixed costs, which we say are fixed as long as the
volume remains within a designated range. For a 100% fixed cost, the relevant range
will be much larger than the ranges within which the medical staff in this example
remains the same.
Semi-fixed costs are also referred to sometimes as "semi-variable costs." However, a
better term for a "step-function" cost such as that of the medical staff is "semi-fixed,"
because true semi-variable costs do not move upward in steps.
Explanation for Choice A:
Semi-fixed costs are fixed over a given range of activity and vary at certain volume
increments. That does not describe the behavior of meals, medicine and supplies costs.
Explanation for Choice C:
Semi-fixed costs are fixed over a given range of activity and vary at certain volume
increments. That does not describe the behavior of the utilities expenses.
Explanation for Choice D:
Hock P2 2020
Section C - Decision Analysis.
Answers
Semi-fixed costs are fixed over a given range of activity and vary at certain volume
increments. That does not explain the behavior of these expenses.
167. Question ID: CMA 692 1.28 (Topic: Using Economics Concepts in Production
Decisions)
The sum of the average fixed costs and the average variable costs for a given output is
known as

 A. Average total cost.correct


 B. Total cost.
 C. Long-run average cost.
 D. Average product.
Question was not answered
Correct Answer Explanation:
Average total cost is the sum of all production costs, both fixed and variable, divided by
the number of units produced. Thus the average total cost is the sum of the average
variable costs and average fixed costs for a given output.
Explanation for Choice B:
Total cost is equal to the sum of the fixed costs and the variable costs for a given
output, not the sum of the average fixed costs and the average variable costs.
Explanation for Choice C:
There is no such thing as "long run average cost." The closest thing to it is "long run
average total cost," which is the average cost per unit of output over the long run, in
which all inputs are considered to be variable. Thus, the definition of long run average
total cost does not include fixed costs since all costs are variable in the long run.
Explanation for Choice D:
Average product is the ratio of total product to the total quantity of an input used to
produce the product. Average product does not have anything to do with costs.
168. Question ID: CMA 692 1.29 (Topic: Using Economics Concepts in Production
Decisions)
The change in total product resulting from the use of one unit more of the variable factor
is known as

 A. Marginal cost.
 B. The point of diminishing average productivity.
 C. Marginal product.correct
 D. The point of diminishing marginal productivity.
Question was not answered
Hock P2 2020
Section C - Decision Analysis.
Answers
Correct Answer Explanation:
The change in total product resulting from the use of one more unit of the variable factor
is known as marginal product.
Explanation for Choice A:
The marginal cost is the additional cost of employing that additional variable factor. The
change in total product resulting from the use of one more unit of the variable factor is
the marginal product, not the marginal cost.
Explanation for Choice B:
The change in total product resulting from the use of one unit more of the variable factor
is not the point of diminishing average productivity, necessarily. Rather, it is the
marginal product.
Explanation for Choice D:
The change in total product resulting from the use of one more unit of the variable
factors is not necessarily the point of diminishing marginal productivity.
169. Question ID: ICMA 19.P2.017 (Topic: Using Economics Concepts in
Production Decisions)
The management of a company is attempting to reduce the cost for Product X by
analyzing the trade-offs between different types of product features and total product
cost. What type of cost reduction strategy is the company using?

 A. Value engineering.correct
 B. Total quality management.
 C. Activity-based costing.
 D. Kaizen.
Question was not answered
Correct Answer Explanation:
Value engineering is an evaluation of all the business functions in a product’s value
chain with the objective of reducing costs while still satisfying customer needs. This
evaluation may lead to design improvements, materials specification changes, or
modifications to manufacturing methods.

Explanation for Choice B:


Total quality management is not a cost-reduction strategy.
Explanation for Choice C:
Activity-based costing is a costing method, not a cost-reduction strategy.
Hock P2 2020
Section C - Decision Analysis.
Answers
Explanation for Choice D:
Kaizen is not a cost-reduction strategy.
170. Question ID: ICMA 10.P2.261 (Topic: Using Economics Concepts in
Production Decisions)
Raymund Inc., a bearings manufacturer, has the capacity to produce 7,000 bearings per
month. The company is planning to replace a portion of its labor intensive production
process with a highly automated process, which would increase Raymund's fixed
manufacturing costs by $30,000 per month and reduce its variable costs by $5 per unit.
Raymund's Income Statement for an average month is as follows.

Sales (5,000 units at $20 per unit) $100,000


Variable manufacturing costs $50,000
Variable selling costs 15,000 65,000
Contribution margin $ 35,000

Fixed manufacturing costs 16,000


Fixed selling costs 4,000 20,000
Operating income $ 15,000
If Raymund installs the automated process, the company's monthly operating income
would be

 A. $10,000.correct
 B. $5,000.
 C. $30,000.
 D. $40,000.
Question was not answered
Correct Answer Explanation:
Presently, variable manufacturing costs are $10 per unit ($50,000 total divided by 5,000
units). If variable manufacturing costs are reduced by $5 per unit, variable costs for the
5,000 units would be $25,000 ($5 × 5,000 units). Fixed manufacturing costs would
increase from $16,000 to $46,000. All other costs would remain the same. The
contribution margin would increase to $60,000 ($100,000 minus $25,000 minus
$15,000). Fixed costs would increase to $50,000 ($46,000 fixed manufacturing costs
plus $4,000 fixed selling costs). The contribution margin of $60,000 minus the fixed
costs of $50,000 equals $10,000 operating income.
Hock P2 2020
Section C - Decision Analysis.
Answers
Explanation for Choice B:
This would be the operating income if the variable costs remained the same and the
only manufacturing and selling fixed costs included were the $30,000 for the new
automation. However, the variable costs will change by $5 per unit, and the original
fixed costs need to be included.
Explanation for Choice C:
This would be the operating income if the new variable costs are calculated using plant
capacity rather than sales, and fixed costs remain unchanged.
Explanation for Choice D:
This would be operating income if the fixed costs remain unchanged. However, fixed
costs will increase by $30,000 if the machine is purchased.
171. Question ID: ICMA 10.P2.232 (Topic: Using Economics Concepts in
Production Decisions)
The total cost of producing 100 units of a good is $800. If a firm’s average variable cost
is $5 per unit, then the firm’s

 A. total variable cost is $300.


 B. average fixed cost is $3.correct
 C. marginal cost is $8.
 D. marginal cost is $3.
Question was not answered
Correct Answer Explanation:
If the average variable cost per unit is $5 and they produced 100 units, the total variable
costs were $500. Given total costs of $800, that means that the fixed costs were a total
of $300, or $3 per unit.
Explanation for Choice A:
If the average variable cost per unit is $5 and they produced 100 units, the total variable
costs were $500, not $300. It is the total fixed costs that are $300, not the total variable
cost.
Explanation for Choice C:
The marginal cost is the cost to produce one more unit — in this case the 101st unit.
This would be equal to the variable cost per unit, which the problem says is $5 per unit,
not $8 per unit.
Explanation for Choice D:
Hock P2 2020
Section C - Decision Analysis.
Answers
The marginal cost is the cost to produce one more unit — in this case the 101st unit.
This would be equal to the variable cost per unit, which the problem says is $5 per unit,
not $3 per unit.
172. Question ID: CMA 1289 1.7 (Topic: Using Economics Concepts in Production
Decisions)
If a firm currently producing 500 units of output incurs total fixed costs of $10,000 and
total variable costs of $15,000, the average total cost per unit is

 A. $30.
 B. $50.correct
 C. $20.
 D. $25.
Question was not answered
Correct Answer Explanation:
Average total cost per unit is the total cost divided by the amount produced. Total cost =
total variable costs + total fixed costs. Therefore for this problem we calculate total cost
per unit as: Total cost = Total Variable Costs of $15,000 + Total Fixed Costs of $10,000
= Total cost of $25,000. Average Total Cost = Total Cost ÷ Quantity produced. Average
Total Cost = $25,000 ÷ 500 = $50.
Explanation for Choice A:
This is the average variable cost per unit.
Explanation for Choice C:
This is the average fixed cost per unit.
Explanation for Choice D:
Average total cost per unit is the total cost divided by the amount produced. Please see
the correct answer for a complete explanation.
173. Question ID: CMA 1292 4.3 (Topic: Make or Buy Decisions)
Richardson Motors uses 10 units of Part No. T305 each month in the production of large
diesel engines. The cost to manufacture one unit of T305 is presented as follows.

Direct materials $ 2,000


Materials handling (20% of direct materials cost) 400
Direct labor 16,000
Manufacturing overhead (150% of direct labor) 24,000
Total manufacturing cost $42,400
Hock P2 2020
Section C - Decision Analysis.
Answers
Materials handling, which is not included in manufacturing overhead, represents the
direct variable costs of the receiving department that are applied to direct materials and
purchased components on the basis of their cost. Richardson's annual manufacturing
overhead budget is one-third variable and two-thirds fixed. Simpson Castings, one of
Richardson's reliable vendors, has offered to supply T305 at a unit price of $30,000.
If Richardson Motors purchases the ten T305 units from Simpson Castings, the capacity
Richardson used to manufacture these parts would be idle. Should Richardson decide
to purchase the parts from Simpson, the out-of-pocket cost per unit of T305 would

 A. Decrease $6,400.
 B. Decrease $12,400.
 C. Increase $3,600.
 D. Increase $9,600.correct
Question was not answered
Correct Answer Explanation:
Total per unit manufacturing cost for the T305 is $42,400. Since the overhead of
$24,000 is 1/3 variable and 2/3 fixed, $16,000 of that would continue and $8,000 would
be avoidable. Thus, the total avoidable variable manufacturing cost per unit is $26,400
($2,000 DM, $16,000 DL, $400 materials handling, and $8,000 variable manufacturing
OH).
If Richardson Motors purchases the units from Simpson Castings then the per unit
variable cost will be $36,000 [$30,000 purchase price + $6,000 material handling cost
applied [20% × $30,000 per unit]. Therefore, by purchasing from Simpson Castings the
per unit cost of the T305 component will increase by $9,600 ($36,000 − $26,400).
Explanation for Choice A:
This answer includes the fixed manufacturing overhead in the costs that would be
avoided if the parts were purchased outside. However, the fixed manufacturing
overhead would continue and so it would be the same whether the parts are
manufactured in-house or purchased outside.
Explanation for Choice B:
This answer includes the fixed manufacturing overhead in the costs that would be
avoided if the parts were purchased outside, and it also fails to include the materials
handling cost as a part of the cost of purchasing the parts outside.
Explanation for Choice C:
This answer does not include the materials handling cost for the parts purchased
outside.
174. Question ID: ICMA 10.P2.225 (Topic: Make or Buy Decisions)
Hock P2 2020
Section C - Decision Analysis.
Answers
Refrigerator Company manufactures ice-makers for installation in refrigerators. The
costs per unit, for 20,000 units of ice-makers, are as follows:

Direct materials $ 7
Direct labor 12
Variable overhead 5
Fixed overhead 10
Total costs $34
Cool Compartments Inc. has offered to sell 20,000 ice-makers to Refrigerator Company
for $28 per unit. If Refrigerator accepts Cool Compartments' offer the plant would be
idled and fixed overhead amounting to $6 per unit could be eliminated. The total
relevant costs associated with the manufacture of ice-makers amount to

 A. $480,000.
 B. $560,000.
 C. $600,000.correct
 D. $680,000.
Question was not answered
Correct Answer Explanation:
The only costs that are not relevant are those that will be there no matter which decision
is made. Those costs are the $4 of fixed overhead that would remain either way. That
leaves $30 of relevant costs per unit associated with the manufacture of ice-makers
($34 − $4), multiplied by 20,000 ice-makers, or $600,000.
Explanation for Choice A:
This is the variable cost to manufacture the ice-makers, but it is not the total relevant
costs associated with their manufacture.
Explanation for Choice B:
This is the cost of purchasing the ice-makers, but this is not the total relevant cost used
to make the decision.
Explanation for Choice D:
This is Refrigerator Company's cost of manufacturing 20,000 ice-makers under the
current structure.
175. Question ID: CMA 1287 5.28 (Topic: Make or Buy Decisions)
Leland Manufacturing uses 10 units of Part Number KJ37 each month in the production
of radar equipment. The unit cost to manufacture 1 unit of KJ37 is presented below.
Hock P2 2020
Section C - Decision Analysis.
Answers
Direct materials $ 1,000
Materials handling (20% of direct material cost) 200
Direct labor 8,000
Manufacturing overhead (150% of direct labor) 12,000
Total manufacturing cost $21,200
Material handling represents the direct variable costs of the Receiving Department that
are applied to direct materials and purchased components on the basis of their cost.
This is a separate charge in addition to manufacturing overhead. Leland's annual
manufacturing overhead budget is one-third variable and two-thirds fixed. Scott Supply,
one of Leland's reliable vendors, has offered to supply Part Number KJ37 at a unit price
of $15,000.
If Leland purchases the KJ37 units from Scott, the capacity Leland used to manufacture
these parts would be idle. Should Leland decide to purchase the parts from Scott, the
unit cost of KJ37 would

 A. Decrease by $6,200.
 B. Change by some amount other than those given.
 C. Increase by $4,800.correct
 D. Decrease by $3,200.
Question was not answered
Correct Answer Explanation:
Even though the manufacturing overhead is being applied on the basis of direct labor
hours and no direct labor hours would be required for production if the KJ37 is
purchased outside, the fixed portion of manufacturing overhead would continue to be
incurred even if Leland purchases the KJ37 from Scott Supply. Therefore, the fixed
component of the manufacturing overhead (2/3 of $12,000, or $8,000) is not relevant to
the decision and can be ignored.
Total variable manufacturing cost per unit for Leland is $13,200 ($1,000 DM + $200
materials handling + $8,000 DL + $4,000 variable overhead [1/3 of $12,000]). If Leland
purchases the units from Scott then the per unit cost will be $18,000 ($15,000 purchase
price + $3,000 materials handling charge [20% of $15,000]). By purchasing from Scott
the per unit cost increase of component KJ37 will be $4,800 ($18,000 − $13,200).
Explanation for Choice A:
This answer assumes that there are no other costs involved in the decision besides the
initial purchase price of $15,000 ($21,200 − $15,000 = $6,200). Other relevant costs
would include unavoidable variable cost and unavoidable fixed cost.
Hock P2 2020
Section C - Decision Analysis.
Answers
Explanation for Choice B:
The correct answer is one of the answers that is given.
Explanation for Choice D:
This is the difference between the total manufacturing cost and the total cost to
purchase the units. It does not take into consideration the fact that the fixed
manufacturing overhead is unavoidable and would continue even if the KJ37 is not
being manufactured. Since the fixed manufacturing overhead is the same whether
Leland manufactures the KJ37 or purchases it, it is an irrelevant cost and should not be
included in the total manufacturing cost used to compare with the cost to purchase the
item.
176. Question ID: ICMA 19.P2.011 (Topic: Make or Buy Decisions)
A manufacturing company is considering a new product for the coming year, which
requires the production of an electric motor. The company can purchase the motor from
a reliable vendor for $21 per unit, or manufacture it internally. The company has excess
capacity to manufacture the 30,000 motors needed in the coming year except for
manufacturing space and special machinery. The machinery can be leased for $45,000
annually. The company has finished goods warehouse space that it currently leases for
$39,000, which can be converted and used to manufacture the motors. To replace the
finished goods warehouse, additional off-site space can be leased at an annual cost of
$54,000. The estimated unit costs for manufacturing the motors internally, exclusive of
the leasing costs itemized above, are shown below.

Direct material $ 8.00


Direct labor 4.00
Variable manufacturing overhead 3.00
Allocated fixed manufacturing overhead 5.00
Should the company make or buy the electric motors, and why?

 A. Make, because manufacturing the motors internally would save $81,000.correct


 B. Buy, because purchasing from the outside vendor would save $54,000.
 C. Buy, because purchasing from the outside vendor would save $69,000.
 D. Make, because manufacturing the motors internally would save $96,000.
Question was not answered
Correct Answer Explanation:
The cost of purchasing the units from the outside supplier is $630,000. This is
calculated as the per-unit cost of $21 multiplied by the 30,000 units that are needed.
Hock P2 2020
Section C - Decision Analysis.
Answers
The cost of producing the units internally is $549,000. This is made up of three
elements:
1) $45,000 to lease the necessary equipment.
2) $54,000 to lease the necessary space. We use $54,000 because this is what the
company needs to pay to replace the space that will be used to produce these units.
This is the additional cost that the company will incur as a result of producing these
units.
3) The actual variable cost of production of the units of $450,000. This is a $15 per unit
charge multiplied by the 30,000 units needed. The fixed manufacturing costs are
ignored because those costs would still be incurred even if they do not produce the
units internally.
The cost savings of producing the units internally is $81,000.
Explanation for Choice B:
This is not the correct answer. Please see the correct answer for an explanation.
We have been unable to determine how to calculate this incorrect answer choice. If you
have calculated it, please let us know how you did it so we can create a full explanation
of why this answer choice is incorrect. Please send us an email at
support@hockinternational.com. Include the full Question ID number and the actual
incorrect answer choice -- not its letter, because that can change with every study
session created. The Question ID number appears at the top of the question. Thank you
in advance for helping us to make your HOCK study materials better.
Explanation for Choice C:
This is not the correct answer. Please see the correct answer for an explanation.
We have been unable to determine how to calculate this incorrect answer choice. If you
have calculated it, please let us know how you did it so we can create a full explanation
of why this answer choice is incorrect. Please send us an email at
support@hockinternational.com. Include the full Question ID number and the actual
incorrect answer choice -- not its letter, because that can change with every study
session created. The Question ID number appears at the top of the question. Thank you
in advance for helping us to make your HOCK study materials better.
Explanation for Choice D:
This answer fails to take into account the full $54,000 that must be paid for the space
that will be used. Even though the actual space that is being used has a cost of only
$39,000, the company must rent replacement space that will cost $54,000. Therefore,
$54,000 is the additional cost that the company will incur as a result of producing these
units, and the amount that needs to be used in this calculation. See the correct answer
for a full explanation.
Hock P2 2020
Section C - Decision Analysis.
Answers
177. Question ID: CMA 691 4.15 (Topic: Make or Buy Decisions)
Regis Company manufactures plugs used in its manufacturing cycle at a cost of $36 per
unit that includes $8 of fixed overhead. Regis needs 30,000 of these plugs annually,
and Orlan Company has offered to sell these units to Regis at $33 per unit. If Regis
decides to purchase the plugs, $60,000 of the annual fixed overhead applied will be
eliminated, and the company may be able to rent the facility previously used for
manufacturing the plugs.
If the plugs are purchased and the facility rented, Regis Company wishes to realize
$100,000 in savings annually. To achieve this goal, the minimum annual rent on the
facility must be

 A. $40,000.
 B. $10,000.
 C. $70,000.
 D. $190,000.correct
Question was not answered
Correct Answer Explanation:
The variable cost of the plugs is $28 per unit ($36 − $8). In addition, the question states
that $60,000 in annual fixed overhead would be saved, or $2 per unit ($60,000 ÷ 30,000
units). The total amount saved by purchasing from Orlan would thus be $30 per unit.
Orlan would charge $33 per unit. Purchasing the units from Orlan, Regis would lose $3
per unit ($30 − $33).
Therefore, Regis will have a total loss of $90,000 (30,000 units × $3 loss per unit) if it
purchases the plugs. If Regis wishes to realize a profit of $100,000 from purchasing the
plugs, then the minimum annual rent that it must receive for renting the facility will be
$190,000, calculated as follows:
($90,000) + X = $100,000
To solve for X, add $90,000 to both sides. The result is
X = $190,000
Explanation for Choice A:
This is not the correct answer. Please see the correct answer for an explanation.
We have been unable to determine how to calculate this incorrect answer choice. If you
have calculated it, please let us know how you did it so we can create a full explanation
of why this answer choice is incorrect. Please send us an email at
support@hockinternational.com. Include the full Question ID number and the actual
incorrect answer choice -- not its letter, because that can change with every study
session created. The Question ID number appears at the top of the question. Thank you
in advance for helping us to make your HOCK study materials better.
Hock P2 2020
Section C - Decision Analysis.
Answers
Explanation for Choice B:
This is not the correct answer. Please see the correct answer for an explanation.
We have been unable to determine how to calculate this incorrect answer choice. If you
have calculated it, please let us know how you did it so we can create a full explanation
of why this answer choice is incorrect. Please send us an email at
support@hockinternational.com. Include the full Question ID number and the actual
incorrect answer choice -- not its letter, because that can change with every study
session created. The Question ID number appears at the top of the question. Thank you
in advance for helping us to make your HOCK study materials better.
Explanation for Choice C:
This answer results from using $6 per unit of fixed cost as the avoidable fixed cost.
However, the avoidable fixed cost is $2 per unit ($60,000 ÷ 30,000 units). See correct
answer for a full explanation.
178. Question ID: CMA 691 4.9 (Topic: Make or Buy Decisions)
A company's approach to a make-or-buy decision

 A. Depends on whether the company is operating at or below normal volume.


 B. Involves an analysis of avoidable costs.correct
 C. Depends on whether the company is operating at or below breakeven.
 D. Should use absorption (full) costing.
Question was not answered
Correct Answer Explanation:
In its analysis a company would be looking only at relevant costs, not total costs.
Relevant costs in a make-or-buy decision would include only costs that would be
avoidable (both variable and fixed) if the company buys the item instead of making it.
Explanation for Choice A:
Whether the company is operating at or below normal volume is not relevant to a make-
or-buy decision.
Explanation for Choice C:
Whether the company is operating at or below breakeven is not relevant to a make-or-
buy decision.
Explanation for Choice D:
Absorption costing includes all variable and fixed manufacturing cost, of which not all
are relevant to the decision whether to make-or-buy decision. Relevant costs in a make-
or-buy decision would include only costs that would be avoidable (both variable and
fixed) if the company buys the item instead of making it.
Hock P2 2020
Section C - Decision Analysis.
Answers
179. Question ID: CMA 1287 5.29 (Topic: Make or Buy Decisions)
Leland Manufacturing uses 10 units of Part Number KJ37 each month in the production
of radar equipment. The unit cost to manufacture 1 unit of KJ37 is presented below.

Direct materials $ 1,000


Materials handling (20% of direct material cost) 200
Direct labor 8,000
Manufacturing overhead (150% of direct labor) 12,000
Total manufacturing cost $21,200
Material handling represents the direct variable costs of the Receiving Department that
are applied to direct materials and purchased components on the basis of their cost.
This is a separate charge in addition to manufacturing overhead. Leland's annual
manufacturing overhead budget is one-third variable and two-thirds fixed. Scott Supply,
one of Leland's reliable vendors, has offered to supply Part Number KJ37 at a unit price
of $15,000.
Assume Leland Manufacturing is able to rent all idle capacity for $25,000 per month. If
Leland decides to purchase the 10 units from Scott Supply, Leland's monthly cost for
KJ37 would

 A. Change by some amount other than those given.


 B. Increase $23,000.correct
 C. Increase $48,000.
 D. Decrease $7,000.
Question was not answered
Correct Answer Explanation:
The variable cost per unit to manufacture Part Number KJ37 is:
Direct Materials: $1,000
Material Handling (20% of direct materials, or $1,000 × 0.20): $200
Direct Labor: $8,000
Variable OH (1/3 of $12,000): $4,000
Total: $13,200
The variable cost per unit to purchase Part Number KJ37 is:
Part KJ37, which has now become direct materials: $15,000
Material Handling (20% of direct materials, or $15,000 × 0.20): $3,000
Total: $18,000
Hock P2 2020
Section C - Decision Analysis.
Answers
The difference is $18,000 − $13,200, or $4,800 per unit. If 10 units of Part Number KJ37
are needed each month, then the total increase in variable cost related to purchasing
KJ37 outside will be $4,800 × 10, or $48,000.
If Leland is able to rent the idle capacity for $25,000 per month, then the net increase in
Leland's monthly cost if the company purchases the units would be $23,000, calculated
as: ($48,000 − $25,000).
Explanation for Choice A:
The amount of change is one of the answer choices.
Explanation for Choice C:
This is the total increase in costs related to purchasing KJ37 outside. ([$15,000
purchase cost + $3,000 handling cost] − [$21,200 total manufacturing cost − $8,000
unavoidable fixed cost]) × 10 units. However, the income from renting the idle capacity
is not netted out.
Explanation for Choice D:
This answer results from omitting the material handling cost in the calculation of variable
cost per unit to purchase Part KJ37. If Part KJ37 is purchased, it will become a direct
materials/purchased components item that will have a material handling cost equal to
20% of the purchase price.
180. Question ID: CMA 692 4.25 (Topic: Make or Buy Decisions)
Laurel Corporation has its own cafeteria with the following annual costs:

Food $100,000
Labor 75,000
Overhead 110,000
Total $285,000
The overhead is 40% fixed. Of the fixed overhead, $25,000 is the salary of the cafeteria
supervisor. The remainder of the fixed overhead has been allocated from total company
overhead. Assuming the cafeteria supervisor will remain and the Laurel will continue to
pay his/her salary, the maximum cost Laurel will be willing to pay an outside firm to
service the cafeteria is

 A. $175,000.
 B. $219,000.
 C. $285,000.
 D. $241,000.correct
Question was not answered
Hock P2 2020
Section C - Decision Analysis.
Answers
Correct Answer Explanation:
In order to find the most Laurel would be willing to pay an outside company, we need to
look at avoidable variable costs only, since all of the fixed costs are unavoidable. Total
avoidable cost is $241,000 ($100,000 food + $75,000 labor + 66,000 of avoidable
variable overhead).
Explanation for Choice A:
This is incorrect since the $175,000 includes only food and labor. It does not include the
avoidable portion of overhead costs.
Explanation for Choice B:
This answer incorrectly assumes that $44,000 of fixed overhead costs are avoidable
($110,000 X 40%) and that variable overhead costs are unavoidable. Fixed overhead
costs are unavoidable and variable overhead costs are avoidable.
Explanation for Choice C:
$285,000 is the total cost. This is similar to a make or buy decision because the
decision is between whether to provide the service in-house or outsource it. The
relevant costs are costs that would be avoidable if the service is outsourced because
unavoidable costs will continue whether the service is outsourced or not. Avoidable
costs include the food, the labor, and the variable portion of the overhead costs.
181. Question ID: CMA 686 5.26 (Topic: Make or Buy Decisions)
Stewart Industries has been producing two bearings, components B12 and B18, for use
in production.

B12 B18
Machine hours required per unit 2.5 3.0
Standard cost per unit:
Direct material $ 2.25 $ 3.75
Direct labor 4.00 4.50
Manufacturing overhead:
Variable (See Note 1) 2.00 2.25
Fixed (See Note 2) 3.75 4.50
$12.00 $15.00
Stewart's annual requirement for these components is 8,000 units of B12 and 11,000
units of B18. Recently, Stewart's management decided to devote additional machine
time to other product lines resulting in only 41,000 machine hours per year that can be
Hock P2 2020
Section C - Decision Analysis.
Answers
dedicated to the production of the bearings. An outside company has offered to sell
Stewart the annual supply of the bearings at prices of $11.25 for B12 and $13.50 for
B18. Stewart wants to schedule the otherwise idle 41,000 machine hours to produce
bearings so that the company can minimize its costs (maximize its net benefits).
Note 1. Variable manufacturing overhead is applied on the basis of direct labor hours.
Note 2. Fixed manufacturing overhead is applied on the basis of machine hours.
The net benefit (loss) per machine hour that would result if Stewart accepts the
supplier's offer of $13.50 per unit for Component B18 is

 A. $(1.75)
 B. $(1.00)correct
 C. $0.50.
 D. Some amount other than those given.
Question was not answered
Correct Answer Explanation:
Whenever a resource is constrained, as the machine hours are in this problem, we need
to work with cost or contribution margin per unit of the constrained
resource, whichever is appropriate. Here, the "cost" is the amount that the outside
supplier will charge over and above the total variable cost to produce the component in-
house. And the "per unit of the constrained resource" is per machine hours. For this
problem, we need to find the net benefit or cost per machine hour for B18 if the units are
purchased from the outside supplier.
The first step is to calculate the total variable cost for B18. That is $3.75 for direct
materials + $4.50 for direct labor + $2.25 for variable manufacturing overhead, for a
total variable cost of $10.50.
The second step is to determine the difference between the outside company's quote
and the total variable cost and whether it is a benefit or a cost. The outside company's
quote is $13.50, and that is $3.00 greater than the total variable cost of $10.50. So the
total cost per unit to buy from the outside supplier is $3.00 per unit.
The final step is to determine the total cost per unit of the constrained resource, which is
machine hours. To manufacture one unit of B18, it requires 3.0 machine hours. So we
divide the total cost per unit to buy from the outside supplier ($3.00) by the number of
machine hours required to manufacture one unit (3.0 machine hours), and we have the
cost per machine hour to purchase the units outside, which is $(1.00).
Explanation for Choice A:
This answer is a result of totalling direct material cost ($3.75) and direct labor cost
($4.50) per unit produced, subtracting the quote of $13.50 from the total ($8.25), and
Hock P2 2020
Section C - Decision Analysis.
Answers
then dividing by the number of machine hours required per unit (3.0). However, variable
manufacturing overhead should be included in the calculation.
Explanation for Choice C:
This answer results from comparing the total cost (variable and fixed) of $15.00 per unit
with the quote of $13.50 per unit and then dividing by 3 machine hours per unit. $15.00
− $13.50 = $1.50 ÷ 3 machine hrs. per unit = $0.50 per unit per machine hrs. However,
this is not the correct way to find the net benefit or loss per machine hour that would
result if Stewart accepts the supplier's offer of $13.50 per unit instead of manufacturing
the components itself.
Explanation for Choice D:
The correct answer is one of those given.
182. Question ID: ICMA 10.P2.264 (Topic: Make or Buy Decisions)
Lark Industries accepted a contract to provide 30,000 units of Product A and 20,000
units of Product B. Lark's staff developed the following information with regard to
meeting this contract.

Product A Product B Total


Selling Price $75 $125
Variable costs $30 $48
Fixed overhead $1,600,000
Machine hours required 3 5
Machine hours available 160,000
Cost if outsourced $45 $60
Lark's operations manager has identified the following alternatives. Which alternative
should be recommended to Lark's management?

 A. Make 20,000 units of Product A, utilize the remaining capacity to make Product B, and
outsource the remainder.
 B. Make 25,000 units of Product A, utilize the remaining capacity to make Product B, and
outsource the remainder.
 C. Rent additional capacity of 30,000 machine hours which will increase fixed costs by
$150,000.
 D. Make 30,000 units of Product A, utilize the remaining capacity to make Product B, and
outsource the remainder.correct
Question was not answered
Correct Answer Explanation:
Hock P2 2020
Section C - Decision Analysis.
Answers
This is a comparison between the cost to manufacture versus the cost to purchase
outside. The revenue to be received and the $1,600,000 of fixed overhead will be the
same whether the product is manufactured or outsourced, so those factors are
irrelevant to this decision. In this case, the contribution margin earned per machine hour
is also not relevant. Looking only at the contribution margin per machine hour for each
product provides an incomplete analysis since it does not include comparative
information on the cost to outsource. We need to find the least cost alternative for
acquiring or producing this order.
With 160,000 machine hours available, the company can use 90,000 of those hours to
manufacture all of the 30,000 units needed of Product A (30,000 × 3 hours per unit).
With the 70,000 remaining machine hours, it can manufacture 14,000 units of Product B
(70,000 ÷ 5 hours per unit). The remaining 6,000 units needed of Product B would be
outsourced. The total variable cost would be ($30 × 30,000) + ($48 × 14,000) + ($60 ×
6,000) = $1,932,000.
This is the least cost alternative.
Explanation for Choice A:
This is a comparison between the cost to manufacture versus the cost to purchase
outside. Since the revenue to be received and the $1,600,000 fixed manufacturing costs
will be the same regardless of the decision that is made, those factors are irrelevant to
this decision. In this case, the contribution margin earned per machine hour is also not
relevant. Looking only at the contribution margin per machine hour for each product
provides an incomplete analysis since it does not include comparative information on
the cost to outsource. We need to find the least cost alternative for acquiring or
producing this order.
If Lark chooses this alternative, its total variable cost for the order will be $2,010,000, as
follows:
20,000 units of A × 3 hours per unit = 60,000 hours, leaving 100,000 hours (160,000
hours − 60,000 hours) available, which could be used to manufacture all 20,000 units of
B required (100,000 hours ÷ 5 hours per unit). The total variable cost of the products
manufactured will be (20,000 × $30) + (20,000 × $48) = $1,560,000.
Outsourced units will be 10,000 units of A at a cost of $45 per unit, or $450,000.
Total variable cost will be $1,560,000 + $450,000 = $2,010,000.
This is not the lowest-cost alternative.
Explanation for Choice B:
This is a comparison between the cost to manufacture versus the cost to purchase
outside. Since the revenue to be received and the $1,600,000 fixed manufacturing costs
will be the same regardless of the decision that is made, those factors are irrelevant to
this decision. In this case, the contribution margin earned per machine hour is also not
Hock P2 2020
Section C - Decision Analysis.
Answers
relevant. Looking only at the contribution margin per machine hour for each product
provides an incomplete analysis since it does not include comparative information on
the cost to outsource. We need to find the least cost alternative for acquiring or
producing this order.
If Lark chooses this alternative, its total variable cost for the order will be $1,971,000, as
follows:
25,000 units of A × 3 hours per unit = 75,000 hours, leaving 85,000 hours (160,000
hours − 75,000 hours) available, which could be used to manufacture 17,000 units of B
(85,000 hours ÷ 5 hours per unit). The total variable cost of the products manufactured
will be (25,000 × $30) + (17,000 × $48) = $1,566,000.
Outsourced units will be 5,000 units of A at a cost of $45 per unit and 3,000 units of B at
a cost of $60 per unit, or (5,000 × $45) + (3,000 × $60) = $405,000.
Total variable cost will be $1,566,000 + $405,000 = $1,971,000.
This is not the lowest-cost alternative.
Explanation for Choice C:
This is a comparison between the cost to manufacture versus the cost to purchase
outside. Since the revenue to be received and the $1,600,000 fixed manufacturing costs
will be the same regardless of the decision that is made, those factors are irrelevant to
this decision. In this case, the contribution margin earned per machine hour is also not
relevant. Looking only at the contribution margin per machine hour for each product
provides an incomplete analysis since it does not include comparative information on
the cost to outsource or, for this answer choice, the cost to rent additional capacity. We
need to find the least cost alternative for acquiring or producing this order.
If Lark chooses this alternative, the company will be able to manufacture the whole
order in-house. Its total variable and incremental fixed cost for the order will be
$2,010,000, as follows:
The total variable cost of the products manufactured will be (30,000 × $30) + (20,000 ×
$48) = $1,860,000.
Incremental fixed cost for the rented capacity will be $150,000.
Total variable and incremental fixed cost will be $1,860,000 + $150,000 = $2,010,000.
This is not the lowest-cost alternative.
183. Question ID: ICMA 19.P2.010 (Topic: Make or Buy Decisions)
A company currently manufactures subcomponent XT9, a part in the company’s main
product ZL10. Management has found an outside supplier that could sell the company
100,000 XT9 subcomponents next year for $45 each. The company’s production of XT9
costs per unit are shown below.
Hock P2 2020
Section C - Decision Analysis.
Answers
Direct materials $17.00
Direct labor 16.00
Variable manufacturing overhead 4.50
Fixed manufacturing overhead 6.00
Total cost per unit $43.50
If the subcomponent is purchased from the outside supplier, all variable production
costs would be eliminated and 70% of the fixed production costs would be eliminated.
Management has found that the space used for the XT9 subcomponent could be used
to produce a new product that would generate $300,000 in net operating income each
year. If the company purchases XT9 from the outside supplier, operating income would
decrease by

 A. $270,000.
 B. $30,000.correct
 C. $630,000.
 D. $330,000.
Question was not answered
Correct Answer Explanation:
The current cost of producing this product is $4,350,000. This needs to be compared to
the total cost if this part is purchased for $45 per unit from an outside supplier.
The cost of purchasing 100,000 units would be $4,500,000. In addition to this, there
would also be $180,000 of fixed costs that would still be incurred because 30% of the
fixed costs would remain. However, the factory space would be able to be used for a
purpose that would provide $300,000 of net operating income. This is a reduction of the
cost of purchasing from an outside supplier because it is a profit that the company will
have only if they purchase from an outside supplier.
Adding these three amounts together, the total cost of purchasing from the outside
supplier is $4,380,000 ($4,500,000 + $180,000 - $300,000).
This is $30,000 more than the current of producing internally.
Explanation for Choice A:
This answer treats both the fixed costs that will remain and the net income that will
come from the alternative use of the factory incorrectly. It adds the net income to the
cost and subtracts the fixed costs that remain. See the correct answer for a full
explanation of this question.
Explanation for Choice C:
Hock P2 2020
Section C - Decision Analysis.
Answers
This answer treats the income received from an alternative use of the factory as an
additional expense. See the correct answer for a full explanation of this question.
Explanation for Choice D:
This answer does not include the $300,000 of operating income that would be received
from an alternative use of the factory space if the company purchased from an outside
supplier. See the correct answer for a full explanation of this question.
184. Question ID: CMA 686 5.27 (Topic: Make or Buy Decisions)
Stewart Industries has been producing two bearings, components B12 and B18, for use
in production.

B12 B18
Machine hours required per unit 2.5 3.0
Standard cost per unit:
Direct material $ 2.25 $ 3.75
Direct labor 4.00 4.50
Manufacturing overhead:
Variable (See Note 1) 2.00 2.25
Fixed (See Note 2) 3.75 4.50
$12.00 $15.00
Stewart's annual requirement for these components is 8,000 units of B12 and 11,000
units of B18. Recently, Stewart's management decided to devote additional machine
time to other product lines resulting in only 41,000 machine hours per year that can be
dedicated to the production of the bearings. An outside company has offered to sell
Stewart the annual supply of the bearings at prices of $11.25 for B12 and $13.50 for
B18. Stewart wants to schedule the otherwise idle 41,000 machine hours to produce
bearings so that the company can minimize its costs (maximize its net benefits).
Note 1. Variable manufacturing overhead is applied on the basis of direct labor hours.
Note 2. Fixed manufacturing overhead is applied on the basis of machine hours.
Stewart will maximize its net benefits by

 A. Purchasing 8,000 units of B12 and manufacturing 11,000 units of B18.


 B. Purchasing 11,000 units of B18 and manufacturing 8,000 units of B12.
 C. Purchasing 4,800 units of B12 and manufacturing the remaining bearings.
 D. Purchasing 4,000 units of B18 and manufacturing the remaining bearings.correct
Question was not answered
Hock P2 2020
Section C - Decision Analysis.
Answers
Correct Answer Explanation:
Whenever a resource is constrained, as the machine hours are in this problem, we need
to work with cost or contribution margin per unit of the constrained
resource, whichever is appropriate. Here, the "cost" is the amount that the outside
supplier will charge over and above the total variable cost to produce the component in-
house. And the "per unit of the constrained resource" is per machine hours. For this
problem, we need to find the increased cost per machine hour for each of the products
if the units are purchased from the outside supplier. The product with the lower
increased cost per machine hour is the one that should be purchased outside, and the
other product should be manufactured.
The first thing needed is to determine the loss per unit of the constrained resource if
each product is purchased outside instead of being manufactured in-house. To do this,
we total all of the variable costs per unit ($2.25 + $4.00 + $2.00 = $8.25 for B12, and
$3.75 + $4.50 + $2.25 = $10.50 for B18). We then subtract the quoted price from the
supplier for each of the components from their variable costs to produce in-house. This
gives us $8.25 − $11.25, or $(3.00) for B12, and $10.50 − $13.50, or $(3.00) for B18.
This is the increased cost or loss per component to purchase the parts from the outside
supplier. Then, we divide each of those total costs per component by the number of
machine hours required to produce each product to calculate the cost per machine
hour, which will be the cost per unit of the constrained resource.
By purchasing B12 outside, the company's cost will increase by $3.00 per unit, or $1.20
per machine hour ($3.00 ÷ 2.5 machine hours per unit). By purchasing B18 outside, the
cost will increase by $3.00 per unit, or $1.00 per machine hour ($3.00 ÷ 3.0 machine
hours per unit). Since the loss per machine hour (the constrained resource) is greater to
purchase B12, the company would be better off to manufacture all 8,000 units needed
of B12. With the constraint of 41,000 machine hours, the company would have 21,000
hours that would be available to produce B18 (8,000 units of B12 × 2.5 machine hours
per unit = 20,000 hours. 41,000 − 20,000 = 21,000 hours remaining). Based on the
21,000 hours of available machine hours left, the company would be able to produce
7,000 units of B18 (21,000 ÷ 3 machine hours per unit = 7,000 units). The remaining
units, 11,000 needed minus 7,000 produced, or 4,000 units, would need to be
purchased outside.
Explanation for Choice A:
This is not the most cost effective method available to the company. Not all available
machine hours would be used in the manufacturing process.
Explanation for Choice B:
After manufacturing 8,000 units of B12 the company would still have enough machine
hours available to produce some B18.
Explanation for Choice C:
Hock P2 2020
Section C - Decision Analysis.
Answers
This is not the most cost effective method available to the company. The company will
be paying $3.00 more per unit to manufacture B12 units ($11.25 − $8.25 = $3.00).
185. Question ID: ICMA 10.P2.244 (Topic: Make or Buy Decisions)
Refrigerator Company manufactures ice-makers for installation in refrigerators. The
costs per unit, for 20,000 units of ice-makers, are as follows:

Direct materials $ 7
Direct labor 12
Variable overhead 5
Fixed overhead 10
Total costs $34
Cool Compartments Inc. has offered to sell 20,000 ice-makers to Refrigerator Company
for $28 per unit. If Refrigerator accepts Cool Compartments' offer, the facilities used to
manufacture ice-makers could be used to produce water filtration units. Revenues from
the sale of water filtration units are estimated at $80,000, with variable costs amounting
to 60% of sales. In addition, $6 per unit of the fixed overhead associated with the
manufacture of ice-makers could be eliminated.
For Refrigerator Company to determine the most appropriate action to take in this
situation, the total relevant costs of make vs. buy, respectively, are

 A. $680,000 vs. $440,000.


 B. $648,000 vs. $528,000.
 C. $600,000 vs. $528,000.correct
 D. $600,000 vs. $560,000.
Question was not answered
Correct Answer Explanation:
This is a differential analysis. In a differential analysis, we look only at costs that will be
different between (or among, if more than one option is being considered) the options.
If Refrigerator Company manufactures the ice-makers, its differential costs (costs that
will differ from the costs that they would incur if they buy the ice makers instead) will
include the variable costs to manufacture the ice makers and the fixed costs that will be
different because they have chosen to manufacture the ice makers. Those fixed costs
are the avoidable fixed costs, i.e., the fixed costs that would not exist if they choose to
buy the ice makers instead. The problem tells us that if they buy the units, $6 per unit of
the fixed overhead associated with the manufacture of ice makers would be eliminated,
so that is the avoidable fixed cost.
The differential cost to make the part is:
Hock P2 2020
Section C - Decision Analysis.
Answers
Variable cost: $24 × 20,000 $480,000
Avoidable fixed cost: $6 × 20,000 120,000
Total $600,000
If Refrigerator Company buys the ice makers, its differential cost will be the cost to buy
the part reduced by any contribution margin that they can earn from manufacturing and
selling the filtration units instead.
The differential cost to buy the part is:

Variable cost: $28 × 20,000 $560,000


Less: Contribution margin from sales of filtration units, $80,000 × (1 − 0.60) ( 32,000)
Total $528,000
Explanation for Choice A:
$680,000 includes $10 per unit in fixed costs for producing the ice-makers. Only $6 of
the fixed cost is avoidable if Refrigerator Company does not manufacture the ice-
makers. Therefore, the other $4 of the fixed cost is irrelevant because it will be there
regardless of which decision is made. $440,000 is the variable cost to purchase the ice-
makers minus the avoidable fixed cost of $6 × 20,000, or $120,000. Thus, the avoidable
fixed cost has been added to the cost to manufacture (along with the irrelevant fixed
cost) and also subtracted from the cost to purchase. It cannot be used as an adjustment
to both.
Furthermore, this answer does not consider the contribution margin that can be earned
from manufacturing and selling filtration units if Refrigerator Company buys the ice-
makers from Cool Compartments.
Explanation for Choice B:
This answer correctly subtracts the contribution margin that could be earned from the
sales of the filtration units from the variable cost to buy the ice-makers in calculating the
differential cost to buy them. However, in calculating the differential cost of producing
the ice-makers, the variable cost to produce the filtration units ($80,000 × 0.60) is added
to the variable cost and the avoidable fixed cost of producing the ice-makers, and that is
incorrect.
Explanation for Choice D:
This answer does not consider the contribution margin that can be earned from
manufacturing and selling filtration units if Refrigerator Company buys the ice-makers
from Cool Compartments.
186. Question ID: CMA 691 4.14 (Topic: Make or Buy Decisions)
Hock P2 2020
Section C - Decision Analysis.
Answers
Regis Company manufactures plugs used in its manufacturing cycle at a cost of $36 per
unit that includes $8 of fixed overhead. Regis needs 30,000 of these plugs annually,
and Orlan Company has offered to sell these units to Regis at $33 per unit. If Regis
decides to purchase the plugs, $60,000 of the annual fixed overhead applied will be
eliminated, and the company may be able to rent the facility previously used for
manufacturing the plugs.
If Regis Company purchases the plugs but does not rent the unused facility, the
company would

 A. Lose $6.00 per unit.


 B. Lose $3.00 per unit.correct
 C. Save $2.00 per unit.
 D. Save $3.00 per unit.
Question was not answered
Correct Answer Explanation:
The variable cost of plugs is $28 per unit ($36 − $8). Ordinarily, this should be the most
Regis would be willing to pay to an outside supplier, because usually fixed costs remain
the same whether a given item is manufactured or not. However, this question states
that $60,000 in annual fixed overhead would be saved by outsourcing the item, which is
equal to a savings of $2 per unit ($60,000 ÷ 30,000 units). Taking this into account, the
most that Regis would then be willing to pay is $30 per unit, because the avoidable cost
is $30 per unit ($28 + $2). By purchasing the units from Orlan at $33, Regis would lose
$3 per unit ($30 − $33).
Explanation for Choice A:
This is the total fixed cost that are unavoidable [$8.00 − ($60,000 ÷ 30,000)].
Explanation for Choice C:
This is the fixed cost that will be avoidable by purchasing from Orlan ($60,000 ÷
30,000).
Explanation for Choice D:
This answer is the difference between the manufacturing cost of $36 and the supplier's
cost of $33.
187. Question ID: CMA 1287 5.30 (Topic: Make or Buy Decisions)
Leland Manufacturing uses 10 units of Part Number KJ37 each month in the production
of radar equipment. The unit cost to manufacture 1 unit of KJ37 is presented below.

Direct materials $ 1,000


Materials handling (20% of direct material cost) 200
Hock P2 2020
Section C - Decision Analysis.
Answers
Direct labor 8,000
Manufacturing overhead (150% of direct labor) 12,000
Total manufacturing cost $21,200
Material handling represents the direct variable costs of the Receiving Department that
are applied to direct materials and purchased components on the basis of their cost.
This is a separate charge in addition to manufacturing overhead. Leland's annual
manufacturing overhead budget is one-third variable and two-thirds fixed. Scott Supply,
one of Leland's reliable vendors, has offered to supply Part Number KJ37 at a unit price
of $15,000.
Assume that Leland Manufacturing does not wish to rent its unused capacity but could
use the idle capacity to manufacture another product that would contribute $52,000 per
month. If Leland elects to manufacture KJ37 in order to maintain quality control,
Leland's opportunity cost is

 A. $18,000.
 B. $(20,000).
 C. Some amount other than those given.
 D. $4,000.correct
Question was not answered
Correct Answer Explanation:
The variable cost per unit to manufacture Part Number KJ37 is:
Direct Materials: $1,000
Material Handling (20% of direct materials, or $1,000 × 0.20): $200
Direct Labor: $8,000
Variable OH (1/3 of $12,000): $4,000
Total: $13,200
The variable cost per unit to purchase Part Number KJ37 is:
Part KJ37, which has now become direct materials: $15,000
Material Handling (20% of direct materials, or $15,000 × 0.20): $3,000
Total: $18,000
The difference is $18,000 − $13,200, or $4,800 per unit. If 10 units of Part Number KJ37
are needed each month, then the total increase in variable expense related to
purchasing KJ37 outside will be $4,800 × 10, or $48,000.
The opportunity cost is the lost benefit for not manufacturing another product. Lost
benefit is calculated by subtracting the additional cost of purchasing the product from
the potential contribution of manufacturing another product. If Leland has the
opportunity to produce another product that would contribute $52,000 per month, then
Hock P2 2020
Section C - Decision Analysis.
Answers
the opportunity cost for continuing to manufacture KJ37 would be $4,000, calculated as
follows: ($52,000 − $48,000).
Explanation for Choice A:
The opportunity cost is the lost benefit for not manufacturing another product. Lost
benefit is calculated by subtracting the additional cost of purchasing the product instead
of manufacturing it from the potential contribution of manufacturing another product.
Explanation for Choice B:
The opportunity cost is the lost benefit for not manufacturing another product. Lost
benefit is calculated by subtracting the additional cost of purchasing the product instead
of manufacturing it from the potential contribution of manufacturing another product.
Explanation for Choice C:
The correct answer is one of the answer choices given.
188. Question ID: CMA 1288 5.15 (Topic: Make or Buy Decisions)
Geary Manufacturing has assembled the data pertaining to two popular products as
follows. Past experience has shown that the fixed manufacturing overhead component
included in the cost per machine hour averages $10. Geary has a policy of filling all
sales orders, even if it means purchasing units from outside suppliers.

Blender Electric Mixer


Direct materials $6 $ 11
Direct labor 4 9
Factory overhead at $16 per hour 16 32
Cost if purchased from an outside supplier 20 38
Annual demand (units) 20,000 28,000
If 50,000 machine hours are available, and Geary Manufacturing desires to follow an
optimal strategy, it should

 A. Produce 20,000 blenders and 15,000 electric mixers, and purchase all other units as
needed.correct
 B. Produce 25,000 electric mixers and purchase all other units as needed.
 C. Purchase all units as needed.
 D. Produce 20,000 blenders and purchase all other units as needed.
Question was not answered
Correct Answer Explanation:
Hock P2 2020
Section C - Decision Analysis.
Answers
In order to find the correct answer, we first need to calculate the necessary machine
hours per unit. The blender is allowed 1 machine hour per unit ($16 overhead ÷ $16 per
hour). The mixer is allowed 2 machine hours per unit ($32 overhead ÷ $16 per hr.).
Based on the information that Geary has 50,000 machine hours available, the company
can produce either 50,000 blenders or 25,000 mixers (or a combination), but some units
of one or the other will need to be purchased.
Next it is necessary to separate the fixed portion of overhead from the variable portion.
The problem tells us that the fixed manufacturing overhead component included in the
overhead cost per machine hour averages $10 per machine hour. Therefore, the
variable portion of overhead for the blender is $6 per unit [($16 × 1) − ($10 × 1)]. The
variable portion of overhead for the mixer is $12 per unit [($16 × 2) − ($10 × 2)].
Therefore, total variable cost for the blender is $16 ($6 DM + $4 DL + $6 VOH) and for
the mixer it is $32 ($11 DM + $9 DL + $12 VOH).
The purchase price of the blender from an outside supplier is $20, so the cost savings
to produce the blender is $4 ($20 purchase price − $16 VC). The cost savings per
machine hour (since machine hours are the constrained resource) would be $4 ($4 cost
savings ÷ 1 machine hour per unit). For the mixer the purchase price from an outside
supplier is $38, so the cost savings to produce the mixer is $6 ($38 purchase price −
$32 VC). The cost savings per machine hour is $3 ($6 cost savings ÷ 2 machine hours
per unit).
In this case, the company will be better off to produce all the necessary blenders first,
since the cost savings per machine hour to produce blenders is $1 more than the cost
savings per machine hour to produce mixers ($4 − $3). The company needs 20,000
blenders per year. Since blenders require 1 machine hour per unit, producing the full
amount needed will require 20,000 machine hours per year. Geary will have 30,000
machine hours available to produce mixers (50,000 total machine hours − 20,000 hours
needed to produce blenders). Since 30,000 machine hours are available to produce
blenders, 15,000 blenders can be produced (30,000 hrs ÷ 2 machine hours per unit).
Therefore, the optimal solution is to produce 20,000 blenders, 15,000 mixers and
purchase the remaining 13,000 mixers needed.
Explanation for Choice B:
This is not the most optimal strategy. See the correct answer for a full explanation.
Explanation for Choice C:
This is not the most optimal strategy. See the correct answer for a full explanation.
Explanation for Choice D:
This is not the most optimal strategy. See the correct answer for a full explanation.
189. Question ID: ICMA 10.P2.243 (Topic: Make or Buy Decisions)
Hock P2 2020
Section C - Decision Analysis.
Answers
Synergy Inc. produces a component that is popular in many refrigeration systems. Data
on three of the five different models of this component are as follows:

Model
A B C
Volume needed (units) 5,000 6,000 3,000
Manufacturing costs:
Variable direct costs $10 $24 $20
Variable overhead 5 10 15
Fixed overhead 11 20 17
Total manufacturing costs $26 $54 $52

Cost if purchased $21 $42 $39


Synergy applies variable overhead on the basis of machine hours at the rate of $2.50
per hour. Models A and B are manufactured in the Freezer Department, which has a
capacity of 28,000 machine processing hours. Which one of the following options
should be recommended by Synergy's management?

 A. The Freezer Department's manufacturing plan should include 5,000 units of Model A
and 4,500 units of Model B.correct
 B. Manufacture all three products in the quantities required.
 C. The Freezer Department's manufacturing plan should include 2,000 units of Model A
and 6,000 units of Model B.
 D. Purchase all three products in the quantities required.
Question was not answered
Correct Answer Explanation:
We do not have sales price information, so we cannot use that to develop a contribution
margin or a contribution margin per unit of the constrained resource (machine hours).
However, the decision is between manufacturing products internally or buying them
outside. The sales price would be the same whether the products are manufactured
internally or purchased. Therefore, the sales price is really not relevant to this decision,
because it would be no different between alternatives.
This analysis can be done by comparing the cost per machine hour to produce each
product internally with the cost per machine hour to buy the product outside. Even
though products purchased outside would technically have no cost per machine hour,
Hock P2 2020
Section C - Decision Analysis.
Answers
expressing the costs that way makes it possible to compare the cost to buy with the cost
to make.
Since the number of machine hours required for each product is not given, we must
calculate it using the variable overhead application rate given and the amount of
variable overhead applied to each product. Product A gets $5 of overhead applied; and
since overhead is applied at the rate of $2.50 per hour, Product A must require 2
machine hours to produce. Product B gets $10 of overhead applied, so it must require 4
machine hours to produce. We exclude Model C from this, because we are not given
any data on machine hours required to produce it, since it is not manufactured in the
Freezer Department.
Next, calculate the difference in dollars between the variable cost to produce each
product internally and the cost to buy that product outside. (Fixed costs are excluded,
because they will be the same regardless of what is done.) The difference for Model A
is $6 ($21 − $15), and the difference for Model B is $8 ($42 − $34). To calculate the
difference per machine hour, divide the difference in cost for each model by the number
of machine hours required to manufacture it. For Model A, this is $6 ÷ 2, or $3 per
machine hour. For Model B, it is $8 ÷ 4, or $2 per machine hour.
The model with the greater difference in cost per machine hour should be given priority
in manufacturing, because more can be saved by manufacturing it internally. Product
A's difference per machine hour ($3) is greater than Product B's ($2). Therefore, the
company should manufacture all of the 5,000 units needed of Model A first.
Model A requires 2 hours of machine time per unit, so manufacturing the 5,000 units
needed will require 10,000 machine hours. Since the Freezer Department has a
capacity of 28,000 machine hours, that will leave 18,000 machine hours for production
of Model B. 18,000 machine hours ÷ 4 machine hours per unit to manufacture Model B
equals 4,500 units of Model B to be manufactured.
Explanation for Choice B:
The Freezer Department, the only department for which we have information on
available processing capacity, does not have enough capacity to manufacture all three
products in the quantities required. Nor does the Freezere Department have enough
capacity to manufacture the two models manufactured in that department in the
quantities required.
Explanation for Choice C:
This is not the most economical decision.
This analysis can be done by comparing the cost per machine hour to produce each
product internally with the cost per machine hour to buy the product outside. Even
though products purchased outside would technically have no cost per machine hour,
expressing the costs that way makes it possible to compare the cost to buy with the cost
to make.
Hock P2 2020
Section C - Decision Analysis.
Answers
Explanation for Choice D:
Purchasing all three products would be more costly than manufacturing some of the
required number of units internally, because fixed overhead would still be incurred, even
if nothing were being manufactured.
190. Question ID: CMA 1296 4.22 (Topic: Make or Buy Decisions)
In a make versus buy decision, the relevant costs include variable manufacturing costs
as well as

 A. depreciation costs.
 B. avoidable fixed costs.correct
 C. general office costs.
 D. factory management costs.
Question was not answered
Correct Answer Explanation:
In the decision to make-versus-buy decision you want to look only at relevant costs that
differ among the options. Relevant costs include both avoidable fixed and variable
costs.
Explanation for Choice A:
Depreciation costs will generally not differ among the options, therefore they are not
relevant.
Explanation for Choice C:
General office costs will generally not differ among the options, therefore they are not
relevant.
Explanation for Choice D:
Factory management costs will generally not differ among the options, therefore they
are not relevant.
191. Question ID: CMA 1290 4.10 (Topic: Make or Buy Decisions)
Costs relevant to a make-or-buy decision include variable manufacturing costs as well
as

 A. Factory depreciation.
 B. Property taxes.
 C. Avoidable fixed costs.correct
 D. Unavoidable costs.
Question was not answered
Hock P2 2020
Section C - Decision Analysis.
Answers
Correct Answer Explanation:
Relevant costs are those costs that differ among related choices. Fixed costs that would
be avoidable if a decision is made to buy are relevant to a make-or-buy decision.
Explanation for Choice A:
Factory depreciation is a fixed cost and is not relevant, unless it is avoidable.
Explanation for Choice B:
Property taxes are fixed costs, so therefore, not relevant to the decision.
Explanation for Choice D:
Unavoidable costs will continue regardless of the decision to make-or-buy, so therefore,
these costs are not relevant to the decision.
192. Question ID: CMA 696 4.21 (Topic: Make or Buy Decisions)
Costs relevant to a make-or-buy decision include variable labor and variable materials
as well as

 A. depreciation.
 B. unavoidable fixed costs.
 C. avoidable fixed costs.correct
 D. property taxes.
Question was not answered
Correct Answer Explanation:
Relevant costs are those costs that differ among related choices. Thus, the only costs
that are relevant in a make-or-buy decision would include variable labor, variable
materials and avoidable fixed costs.
Explanation for Choice A:
Depreciation expense is fixed and will continue regardless of the decision to make-or-
buy, so therefore is not relevant.
Explanation for Choice B:
Unavoidable fixed costs will continue regardless of the decision to make-or-buy, so
therefore, not relevant.
Explanation for Choice D:
Property taxes are fixed and will continue regardless of the decision to make-or-buy.
193. Question ID: CMA 1292 4.4 (Topic: Make or Buy Decisions)
Richardson Motors uses 10 units of Part No. T305 each month in the production of large
diesel engines. The cost to manufacture one unit of T305 is presented as follows.
Hock P2 2020
Section C - Decision Analysis.
Answers
Direct materials $ 2,000
Materials handling (20% of direct materials cost) 400
Direct labor 16,000
Manufacturing overhead (150% of direct labor) 24,000
Total manufacturing cost $42,400
Materials handling, which is not included in manufacturing overhead, represents the
direct variable costs of the receiving department that are applied to direct materials and
purchased components on the basis of their cost. Richardson's annual manufacturing
overhead budget is one-third variable and two-thirds fixed. Simpson Castings, one of
Richardson's reliable vendors, has offered to supply T305 at a unit price of $30,000.
Assume Richardson Motors is able to rent all idle capacity for $50,000 per month. If
Richardson decides to purchase the 10 units from Simpson Castings, Richardson's
monthly cost for T305 would

 A. Decrease $14,000.
 B. Increase $46,000.correct
 C. Increase $96,000.
 D. Decrease $64,000.
Question was not answered
Correct Answer Explanation:
Total per unit manufacturing cost for the T305 is $42,400. Since the overhead of
$24,000 is 1/3 variable and 2/3 fixed, $16,000 of that would be fixed and would continue
if Richardson purchases the T305 outside and $8,000 of it would be variable and would
be avoidable. Thus, the total avoidable variable manufacturing cost per unit is $26,400
($2,000 DM, $16,000 DL, $400 materials handling, and $8,000 variable manufacturing
OH).
If Richardson Motors purchases the units from Simpson Castings then the per unit
variable cost will be $36,000 ($30,000 purchase price + $6,000 material handling cost
applied [20% handling cost × $30,000 per unit]. Therefore, by purchasing from Simpson
Castings the per unit cost of the T305 component will increase $9,600 ($36,000 −
$26,400). By purchasing 10 units per month, total monthly cost would increase by
$96,000 ($9,600 × 10). But, if the idle capacity can be rented for $50,000 per month,
then the monthly net cost would be reduced to $46,000 ($96,000 − $50,000).
Explanation for Choice A:
An answer of $14,000 does not include the materials handling cost for the purchased
components.
Hock P2 2020
Section C - Decision Analysis.
Answers
Explanation for Choice C:
This answer overlooks the rent of the idle capacity.
Explanation for Choice D:
An answer of $64,000 is the difference between the total cost to manufacture the T305
and the total cost to purchase the T305. However, it does not consider the fixed
manufacturing cost, which will continue whether or not the T305 is being manufactured.
It also does not include the rental income.
194. Question ID: ICMA 13.P2.045 (Topic: Make or Buy Decisions)
GiantCo has received an offer from PatriotCo to produce units that GiantCo currently
produces and sells. The unit price quoted by PatriotCo is higher than GiantCo’s variable
production cost per unit, but lower than the price at which GiantCo can market the units.
Under which circumstance would GiantCo’s profits increase by purchasing units from
PatriotCo?

 A. GiantCo's fixed overhead would remain the same if GiantCo purchased units from
PatriotCo.
 B. GiantCo's administrative costs are zero.
 C. Market demand for the product exceeds GiantCo's capacity.correct
 D. GiantCo has significant sunk costs.
Question was not answered
Correct Answer Explanation:
If the demand for the product exceeds GiantCo’s capacity, GiantCo’s profits would
increase by purchasing from PatriotCo the excess units it could not produce internally.
Even though the contribution margin per unit would be lower than the contribution
margin per unit for internally manufactured units, each unit purchased from PatriotCo
and sold would increase GiantCo’s net before-tax profits by its contribution margin.
Since there would be no additional fixed costs associated with purchasing units from
PatriotCo, all of the increase in the contribution resulting from the increased sales would
flow to profit before tax.
Explanation for Choice A:
Whether or not GiantCo's fixed overhead remains the same, if GiantCo continues to
produce the units interally at its same production level and is not able to sell the
additional units purchased from PatriotCo, GiantCo's profits would not increase as a
result of purchasing units from PatriotCo. The only result would be an increase in
inventory.
If GiantCo replaces their own internal production with units purchased from PatriotCo.,
Giantco's profits would decrease because the price from PatriotCo. Is higher than the
variable cost to produce the units internally.
Hock P2 2020
Section C - Decision Analysis.
Answers
Explanation for Choice B:
The amount of GiantCo's administrative costs is not relevant to determining whether or
not purchasing units from PatriotCo would result in increased profits.
Explanation for Choice D:
Sunk costs are costs that have already been incurred, and they cannot be changed by
any decision made today. Thus the existence of significant sunk costs is not relevant to
determining whether or not purchasing units from PatriotCo would result in increased
profits.
195. Question ID: ICMA 10.P2.246 (Topic: Make or Buy Decisions)
Aril Industries is a multiproduct company that currently manufactures 30,000 units of
Part 730 each month for use in production. The facilities now being used to produce
Part 730 have fixed monthly overhead costs of $150,000, and a theoretical capacity to
produce 60,000 units per month. If Aril were to buy Part 730 from an outside supplier,
the facilities would be idle and 40% of fixed costs would continue to be incurred. There
are no alternative uses for the facilities. The variable production costs of Part 730 are
$11 per unit. Fixed overhead is allocated based on planned production levels.
If Aril Industries continues to use 30,000 units of Part 730 each month, it would realize a
net benefit by purchasing Part 730 from an outside supplier only if the supplier's unit
price is less than

 A. $14.00.correct
 B. $13.00.
 C. $12.00.
 D. $12.50.
Question was not answered
Correct Answer Explanation:
The maximum price that Aril should pay to an outside supplier is the amount per unit
that it would save if it did not manufacture the units internally. The variable cost saved
would be $11 per unit. The fixed cost saved would be, in total, $150,000 × 0.60, or
$90,000. On a per unit basis, that would be $90,000 ÷ 30,000 units manufactured each
month, or $3 per unit. So the maximum price Aril should pay an outside supplier is $11
(the variable manufacturing cost saved) plus $3 (the fixed manufacturing cost saved),
which is $14.
Explanation for Choice B:
This is the variable manufacturing cost saved per unit plus $2 for fixed manufacturing
cost saved per unit. The fixed manufacturing cost saved per unit was calculated as
($150,000 × 0.40) ÷ 30,000 units, which is incorrect. 40% of the fixed costs is the
Hock P2 2020
Section C - Decision Analysis.
Answers
amount that would continue to be incurred if Aril were to buy Part 730 from an outside
supplier, not the amount of fixed costs that could be saved.
Explanation for Choice C:
This is the variable manufacturing cost saved per unit plus $1.00 for fixed manufacturing
cost saved per unit. The fixed manufacturing cost saved per unit was calculated as
($150,000 × 0.40) ÷ 60,000 units, the theoretical capacity. This is incorrect for two
reasons: (1) The total fixed manufacturing cost saved should be divided by the number
of units used each month, not the theoretical capacity. (2) 40% of the fixed costs is the
amount that would continue to be incurred if Aril were to buy Part 730 from an outside
supplier, not the amount of fixed costs that could be saved.
Explanation for Choice D:
This is the variable manufacturing cost saved per unit plus $1.50 for fixed manufacturing
cost saved per unit. The fixed manufacturing cost saved per unit was calculated as
($150,000 × 0.60) ÷ 60,000 units, the theoretical capacity. The total fixed manufacturing
cost saved should be divided by the number of units used each month, not the
theoretical capacity.
196. Question ID: CIA 593 IV.19 (Topic: Make or Buy Decisions)
The ABC Company manufactures components for use in producing one of its finished
products. When 12,000 units are produced, the full cost per unit is $35, separated as
follows:

Direct materials $5
Direct labor 15
Variable overhead 10
Fixed overhead 5
The XYZ Company has offered to sell 12,000 components to ABC for $37 each. If ABC
accepts the offer, some of the facilities currently being used to manufacture the
components can be rented as warehouse space for $40,000. However, $3 of the fixed
overhead currently applied to each component would have to be covered by ABC's
other products. What is the differential cost to the ABC Company of purchasing the
components from the XYZ Company?

 A. $24,000
 B. $8,000
 C. $44,000
 D. $20,000correct
Question was not answered
Correct Answer Explanation:
Hock P2 2020
Section C - Decision Analysis.
Answers
The differential (incremental) costs are those costs that differ between the two options.
The net cost of purchasing the component and renting the warehouse space is
$404,000 [($37.00 × 12,000) − $40,000]. The total cost to manufacture is $384,000
[($30.00 variable cost + $2.00 avoidable fixed cost) × 12,000 units]. The net cost
differential is $20,000 ($404,000 − $384,000).
Explanation for Choice A:
This answer is incorrect because it is simply taking the cost differential between the full
manufacturing cost ($35 × 12,000 = $420,000) and the cost to purchase ($37 × 12,000
= $444,000), ignoring the contribution from rental of the warehouse and the avoidable
fixed overhead cost.
Explanation for Choice B:
This answer results from using $3 of fixed overhead as the avoidable cost that would
not be incurred if XYZ buys the component instead of manufacturing it. However, the
question says that of the $5 of fixed overhead currently being applied to each unit of the
component manufactured, $3 would have to be covered by ABC's other products. That
means that $3 of the $5 cost is unavoidable, and the other $2 is avoidable. It is the $2
avoidable cost that is relevant to this decision, because that amount would not be
incurred if ABC buys the component instead of manufacturing it. The other $3 that is
unavoidable will be the same regardless of which option is chosen, so it is not relevant.
Explanation for Choice C:
The question says, "$3 of the [$5] fixed overhead currently applied to each component
would have to be covered by ABC's other products." That means that $2 of the fixed
overhead being applied to the components ($5 − $3) can be avoided if the XYZ buys the
components instead. This answer ignores the $2 of avoidable fixed overhead cost.
197. Question ID: CMA 1292 4.5 (Topic: Make or Buy Decisions)
Richardson Motors uses 10 units of Part No. T305 each month in the production of large
diesel engines. The cost to manufacture one unit of T305 is presented as follows.

Direct materials $ 2,000


Materials handling (20% of direct materials cost) 400
Direct labor 16,000
Manufacturing overhead (150% of direct labor) 24,000
Total manufacturing cost $42,400
Materials handling, which is not included in manufacturing overhead, represents the
direct variable costs of the receiving department that are applied to direct materials and
purchased components on the basis of their cost. Richardson's annual manufacturing
overhead budget is one-third variable and two-thirds fixed. Simpson Castings, one of
Richardson's reliable vendors, has offered to supply T305 at a unit price of $30,000.
Hock P2 2020
Section C - Decision Analysis.
Answers
Assume Richardson Motors could use the idle capacity to manufacture another product
that would contribute $104,000 per month. If Richardson chooses to manufacture the
ten T305 units in order to maintain quality control, Richardson's opportunity cost is

 A. $88,000.
 B. $8,000.correct
 C. $68,000.
 D. $(96,000).
Question was not answered
Correct Answer Explanation:
Total per unit manufacturing cost for the T305 is $42,400. Since the overhead of
$24,000 is 1/3 variable and 2/3 fixed, $16,000 of that would continue and be
unavoidable. Thus, the avoidable variable manufacturing cost per unit is $26,400
($2,000 direct materials + $400 materials handling + $16,000 direct labor + $8,000
variable overhead).
If Richardson Motors purchases the units from Simpson Castings then the per unit
variable cost will be $36,000 ($30,000 purchase price + $6,000 material handling cost
applied [20% × $30,000 per unit]. Therefore, by purchasing from Simpson Castings the
per unit cost of the T305 component will increase $9,600 ($36,000 − $26,400). By
purchasing 10 units per month, total monthly cost would increase by $96,000 ($9,600 ×
10).
If Richardson Motors could manufacture another product that would contribute $104,000
per month, then the opportunity cost to continue manufacturing the part in-house would
be $8,000 ($104,000 − $96,000).
Explanation for Choice A:
This answer uses a variable overhead amount of 2/3 of the total overhead to calculate
the avoidable variable manufacturing expense, instead of 1/3.
Explanation for Choice C:
This answer fails to include the materials handling cost in the calculation of the net cost
to purchase the T305 from an outside supplier.
Explanation for Choice D:
This is the additional cost to purchase the units outside. However, this answer ignores
the rental opportunity of $104,000.
198. Question ID: ICMA 10.P2.265 (Topic: Make or Buy Decisions)
Aspen Company plans to sell 12,000 units of product XT and 8,000 units of product RP.
Aspen has a capacity of 12,000 productive machine hours. The unit cost structure and
machine hours required for each product is as follows.
Hock P2 2020
Section C - Decision Analysis.
Answers
Unit Costs XT RP
Materials $37 $24
Direct labor 12 13
Variable overhead 6 3
Fixed overhead 37 38

Machine hours required 1.0 1.5


Aspen can purchase 12,000 units of XT at $60 and/or 8,000 units of RP at $45. Based
on the above, which one of the following actions should be recommended to Aspen's
management?

 A. Produce XT internally and purchase RP.correct


 B. Purchase both XT and RP.
 C. Produce both XT and RP.
 D. Produce RP internally and purchase XT.
Question was not answered
Correct Answer Explanation:
Usually with a problem like this, we calculate the contribution margin per machine hour,
and the product with the highest contribution margin per machine hour required to
produce it is the one we should produce first with whatever available production
capacity we have. Here, we don't have any sales prices, so we can't calculate any
contribution margins. However, we do have the cost to purchase each unit externally.
So we can use the difference between the cost to purchase externally and the variable
cost to produce internally in place of the contribution margin, because the cost to
purchase externally can be looked at as the amount saved by producing the units
internally. Furthermore, the sales price will be the same regardless of how the product is
obtained, so revenue is really irrelevant because it does not differ between the
alternatives.
The cost to purchase XT externally is $60, and the variable cost to produce it internally
is $55. So the savings from producing it internally are $5 per unit, and that $5 divided by
the number of machine hours required to produce one unit (1 hour) is the savings per
machine hour, which is $5.
The cost to purchase RP externally is $45, and the variable cost to produce it internally
is $40. So the savings from producing it internally are also $5 per unit. But it takes 1.5
hours to produce one unit of RP internally. Therefore, the savings per machine hour are
$5 divided by 1.5, or $3.333.
Hock P2 2020
Section C - Decision Analysis.
Answers
Since the savings per machine hour for producing XT internally are $5 and the savings
per machine hour for producing RP internally are only $3.33, the company should give
priority to producing XT internally. The company has 12,000 machine hours available. It
requires 12,000 units of XT, and each unit of XT requires 1 machine hour. Therefore,
the company should manufacture all 12,000 units of XT that it requires using its 12,000
available machine hours. It will then need to purchase all of the RP it needs externally.
Explanation for Choice B:
This is not the most cost-effective solution. See correct answer for an explanation.
Explanation for Choice C:
This is not the most cost-effective solution. See correct answer for an explanation.
Explanation for Choice D:
This is not the most cost-effective solution. See correct answer for an explanation.
199. Question ID: CMA 687 5.20 (Topic: Make or Buy Decisions)
Cohasset Company currently manufactures all component parts used in the
manufacture of various hand tools. A handle is used in three different tools. The unit
cost budget for 20,000 handles is

Direct material $ 0.60


Direct labor 0.40
Variable overhead 0.10
Fixed overhead 0.20
Total unit cost $1.30
R&M Steel has offered to supply 20,000 handles to Cohasset for $1.25 each, delivered.
If Cohasset currently has idle capacity that cannot be used, accepting the offer will

 A. Decrease the handle unit cost by $0.05.


 B. Increase the handle unit cost by $0.15.correct
 C. Increase the handle unit cost by $0.05.
 D. Decrease the handle unit cost by $0.15.
Question was not answered
Correct Answer Explanation:
Since the company has idle capacity, the company would be better off producing the
handles as long as the supplier's price is above total variable cost ($1.10). If Cohasset's
price is $1.25, then buying the handles would increase the handle cost by $0.15 per
handle ($1.25 − $1.10).
Hock P2 2020
Section C - Decision Analysis.
Answers
Explanation for Choice A:
This answer results from including fixed overhead as an avoidable cost. There is no
indication that any of the fixed overhead could be avoided if the company buys the
handles outside. Therefore, the only avoidable costs the company has are the variable
costs. Because the company has idle capacity, it is better off manufacturing the handles
unless the supplier's cost is below R&M's variable costs, which it is not.
Explanation for Choice C:
This answer results from including fixed overhead as an avoidable cost and not
including variable overhead as an avoidable cost. Variable overhead is an avoidable
cost while fixed over head is not.
Explanation for Choice D:
This answer results from including fixed overhead as an avoidable cost. There is no
indication that any of the fixed overhead could be avoided if the company buys the
handles outside. Therefore, the only avoidable costs the company has are the variable
costs. Because the company has idle capacity, it is better off manufacturing the handles
unless the supplier's cost is below R&M's variable costs, which it is not.
200. Question ID: CMA 686 5.28 (Topic: Make or Buy Decisions)
Stewart Industries has been producing two bearings, components B12 and B18, for use
in production.

B12 B18
Machine hours required per unit 2.5 3.0
Standard cost per unit:
Direct material $ 2.25 $ 3.75
Direct labor 4.00 4.50
Manufacturing overhead:
Variable (See Note 1) 2.00 2.25
Fixed (See Note 2) 3.75 4.50
$12.00 $15.00
Stewart's annual requirement for these components is 8,000 units of B12 and 11,000
units of B18. Recently, Stewart's management decided to devote additional machine
time to other product lines resulting in only 41,000 machine hours per year that can be
dedicated to the production of the bearings. An outside company has offered to sell
Stewart the annual supply of the bearings at prices of $11.25 for B12 and $13.50 for
B18. Stewart wants to schedule the otherwise idle 41,000 machine hours to produce
bearings so that the company can minimize its costs (maximize its net benefits).
Hock P2 2020
Section C - Decision Analysis.
Answers
Note 1. Variable manufacturing overhead is applied on the basis of direct labor hours.
Note 2. Fixed manufacturing overhead is applied on the basis of machine hours.
Assume that Stewart's idle capacity of 41,000 machine hours has a traceable
unavoidable annual fixed cost of $44,000 that will continue if the capacity is not used.
The maximum price Stewart would be willing to pay a supplier for component B18 is

 A. $10.50.
 B. $14.00.
 C. Some amount other than those given.correct
 D. $14.50.
Question was not answered
Correct Answer Explanation:
The first thing needed is to determine which component should receive priority on the
limited resource of machine hours. To do this, we need to calculate the loss per unit of
the constrained resource if B12 is purchased outside versus being manufactured in-
house, and the loss per unit of the constrained resource if B18 is purchased outside
versus being manufactured in-house. To do this, we total all of the variable costs per
unit ($2.25 + $4.00 + $2.00 = $8.25 for B12, and $3.75 + $4.50 + $2.25 = $10.50 for
B18). We then subtract the quoted price from the supplier for each of the components
from their variable costs to produce in-house. This gives us $8.25 − $11.25, or $(3.00)
for B12, and $10.50 − $13.50, or $(3.00) for B18. The cost per component to purchase
the parts from the outside supplier is $3.00 for both components. Then, we divide each
of those costs per component by the number of machine hours required to produce
each product in-house. The result is the cost per machine hour for each component,
which is the cost per unit of the constrained resource for each component. This gives us
$(3.00) ÷ 2.5, or $(1.20) for B12 and $(3.00) ÷ 3.0, or $(1.00) for B18.
Since the cost per unit of the constrained resource (machine hours) is higher to
purchase B12 outside than it is to purchase B18 outside ($1.20 for B12 versus $1.00 for
B18), Stewart should produce all of the B12 that it needs in-house and as much B18 as
it can produce in the remaining machine time. The amount that it needs of B18 which it
cannot produce itself should be purchased outside. This will minimize the additional cost
that will result from having to purchase some of its components from an outside
supplier.
This will mean producing 8,000 units of B12, requiring 2.5 machine hours each for a
total of 20,000 hours; and in the 21,000 hours left, producing 21,000 ÷ 3, or 7,000 units
of B18; and purchasing the remainder of the B18 units needed.
Since the $44,000 in fixed costs is unavoidable and will continue whether the equipment
is used or not, this fixed cost is not relevant to this decision and should not be included
in any of the calculations.
Hock P2 2020
Section C - Decision Analysis.
Answers
This question asks what is the maximum amount Stewart would be willing to pay an
outside supplier for B18. The alternative to buying some of its requirements of B18
outside is buying some of its requirements of B12 outside. If the additional cost to
purchase B18 outside were to increase above the additional cost to purchase B12
outside, then Stewart would choose to purchase the B12 outside instead. Thus, we
need to consider the additional cost per machine hour that Stewart would have to pay
for the B12 to be the upper limit for additional cost per machine hour to pay for the B18,
because at a price for B18 that is above that additional cost, Stewart will purchase the
B12 outside instead.
Since B18 requires 3 hours of machine time to produce one unit, the maximum amount
that Stewart would pay an outside supplier for B18 would include $1.20 × 3, or $3.60
(the loss per machine hour to produce one unit of B12 × the number of hours required to
produce one unit of B18). Therefore, the total cost that Stewart would pay an outside
supplier for B18 would be $10.50 (the company's variable cost per unit) plus a $3.60
cost to purchase a unit of B12 outside, for a total of $14.10.
Explanation for Choice A:
This is the total variable cost per unit ($3.75 + $4.50 + $2.25). If the company had idle
capacity and could produce all it needed, this would be the most it would pay to an
outside supplier. However, Stewart does not have adequate capacity to produce all that
it needs, and so the company must obtain some of its components from an outside
supplier.
This question asks what is the maximum amount Stewart would be willing to pay an
outside supplier for B18. The alternative to buying some of its requirements of B18
outside is buying some of its requirements of B12 outside. If the additional cost to
purchase B18 outside were to increase above the additional cost to purchase B12
outside, then Stewart would choose to purchase the B12 outside instead. Thus, we
need to consider the additional cost per machine hour that Stewart would have to pay
for the B12 to be the upper limit for additional cost per machine hour to pay for the B18,
because at a price for B18 that is above that additional cost, Stewart will purchase the
B12 outside instead.
Explanation for Choice B:
This is not the correct answer. Please see the correct answer for an explanation.
We have been unable to determine how to calculate this incorrect answer choice. If you
have calculated it, please let us know how you did it so we can create a full explanation
of why this answer choice is incorrect. Please send us an email at
support@hockinternational.com. Include the full Question ID number and the actual
incorrect answer choice -- not its letter, because that can change with every study
session created. The Question ID number appears at the top of the question. Thank you
in advance for helping us to make your HOCK study materials better.
Explanation for Choice D:
Hock P2 2020
Section C - Decision Analysis.
Answers
This is the variable cost of $10.50 for B18 plus the unavoidable fixed cost of $44,000
divided by Stewart's annual requirement of 11,000 units of B18. Since the $44,000 in
fixed cost is unavoidable and will continue whether the equipment is used or not, this
fixed cost is not relevant to this decision and should not be included.
This question asks what is the maximum amount Stewart would be willing to pay an
outside supplier for B18. The alternative to buying some of its requirements of B18
outside is buying some of its requirements of B12 outside. If the additional cost to
purchase B18 outside were to increase above the additional cost to purchase B12
outside, then Stewart would choose to purchase the B12 outside instead. Thus, we
need to consider the additional cost per machine hour that Stewart would have to pay
for the B12 to be the upper limit for additional cost per machine hour to pay for the B18,
because at a price for B18 that is above that additional cost, Stewart will purchase the
B12 outside instead.
201. Question ID: CMA 696 4.11 (Topic: Make or Buy Decisions)
Listed below are a company's monthly unit costs to manufacture and market a particular
product.

Manufacturing costs:
Direct materials $2.00
Direct labor 2.40
Variable indirect 1.60
Fixed indirect 1.00
Marketing costs:
Variable 2.50
Fixed 1.50
The company must decide to continue making the product or buy it from an outside
supplier. The supplier has offered to make the product at the same level of quality that
the company can make it. Fixed marketing costs would be unaffected, but variable
marketing costs would be reduced by 30% if the company were to accept the proposal.
What is the maximum amount per unit that the company can pay the supplier without
decreasing operating income?

 A. $7.75
 B. $5.25
 C. $8.50
 D. $6.75correct
Question was not answered
Hock P2 2020
Section C - Decision Analysis.
Answers
Correct Answer Explanation:
The goal is to find a cost for the part to be purchased externally that makes the total per
unit cost for the externally purchased part no greater than the total per unit cost for the
product purchased internally.
For purposes of this comparison, fixed costs are not relevant because they will be the
same whether the company purchases the product outside or manufactures it internally.
So we look only at variable costs.
The total variable cost to produce the part internally and market it is $8.50 ($2.00 +
$2.40 + $1.60 + $2.50). If the part is purchased outside, the variable marketing cost will
decrease from $2.50 per unit to $1.75 per unit ($2.50 × 0.70, since 30% of the cost is
avoidable).
We need to make sure that the total variable cost to purchase the part externally and
market it are no more than $8.50 per unit. The variable marketing cost associated with
purchasing the part externally would be $1.75, and that would be the only other cost in
addition to the cost for the part. Therefore, the company needs to be able to buy the
part for no more than $8.50 minus $1.75, which is $6.75. If the company pays any more
than $6.75 for the part, its total cost to buy and market the product will be greater than
$8.50 per unit, and that will cause its operating income to decrease.
Another way to look at this is to total the avoidable costs if the company purchases the
part externally. The avoidable costs include direct materials ($2.00), direct labor ($2.40),
variable indirect ($1.60) and $0.75 of variable marketing costs (the difference between
$2.50 and $1.75), for a total of $6.75.
Therefore, the maximum amount per unit that the company would pay an outside
supplier would be $6.75.
Explanation for Choice A:
This is the variable manufacturing costs plus the reduced amount of the variable
marketing costs ($2.50 × [1 − 0.30). The amount added for the variable marketing costs
should be the amount of the reduction in the variable marketing costs. See the correct
answer explanation for a complete explanation.
Explanation for Choice B:
This is the variable manufacturing costs minus the amount of the reduction in the
variable marketing cost ($0.75). The amount of the reduction in the variable marketing
cost needs to be added instead of subtracted. See the correct answer explanation for a
complete explanation.
Explanation for Choice C:
This is the total variable cost to produce and market one unit at present. A cost of $8.50
per unit would result in a decrease in operating income, because if the company pays
an outside supplier $8.50 per unit, the company will still need to pay the variable
Hock P2 2020
Section C - Decision Analysis.
Answers
marketing cost of $1.75 per unit ($2.50 × [1 − 0.30]) separately. The marketing costs
added to this cost of $8.50 per unit would cause the total variable cost per unit to
increase above its present amount. The amount of the revised variable marketing cost
needs to be subtracted from the total variable cost to produce and market one unit at
present in order to allow for its separate payment without increasing total variable cost.
202. Question ID: ICMA 1603.P2.055 (Topic: Marginal Analysis-Other Decisions)
A company has a portfolio of four products and incurs $175,000 of allocated fixed costs
per year. Financial data for the four products are shown below.

Product A Product B Product C Product D


Units sold 25,000 18,750 3,750 2,500
Revenue $750,000 $600,000 $150,000 $100,000
Unit variable costs $24 $24 $37 $41
Which product should the company discontinue?

 A. Product B.
 B. Product A.
 C. Product D.correct
 D. Product C.
Question was not answered
Correct Answer Explanation:
The contribution (revenue less variable costs) generated by each product that can go
toward covering the fixed costs is as follows:

Product A Product B Product C Product D


Units sold 25,000 18,750 3,750 2,500
Revenue $750,000 $600,000 $150,000 $100,000
Unit variable costs $24 $24 $37 $41
Variable costs $600,000 $450,000 $138,750 $102,500
Contribution margin $150,000 $150,000 $ 11,250 $ (2,500)
The product that should be discontinued is Product D because it is not contributing
anything toward covering the fixed costs.
Explanation for Choice A:
Hock P2 2020
Section C - Decision Analysis.
Answers
Product B should not be discontinued. It is contributing $150,000 toward covering the
fixed costs of the company: Revenue of $600,000 less variable costs of (18,750 × $24)
= $150,000.
Explanation for Choice B:
Product A should not be discontinued. It is contributing $150,000 toward covering the
fixed costs of the company: Revenue of $750,000 less variable costs of (25,000 × $24)
= $150,000.
Explanation for Choice D:
Product C should not be discontinued. It is contributing $11,250 toward covering the
fixed costs of the company: Revenue of $150,000 less variable costs of (3,750 × $37) =
$11,250.
203. Question ID: ICMA 1603.P2.034 (Topic: Marginal Analysis-Other Decisions)
A company produces ready-to-bake pie crusts. In deciding whether to process this
product further into complete ready-to-bake pies by adding filling, relevant dollar
amounts to consider would include all of the following except the

 A. selling price of the crusts.


 B. cost to add the filling.
 C. selling price of the complete pies.
 D. cost to manufacture the crusts.correct
Question was not answered
Correct Answer Explanation:
The difference between the additional revenue to be received (by processing the crust
further into ready-to-bake pies) and the additional cost (to add the filling) is relevant to
the decision as to whether to process the product further into complete ready-to-bake
pies by adding filling. The cost to manufacture the crusts is a sunk cost. It cannot be
changed by the decision whether to process the crusts further, so it is not relevant to the
decision.
Explanation for Choice A:
The difference between the additional revenue to be received (by processing the crust
further into ready-to-bake pies) and the additional cost (to add the filling) is relevant to
the decision as to whether to process the product further. Therefore, the selling price of
the crusts is relevant to the decision because it is needed to determine the additional
revenue to be received.
Explanation for Choice B:
The difference between the additional revenue to be received (by processing the crust
further into ready-to-bake pies) and the additional cost (to add the filling) is relevant to
Hock P2 2020
Section C - Decision Analysis.
Answers
the decision as to whether to process the product further. Therefore, the cost to add the
filling is relevant to the decision because it is needed to determine the difference
between the additional revenue to be received and the additional cost.
Explanation for Choice C:
The difference between the additional revenue to be received (by processing the crust
further into ready-to-bake pies) and the additional cost (to add the filling) is relevant to
the decision as to whether to process the product further. Therefore, the selling price of
the complete pies is relevant to the decision because it is needed to determine the
additional revenue to be received.
204. Question ID: ICMA 19.P2.008 (Topic: Marginal Analysis-Other Decisions)
A company’s budget indicated the following cost per unit for the company’s most
popular product.

Variable manufacturing costs $64


Fixed manufacturing overhead 45
Sales commissions 3
Fixed selling and administrative costs 32
Although this product normally sells for $160 per unit, the company received a special
order from a new customer. If the company has idle capacity, its income would increase
by accepting the order if the selling price per unit for the order was greater than

 A. $109.
 B. $67.correct
 C. $64.
 D. $144.
Question was not answered
Correct Answer Explanation:
When a company has the idle capacity to make a special order without having to cancel
any existing sales, the price that they should charge is equal to the variable costs of
filling that order.
In this question, those costs are $64 of variable manufacturing costs and $3 of
commission. This is $67 and this is the minimum price that they should charge.
There is no information in the question that says that the commission will not be paid on
this order, so we must treat it as a variable cost of filling this order.
Explanation for Choice A:
Hock P2 2020
Section C - Decision Analysis.
Answers
This choice incorrectly includes the fixed manufacturing costs in the costs that need to
be covered with the selling price of this special order.
Explanation for Choice C:
This answer does not include the commission as a cost of filling this order.
Explanation for Choice D:
This choice incorrectly includes the fixed manufacturing costs and the fixed selling and
administrative costs in the costs that need to be covered with the selling price of this
special order.
205. Question ID: CMA 1293 4.1 (Topic: Marginal Analysis-Other Decisions)
Copeland, Inc. produces X-547 in a joint manufacturing process. The company is
studying whether to sell X-547 at the split-off point or upgrade the product to become
Xylene. The following information has been gathered.

1. Selling price per pound of X-547.


2. Variable manufacturing costs of the upgrade process.
3. Avoidable fixed costs of the upgrade process.
4. Selling price per pound of Xylene.
5. Joint manufacturing costs to produce X-547.
Which of these items should be reviewed when making the upgrade decision?

 A. 1, 2, 3, 4, and 5.
 B. 1, 2, and 4.
 C. 1, 2, 3, and 4.correct
 D. 1, 2, 4, and 5.
Question was not answered
Correct Answer Explanation:
When reviewing the data in order to make a decision whether to upgrade or not, only
relevant, or incremental revenues and costs are considered. Relevant revenues and
costs are those that will be different between or among the various alternatives being
considered.

1. The selling price per pound of X-547 is relevant because the selling price will be
different if the company decides to upgrade the product to Xylene.
2. Variable manufacturing costs of the upgrade process are relevant, because they
will be different if the company decides to upgrade the product.
3. Avoidable fixed costs of the upgrade process are relevant, because they will be
different if the company decides to upgrade the product.
Hock P2 2020
Section C - Decision Analysis.
Answers
4. Selling price per pound of Xylene is relevant, because the sellling price will be
different if the company decides to upgrade the X-547 to Xylene.
Thus, relevant (incremental) items include 1, 2, 3 and 4. Joint manufacturing costs are
sunk costs, so they are not relevant to the decision.
Explanation for Choice A:
This answer includes joint manufacturing costs, which are sunk costs and are not
relevant to the decision of whether to upgrade the product or not.
Explanation for Choice B:
This answer does not include all of the items that are relevant to the decision whether to
upgrade the product or not.
Explanation for Choice D:
Avoidable fixed costs are relevant to the decision, but sunk costs are not.
206. Question ID: ICMA 10.P2.235 (Topic: Marginal Analysis-Other Decisions)
Johnson Company manufactures a variety of shoes and has received a special one-
time-only order directly from a wholesaler. Johnson has sufficient idle capacity to accept
the special order to manufacture 15,000 pairs of sneakers at a price of $7.50 per pair.
Johnson's normal selling price is $11.50 per pair of sneakers. Variable manufacturing
costs are $5.00 per pair and fixed manufacturing costs are $3.00 a pair. Johnson's
variable selling expense (not including shipping-out expense) to obtain an order for its
normal line of sneakers is $1.00 per pair. What would the effect on Johnson’s operating
income be if the company accepted the special order?

 A. Increase by $37,500.correct
 B. Decrease by $60,000.
 C. Increase by $52,500.
 D. Increase by $22,500.
Question was not answered
Correct Answer Explanation:
The incremental contribution margin per pair of sneakers is $7.50 selling price minus
$5.00 variable manufacturing cost, or $2.50. Fixed manufacturing costs are irrelevant,
because they will be the same whether the order is accepted or not accepted. The
variable selling expense for this special order would not be the same as the variable
selling expense for the company’s normal line of sneakers. The question states that the
$1.00 variable selling expense does not include shipping-out expense. Other typical
variable selling expenses are sales representative’s commissions, advertising, etc. But
in this question it is stated that the order was received directly from the wholesaler, so
those other selling expenses for the company’s normal line of sneakers would not apply.
Hock P2 2020
Section C - Decision Analysis.
Answers
Therefore, the effect on operating income if the company accepts the special order will
be $2.50 × 15,000 pairs, or $37,500.
Explanation for Choice B:
This is the difference between Johnson's normal selling price of $11.50 per pair and the
special order price of $7.50 per pair, multiplied by 15,000 pairs. This answer assumes
the company will have a loss equal to the amount of the discount on each pair of
sneakers because it could have sold them for $11.50 instead of $7.50. That is not the
case, because the company's facilities would be idle if it does not accept this order. In
other words, it cannot sell these sneakers for $11.50. Thus there is no opportunity cost
in accepting this order.
Furthermore, the company's normal fixed costs and normal variable selling costs do not
apply to this order.
(1) Fixed manufacturing costs are irrelevant, because they will be the same whether the
order is accepted or not accepted.
(2) The variable selling expense for this special order would not be the same as the
selling expense for the company’s normal line of sneakers. The question states that the
$1.00 variable selling expense does not include shipping-out expense. Other typical
variable selling expenses are sales representative’s commissions, advertising, etc. But
in this question it is stated that the order was received directly from the wholesaler, so
those other selling expenses for the company’s normal line of sneakers would not apply.
Explanation for Choice C:
This is the special order price of $7.50 per unit minus the fixed cost per unit of $3.00 per
pair minus the normal variable selling cost of $1.00 per pair, the difference multiplied by
15,000 units. This is incorrect because
(1) Fixed manufacturing costs are irrelevant, because they will be the same whether the
order is accepted or not accepted;
(2) The variable manufacturing cost per unit has not been taken into consideration; and
(3) The variable selling cost for the company's normal line of sneakers has been
included. The selling expense for this special order would not be the same as the
selling expense for the company’s normal line of sneakers. The question states that the
$1.00 variable selling expense does not include shipping-out expense. Other typical
variable selling expenses are sales representative’s commissions, advertising, etc. But
in this question it is stated that the order was received directly from the wholesaler, so
those other selling expenses for the company’s normal line of sneakers would not apply.
Explanation for Choice D:
This answer results from including the $1.00 per unit variable selling expense for the
normal line of sneakers as a cost of the order.
Hock P2 2020
Section C - Decision Analysis.
Answers
The key to getting this question right is to not assume that the variable selling expense
for this special order would be the same as the variable selling expense for the
company’s normal line of sneakers. The question states that the $1.00 variable selling
expense does not include shipping-out expense. Other typical variable selling expenses
are sales representative’s commissions, advertising, etc. But in this question it is stated
that the order was received directly from the wholesaler, so those other selling
expenses for the company’s normal line of sneakers would not apply.
207. Question ID: ICMA 1603.P2.033 (Topic: Marginal Analysis-Other Decisions)
A small delivery company received an order that requires nine deliveries lasting two
hours each on the same day. The company owns two vans that together can make
eight trips per day. The company can rent a van on a daily eight-hour basis for $72, and
the fuel cost is $20 per trip. The company has several van drivers, each of whom earns
$30,000 annually and is expected to make 1,000 deliveries each year. The marginal
cost of the ninth delivery is

 A. $92.correct
 B. $38.
 C. $28.
 D. $122.
Question was not answered
Correct Answer Explanation:
The marginal cost of the ninth delivery includes the $72 eight-hour rental fee for the
third van, since the van must be rented on an eight-hour basis and cannot be rented by
the hour. The marginal cost also includes the fuel cost of one trip, or $20. The salary of
the van driver is a fixed cost and will not increase as a result of the trip, so it is not
relevant. Thus the marginal cost of the ninth delivery is $72 + $20 = $92.
Explanation for Choice B:
This answer results from including the cost of the rental van at its hourly rate, assuming
two hours for the delivery. However, the van must be rented on an eight-hour basis and
cannot be rented by the hour, so the marginal cost of the ninth delivery must include the
full eight-hour rental fee for the van.
Explanation for Choice C:
This is not the correct answer. Please see the correct answer for an explanation.
We have been unable to determine how to calculate this incorrect answer choice. If you
have calculated it, please let us know how you did it so we can create a full explanation
of why this answer choice is incorrect. Please send us an email at
support@hockinternational.com. Include the full Question ID and the actual incorrect
answer choice—not its letter, because that can change with every study session
Hock P2 2020
Section C - Decision Analysis.
Answers
created. The Question ID appears at the top of the question. Thank you in advance for
helping us to make your HOCK study materials better.
Explanation for Choice D:
This answer results from including a pro-rata cost of the delivery van driver in the cost of
the ninth delivery. The salary of the van driver is a fixed cost and will not increase as a
result of the trip, so it is not relevant.
208. Question ID: CMA 1296 4.8 (Topic: Marginal Analysis-Other Decisions)
Whitehall Corporation produces chemicals used in the cleaning industry. During the
previous month, Whitehall incurred $300,000 of joint costs in producing 60,000 units of
AM-12 and 40,000 units of BM-36. Whitehall uses the units-of-production method to
allocate joint costs. Currently, AM-12 is sold at split-off for $3.50 per unit. Flank
Corporation has approached Whitehall to purchase all of the production of AM-12 after
further processing. The further processing will cost Whitehall $90,000.
Assume that Whitehall Corporation agreed to sell AM-12 to Flank Corporation for $5.50
per unit after further processing. During the first month of production, Whitehall sold
50,000 units with 10,000 units remaining in inventory at the end of the month. With
respect to AM-12, which one of the following statements is correct?

 A. The operating profit last month was $125,000, and the inventory value is $30,000.
 B. The operating profit last month was $50,000, and the inventory value is $15,000.
 C. The operating profit last month was $200,000, and the inventory value is $30,000.
 D. The operating profit last month was $50,000, and the inventory value is $45,000.correct
Question was not answered
Correct Answer Explanation:
The total per unit cost of the 60,000 units of AM-12 after processing further consists of
an allocation of the joint cost plus the additional processing costs, which are separable
costs. 60% (60,000 ÷ [60,000 + 40,000]) of the joint cost of $300,000 is allocated to the
AM-12, or $180,000. That plus the additional processing costs of $90,000 equals total
cost of $270,000 for the 60,000 units of AM-12 after further processing, or $4.50 per
unit.
The sale price is $5.50 per unit, so the contribution margin per unit is $5.50 − $4.50, or
$1.00 per unit. The operating profit for sale of 50,000 units is 50,000 × $1.00, or
$50,000.
The cost of the 10,000 units remaining in inventory (60,000 units produced − 50,000
units sold) is 10,000 × $4.50, or $45,000.
Explanation for Choice A:
This answer results from using only the joint cost as the cost of sales. The joint cost
plus the separable cost is the cost of the units.
Hock P2 2020
Section C - Decision Analysis.
Answers
Explanation for Choice B:
This answer results from using an inventory value of $1.50 per unit for the unsold units
of AM-12, which represents only the separable costs. The inventory value consists of
both the joint costs and the separable costs.
Explanation for Choice C:
This is not the correct answer. Please see the correct answer for an explanation.
We have been unable to determine how to calculate this incorrect answer choice. If you
have calculated it, please let us know how you did it so we can create a full explanation
of why this answer choice is incorrect. Please send us an email at
support@hockinternational.com. Include the full Question ID number and the actual
incorrect answer choice -- not its letter, because that can change with every study
session created. The Question ID number appears at the top of the question. Thank you
in advance for helping us to make your HOCK study materials better.
209. Question ID: ICMA 13.P2.047 (Topic: Marginal Analysis-Other Decisions)
Joe Cooper owns and operates an ice cream truck that he drives through residential
neighborhoods to sell five different treats to the area’s children. On average, Cooper
sells 100 of each type of treat per day for the 120 days per year when the weather is
warm enough to generate sales. Four of his products are profitable, but the other,
Creamy Delight, indicates a loss as follows:

Selling price/unit $ 1.75


Cost of each treat 0.80
Truck operating costs/unit 0.37
Joe's salary/unit 0.60
Administrative costs/unit 0.08
Loss/unit $(0.10)
If Cooper cannot raise his selling price, he should

 A. discontinue the sales of Creamy Delight to increase his profits by $240.


 B. continue to sell Creamy Delight to avoid a decrease in profit of $11,400.correct
 C. discontinue the sales of Creamy Delight to increase his profits by $1,200.
 D. continue to sell Creamy Delight to avoid a decrease in profit of $6,960.
Question was not answered
Correct Answer Explanation:
Only the variable revenue (selling price per unit) and variable cost (cost of each treat)
are relevant to this decision. Truck operating costs, Joe's salary, and administrative
Hock P2 2020
Section C - Decision Analysis.
Answers
costs are fixed costs. The fixed costs will be the same whether or not Cooper sells
Creamy Delight. Each unit of Creamy Delight sold contributes $0.95 to covering fixed
costs ($1.75 − $0.80). If Creamy Delight were discontinued, $0.95 × 100 sales per day ×
120 days per year or $11,400 in total contribution would be lost, and net profit would
decline by that amount.
Explanation for Choice A:
Only the variable revenue (selling price per unit) and variable cost (cost of each treat)
are relevant to this decision. Truck operating costs, Joe's salary, and administrative
costs are fixed costs. The fixed costs will be the same whether or not Cooper sells
Creamy Delight.
Because the variable cost (the cost of each treat in this question) is less than the selling
price, Cooper should continue to sell the Creamy Delight. If he were to stop selling it, his
profit would go down.
Explanation for Choice C:
Only the variable revenue (selling price per unit) and variable cost (cost of each treat)
are relevant to this decision. Truck operating costs, Joe's salary, and administrative
costs are fixed costs. The fixed costs will be the same whether or not Cooper sells
Creamy Delight.
Because the variable cost (the cost of each treat in this question) is less than the selling
price, Cooper should continue to sell the Creamy Delight. If he were to stop selling it, his
profit would go down.
Explanation for Choice D:
Cooper should continue to sell Creamy Delight. However, this calculation of the
decrease in profits if he were not to sell it is incorrect.
Only the variable revenue (selling price per unit) and variable cost (cost of each treat)
are relevant to this decision. Truck operating costs, Joe's salary, and administrative
costs are fixed costs. The fixed costs will be the same whether or not Cooper sells
Creamy Delight.
This answer includes the costs of operating the truck in the calculation of the
contribution from Creamy Delight.
210. Question ID: CMA 694 4.27 (Topic: Marginal Analysis-Other Decisions)
Condensed monthly operating income data for Korbin Inc. for May follows:

Urban Suburban
Store Store Total
Sales $80,000 $120,000 $200,000
Variable costs 32,000 84,000 116,000
Hock P2 2020
Section C - Decision Analysis.
Answers
Contribution margin $48,000 $36,000 $84,000
Direct fixed costs 20,000 40,000 60,000
Store segment margin $28,000 $(4,000) $24,000
Common fixed cost 4,000 6,000 10,000
Operating income $24,000 $(10,000) $14,000
Additional information regarding Korbin's operations follows:

 One-fourth of each store's direct fixed costs would continue if either store were
closed.
 Korbin allocates common fixed costs to each store on the basis of sales dollars.
 Management estimates that closing the Suburban Store would result in a 10%
decrease in the Urban Store's sales, while closing the Urban Store would not affect
the Suburban Store's sales.
 The operating results for May are representative of all months.
One-half of the Suburban Store's dollar sales are from items sold at variable cost to
attract customers to the store. Korbin is considering the deletion of these items, a move
that would reduce the Suburban Store's direct fixed expenses by 15% and result in a
20% loss of Suburban Store's remaining sales volume. This change would not affect the
Urban Store. A decision by Korbin to eliminate the items sold at cost would result in a
monthly increase (decrease) in Korbin's operating income of

 A. $(5,200)
 B. $2,000
 C. $(7,200)
 D. $(1,200)correct
Question was not answered
Correct Answer Explanation:
Since 50% of the sales are derived from selling at variable cost, the contribution margin
must come from the other 50% of sales (since no contribution margin results from a
sales price equal to the variable cost). Therefore, if the other sales decrease by 20%,
then the contribution margin must decrease by $7,200 ($36,000 × 20%). If fixed costs
decrease by 15%, or $6,000 ($40,000 × 15%) then the monthly net decrease in Korbin's
operating income would be $1,200.
Explanation for Choice A:
This answer $(5,200) represents the new segment margin for Suburban Store ($28,800
− $34,000).
Hock P2 2020
Section C - Decision Analysis.
Answers
Explanation for Choice B:
This is not the correct answer. Please see the correct answer for an explanation.
We have been unable to determine how to calculate this incorrect answer choice. If you
have calculated it, please let us know how you did it so we can create a full explanation
of why this answer choice is incorrect. Please send us an email at
support@hockinternational.com. Include the full Question ID number and the actual
incorrect answer choice -- not its letter, because that can change with every study
session created. The Question ID number appears at the top of the question. Thank you
in advance for helping us to make your HOCK study materials better.
Explanation for Choice C:
This answer represents the reduction in the contribution margin ($36,000 × 20%).
211. Question ID: CMA 692 4.28 (Topic: Marginal Analysis-Other Decisions)
Power Systems, Inc. manufactures jet engines for the United States armed forces on a
cost-plus basis. The cost of a particular jet engine the company manufactures is shown
as follows.

Direct materials $200,000


Direct labor 150,000
Overhead:
Supervisor's salary 20,000
Fringe benefits on direct labor 15,000
Depreciation 12,000
Rent 11,000
Total cost $408,000
If production of this engine were discontinued, the production capacity would be idle and
the supervisor would be laid off. When asked to bid on the next contract for this engine,
the minimum unit price that Power Systems should bid is

 A. $385,000.correct
 B. $365,000.
 C. $408,000.
 D. $397,000.
Question was not answered
Correct Answer Explanation:
Hock P2 2020
Section C - Decision Analysis.
Answers
In this problem want to calculate all avoidable cost. Therefore, the minimal unit price
that Power Systems should bid is $385,000 ($200,000 DM + $150,000 DL + $20,000
Supervisors salary + $15,000 Fringe benefits on DL).
Explanation for Choice B:
This answer does not include the salary of the supervisor who would be laid off if
production of the engine were discontinued. This is an avoidable cost and should be
included in the calculation.
Explanation for Choice C:
This answer includes unavoidable cost (depreciation and rent) that should not be
included.
Explanation for Choice D:
This answer includes depreciation which is an unavoidable cost, and should not be
included.
212. Question ID: ICMA 10.P2.256 (Topic: Marginal Analysis-Other Decisions)
The Doll House, a very profitable company, plans to introduce a new type of doll to its
product line. The sales price and costs for the new dolls are as follows.

Selling price per doll $100


Variable cost per doll $60
Incremental annual fixed costs $456,000
Income tax rate 30%
If 10,000 of the new dolls are produced and sold, the effect on Doll House's profit (loss)
would be

 A. $(176,000).
 B. $(39,200).correct
 C. $280,000.
 D. $(56,000).
Question was not answered
Correct Answer Explanation:
The incremental revenues and costs from the manufacture and sale of the new doll are
as follows:

Incremental sales: $100 × 10,000 $1,000,000


Incremental variable cost: $60 × 10,000 600,000
Hock P2 2020
Section C - Decision Analysis.
Answers
Incremental contribution margin: $40 × 10,000 $ 400,000
Incremental fixed costs 456,000
Net incremental operating loss $ (56,000)
Plus income tax effect of incremental loss: 30% of $56,000 16,800
Net incremental after-tax operating loss $(39,200)
Explanation for Choice A:
This is the net incremental before-tax operating loss of $(56,000) minus $120,000,
which is 30% of the incremental contribution margin of $400,000. The incremental tax
effect of the $(56,000) incremental operating loss is 30% of the ($56,000) incremental
operating loss, not 30% of the incremental contribution margin. Furthermore, the tax
effect of a loss reduces the loss instead of increasing it. Assuming the company's other
operations are profitable, the loss will shield other net income from tax, reducing the
overall tax due.
Explanation for Choice C:
This answer does not include any deductions for the incremental fixed costs and the tax
effect of the fixed costs.
Explanation for Choice D:
This is the incremental operating loss before the tax effect of the loss is taken into
consideration.
213. Question ID: ICMA 19.P2.015 (Topic: Marginal Analysis-Other Decisions)
A company sells two products that are manufactured in the same production
department on two different machines. The contribution margin per unit of the two
products is $120 and $80, respectively. When deciding if the second product should be
discontinued, which one of the following pieces of information is needed to make the
correct decision?

 A. Production department manager’s salary.


 B. Commissions paid on the second product’s sales.
 C. Depreciation expense of the second product’s machinery.
 D. Alternative use of the second product’s space.correct
Question was not answered
Correct Answer Explanation:
The alternative use of the second product's space is a relevant item in the decision of
whether or not to discontinue the second product. The benefit from an alternative use of
the space may be greater than the benefit received from the second product using that
space.
Hock P2 2020
Section C - Decision Analysis.
Answers
Explanation for Choice A:
The production manager's salary is a fixed cost and not relevant to the decision about
the second product.
Explanation for Choice B:
Commissions paid on the sales of the second product are a variable cost that is already
included in the contribution margin of the second product.
Explanation for Choice C:
The machinery that is used for the second product is essentially a fixed cost and if the
second product is not produced, the cost of the machinery will remain unchanged.
214. Question ID: ICMA 1603.P2.064 (Topic: Marginal Analysis-Other Decisions)
A retail company has three segments with total operating income of $500,000. Selected
financial information for Segment 1 is presented below.

Segment 1
Unit sales 28,000
Sales revenue $700,000
Cost of sales 420,000
Administrative expenses 144,000
Commissions 14,000
Rent 140,000
Salaries 32,000

 Administrative expenses are allocated to the three segments equally.


 Commissions are paid to the salespersons in each segment based on 2% of gross
sales.
 The company rents the entire building and allocates the rent to the three segments
based on the square footage occupied by each.
 Salaries represent payments to the employees in the segment.
The controller has expressed concern about the operating loss for Segment 1 and has
suggested that it be closed. If the segment is closed, none of the employees would be
retained. Should the company drop Segment 1?

 A. No, because total operating income would decrease by $94,000.


 B. No, because total operating income would decrease by $234,000.correct
 C. No, because total operating income would decrease by $126,000.
Hock P2 2020
Section C - Decision Analysis.
Answers
 D. Yes, because total operating income would increase by $50,000.
Question was not answered
Correct Answer Explanation:
Segment 1 is providing a $234,000 contribution toward covering the company's
common (allocated) costs, calculated as $700,000 sales revenue less $420,000 cost of
sales, less $14,000 commissions, less $32,000 salaries. Those are the items that would
go away if Segment 1 were dropped.
Segment 1's apparent operating loss is caused by the $144.000 of common
administrative expenses being allocated to it and by the $140,000 in common rental
expense being allocated to it. The common expenses that are being allocated would not
change in total for the company if Segment 1 were dropped. Those common fixed costs
would continue to be paid if Segment 1 were dropped—they would just be reallocated to
the remaining operating segments.
Therefore, the company should not drop Segment 1 because doing so would cause
operating income to decrease by $234,000.
Explanation for Choice A:
This is Segment 1's sales revenue less its cost of sales, less its commissions, less rent
allocated to it, and less salaries. Rent is not an expense that would go away if Segment
1 were dropped because the company rents the entire building and allocates the rent.
The company would continue to rent the entire building if Segment 1 were dropped and
so the rent expense would continue. It would just be reallocated to the remaining
operating segments. Therefore it should not be included in the calculation.
Explanation for Choice C:
This is Segment 1's sales revenue less its cost of sales, less its commissions, and less
rent allocated to it. Salaries are an expense that would go away if Segment 1 were
dropped, and so they should be included in the calculation. Rent is not an expense that
would go away if Segment 1 were dropped because the company rents the entire
building and allocates the rent. The company would continue to rent the entire building if
Segment 1 were dropped and so the rent expense would continue. It would just be
reallocated to the remaining operating segments. Therefore it should not be included in
the calculation.
Explanation for Choice D:
This the amount of operating loss attributed to Segment 1 after allocation of common
administrative expenses and allocation of common rent expense. Those common costs
would continue to be paid if Segment 1 were dropped—they would just be reallocated to
the remaining operating segments. Since the common costs would not go away, they
are not relevant to the decision and should not be considered in the analysis of whether
to drop the segment.
Hock P2 2020
Section C - Decision Analysis.
Answers
215. Question ID: CMA 691 4.13 (Topic: Marginal Analysis-Other Decisions)
American Coat Company estimates that 60,000 special zippers will be used in the
manufacture of men's jackets during the next year. Reese Zipper Company has quoted
a price of $0.60 per zipper. American would prefer to purchase 5,000 units per month,
but Reese is unable to guarantee this delivery schedule. In order to ensure availability
of these zippers, American is considering the purchase of all 60,000 units at the
beginning of the year. Assuming American can invest cash at 8%, the company's
opportunity cost of purchasing the 60,000 units at the beginning of the year is

 A. $1,440.
 B. $1,500.
 C. $2,640.
 D. $1,320.correct
Question was not answered
Correct Answer Explanation:
The total cost of the zippers for one year is $36,000 (60,000 × $0.60). The company has
a choice: It can purchase all of the zippers at the beginning of the year, or it can
purchase them as it needs them, which is 5,000 per month.
If American Coat Company purchases 5,000 zippers monthly, it will pay $3,000 per
month for them (5,000 × $0.60).
If American Coat Company decides to purchase all the zippers at the beginning of the
year then the total extra cash outlay for the company in January over and above what
they would pay to purchase only enough for one month will be $33,000 [$36,000 −
(5,000 zippers needed the first month × $0.60)]
The money not needed for zipper purchases could be invested at 8%. The amount of
money not needed for zipper purchases and thus invested would start the year at
$33,000 and decline each month until at the end of the year, it would be zero.
Therefore, the average balance of the investable funds would be the beginning balance
plus the ending balance divided by 2 ([$33,000 + $0] ÷ 2), which is $16,500. An average
balance of $16,500 invested at 8% per annum for one year equals income of $1,320
over the one-year period. That is the income given up by purchasing all of the zippers at
the beginning of the year, and that is the opportunity cost.
Explanation for Choice A:
This answer results from using an average investable balance of $18,000 for the year,
calculated as ($36,000 + $0) / 2. However, the investable balance begins the year at
$33,000, not $36,000, because $33,000 is the difference between the cost to purchase
in January all of the zippers at the same time ($36,000) and the cost to purchase only
the first month's requirement ($3,000).
Explanation for Choice B:
Hock P2 2020
Section C - Decision Analysis.
Answers
This is not the correct answer. Please see the correct answer for an explanation.
We have been unable to determine how to calculate this incorrect answer choice. If you
have calculated it, please let us know how you did it so we can create a full explanation
of why this answer choice is incorrect. Please send us an email at
support@hockinternational.com. Include the full Question ID number and the actual
incorrect answer choice -- not its letter, because that can change with every study
session created. The Question ID number appears at the top of the question. Thank you
in advance for helping us to make your HOCK study materials better.
Explanation for Choice C:
This answer results from multiplying $33,000 (the difference between the cost to buy
60,000 zippers and the cost to buy 5,000 zippers) by the interest rate at which the
unused funds could be invested. However, the investable funds amount will decline
each month because additional zippers will need to be purchased each month, if the
company purchases the zippers it needs monthly. Therefore, the amount of income lost
should be based on the average balance of the investable funds over the course of one
year, not the amount of investable funds in January.
216. Question ID: ICMA 10.P2.226 (Topic: Marginal Analysis-Other Decisions)
Edwards Products has just developed a new product with a manufacturing cost of $30.
The Marketing Director has identified three marketing approaches for this new product.

Approach Set a selling price of $36 and have the firm's sales staff sell the product at a 10%
X commission with no advertising program. Estimated annual sales would be
10,000 units.
Approach Set a selling price of $38, have the firm's sales staff sell the product at a 10%
Y commission, and back them up with a $30,000 advertising program. Estimated
annual sales would be 12,000 units.
Approach Rely on wholesalers to handle the product. Edwards would sell the new product
Z to the wholesalers at $32 per unit and incur no selling expenses. Estimated
annual sales would be 14,000 units.
Rank the three alternatives in order of net profit, from highest net profit to lowest.

 A. X, Y, Z.
 B. Z, X, Y.correct
 C. Z, Y, X.
 D. Y, Z, X.
Question was not answered
Correct Answer Explanation:
Hock P2 2020
Section C - Decision Analysis.
Answers
The best way to solve this is to create income statements for each of the three
approaches. The sales values are calculated based on selling price multiplied by units
sold. Manufacturing costs are the $30 given in the problem multiplied by the number of
units sold. Commissions under Approach X and Y are 10% of sales, and only Approach
Y has advertising expenses. Here are the income statements:

X Y Z
Sales $360,000 $456,000 $448,000
Manufacturing Costs 300,000 360,000 420,000
Commissions 36,000 45,600 0
Advertising 0 30,000 0
Operating Income $ 24,000 $ 20,400 $ 28,000
The highest operating income is with Approach Z, next highest is Approach X, and the
lowest is Approach Y.
Explanation for Choice A:
The best way to solve this is to create income statements for each of the three
approaches. This answer could result from any error in calculations or in evaluating the
results of the calculations. See correct answer for details.
Explanation for Choice C:
The best way to solve this is to create income statements for each of the three
approaches. This answer could result from any error in calculations or in evaluating the
results of the calculations. See correct answer for details.
Explanation for Choice D:
The best way to solve this is to create income statements for each of the three
approaches. This answer could result from any error in calculations or in evaluating the
results of the calculations. See correct answer for details.
217. Question ID: CIA 1192 IV.18 (Topic: Marginal Analysis-Other Decisions)
A manufacturer has been approached by a new customer who wants to place a one-
time order for a component similar to one that the manufacturer makes for another
customer. Existing sales will not be affected by acceptance of this order. The
manufacturer has a policy of setting its targeted selling price at 60% over full
manufacturing cost. The manufacturing costs and the targeted selling price for the
existing product are presented as follows.

Direct materials $ 2.30


Direct labor 3.60
Hock P2 2020
Section C - Decision Analysis.
Answers
Variable manufacturing overhead (applied at 75% of direct labor cost) 2.70
Fixed manufacturing overhead (applied at 150% of direct labor cost) 5.40
Total manufacturing cost $14.00
Markup (60% of full manufacturing cost) 8.40
Targeted selling price $22.40
The manufacturer has excess capacity to produce the quantity of the component
desired by the new customer. The direct materials used in the component for the new
customer would cost the manufacturer $0.25 less than those used in the component
currently being made. The variable selling expenses (packaging and shipping) would be
the same, or $0.90 per unit.
Under these circumstances, the minimum unit price at which the manufacturer would
accept the special order is one exceeding:

 A. $14.80
 B. $14.00
 C. $9.25correct
 D. $8.35
Question was not answered
Correct Answer Explanation:
The minimum unit price will be the sum of the relevant variable costs of the process, as
follows:
Direct materials = $2.30 − $0.25 = $2.05
Direct labor = $3.60
Variable manufacturing overhead = $2.70
Variable selling expenses = $0.90
Total variable cost = $2.05 + $3.60 + $2.70 + $0.90 = $9.25
Note that the question asks only for the minimum unit price at which the manufacturer
would accept the special order. At any price over $9.25, total profit for the company will
be increased, but this is so only because the manufacturer has excess unused capacity.
If the manufacturer did not have excess capacity, it would have to not manufacture
something else in order to accept this order. Not manufacturing something else would
cause the company to incur an opportunity cost in taking the new order equal to the
contribution margin lost because of not having the other product available to sell. That
opportunity cost would need to be included as a variable cost of the special order
Hock P2 2020
Section C - Decision Analysis.
Answers
product in calculating the minimum unit price at which the manufacturer would accept
the special order.
Explanation for Choice A:
This answer includes the 60% markup over the full manufacturing cost. The question
asks for the minimum unit price at which the manufacturer would accept the special
order. The minimum price does not include the markup.
Explanation for Choice B:
The answer $14.00 includes all manufacturing cost (fixed and variable) but does not
include the additional selling and marketing cost that would be incurred. It also does not
include the revised cost of material ($2.30 − $0.25).
Explanation for Choice D:
The answer $8.35 does not include the additional selling expense of $0.90.
218. Question ID: ICMA 10.P2.222 (Topic: Marginal Analysis-Other Decisions)
Jack Blaze wants to rent store space in a new shopping mall for the three month holiday
shopping season. Blaze believes he has a new product available which has the
potential for good sales. The product can be obtained on consignment at the cost of $20
per unit and he expects to sell the item for $100 per unit. Due to other business
ventures, Blaze's risk tolerance is low. He recognizes that, as the product is entirely
new, there is an element of risk. The mall management has offered Blaze three rental
options: (1) a fixed fee of $8,000 per month, (2) a fixed fee of $3,990 per month plus
10% of Blaze's revenue, or (3) 30% of Blaze's revenues. Which one of the following
actions would you recommend to Jack Blaze?

 A. Choose the third option no matter what Blaze expects the revenues to be.correct
 B. Choose the second option only if Blaze expects revenues to exceed $5,700.
 C. Choose the second option no matter what Blaze expects the revenues to be.
 D. Choose the first option no matter what Blaze expects the revenues to be.
Question was not answered
Correct Answer Explanation:
The most important fact given in this question is that Jack Blaze's risk tolerance is low.
The most important fact not given is sales volume, which is not known. If he were to
commit to paying any rental amount that includes a fixed amount, he might not sell
enough to cover the fixed cost and could lose money. So he should choose option 3,
because with that option, he is keeping all of his expenses variable. The selling price
will be $100 per unit, the variable cost is $20 per unit, and his rent will be 30% of his
revenue, or $30 per unit. As long as he keeps any other expenses variable and under
$50 per unit ($100 − $20 − $30), he will be virtually assured of making a profit. His risk
of losing money is very low.
Hock P2 2020
Section C - Decision Analysis.
Answers
Explanation for Choice B:
The most important fact given in this question is that Jack Blaze's risk tolerance is low.
The most important fact not given is sales volume, which is not known. And expected
sales volume is only that: expected. Expected sales volume might not be realized. If he
were to commit to paying any rental amount that includes a fixed amount, he might not
sell enough to cover the fixed cost and could lose money.
Explanation for Choice C:
The most important fact given in this question is that Jack Blaze's risk tolerance is low.
The most important fact not given is sales volume, which is not known. If he were to
commit to paying any rental amount that includes a fixed amount, he might not sell
enough to cover the fixed cost and could lose money.
Explanation for Choice D:
The most important fact given in this question is that Jack Blaze's risk tolerance is low.
The most important fact not given is sales volume, which is not known. If he were to
commit to paying any rental amount that includes a fixed amount, he might not sell
enough to cover the fixed cost and could lose money.
219. Question ID: CIA 594 III.45 (Topic: Marginal Analysis-Other Decisions)
A company has 7,000 obsolete toys carried in inventory at a manufacturing cost of $6
per unit. If the toys are reworked for $2 per unit, they could be sold for $3 per unit. If the
toys are scrapped, they could be sold for $1.85 per unit. Which alternative is more
desirable (rework or scrap) and what is the total dollar amount of the advantage of that
alternative?

 A. Rework, $8,050.
 B. Scrap, $47,950.
 C. Rework, $36,050.
 D. Scrap, $5,950.correct
Question was not answered
Correct Answer Explanation:
The manufacturing cost of $6.00 is a sunk cost and not relevant to the decision. The net
revenue from reworking the obsolete toys is $7,000 [($3.00 − $2.00) × 7,000)]. Revenue
if the toys are scrapped is $12,950 ($1.85 × 7,000). The dollar advantage for selling as
scrap is $5,950 ($12,950 − $7,000).
This could also be solved by multiplying the per unit difference by 7,000. The net
advantage to reworking and selling the inventory is $1 per unit ($3 sales price minus $2
rework cost). The net advantage to selling the inventory for scrap is $1.85 per unit.
Selling the inventory for scrap will net the company $0.85 per unit more than reworking
it to sell. $0.85 × 7,000 = $5,950.
Hock P2 2020
Section C - Decision Analysis.
Answers
Explanation for Choice A:
This answer results from subtracting $1.85 from $3.00 and multiplying the difference by
7,000. This ignores the cost of the rework.
Explanation for Choice B:
This is the rework cost ($2) plus the sale price of the reworked units ($3) plus the
revenue from selling them for scrap ($1.85) multiplied by 7,000. That is not the total
dollar amount of the advantage of one of the alternatives.
Explanation for Choice C:
This is the present inventory cost of $6 plus the net realizable value of $1 minus the
scrap revenue of $1.85, the quantity multiplied by 7,000. That is not the total dollar
amount of the advantage of one of the alternatives.
220. Question ID: CMA 1296 4.7 (Topic: Marginal Analysis-Other Decisions)
Whitehall Corporation produces chemicals used in the cleaning industry. During the
previous month, Whitehall incurred $300,000 of joint costs in producing 60,000 units of
AM-12 and 40,000 units of BM-36. Whitehall uses the units-of-production method to
allocate joint costs. Currently, AM-12 is sold at split-off for $3.50 per unit. Flank
Corporation has approached Whitehall to purchase all of the production of AM-12 after
further processing. The further processing will cost Whitehall $90,000.
Concerning AM-12, which one of the following alternatives is most advantageous?

 A. Whitehall should process further and sell to Flank if the total selling price per unit after
further processing is greater than $5.00.correct
 B. Whitehall should process further and sell to Flank if the total selling price per unit after
further processing is greater than $5.25, which maintains the same gross profit
percentage.
 C. Whitehall should continue to sell at split-off unless Flank offers at least $4.50 per unit
after further processing, which covers Whitehall's total costs.
 D. Whitehall should process further and sell to Flank if the total selling price per unit after
further processing is greater than $3.00, which covers the joint costs.
Question was not answered
Correct Answer Explanation:
Joint costs prior to the split-off are not relevant to the decision as to whether to process
further or sell at the split-off, because they will be the same regardless of which choice
Whitehall makes.
We know that AM-12 can be sold for $3.50 per unit at the split-off. Further processing
will cost $90,000 for 60,000 units, so the additional cost per unit for the additional
processing would be $1.50 ($90,000 ÷ 60,000 units of AM-12). Therefore, the minimum
Hock P2 2020
Section C - Decision Analysis.
Answers
price that Whitehall would accept must be a price greater than $5.00 per unit ($3.50 +
$1.50).
Why would a price greater than $5.00 per unit be more advantageous than a price
greater than $5.25 per unit, as suggested by another answer choice? If Whitehall could
get $5.25 per unit, that would increase its gross profit by more than if it could get $5.01
per unit, certainly. However, if Whitehall refuses to accept a price between $5.01 and
$5.24 for the further-processed AM-12, it could price itself out of the market. Flank
Corporation might decide it would not pay that much and might walk away from the
deal, leaving Whitehall to sell the AM-12 without further processing for $3.50.
Any price of $5.01 or greater would be advantageous to Whitehall because it would
increase gross profits for the company as a whole. Revenue would increase by a
minimum of $1.51 per unit sold ($5.01 − $3.50), while cost of goods sold expense would
increase by $1.50 per unit sold. Thus, gross profit for the company would increase by a
minimum of $0.01 per unit sold, or by at least $600 (60,000 units × $0.01).
On the other hand, requiring a price of $5.25 per unit could cause the opportunity —
and the additional gross profit — to evaporate.
Explanation for Choice B:
If Whitehall processes the AM-12 further, cost of goods sold expense per unit will
increase by $1.50 for the AM-12 ($90,000 ÷ 60,000 units). Thus, any price of $5.01 or
greater ($3.50 + $1.50 + $0.01) for the further-processed AM-12 would be
advantageous to Whitehall because it would increase gross profits for the company as a
whole. Revenue would increase by a minimum of $1.51 per unit sold ($5.01 − $3.50),
while cost of goods sold expense would increase by $1.50 per unit sold. Thus, gross
profit for the company would increase by a minimum of $0.01 per unit sold, or by at
least $600 (60,000 units × $0.01).
Therefore, the statement that this would maintain the same gross profit percentage is
not accurate, because the gross profit percentage for the company as a whole would
increase, not remain the same.
If Whitehall could get $5.25 per unit, that would increase its gross profit by more than if
it could get $5.01 per unit, certainly. However, if Whitehall refuses to accept a price
between $5.01 and $5.24 for the further-processed AM-12, it could price itself out of the
market. Flank Corporation might decide it would not pay that much and might walk away
from the deal, leaving Whitehall to sell the AM-12 without further processing — and
without any additional gross profit — for $3.50.
Explanation for Choice C:
A price of $4.50 per unit after further processing would not be adequate, because it
would cause a reduction in total profits.
Explanation for Choice D:
Hock P2 2020
Section C - Decision Analysis.
Answers
Joint costs are irrelevant to the decision. A price of $3.00 per unit after further
processing would not be adequate, because it would cause a reduction in total profits.
221. Question ID: ICMA 10.P2.239 (Topic: Marginal Analysis-Other Decisions)
Raymund Inc. currently sells its only product to Mall-Stores. Raymund has received a
one-time-only order for 2,000 units from another buyer. Sale of the special order items
will not require any additional selling effort. Raymund has a manufacturing capacity to
produce 7,000 units. Raymund has an effective income tax rate of 40%. Raymund’s
Income Statement, before consideration of the one-time-only order, is as follows.

Sales (5,000 units at $20 per unit) $100,000


Variable manufacturing costs $50,000
Variable selling costs 15,000 65,000
Contribution margin 35,000

Fixed manufacturing costs 16,000


Fixed selling costs 4,000 20,000
Operating income 15,000

Income taxes 6,000


Net income $9,000
In negotiating a price for the special order, Raymund should set the minimum per unit
selling price at

 A. $18.
 B. $10.correct
 C. $13.
 D. $17.
Question was not answered
Correct Answer Explanation:
The plant has the capacity to produce 7,000 units. It is currently producing and selling
5,000 units; so it has adequate capacity to accept this one-time-only special order for
2,000 units. There will be no opportunity cost for loss of other business that needs to be
considered in this price. Also, this sale will not require any additional selling effort, so
there will be no variable selling costs associated with it. The only additional costs
associated with the order will be the variable manufacturing costs. Variable
Hock P2 2020
Section C - Decision Analysis.
Answers
manufacturing cost per unit is $50,000 ÷ 5,000 units, which is $10 per unit. That is the
minimum per unit selling price that Raymund should charge.
Explanation for Choice A:
This is the variable cost per unit of $10 plus 40% of the current $20 sales price ($8). If
Raynaud were to sell the special order for $18 per unit and the variable cost associated
with the order were $10, Raynaud would earn a before-tax profit on each unit of $8.
Income tax would be payable at the rate of 40% on the $8 profit (not on the sales price),
or $3.20 per unit, and the company would have an after-tax profit of $4.80 per unit.
Since Raynaud has the necessary capacity to produce the 2,000-unit order, its
minimum per unit selling price should be equal to its variable cost. At that price, the
company would break even on the order. There would be no tax effect because at a
break-even level, the order would earn no taxable income.
Explanation for Choice C:
This is the variable manufacturing cost plus the variable selling cost per unit. The
problem says that sale of the special order items will not require any additional selling
effort. Therefore, the variable selling cost should not be included.
Explanation for Choice D:
This is the variable manufacturing cost plus the variable selling cost plus the fixed
manufacturing cost plus the fixed selling cost per unit.
The problem says that sale of the special order items will not require any additional
selling effort. Therefore, the variable selling cost should not be included.
The fixed costs will be same whether this special order is accepted or not, so they
should not be included.
222. Question ID: ICMA 1603.P2.065 (Topic: Marginal Analysis-Other Decisions)
A company uses a joint process to produce three products: A, B and C, all derived from
one input. The company can sell these products at the point of split-off or process them
further. The joint production costs during October were $10,000. The company allocates
joint costs to the products in proportion to the relative physical volume of output.
Additional information is presented below.

If Processed Further
Unit Sales
Units Price at Unit Unit
Product Produced Split-off Sales Price Addt'l Cost
A 1,000 $4.00 $5.00 $0.75
B 2,000 2.25 4.00 1.20
Hock P2 2020
Section C - Decision Analysis.
Answers
C 1,500 3.00 3.75 0.90
Assuming sufficient demand exists to maximize profits, the company should sell

 A. product A at split-off and perform additional processing on products B and C.


 B. product C at split-off and perform additional processing on products A and B.correct
 C. products A, B, and C at split-off.
 D. product B at split-off and perform additional processing on products C and A.
Question was not answered
Correct Answer Explanation:
The relevant amounts are (1) the increase in the sales price of each product as a result
of processing further, and (2) the cost to process each product further. If the increase in
the sales price of a product as a result of processing further is greater than the cost to
process further, the product should be processed further because doing so would
increase operating income.
Product A's unit sales price would be increased by $1.00 ($5.00 − $4.00) by the
additional processing, while the additional processing would cost $0.75 per unit, an
increase of $0.25 operating income per unit, so Product A should be processed further.
Product B's unit sales price would be increased by $1.75 ($4.00 − $2.25) by the
additional processing, while the additional processing would cost $1.20 per unit, an
increase of $0.55 operating income per unit, so Product B should be processed further.
However, the increase in C's unit sales price is only $0.75 ($3.75 − $3.00) compared
with an additional processing cost of $0.90, a decrease of $0.15 operating income per
unit, so Product C should be sold at the split-off point.
Explanation for Choice A:
The relevant amounts are (1) the increase in the sales price of each product as a result
of processing further, and (2) the cost to process each product further. If the increase in
the sales price of a product as a result of processing further is greater than the cost to
process further, the product should be processed further because doing so would
increase operating income.
This is not the decision that would maximize profits.
Explanation for Choice C:
The relevant amounts are (1) the increase in the sales price of each product as a result
of processing further, and (2) the cost to process each product further. If the increase in
the sales price of a product as a result of processing further is greater than the cost to
process further, the product should be processed further because doing so would
increase operating income.
Hock P2 2020
Section C - Decision Analysis.
Answers
This is not the decision that would maximize profits.
Explanation for Choice D:
The relevant amounts are (1) the increase in the sales price of each product as a result
of processing further, and (2) the cost to process each product further. If the increase in
the sales price of a product as a result of processing further is greater than the cost to
process further, the product should be processed further because doing so would
increase operating income.
This is not the decision that would maximize profits.
223. Question ID: ICMA 10.P2.242 (Topic: Marginal Analysis-Other Decisions)
Green Corporation builds custom-designed machinery. A review of selected data and
the company's pricing policies revealed the following.

 A 10% commission is paid on all sales orders.


 Variable and fixed factory overheads total 40% and 20%, respectively, of direct
labor.
 Corporate administrative costs amount to 10% of direct labor.
 When bidding on jobs, Green adds a 25% markup to the total of all factory and
administrative costs to cover income taxes and produce a profit.
 The firm's income tax rate is 40%.
The company expects to operate at a maximum of 80% of practical capacity.
Green recently received an invitation to bid on the manufacture of some custom
machinery for Kennendale, Inc. For this project, Green's production accountants
estimate the material and labor costs will be $66,000 and $120,000, respectively.
Accordingly, Green submitted a bid to Kennendale in the amount of $375,000. Feeling
Green's bid was too high, Kennendale countered with a price of $280,000. Which one of
the following options should be recommended to Green's management?

 A. Reject the counteroffer because the order will decrease operating income.
 B. Accept the counteroffer even though the order will decrease operating income.
 C. Reject the counteroffer even though the order will increase operating income.
 D. Accept the counteroffer because the order will increase operating income.correct
Question was not answered
Correct Answer Explanation:
The company has unused capacity, so any price over and above the company's
variable costs will increase operating income. The company's variable costs are:

Labor $120,000
Hock P2 2020
Section C - Decision Analysis.
Answers
Material 66,000
Variable overhead (40% of direct labor) 48,000
Administrative costs (10% of direct labor) 12,000
Subtotal $246,000
Commission on $280,000 price 28,000
Total variable cost $274,000
The total variable cost is lower than the price of $280,000, so the company should
accept the order at a price of $280,000.
Explanation for Choice A:
The counteroffer should not be rejected.
Explanation for Choice B:
The order will not decrease operating income.
Explanation for Choice C:
The counteroffer should not be rejected.
224. Question ID: CMA 696 4.17 (Topic: Marginal Analysis-Other Decisions)
When management must decide to accept or reject a one-time-only special order, given
sufficient idle capacity, which one of the following is not relevant to the decision?

 A. Absorption costs.correct
 B. Variable costs.
 C. Direct costs.
 D. Differential costs.
Question was not answered
Correct Answer Explanation:
Relevant costs are those costs that will differ among the decision choices. Absorption
costs include some costs (fixed factory overhead) that will be the same whether the
order is accepted or not.
Explanation for Choice B:
Variable costs will be incurred only if the special order is accepted or rejected, so they
are relevant.
Explanation for Choice C:
Hock P2 2020
Section C - Decision Analysis.
Answers
Direct costs are costs that will be incurred only if the special order is accepted or
rejected, so they are relevant.
Explanation for Choice D:
Differential costs will differ among the decision choices and therefore are relevant.
225. Question ID: CIA 1193 IV.24 (Topic: Marginal Analysis-Other Decisions)
There is a market for both product X and product Y. Which of the following costs and
revenues would be most relevant in deciding whether to sell product X or process it
further to make product Y?

 A. Total cost of making X and the revenue from sale of X and Y.


 B. Total cost of making Y and the revenue from sale of Y.
 C. Additional cost of making X, given the cost of making Y, and additional revenue from Y.
 D. Additional cost of making Y, given the cost of making X, and additional revenue from
Y.correct
Question was not answered
Correct Answer Explanation:
Relevant revenues and costs are those that will differ among the related options. The
relevant revenue and cost in this problem are the additional cost of making and selling
product Y and the additional revenue (over and above the revenue from X) to be earned
from selling product Y.
Explanation for Choice A:
The cost of making product X is a sunk cost and not relevant.
Explanation for Choice B:
Only incremental (differential) costs are considered relevant.
Explanation for Choice C:
Product Y can only be processed after product X is completed.
226. Question ID: ICMA 19.P2.007 (Topic: Marginal Analysis-Other Decisions)
A fuel company can sell 8 units of product at a selling price of $450. However, at a
selling price of $445 the company can sell 9 units. What is the marginal revenue that is
derived from selling the 9th unit?

 A. ($5).
 B. $405.correct
 C. $4,005.
 D. $445.
Hock P2 2020
Section C - Decision Analysis.
Answers
Question was not answered
Correct Answer Explanation:
The marginal revenue is the revenue that comes from selling one more unit. If the
company sells 8 units, the selling price is $450 per unit and total revenues are $3,600.
In order to sell 9 units, however, the price must be lowered to $445. At $445 per unit,
the revenue from 9 units is $4,005.
The increase of $405 from $3,600 of revenue at 8 units to $4,005 of revenue at 9 units
is the marginal revenue of the 9th unit.
Explanation for Choice A:
This is simply the decrease in the selling price to sell eight units and the selling price to
sell nine units.
Explanation for Choice C:
This is the total revenue for selling nine units. But, this is not the marginal revenue of
the 9th unit.
Explanation for Choice D:
This is the selling price that is required to be able to sell nine units. But, this is not the
marginal revenue of the 9th unit.
227. Question ID: ICMA 10.P2.241 (Topic: Marginal Analysis-Other Decisions)
Gardener Company currently is using its full capacity of 25,000 machine hours to
manufacture product XR-2000. LJB Corporation placed an order with Gardener for the
manufacture of 1,000 units of KT-6500. LJB would normally manufacture this
component. However, due to a fire at its plant, LJB needs to purchase these units to
continue manufacturing other products. This is a one time special order. The following
reflects unit cost data, and selling prices.

KT-6500 XR-2000
Material $ 27 $ 24
Direct labor 12 10
Variable overhead 6 5
Fixed overhead 48 40
Variable selling & administrative 5 4
Fixed selling & administrative 12 10
Hock P2 2020
Section C - Decision Analysis.
Answers
Normal selling price $125 $105

Machine hours required 3 4


What is the minimum unit price that Gardener should charge LJB to manufacture 1,000
units of KT-6500?

 A. $125.00.
 B. $96.50.correct
 C. $93.00.
 D. $110.00.
Question was not answered
Correct Answer Explanation:
Because Gardner is at full capacity manufacturing only XR-2000, they would need to
pull machine hours away from XR-2000 to produce KT-6500. Therefore, the minimum
price they should charge would be equal to the variable cost of KT-6500 plus the
contribution margin lost on the sacrificed units of XR-2000.
The variable costs for KT-6500 are $50 ($27 material + $12 direct labor + $6 variable
overhead + $5 variable selling & administrative). The total variable cost for 1,000 units
would be $50,000.
The contribution margin on the sacrificed units of XR-2000 is an opportunity cost that
needs to be included. Each unit of KT-6500 requires 3 hours of machine time, so for
1,000 units, Gardner would have to pull 3,000 hours away from producing XR-2000.
3,000 hours divided by the 4 hours used to produce 1 unit of XR-2000 means Gardner
would be giving up 750 units of XR-2000. Contribution margin is calculated as sales
minus variable costs. For XR-2000, the selling price is $105 and variable costs are $43
($24 material + $10 direct labor + $5 variable overhead + $4 variable selling &
administrative), which is a $62 contribution margin per unit. 750 units at a contribution
margin of $62 per unit is $46,500 that would be given up in order to produce the KT-
6500 for this order.
$50,000 of total variable cost for 1,000 units of KT-6500 + $46,500 of lost contribution
margin for XR-2000 equals $96,500 total cost, or $96.50 per unit. Therefore, $96.50 per
unit is the minimum price that Gardener should charge.
Explanation for Choice A:
This is the normal selling price, but not necessarily the minimum unit price that should
be charged for this special order.
Explanation for Choice C:
Hock P2 2020
Section C - Decision Analysis.
Answers
This is the material, direct labor, variable overhead and fixed overhead cost of one unit
of KT-6500 (27 + 12 + 6 + 48). The fixed overhead cost should not be included, while
variable selling and administrative costs plus an opportunity cost for the units of XR-
2000 that cannot be manufactured need to be included.
Explanation for Choice D:
This is the total manufacturing, selling and administrative costs (fixed and variable) for
one unit of KT-6500. However, the fixed costs should not be included, and an
opportunity cost for the units of XR-2000 that cannot be produced should be included.
228. Question ID: ICMA 19.P2.012 (Topic: Marginal Analysis-Other Decisions)
A company currently has a four-stage manufacturing process in the following order:
Processing, Smoothing, Shaping, and Painting. There is a market for the output of each
stage. A newly- appointed management accountant has been examining the company’s
operations, and has prepared the following information below.

Manufacturing Total Selling Price of Incremental Variable Cost Per


Stage Output Stage
Processing $10 $8
Smoothing 12 1
Shaping 18 5
Painting 20 3
Given the above information, selling the output after which one of the following stages
will yield the greatest contribution margin?

 A. Painting.
 B. Shaping.correct
 C. Smoothing.
 D. Processing.
Question was not answered
Correct Answer Explanation:
The company should produce any stage for which the incremental revenue exceeds the
incremental revenue. For the Processing, Smoothing and Shaping stages, the
incremental revenue is greater than the incremental cost. However, for the Painting
stage, the incremental costs are $3, but they can raise the price only by $2. The
company should stop production after the Shaping stage.
Explanation for Choice A:
By producing through the Painting stage, the company is not maximizing its
contribution.
Hock P2 2020
Section C - Decision Analysis.
Answers
Explanation for Choice C:
If the company stopped production after the Smoothing stage, they would not maximize
contribution.
Explanation for Choice D:
If the company stopped production after the Processing stage, they would not maximize
contribution.
229. Question ID: CMA 1286 5.17 (Topic: Marginal Analysis-Other Decisions)
Hermo Company has just completed a hydro-electric plant at a cost of $21,000,000.
The plant will provide the company's power needs for the next 20 years. Hermo will use
only 60% of the power output annually. At this level of capacity, Hermo's annual
operating costs will amount to $1,800,000, of which 80% are fixed.
Quigley Company currently purchases its power from MP Electric at an annual cost of
$1,200,000. Hermo could supply this power, thus increasing output of the plant to 90%
of capacity. This would reduce the estimated life of the plant to 14 years.
If Hermo decides to supply power to Quigley, it wants to be compensated for the
decrease in the life of the plant and the appropriate variable costs. Hermo has decided
that the charge for the decreased life should be based on the original cost of the plant
calculated on a straight-line basis. The minimum annual amount that Hermo would
charge Quigley would be

 A. $990,000.
 B. $630,000.correct
 C. Some amount other than those given.
 D. $450,000.
Question was not answered
Correct Answer Explanation:
This calculation has to include any additional increases in FC and VC that arise from the
sale. The increased FC is $450,000, which was calculated by taking the difference
between depreciation over a 14-year life and depreciation over a 20-year life
($1,500,000 − $1,050,000).
The variable cost of operating the plant at 60% capacity is $360,000. That is calculated
as the total operating cost of $1,800,000 at an output of 60% minus the 80% proportion
of that amount represented by fixed costs, which is $1,440,000. The increase in output
from 60% of capacity to 90% of capacity is a 50% increase (the 30% increase is 50% of
60%). Therefore, variable costs will increase by 50% of $360,000, or $180,000.
Thus, total increased cost would be $630,000 ($450,000 + $180,000).
Explanation for Choice A:
Hock P2 2020
Section C - Decision Analysis.
Answers
This answer results from taking the increase in FC and adding the total VC of operating
at 90% capacity ($450,000 + $540,000). Only the amount of increase in variable cost
should be included in the calculation.
The existing variable cost is $360,000 (20% of $1,800,000). Hermo will need to pay
variable costs of $360,000 whether or not they increase their production to 90% of
capacity. Therefore, only the amount of increase in variable costs that would result from
the increase in usage to 90% is relevant to this decision.
Explanation for Choice C:
The correct answer is among those given.
Explanation for Choice D:
This is the amount of increase in depreciation when the plant is depreciated over a 14-
year life as opposed to a 20-year life. The increased FC is $450,000, which was
calculated by taking the difference between depreciation over a 14-year life and
depreciation over a 20-year life [($21,000,000 ÷ 14-year life) − ($21,000,000 ÷ 20-year
life)].
The minimum annual amount that Hermo would charge Quigley will include this
increase in the fixed cost of depreciation, but it also needs to include the increase in
variable operating costs that will result from increasing the plant's output from 60% of
capacity to 90% of capacity.
230. Question ID: ICMA 10.P2.253 (Topic: Marginal Analysis-Other Decisions)
Reynolds Inc. manufactures several different products, including a premium lawn
fertilizer and weed killer that is popular in hot, dry climates. Reynolds is currently
operating at less than full capacity because of market saturation for lawn fertilizer. Sales
and cost data for a 40-pound bag of Reynolds lawn fertilizer is as follows.

Selling price $18.50


Production cost:
Materials and labor $12.25
Variable overhead 3.75
Allocated fixed overhead 4.00 20.00
Income (loss) per bag $ (1.50)
On the basis of this information, which one of the following alternatives should be
recommended to Reynolds management?

 A. Increase output and spread fixed overhead over a larger volume base.
 B. Drop this product from its product line.
Hock P2 2020
Section C - Decision Analysis.
Answers
 C. Select a different cost driver to allocate its overhead.
 D. Continue to produce and market this product.correct
Question was not answered
Correct Answer Explanation:
Because all of its variable costs and some of its allocated fixed overhead costs as well
are being covered by the sales price, continuing to produce and market the fertilizer is a
good idea, at least for the short term. This product is covering its variable costs and also
a portion of the fixed overhead allocated to it. If this product were dropped, the amount
of allocated fixed overhead costs being covered by it would no longer be covered.
Overall net income would actually be lower unless another more profitable product
could be found to take the place of the fertilizer in Reynolds' product line. Finding a
more profitable product to replace the fertilizer is a possible solution for the long term,
but for the short term, the best solution is to continue to produce and market this
fertilizer.
Explanation for Choice A:
The problem tells us that Raynolds is currently operating at less than full capacity
because of market saturation for lawn fertilizer. Because of this market saturation,
increasing output is not a good solution, since there would not be adequate market
demand for the increased output. The increased output would either pile up, unsold, or it
would have to be sold at a lower price, which could result in a larger loss per unit sold.
Either way, the company's net cash flow could be decreased because of the increased
output without increased demand.
Explanation for Choice B:
If this product is dropped, the company's fixed overhead costs would be unchanged in
total. This product is covering its variable costs and also a portion of the fixed overhead
allocated to it. If this product were dropped, the amount of allocated fixed costs being
covered by it would no longer be covered, and overall net income would actually be
lower.
Explanation for Choice C:
The problem does not give any information on the cost driver being used to allocate
fixed overhead. Therefore, this answer does not make sense.
231. Question ID: ICMA 10.P2.238 (Topic: Marginal Analysis-Other Decisions)
The loss of a key customer has temporarily caused Bedford Machining to have some
excess manufacturing capacity. Bedford is considering the acceptance of a special
order, one that involves Bedford's most popular product. Consider the following types of
costs.

I. Variable costs of the product


Hock P2 2020
Section C - Decision Analysis.
Answers
II. Fixed costs of the product
III. Direct fixed costs associated with the order
IV. Opportunity cost of the temporarily idle capacity
Which one of the following combinations of cost types should be considered in the
special order acceptance decision?

 A. I and II.
 B. I and IV.
 C. II and III.
 D. I, III, and IV.correct
Question was not answered
Correct Answer Explanation:
Variable costs, fixed costs directly associated with the offer, and the opportunity costs of
the idle capacity all need to be taken into consideration, as they are all relevant costs.
The variable costs of the product and the direct fixed costs associated with this order
are costs that will definitely differ between the two alternatives of accepting the special
order or not accepting the special order.
The opportunity cost is a potentially relevant cost that must be considered as well. The
question says only that Bedford has some excess capacity. It does not say that Bedford
has enough excess capacity to produce the special order without having to stop
manufacturing some other order or orders. Therefore, whether or not there is an actual
opportunity cost is unknown. Thus the potential for an opportunity cost must be
considered in deciding whether or not to accept the special order.
Only the fixed costs of the product, which will be there whether the offer is accepted or
not accepted, are not relevant to the decision.
Explanation for Choice A:
When deciding whether to accept or not accept a special order, the focus needs to be
on costs that will be different between one choice and the other. Variable costs will
always be a consideration, as they would not exist if the order is not accepted.
However, fixed costs are not relevant, because they will be the same whether the
special order is accepted or not.
Explanation for Choice B:
These costs are relevant to the decision, but they are not all of the relevant costs.
Explanation for Choice C:
When deciding whether to accept a special order, fixed costs are not relevant as they
will be the same whether the special order is accepted or not. However, direct fixed
Hock P2 2020
Section C - Decision Analysis.
Answers
costs associated with the order will need to be considered, as they would not be
incurred if the special order is not accepted.
232. Question ID: ICMA 19.P2.014 (Topic: Marginal Analysis-Other Decisions)
A company manufactures three products, T1, T2 and T3. Their financial information is
shown below.

T1 T2 T3
Sales $60,000 $90,000 $24,000
Variable costs 36,000 48,000 15,000
Contribution margin 24,000 42,000 9,000
Fixed costs:
Avoidable 9,000 18,000 6,000
Unavoidable 6,000 9,000 5,400
Operating income $9,000 $15,000 ($2,400)
Management is concerned about the financial performance of T3. If the company drops
the T3 product line, the operating income will

 A. decrease by $3,000.correct
 B. decrease by $9,000.
 C. increase by $2,400.
 D. increase by $3,000.
Question was not answered
Correct Answer Explanation:
In evaluating the effect of dropping the T3 product line, we are interested in what costs
will not be incurred if T3 is dropped (these are savings) and what contribution will be lost
if T3 is dropped.
If T3 is dropped, the company will lose $9,000 of contribution that T3 currently
provides.
However, there are also $6,000 of fixed costs that are avoidable if TS were dropped.
These are savings that the company would have if it dropped T3.
Combining these two amounts, we calculate that the operating income of the company
will decrease by $3,000 id the T3 product line is dropped.
Explanation for Choice B:
Hock P2 2020
Section C - Decision Analysis.
Answers
This answer ignores the savings that will result from some of the fixed costs being
avoided if T3 is dropped. See the correct answer for a full explanation of this question.
Explanation for Choice C:
This answer does not take into account the contribution that will be lost and the fixed
costs that will be avoided if T3 is dropped. See the correct answer for a full explanation
of this question.
Explanation for Choice D:
This answer treats the lost contribution and avoided fixed costs incorrectly. See the
correct answer for a full explanation of this question.
233. Question ID: ICMA 10.P2.237 (Topic: Marginal Analysis-Other Decisions)
Basic Computer Company (BCC) sells its micro-computers using bid pricing. It develops
bids on a full cost basis. Full cost includes estimated material, labor, variable
overheads, fixed manufacturing overheads, and reasonable incremental computer
assembly administrative costs, plus a 10% return on full cost. BCC believes bids in
excess of $925 per computer are not likely to be considered.
BCC's current cost structure, based on its normal production levels, is $500 for
materials per computer and $20 per labor hour. Assembly and testing of each computer
requires 12 labor hours. BCC's variable manufacturing overhead is $2 per labor hour,
fixed manufacturing overhead is $3 per labor hour, and incremental administrative costs
are $8 per computer assembled.
The company has received a request from the School Board for 500 computers. BCC's
management expects heavy competition in bidding for this job. As this is a very large
order for BCC, and could lead to other educational institution orders, management is
extremely interested in submitting a bid which would win the job, but at a price high
enough so that current net income will not be unfavorably impacted. Management
believes this order can be absorbed within its current manufacturing facility. Which one
of the following bid prices should be recommended to BCC's management?

 A. $888.80.
 B. $764.00.
 C. $849.20.
 D. $772.00.correct
Question was not answered
Correct Answer Explanation:
The company wants to bid a price that will win the job because it could lead to other
educational institution orders. Therefore, it will not use its usual bidding structure of all
variable and fixed costs plus a 10% profit. However, it does not want its current net
income to decrease because of this job. Therefore, it will need to submit a bid that
Hock P2 2020
Section C - Decision Analysis.
Answers
covers its variable costs. Its variable costs are materials, labor, variable overhead, and
incremental administrative costs. Fixed overhead is not included, because it will not
change as a result of this order. The bid should be:

Labor: 12 hours @ $20 $240


Materials 500
Variable overhead: $2 × 12 labor hours 24
Incremental assembly administrative costs 8
Total bid per computer $772
Explanation for Choice A:
This is the full cost plus a 10% return on the full cost. The company wants to bid a price
that will win the job because it could lead to other educational institution orders.
Therefore, it should not use its usual bidding structure of all variable and fixed costs
plus a 10% profit.
Explanation for Choice B:
This is the total cost for materials, labor and variable overhead. BCC does not want its
current net income to decrease because of this job. Therefore, it will need to submit a
bid that covers all of its variable costs. Its variable costs are materials, labor, variable
overhead, and incremental administrative costs.
Explanation for Choice C:
This is BCC's variable costs plus a 10% profit. The company wants to bid a price that
will win the job because it could lead to other educational institution orders. Therefore, it
should not include its usual 10% profit.
234. Question ID: CMA 1296 4.4 (Topic: Marginal Analysis-Other Decisions)
Kator Co. is a manufacturer of industrial components. One of their products that is used
as a subcomponent in auto manufacturing is KB-96. This product has the following
financial structure per unit:

Selling price $150


Direct materials $20
Direct labor 15
Variable manufacturing overhead 12
Fixed manufacturing overhead 30
Shipping and handling 3
Hock P2 2020
Section C - Decision Analysis.
Answers
Fixed selling and administrative 10
Total costs $90
Kator Co. has received a special, one-time, order for 1,000 KB-96 parts. Assume that
Kator is operating at full capacity and that the contribution margin of the output that
would be displaced by the special order is $10,000. The minimum price that is
acceptable, using the original data, for this one-time special order is in excess of

 A. $87
 B. $60correct
 C. $70
 D. $100
Question was not answered
Correct Answer Explanation:
If the company is operating at full capacity, then in order to produce the special order
the company will lose $10,000 of potential contribution, or $10 per unit ($10,000 lost
contribution ÷ 1,000 units in the special order). This lost opportunity has to be reflected
in the price that will be charged for the special order, which will be $20 DM + $15 DL +
$12 VOH + $3 shipping and handling + $10 opportunity cost, for a total of $60. $60 is
the minimum price that is acceptable.
Explanation for Choice A:
An answer of $87 includes fixed manufacturing, but does not include shipping and
handling.
Explanation for Choice C:
This answer is including fixed selling and administrative cost which are unavoidable and
should not be included.
Explanation for Choice D:
The answer $100 is calculated by the use of the full cost ($90 per unit) plus the
opportunity cost of $10 per unit to calculate the minimum acceptable price ($90 full cost
of KB-96 + $10 opportunity cost). The answer should include only incremental cost plus
the opportunity cost.
235. Question ID: ICMA 10.P2.249 (Topic: Marginal Analysis-Other Decisions)
Oakes Inc. manufactured 40,000 gallons of Mononate and 60,000 gallons of Beracyl in
a joint production process, incurring $250,000 of joint costs. Oakes allocates joint costs
based on the physical volume of each product produced. Mononate and Beracyl can
each be sold at the split-off point in a semifinished state or, alternatively, processed
further. Additional data about the two products are as follows.
Hock P2 2020
Section C - Decision Analysis.
Answers
Mononate Beracyl
Sales price per gallon at split-off $ 7 $15
Sales price per gallon if processed further $10 $18
Variable production costs if processed further $125,000 $115,000
An assistant in the company's cost accounting department was overheard saying that
"....when both joint and separable costs are considered, the firm has no business
processing either product beyond the split-off point. The extra revenue is simply not
worth the effort." Which of the following strategies should be recommended for Oakes?

 A. Sell Mononate at split-off; process Beracyl further.correct


 B. Sell Mononate at split-off; sell Beracyl at splitoff.
 C. Process Mononate further; process Beracyl further.
 D. Process Mononate further; sell Beracyl at split-off.
Question was not answered
Correct Answer Explanation:
The additional revenue to be received from processing each product further should be
compared with the costs of processing the product further. The joint costs are irrelevant
to this decision because they have already been incurred.
Mononate:
40,000 gallons × $3 additional revenue = $120,000 additional revenue if processed
further
Variable cost to process further = 125,000
Loss from processing further = $(5,000)
Beracyl:
60,000 gallons × $3 additional revenue = $180,000 additional revenue if processed
further
Variable cost to process further = $115,000
Profit from processing further = $65,000
Mononate should not be processed further but should be sold at the split-off point,
because to process it further would mean the company would lose $5,000. However,
Beracyl should be processed further because the company can earn additional
operating income of $65,000 by processing it further.
Explanation for Choice B:
This would not result in the highest operating income.
Explanation for Choice C:
This would not result in the highest operating income.
Hock P2 2020
Section C - Decision Analysis.
Answers
Explanation for Choice D:
This would not result in the highest operating income.
236. Question ID: ICMA 13.P2.046 (Topic: Marginal Analysis-Other Decisions)
Ross Inc. uses a joint process which yields two products, X and Y. Each product can be
sold at its split-off point or processed further. All the additional processing costs are
variable and can be traced to each product. Joint production costs are $35,000. Other
sales and cost data are as follows.

Product X Product Y
Sales value at split-off point $60,000 $35,000
Final sales value if processed further 80,000 50,000
Additional costs beyond split-off 14,000 18,000
Management wants to know whether to sell each product at the split-off point or to
process the products further. Which one of the following options should be
recommended to Ross' management?

 A. Process Product X further and sell Product Y at split-off.correct


 B. Process both products further.
 C. Sell Product X at split-off and process Product Y further.
 D. Sell both products at the split-off point.
Question was not answered
Correct Answer Explanation:
For each product, the increased revenues attainable by processing further should be
balanced against the increased costs to process further.
The increased revenue from processing Product X further is $20,000 ($80,000 −
$60,000), and the cost to process further is $14,000. The additional revenue is $6,000
greater than the additional cost. Thus Product X should be processed further because
doing so will result in $6,000 additional net income before tax.
The increased revenue from processing Product Y further will be $15,000 ($50,000 −
$35,000), and the cost to process further is $18,000. The additional revenue is $3,000
less than the additional cost. Thus Product Y should not be processed further because
doing so will result in a $3,000 decrease in net income before tax.
Explanation for Choice B:
For each product, the increased revenues attainable by processing further should be
balanced against the increased costs to process further.
Hock P2 2020
Section C - Decision Analysis.
Answers
The increased revenue from processing Product X further is $20,000 ($80,000 −
$60,000), and the cost to process further is $14,000. The additional revenue is $6,000
greater than the additional cost. Thus Product X should be processed further because
doing so will result in $6,000 additional net income before tax.
The increased revenue from processing Product Y further will be $15,000 ($50,000 −
$35,000), and the cost to process further is $18,000. The additional revenue is $3,000
less than the additional cost. Thus Product Y should not be processed further because
doing so will result in a $3,000 decrease in net income before tax.
Explanation for Choice C:
For each product, the increased revenues attainable by processing further should be
balanced against the increased costs to process further.
The increased revenue from processing Product X further is $20,000 ($80,000 −
$60,000), and the cost to process further is $14,000. The additional revenue is $6,000
greater than the additional cost. Thus Product X should be processed further because
doing so will result in $6,000 additional net income before tax.
The increased revenue from processing Product Y further will be $15,000 ($50,000 −
$35,000), and the cost to process further is $18,000. The additional revenue is $3,000
less than the additional cost. Thus Product Y should not be processed further because
doing so will result in a $3,000 decrease in net income before tax.
Explanation for Choice D:
For each product, the increased revenues attainable by processing further should be
balanced against the increased costs to process further.
The increased revenue from processing Product X further is $20,000 ($80,000 −
$60,000), and the cost to process further is $14,000. The additional revenue is $6,000
greater than the additional cost. Thus Product X should be processed further because
doing so will result in $6,000 additional net income before tax.
The increased revenue from processing Product Y further will be $15,000 ($50,000 −
$35,000), and the cost to process further is $18,000. The additional revenue is $3,000
less than the additional cost. Thus Product Y should not be processed further because
doing so will result in a $3,000 decrease in net income before tax.
237. Question ID: ICMA 13.P2.044 (Topic: Marginal Analysis-Other Decisions)
National Technology Corporation manufactures integrated computer components. Its
unit cost structure, based upon a volume of 300,000 units, is as follows.

Variable Costs Fixed Costs Total Costs


Direct material $ 3.50 $ 3.50
Direct labor 9.00 9.00
Hock P2 2020
Section C - Decision Analysis.
Answers
Packaging 2.00 2.00
Manufacturing O/H 3.00 $ 6.50 9.50
Marketing costs 2.50 8.00 10.50
Administrative costs 4.00 4.50 8.50
Total costs $24.00 $19.00 $43.00
A foreign company recently approached National with an order of 50,000 units of a
specially designed component at $35 per unit. The order will require specialized
procurement costs of $150,000 and only one-half of the variable costs associated with
the administrative area will be needed. Otherwise, cost behavior will remain the same.
Adequate capacity is available to handle this request. What is the relevant unit cost to
be considered by National in making a decision on this offer?

 A. $25.00.correct
 B. $43.00.
 C. $24.00.
 D. $22.00.
Question was not answered
Correct Answer Explanation:
The relevant unit cost will be the variable costs per unit plus the specialized
procurement costs per unit.
Variable costs per unit, adjusted for the fact that only one-half of the variable costs
associated with the administrative area will be incurred, are:

Direct material $ 3.50


Direct labor 9.00
Packaging 2.00
Variable manufacturing O/H 3.00
Variable marketing costs 2.50
Variable administrative costs 2.00
Total variable costs $22.00
Specialized procurement costs will be $150,000 in total. On a per-unit basis, this equals
$3 per unit ($150,000 ÷ 50,000 units).
Hock P2 2020
Section C - Decision Analysis.
Answers
Therefore, the total relevant cost per unit to be considered by National in making a
decision on this offer is $22 variable costs plus $3 fixed costs, for a total of $25.
Explanation for Choice B:
The relevant unit cost will be the variable costs per unit plus the specialized
procurement costs per unit. This answer includes fixed costs as well.
Explanation for Choice C:
The relevant unit cost will be the variable costs per unit plus the specialized
procurement costs per unit. This answer does not include the specialized procurement
costs per unit and it also includes too much administrative costs per unit as we are told
in the question that administrative costs for this order are $2 less than usual.
Explanation for Choice D:
The relevant unit cost will be the variable costs per unit plus the specialized
procurement costs per unit. This answer does not include the specialized procurement
costs per unit.
238. Question ID: ICMA 1603.P2.025 (Topic: Marginal Analysis-Other Decisions)
A company currently sells 6,000 units per month and has received a special order from
an international customer. The international customer would like to purchase 1,500 units
for a price of $80 per unit. The company currently sells the product to regular customers
for $95 per unit. The company has excess capacity to produce the special order. The
product unit cost is shown below.

Direct materials $49.50


Direct labor 16.50
Variable overhead 9.50
Fixed overhead 3.50
Fixed manufacturing overhead totals $35,000 per month. Management has determined
that the additional shipping costs for the international delivery would be $4 per unit.
Should the company accept the special order?

 A. No, because operating income will decrease by $4,500.


 B. No, because operating income will decrease by $21,000.
 C. Yes, because operating income will increase by $6,750.
 D. Yes, because operating income will increase by $750.correct
Question was not answered
Correct Answer Explanation:
Hock P2 2020
Section C - Decision Analysis.
Answers
The company has excess capacity, so it will not have to forgo manufacturing any other
orders and thus will not have any opportunity cost for accepting this order.
The contribution margin per unit will be:

Revenue $80.00
Direct materials (49.50)
Direct labor (16.50)
Variable overhead ( 9.50)
Shipping cost ( 4.00)
Contribution margin $ 0.50
For an order of 1,500 units, operating income will increase by 1,500 × $0.50 = $750.
Explanation for Choice A:
This answer results from including an allocation of fixed overhead to the special order
units. Fixed overhead does not change in total as long as the activity level remains
within the relevant range. The question does not say that the special order will cause
activity to exceed the relevant range, so we assume it will not. Therefore, it is not
appropriate to include an allocation of fixed overhead to the special order units in
calculating the effect on operating income.
Explanation for Choice B:
This answer results from two errors:
(1) Including direct materials and direct material only, plus shipping costs for the special
order in the calculation of the contribution margin per unit for the special order. The
variable overhead should also be included.
(2) Including an opportunity cost of $29 per unit (regular selling price less direct
materials and direct material) in the calculation of the contribution margin per unit for the
special order. The company has excess capacity, so it will not have to forgo
manufacturing any other orders and thus will not have any opportunity cost for
accepting this order.
Explanation for Choice C:
This answer results from failing to include the additional shipping costs for the
international delivery as variable costs for the special order.
239. Question ID: CMA Sample Q4.2 (Topic: Marginal Analysis-Other Decisions)
All of the following costs are relevant to a decision to accept or reject an order except

 A. Sunk costs.correct
Hock P2 2020
Section C - Decision Analysis.
Answers
 B. Replacement costs.
 C. Out-of-pocket costs.
 D. Differential costs.
Question was not answered
Correct Answer Explanation:
Sunk costs are costs that have already been incurred. They will not vary with the related
options, therefore they are are irrelevant.
Explanation for Choice B:
Replacement cost is the cost to replace an existing asset. It is relevant to the decision
making process.
Explanation for Choice C:
Out-of-pocket costs are actual cash outflows that have to be incurred and are relevant
to the decision making process.
Explanation for Choice D:
Differential costs are costs that differ between alternatives. They are relevant to the
decision whether to accept or reject.
240. Question ID: ICMA 10.P1.185 (Topic: Marginal Analysis-Other Decisions)
Fitzpatrick Corporation uses a joint manufacturing process in the production of two
products, Gummo and Xylo. Each batch in the joint manufacturing process yields 5,000
pounds of an intermediate material, Valdene, at a cost of $20,000. Each batch of
Gummo uses 60% of the Valdene and incurs $10,000 of separate costs. The resulting
3,000 pounds of Gummo sells for $10 per pound. The remaining Valdene is used in the
production of Xylo which incurs $12,000 of separable costs per batch. Each batch of
Xylo yields 2,000 pounds and sells for $12 per pound. Fitzpatrick uses the net realizable
value method to allocate the joint material costs. The company is debating whether or
not to process Xylo further into a new product, Zinten, which would incur an additional
$4,000 in costs and sell for $15 per pound. If Zinten is produced, income would increase
by

 A. $5,760 per batch produced.


 B. $26,000 per batch produced.
 C. $14,000 per batch produced.
 D. $2,000 per batch produced.correct
Question was not answered
Correct Answer Explanation:
Hock P2 2020
Section C - Decision Analysis.
Answers
This seems like a difficult question, but it really is not. All of the information about the
joint costs and their allocation is irrelevant. The only thing we need to determine is how
much income would increase if the Xylo is processed further into Zinten instead of being
sold as Xylo without further processing. Therefore, this is a sell or process further
decision, and the only amounts that are relevant are the price Xylo sells for per pound,
the price Zinten would sell for per pound, and the additional cost to process Xylo into
Zinten.
Xylo sells for $12 per pound, and 2,000 pounds are manufactured in each batch, so
total revenue for Xylo would be $12 × 2,000, or $24,000. Total cost for Xylo at this point
is irrelevant, because it will be the same whether Xylo is processed further or not
processed further. If Xylo is processed further into Zinten, total revenue for Zinten would
be $15 × 2,000, or $30,000. The additional cost to process further is $4,000. Therefore,
the difference in net income per batch if Zinten is manufactured instead of selling the
product as Xylo is ($30,000 − $4,000) − $24,000, or $26,000 − $24,000, which is $2,000
per batch.
Explanation for Choice A:
This is not the correct answer. Please see the correct answer for an explanation.
We have been unable to determine how to calculate this incorrect answer choice. If you
have calculated it, please let us know how you did it so we can create a full explanation
of why this answer choice is incorrect. Please send us an email at
support@hockinternational.com. Include the full Question ID number and the actual
incorrect answer choice -- not its letter, because that can change with every study
session created. The Question ID number appears at the top of the question. Thank you
in advance for helping us to make your HOCK study materials better.
Explanation for Choice B:
$26,000 per batch produced is the revenue of $30,000 from selling 2,000 pounds of
Zinten minus the $4,000 it would cost to process the Xylo into Zinten. This does not take
into account the revenue that would be earned by selling the product as Xylo without
further processing.
Explanation for Choice C:
This answer results from summing the separable cost of Xylo ($12,000) and the
additional cost to process Xylo into Zinten ($4,000) and dividing the result by the
number of pounds of Zinten produced (assumed to be 2,000, the same as the number
of pounds of Xylo) to get a cost per pound of $8. Then the selling price of Zinten ($15
per pound) minus the $8 cost per pound is multiplied by 2,000 pounds to equal $14,000.
This is incorrect because (1) the separable cost of Xylo ($12,000) used in calculating
this incorrect answer is irrelevant to the decision because that same cost will be
incurred regardless of the decision made; and (2) the revenue from producing and
selling Xylo ($12 × 2,000) is not used in calculating this incorrect answer. The revenue
Hock P2 2020
Section C - Decision Analysis.
Answers
from Xylo is needed because the question is asking how much net income would
increase if Zinten is produced and sold instead of Xylo. To calculate an amount of
change, we must compare the new amount with the old amount.
241. Question ID: ICMA 10.P2.254 (Topic: Marginal Analysis-Other Decisions)
Following are the operating results of the two segments of Parklin Corporation.

Segment A Segment B Total


Sales $10,000 $15,000 $25,000
Variable cost of goods sold 4,000 8,500 12,500
Fixed costs of goods sold 1,500 2,500 4,000
Gross margin $ 4,500 4,000 8,500
Variable selling and administrative 2,000 3,000 5,000
Fixed selling and administrative 1,500 1,500 3,000
Operating income (loss) $ 1,000 $ (500) $ 500
Variable costs of goods sold are directly related to the operating segments. Fixed costs
of goods sold are allocated to each segment based on the number of employees. Fixed
selling and administrative expenses are allocated equally. If Segment B is eliminated,
$1,500 of fixed costs of goods sold would be eliminated. Assuming Segment B is
closed, the effect on operating income would be

 A. a decrease of $2,000.correct
 B. a decrease of $2,500.
 C. an increase of $500.
 D. an increase of $2,000.
Question was not answered
Correct Answer Explanation:
The fastest way to calculate the answer to this question is to do an incremental
analysis, as follows.

Fixed cost eliminated if Segment B is closed $ 1,500


Less: Contribution margin lost if Segment B is closed (3,500) *
Change in operating income $(2,000)
*$15,000 in sales less $8,500 in variable COGS less $3,000 in variable selling and
administrative expense.
Hock P2 2020
Section C - Decision Analysis.
Answers
Another way to calculate the answer is to recalculate the income statement based on
having only one segment, since Segment B will be closed. Segment B's sales and
variable costs will be gone, but only $1,500 of Segment B's fixed cost of goods sold will
be eliminated after Segment B is closed. Since Segment B's fixed cost of goods sold is
$2,500, $1,000 of fixed costs will remain and be absorbed by Segment A. $1,500 of
Segment B's fixed selling and administrative expenses will also remain after Segment B
is closed.

Sales $10,000
Variable cost of goods sold 4,000
Fixed cost of goods sold 2,500
Gross Margin $ 3,500

Variable selling and administrative 2,000


Fixed selling and administrative 3,000
Operating income (loss) $(1,500)
Total operating income including Segment B is $500. Without Segment B, total
operating income will decrease to a loss of $1,500, which is a $2,000 decrease.
Explanation for Choice B:
This answer results from assuming that $1,500 of Fixed COGS would remain after
Segment B is closed, rather than $1,000 ($2,500 − $1,500 of eliminated costs).
Explanation for Choice C:
This assumes the $500 loss for Segment B would disappear and have no impact on
Segment A. However, the fixed costs would still remain in total for selling and
administrative and in part for COGS.
Explanation for Choice D:
This answer results from eliminating fixed cost of goods sold in total. However, all of
Segment A’s fixed cost of goods sold would remain, and a portion of B’s fixed cost of
goods sold would remain.
242. Question ID: ICMA 1603.P2.032 (Topic: Marginal Analysis-Other Decisions)
A retailer planned to purchase 55,000 units and sell 50,000 units, yielding the following
operating income.

Sales $50,000,000
Cost of goods sold 33,000,000
Hock P2 2020
Section C - Decision Analysis.
Answers
Variable selling costs 5,000,000
Fixed selling and administrative costs 7,000,000
Operating income $ 5,000,000
The company expects to receive an additional order that would allow it to sell all 55,000
units it purchased. If the company accepts this order, its operating income will be

 A. $9,500,000.
 B. $6,700,000.
 C. $5,500,000.
 D. $6,200,000.correct
Question was not answered
Correct Answer Explanation:
The contribution margin per unit is $240, calculated as $50,000,000 sales revenue −
$33,000,000 COGS − $5,000,000 variable selling cost, which equals $12,000,000,
divided by 50,000 units. Selling an additional 5,000 units would increase operating
income by 5,000 × $240, or $1,200,000. Current operating income of $5,000,000 plus
the additional $1,200,000 equals operating income of $6,200,000.
Explanation for Choice A:
This answer results from calculating the revised operating income without including the
increase in cost of goods sold that is to be expected as a result of the additional sale.
Explanation for Choice B:
This answer results from calculating the revised operating income without including the
increase in variable selling costs that is to be expected as a result of the additional sale.
Explanation for Choice C:
This answer results from increasing fixed selling and administrative costs by $700,000
to reflect the increased sales revenue. Fixed costs do not change in total with changes
in activity such as sales,1 and so the fixed cost should not be increased in calculating
the revised operating income with the additional sale.
1Fixed costs do not change in total with changes in activity as long as the activity
remains within the relevant range. Unless an exam question states otherwise, we
assume that activity that increases or decreases remains within the relevant range and
thus fixed costs do not change as a result of the change in activity.
243. Question ID: ICMA 1603.P2.029 (Topic: Marginal Analysis-Other Decisions)
Which one of the following factors is least likely to be taken into account when
analyzing a company’s selling expenses?
Hock P2 2020
Section C - Decision Analysis.
Answers
 A. Possibility that the expense, such as sales promotion expense, will yield future benefits.
 B. Changes in the unit selling prices.correct
 C. Variance of the expense compared with prior years.
 D. Percentage of variable and fixed selling expenses in relation to revenue.
Question was not answered
Correct Answer Explanation:
Changes in the per unit selling prices are less likely than the other answer choices to be
taken into account when analyzing a company’s selling expenses because the unit
selling price represents revenue, not a selling expense.
It is true that if sales commissions are tied to selling price, changes in the unit selling
prices will affect the amount of sales commissions. However, this answer is the best
answer to the question from among those given, even if it is not a perfect answer.
Explanation for Choice A:
The possibility that an expense such as sales promotion expense will yield future
benefits is likely to be taken into account when analyzing a company’s selling expenses
because the future benefits need to be taken into consideration in determining whether
an expense is justified.
Explanation for Choice C:
The variance of the expense compared with prior years is likely to be taken into account
when analyzing a company’s selling expenses because the trend of expenses over time
is an important factor in determining whether the expenses are reasonable.
Furthermore, an unreasonable increase in selling expenses over time might be an
indication of fraud.
Explanation for Choice D:
The percentage of variable and fixed selling expenses in relation to revenue is an
important factor in analyzing selling expenses, as it impacts operating income.
244. Question ID: CIA 596 III.80 (Topic: Marginal Analysis-Other Decisions)
A manufacturing company's primary goals include product quality and customer
satisfaction. The company sells a product, for which the market demand is strong, for
$50 per unit. Due to the capacity constraints in the Production Department, only
300,000 units can be produced per year. The current defective rate is 12% (i.e., of the
300,000 units produced, only 264,000 units are sold and 36,000 units are scrapped).
There is no revenue recovery when defective units are scrapped. The full manufacturing
cost of a unit is $29.50, including:

Direct materials $17.50


Direct labor 4.00
Hock P2 2020
Section C - Decision Analysis.
Answers
Fixed manufacturing overhead 8.00
The company's designers have estimated that the defective rate can be reduced to 2%
by using a different direct material. However, this will increase the direct materials cost
by $2.50 per unit to $20 per unit. The net benefit of using the new material to
manufacture the product will be:

 A. $1,425,000
 B. $750,000correct
 C. $(120,000)
 D. $120,000
Question was not answered
Correct Answer Explanation:
If the defect rate is dropped from 12% to 2%, 30,000 more units will be able to be sold
(300,000 × [12% − 2%]). At a sales price of $50, this will lead to $1,500,000 in additional
revenue for the company ($50 × 30,000). However, in order to obtain this increase in
revenue caused by the decrease in the defective rate, the cost of production for all units
produced will increase by $2.50 per unit. Since 300,000 units are produced a year, this
is an incremental production cost of $750,000 ($2.50 × 300,000). In total, the better
materials that would reduce the defect rate would create a net benefit of $750,000
($1,500,000 increased revenue minus $750,000 increased cost).
Explanation for Choice A:
This answer assumes that the more expensive materials are used only for the additional
good units produced. The more expensive materials need to be used for every unit
produced during the period.
Explanation for Choice C:
This answer results from applying the cost per unit to only the good units and then
subtracting the operating income calculated using the more expensive direct materials
from the operating income calculated using the less expensive direct materials. This is
incorrect for two reasons: (1) The costs to manufacture the defective units have also
been spent and must be included in the calculations. (2) the operating income using he
less expensive direct materials should be subtracted from the operating income using
the more expensive direct materials.
Explanation for Choice D:
This answer results from applying the cost per unit to only the good units. However, the
costs to manufacture the defective units have also been spent and must be included in
the calculations.
245. Question ID: ICMA 10.P2.257 (Topic: Marginal Analysis-Other Decisions)
Hock P2 2020
Section C - Decision Analysis.
Answers
The Furniture Company currently has three divisions: Maple, Oak, and Cherry. The oak
furniture line does not seem to be doing well and the president of the company is
considering dropping this line. If it is dropped, the revenues associated with the Oak
Division will be lost and the related variable costs saved. Also, 50% of the fixed costs
allocated to the oak furniture line would be eliminated. The income statements, by
divisions, are as follows.

Maple Oak Cherry


Sales $55,000 $85,000 $100,000
Variable Costs 40,000 72,000 82,000
Contribution Margin 15,000 13,000 18,000
Fixed costs 10,000 14,000 10,200
Operating profit (loss) $ 5,000 $(1,000) $ 7,800
Which one of the following options should be recommended to the president of the
company?

 A. Continue operating the Oak Division as discontinuance would result in a total operating
loss of $1,200.
 B. Discontinue the Oak Division which would result in a $7,000 increase in operating
profits.
 C. Discontinue the Oak Division which would result in a $1,000 increase in operating
profits.
 D. Continue operating the Oak Division as discontinuance would result in a $6,000 decline
in operating profits.correct
Question was not answered
Correct Answer Explanation:
Without the Oak Division, the contribution margin for the company as a whole would be
$13,000 lower, the amount of the Oak Division's contribution margin ($85,000 −
$72,000). $7,000 of fixed costs would be saved (50% of the fixed costs allocated to the
oak furniture line) by discontinuing the Oak Division. Therefore, without the Oak
Division, the operating profit for the company as a whole would be decreased by
$13,000 and increased by $7,000. Those two amounts equal a net decrease of $6,000
in profits. Therefore, the company should continue operating the Oak Division, because
operating profits would decrease by $6,000 if the Oak Division were dropped.
Explanation for Choice A:
This answer assumes that all of the fixed costs presently being allocated to the Oak
Division would continue to be incurred if the division were discontinued. $7,000 of fixed
Hock P2 2020
Section C - Decision Analysis.
Answers
costs would be saved (50% of the fixed costs allocated to the oak furniture line) by
discontinuing the Oak Division.
Explanation for Choice B:
This is the amount of the fixed costs that would be saved if the Oak Division were
discontinued. However, the company would also lose the contribution margin being
provided by the Oak Division.
Explanation for Choice C:
This answer assumes that all of the fixed costs allocated to the Oak Division would be
saved if the division were discontinued. Only $7,000 of fixed costs would be saved (50%
of the fixed costs allocated to the oak furniture line) by discontinuing the Oak Division.
246. Question ID: CIA 1189 IV.25 (Topic: Marginal Analysis-Other Decisions)
Which costs are relevant to the decision to further process a product beyond its current
state?

 A. Absorption costs.
 B. Joint costs.
 C. Fixed factory overhead.
 D. Incremental costs.correct
Question was not answered
Correct Answer Explanation:
The only costs that are relevant to a decision to sell or process further are incremental
costs. Incremental revenues are also relevant. The increased (incremental) revenues
attainable by processing further should be compared with the increased (incremental)
costs to process further. If the incremental revenues are greater than the incremental
costs, net operating income will be increased as a result of the additional processing,
assuming there is a market for the product after the additional processing. The amount
of increase in net operating income as a result of the additional processing is the basis
for the decision to sell or process further.
Explanation for Choice A:
Absorption costing includes all manufacturing cost (variable and fixed). Not all
manufacturing costs will be relevant to the decision whether to process beyond its
current state.
Explanation for Choice B:
Joint costs are not relevant to the decision because they are sunk costs. They have
already been incurred.
Explanation for Choice C:
Hock P2 2020
Section C - Decision Analysis.
Answers
Fixed factory overhead would probably not be relevant to the decision of whether to
process a product beyond its current state. It would be relevent only if it would change
as a result of a decision to process further. There is a better answer choice available.
247. Question ID: ICMA 1603.P2.057 (Topic: Marginal Analysis-Other Decisions)
A tennis equipment company produces two lines of tennis shoes, Professional and
Amateur. Income statement data for the tennis shoes is shown below.

Professional Amateur Total


Sales $550,000 $750,000 $1,300,000
Variable costs 275,000 400,000 675,000
Direct fixed costs 100,000 300,000 400,000
Allocated fixed costs 37,500 112,500 150,000
Operating income $137,500 $(62,500) $ 75,000
Since the Amateur line shows a loss, the company is considering eliminating this line of
tennis shoes. Based on the data provided, should the company drop the Amateur tennis
shoe line?

 A. No, operating income will decrease by $350,000.


 B. Yes, operating income will increase by $62,500.
 C. Yes, operating income will increase by $25,000.
 D. No, operating income will decrease by $50,000.correct
Question was not answered
Correct Answer Explanation:
The Amateur line is providing a $50,000 contribution toward covering the company's
common (allocated) fixed costs, calculated as $750,000 sales revenue less $400,000
variable costs less $300,000 direct fixed costs. Those are the items that would go away
if the Amateur line were dropped.
The Amateur line's apparent operating loss is caused by the $112,500 of common fixed
costs being allocated to it. The common fixed costs that are being allocated would not
change in total for the company if the Amateur line were dropped. Those common fixed
costs would continue to be paid if the Amateur line were dropped—they would just be
reallocated to the remaining Professional line.
Therefore, the company should not drop the Amateur line because doing so would
cause operating income to decrease by $50,000.
Explanation for Choice A:
Hock P2 2020
Section C - Decision Analysis.
Answers
This is the Amateur line's sales revenue of $750,000 less its variable costs of $400,000,
both of which would go away if the Amateur line is dropped. However, the Amateur
line's direct fixed costs of $300,000 would also go away because they are direct costs,
not allocated costs. Thus the direct fixed costs need to be included in the calculation of
the amount by which operating income would decrease if the Amateur line is eliminated.
Explanation for Choice B:
This is the amount of operating loss attributed to the Amateur line after allocation of
common fixed costs. Those common fixed costs would continue to be paid if the
Amateur line were dropped—they would just be reallocated to the remaining
Professional line. Since the common fixed costs would not go away, they are not
relevant to the decision and should not be considered in the analysis of whether to drop
the Amateur tennis shoe line.
Explanation for Choice C:
This is the operating income the company would earn without the Amateur line, not the
amount of increase or decrease in operating income.
248. Question ID: ICMA 10.P2.240 (Topic: Marginal Analysis-Other Decisions)
Two months ago, Hickory Corporation purchased 4,500 pounds of Kaylene at a cost of
$15,300. The market for this product has become very strong, with the price jumping to
$4.05 per pound. Because of the demand, Hickory can buy or sell Kaylene at this price.
Hickory recently received a special order inquiry that would require the use of 4,200
pounds of Kaylene. In deciding whether to accept the order, management must
evaluate a number of decision factors. Without regard to income taxes, which one of the
following combination of factors correctly depicts relevant and irrelevant decision
factors, respectively?

 A. Relevant Decision Factor: 4,500 pounds of Kaylene; Irrelevant Decision Factor:


Remaining 300 pounds of Kaylene.
 B. Relevant Decision Factor: Purchase price of $3.40 per lb.; Irrelevant Decision Factor:
Market price of $4.05 per lb.
 C. Relevant Decision Factor: Remaining 300 pounds of Kaylene; Irrelevant Decision
Factor: Market price of $4.05 per lb.
 D. Relevant Decision Factor: Market price of $4.05 per lb.; Irrelevant Decision Factor:
Purchase price of $3.40 per lb.correct
Question was not answered
Correct Answer Explanation:
Relevant information for decision-making is expected future revenues and costs. The
relevant cost is the current price, because that is what it would cost to replace any
Kaylene used from inventory and also the cost to purchase Kaylene today if needed for
Hock P2 2020
Section C - Decision Analysis.
Answers
current production. The price paid 2 months ago has no bearing on the decision to be
made.
Explanation for Choice A:
The 4,500 pounds of Kaylene purchased in the past is not relevant to the decision.
Relevant information for decision-making is expected future revenues and costs.
Furthermore, the problem does not indicate how much of the 4,500 pounds purchased
is still on hand, so it is not possible to know how much would remain after using 4,200
pounds for the special order.
Explanation for Choice B:
Relevant information for decision-making is expected future revenues and costs.
Historical prices are not relevant to the decision to be made.
Explanation for Choice C:
The potentially remaining 300 pounds of Kaylene is not relevant. The problem does not
indicate how much of the 4,500 pounds purchased is still on hand, so it is not possible
to know how much would remain after using 4,200 pounds.
The current market price is absolutely relevant, because that is what it would cost to
replace any Kaylene used from inventory and also the cost to replace any Kaylene used
from inventory and also the cost to purchase Kaylene today if needed for current
production. The variable cost of the special order would need to include the cost of the
material at its current price.
249. Question ID: CMA 692 4.27 (Topic: Marginal Analysis-Other Decisions)
ABD Realty manages five apartment complexes in a three-state area. Summary income
statements for each apartment complex are shown as follows.

ABD Realty
Summary Income Statements
(in thousands)
One Two Three Four Five
Rental income $1,000 $1,210 $2,347 $1,878 $1,065
Expenses 800 1,300 2,600 2,400 1,300
Profit $ 200 $ (90) $ (253) $ (522) $ (235)
Included in the expenses is $1,200,000 of corporate overhead allocated to the
apartment complexes based on rental income. The apartment complex(es) that ABD
should consider selling is (are)

 A. Apartment complexes Three, Four, and Five.


 B. Apartment complex Four.
Hock P2 2020
Section C - Decision Analysis.
Answers
 C. Apartment complexes Two, Three, Four, and Five.
 D. Apartment complexes Four and Five.correct
Question was not answered
Correct Answer Explanation:
ABD Realty needs to analyze the contribution per complex before corporate overhead.
This is done to see which complex is able to cover its own operating expenses. It would
be best to allocate this corporate overhead based on total Rental Income of $7,500.

ABD Realty
Summary Income Statements
(in thousands)
One Two Three Four Five Total
Rental income $1,000 $1,210 $2,347 $1,878 $1,065 $7,500
% of total 13.33% 16.14% 31.29% 25.04% 14.20%
Overhead allocated $160 $194 $375 $301 $170 $1,200
The next step is to add back the corporate overhead allocation to the individual segment
operating profit to see operating income per complex exclusive of the corporate
overhead allocation.
Profit + Total Overhead Allocation = Operating income

ABD Realty
Summary Income Statements
(in thousands)
One Two Three Four Five
Profit $ 200 $(90) $(253) $(522) $(235)
Plus: Overhead allocated 160 194 375 301 170
Operating income $ 360 $104 $ 122 $(221) $ (65)
Based on the above information, complexes Four and Five are still operating at a loss,
so therefore, need to be sold.
Explanation for Choice A:
The solution is to calculate operating income before corporate overhead allocation. The
complexes with operating losses before the corporate overhead allocation need to be
sold because they are not contributing anything to covering corporate overhead. The
ones with operating income before the corporate overhead allocation should be retained
because they are contributing some income to covering the corporate overhead.
Hock P2 2020
Section C - Decision Analysis.
Answers
Complex Three has an operating income before the overhead allocation and should not
be sold.
Explanation for Choice B:
The solution is to calculate operating income before corporate overhead allocation. The
complexes with operating losses before the corporate overhead allocation need to be
sold because they are not contributing anything to covering corporate overhead. The
ones with operating income before the corporate overhead allocation should be retained
because they are contributing some income to covering the corporate overhead.
Complex Four does have an operating loss before the corporate overhead allocation,
but it is not the only complex with an operating loss.
Explanation for Choice C:
The solution is to calculate operating income before corporate overhead allocation. The
complexes with operating losses before the corporate overhead allocation need to be
sold because they are not contributing anything to covering corporate overhead. The
ones with operating income before the corporate overhead allocation should be retained
because they are contributing some income to covering the corporate overhead.
Complexes Two and Three have an operating income before the overhead allocation
and should not be sold.
250. Question ID: CMA 694 4.25 (Topic: Marginal Analysis-Other Decisions)
Condensed monthly operating income data for Korbin Inc. for May follows:

Urban Suburban
Store Store Total
Sales $80,000 $120,000 $200,000
Variable costs 32,000 84,000 116,000
Contribution margin $48,000 $36,000 $84,000
Direct fixed costs 20,000 40,000 60,000
Store segment margin $28,000 $(4,000) $24,000
Common fixed cost 4,000 6,000 10,000
Operating income $24,000 $(10,000) $14,000
Additional information regarding Korbin's operations follows:

 One-fourth of each store's direct fixed costs would continue if either store were
closed.
 Korbin allocates common fixed costs to each store on the basis of sales dollars.
Hock P2 2020
Section C - Decision Analysis.
Answers
 Management estimates that closing the Suburban Store would result in a 10%
decrease in the Urban Store's sales, while closing the Urban Store would not affect
the Suburban Store's sales.
 The operating results for May are representative of all months.
A decision by Korbin to close the Suburban Store would result in a monthly increase
(decrease) in Korbin's operating income of

 A. $(1,200)
 B. $4,000
 C. $(6,000)
 D. $(10,800)correct
Question was not answered
Correct Answer Explanation:
Total operating income before closing the Suburban store is $14,000. After closing the
Suburban Store, operating income will become $3,200, calculated as follows:
1. Sales for the remaining Urban Store, decreased by 10%, will be $72,000.
2. Variable cost for the remaining Urban Store, also decreased by 10%, will be $28,800.
3. Contribution Margin will be $72,000 − $28,800 = $43,200.
4. Direct fixed cost: $20,000 for Urban Store plus 1/4 or $10,000 of Suburban's direct
fixed cost that would continue will be $30,000.
5. Store segment margin (which will not be meaningful since there will be only one
store) will be $13,200.
6. Common allocated fixed cost, which now will all be deducted from Urban Store's
segment margin will remain $10,000.
7. Operating Income = $3,200
If Korbin closes the store then operating income will decrease to $3,200. The net effect
to Korbin will be to decrease net operating income by $10,800 ($14,000 − $3,200).
Explanation for Choice A:
This is not the correct answer. Please see the correct answer for an explanation.
We have been unable to determine how to calculate this incorrect answer choice. If you
have calculated it, please let us know how you did it so we can create a full explanation
of why this answer choice is incorrect. Please send us an email at
support@hockinternational.com. Include the full Question ID number and the actual
incorrect answer choice -- not its letter, because that can change with every study
Hock P2 2020
Section C - Decision Analysis.
Answers
session created. The Question ID number appears at the top of the question. Thank you
in advance for helping us to make your HOCK study materials better.
Explanation for Choice B:
This answer does not account for the decreased sales volume in the Urban Store that
will result from closing the Suburban store, nor does it account for the portion of
Suburban Store's direct fixed costs that will continue if the store were closed.
Explanation for Choice C:
This answer does not account for the decreased sales volume in the Urban Store that
will result from closing the Suburban store.
251. Question ID: ICMA 19.P2.009 (Topic: Marginal Analysis-Other Decisions)
A company manufactures a product that has the following unit price and costs.

Selling price $300


Costs
Direct materials $40
Direct labor 30
Variable manufacturing overhead 24
Fixed manufacturing overhead 60
Variable selling 6
Fixed selling and administrative 20
Total costs (180)
Operating margin $120
The company received a special order for 1,000 units of the product. The company
currently has excess capacity but has an alternative use for this capacity that will result
in a contribution margin of $20,000. What is the minimum price that the company should
charge for this special order?

 A. $120, because it covers the costs of manufacturing the product and allows the
company to break even.correct
 B. $140, because operating margin will increase by $20,000.
 C. $200, because operating margin will increase by $20,000.
 D. $180, because it covers the costs of manufacturing the product and allows the
company to break even.
Question was not answered
Hock P2 2020
Section C - Decision Analysis.
Answers
Correct Answer Explanation:
When a company receives a special order and they must forego another opportunity in
order to produce this special order, the price that they need to charge needs to provide
at least as much contribution as the missed opportunity that they cannot pursue if they
fill this special order. We are told that if they produce this special order that they will give
up $20,000 of contribution from another opportunity. So, we know that in addition to
covering the variable costs of fulfilling this order (which includes all variable costs
including the variable selling cost), they need to add $20 per unit in order to provide
$20,000 of contribution from this 1,000 unit order.
The variable costs that they need to cover include: direct materials ($40), direct labor
($30), variable manufacturing overhead ($24) and variable selling ($6). These costs
total $100 ad when we add the $20 of contribution per unit, we get the minimum selling
price of $120 per unit
Explanation for Choice B:
This answer incorrectly includes the fixed selling and administrative costs as a cost to
cover. See the correct answer for a full explanation of this question.
Explanation for Choice C:
This answer incorrectly includes all of the fixed costs of the company as a cost to cover.
See the correct answer for a full explanation of this question.
Explanation for Choice D:
This answer incorrectly includes the fixed manufacturing overhead costs as a cost to
cover. See the correct answer for a full explanation of this question.
252. Question ID: ICMA 10.P2.250 (Topic: Marginal Analysis-Other Decisions)
Current business segment operations for Whitman, a mass retailer, are presented
below.

Merchandise Automotive Restaurant Total


Sales $500,000 $400,000 $100,000 $1,000,000
Variable costs 300,000 200,000 70,000 570,000
Fixed costs 100,000 100,000 50,000 250,000
Operating income (loss) $100,000 $100,000 $(20,000) $ 180,000
Management is contemplating the discontinuance of the Restaurant segment since "it is
losing money." If this segment is discontinued, $30,000 of its fixed costs will be
eliminated. In addition, Merchandise and Automotive sales will decrease 5% from their
current levels. What will Whitman's total contribution margin be if the Restaurant
segment is discontinued?
Hock P2 2020
Section C - Decision Analysis.
Answers
 A. $380,000.correct
 B. $220,000.
 C. $367,650.
 D. $160,000.
Question was not answered
Correct Answer Explanation:
The question asks what the contribution margin for the whole company will be if the
Restaurant division is discontinued. If that happens, the Merchandise and Automotive
divisions will also lose 5% of their business.
The present contribution margin for the whole company is $430,000 ($1,000,000 −
$570,000).
The contribution margins for both the Merchandise and the Automotive divisions are
$200,000, for a total of $400,000. The company will lose 5% of that, or $20,000, in its
total contribution margin, from decreases in sales at those divisions. In addition, the
company will lose the $30,000 contribution margin from the Restaurant division
($100,000 − $70,000).
Therefore, the total decrease in the company’s contribution margin from discontinuing
the Restaurant division will be $20,000 + $30,000, or $50,000. Since the present
contribution margin is $430,000, the revised contribution margin after discontinuing the
Restaurant division will be $430,000 − $50,000, which is $380,000.
Explanation for Choice B:
This is what the revised fixed costs would be if the Restaurant segment is eliminated.
The question asks for the company's total contribution margin if the Restaurant segment
is discontinued. The contribution margin is sales minus variable costs. Fixed costs are
not included.
Explanation for Choice C:
This is not the correct answer. Please see the correct answer for an explanation.
We have been unable to determine how to calculate this incorrect answer choice. If you
have calculated it, please let us know how you did it so we can create a full explanation
of why this answer choice is incorrect. Please send us an email at
support@hockinternational.com. Include the full Question ID number and the actual
incorrect answer choice -- not its letter, because that can change with every study
session created. The Question ID number appears at the top of the question. Thank you
in advance for helping us to make your HOCK study materials better.
Explanation for Choice D:
Hock P2 2020
Section C - Decision Analysis.
Answers
This is the revised operating income if the Restaurant segment is eliminated. The
question asks for the company's total contribution margin if the Restaurant segment is
discontinued. The contribution margin is sales minus variable costs. Fixed costs are not
included.
253. Question ID: ICMA 10.P2.248 (Topic: Marginal Analysis-Other Decisions)
Jones Enterprises manufactures 3 products, A, B, and C. During the month of May
Jones' production, costs, and sales data were as follows.

Products
A B C Totals
Units of production 30,000 20,000 70,000 120,000
Joint production costs to split-off point $480,000
Further processing costs $ — $60,000 $140,000
Unit sales price:
At split-off 3.75 5.50 10.25
After further processing — 8.00 12.50
Based on the above information, which one of the following alternatives should be
recommended to Jones' management?

 A. Process Product B further but sell Product C at the split-off point


 B. Process Product C further but sell Product B at the split-off point.correct
 C. Process both Products B and C further.
 D. Sell both Product B and Product C at the split-off point.
Question was not answered
Correct Answer Explanation:
The increased revenue from further processing for Product B would be ($8.00 − $5.50)
× 20,000, or $50,000. The costs to process further would be $60,000. The cost to
process further is greater than the incremental revenue that would be earned.
Therefore, it would not be profitable to process Product B further.
The increased revenue from further processing for Product C would be ($12.50 −
$10.25) × 70,000, or $157,500. The costs to process further would be $140,000. The
incremental revenue that would be earned is greater than the cost to process further.
Therefore, it would be profitable to process Product C further.
So the recommendation should be to process Product C further but sell Product B at the
split-off point. This would result in the highest operating income.
Hock P2 2020
Section C - Decision Analysis.
Answers
Explanation for Choice A:
This would not result in the highest operating income.
Explanation for Choice C:
This would not result in the highest operating income.
Explanation for Choice D:
This would not result in the highest operating income.
254. Question ID: ICMA 10.P2.251 (Topic: Marginal Analysis-Other Decisions)
Current business segment operations for Whitman, a mass retailer, are presented
below.

Merchandise Automotive Restaurant Total


Sales $500,000 $400,000 $100,000 $1,000,000
Variable costs 300,000 200,000 70,000 570,000
Fixed costs 100,000 100,000 50,000 250,000
Operating income (loss) $100,000 $100,000 $(20,000) $ 180,000
Management is contemplating the discontinuance of the Restaurant segment since "it is
losing money." If this segment is discontinued, $30,000 of its fixed costs will be
eliminated. In addition, Merchandise and Automotive sales will decrease 5% from their
current levels. When considering the decision, Whitman’s controller advised that one of
the financial aspects Whitman should review is contribution margin. Which one of the
following options reflects the current contribution margin ratios for each of Whitman's
business segments?

 A. Retailing 60%; Automotive 50%; Restaurant 70%.


 B. Retailing 40%; Automotive 50%; Restaurant 30%.correct
 C. Retailing 60%; Automotive 50%; Restaurant 30%.
 D. Retailing 40%; Automotive 50%; Restaurant 70%.
Question was not answered
Correct Answer Explanation:
The contribution margin is sales – variable costs. The contribution margin ratio is the
contribution margin divided by sales.

 The contribution margin ratio for Merchandise is ($500,000 − $300,000) ÷


$500,000, which is 40%.
 The contribution margin ratio for Automotive is $400,000 − $200,000 ÷ $400,000,
which is 50%.
Hock P2 2020
Section C - Decision Analysis.
Answers
 The contribution margin ratio for Restaurant is ($100,000 − $70,000) ÷ $100,000,
which is 30%.
Explanation for Choice A:
This is variable costs divided by sales for each of the three segments. The contribution
margin ratio is the contribution margin divided by sales. The contribution margin is sales
– variable costs.
Explanation for Choice C:
This is variable costs divided by sales for the retailing and automotive segments. The
contribution margin ratio is the contribution margin divided by sales. The contribution
margin is sales – variable costs.
Explanation for Choice D:
This is variable costs divided by sales for the automotive and restaurant segments. The
contribution margin ratio is the contribution margin divided by sales. The contribution
margin is sales minus variable costs.
255. Question ID: ICMA 10.P2.273 (Topic: Pricing and Pricing Strategy)
Leader Industries is planning to introduce a new product, DMA. It is expected that
10,000 units of DMA will be sold. The full product cost per unit is $300. Invested capital
for this product amounts to $20 million. Leader’s target rate of return on investment is
20%. The markup percentage for this product, based on operating income as a
percentage of full product cost, will be

 A. 233.3%.
 B. 42.9%.
 C. 133.3%.correct
 D. 57.1%.
Question was not answered
Correct Answer Explanation:
The target rate of return on investment is 20%. 20% of $20,000,000 invested is
$4,000,000. So $4,000,000 is the annual amount of increase in operating income that
the company needs from the sale of DMA in order to achieve its target of a 20% return
on investment.
The total annual cost of the new product is $300 × 10,000, or $3,000,000. The markup
amount is the amount of profit: $4,000,000. So the markup percentage, based on
operating income as a percentage of full product cost, is $4,000,000 ÷ $3,000,000,
which is 1.333 or 133.3%.
The markup percentage on cost needs to be 133.3% to achieve a 20% return on
investment.
Hock P2 2020
Section C - Decision Analysis.
Answers
Explanation for Choice A:
This is the revenue that would be required from the sale of 10,000 units of DMA to
achieve a 20% return on the investment ($7,000,000) divided by the cost for 10,000
units ($3,000,000). That is not the way the markup percentage on cost is calculated.
The markup percentage on cost is the amount of profit (the amount by which revenue is
greater than cost) divided by the cost.
The formula for calculating the price when markup on cost pricing is used is:
Price = Item Cost + (Item Cost × Markup Percentage)
Using this formula for 10,000 units, we have
Total Revenue = Total Cost + (Total Cost × Markup Percentage)
A markup percentage on cost of 233.3% will result in revenue for DMA of $9,999,000
($3,000,000 + [$3,000,000 × 2.333). Operating profit from the product will be
$6,999,000 ($9,999,000 − $3,000,000), which is 35% of the project's $20 million cost.
This is much greater than the project's target return of 20% and would probably result in
a price per unit that would discourage sales.
Explanation for Choice B:
When markup on cost pricing is used, the company determines what its costs are and
then adds a standard markup percentage to the cost to arrive at the price for the
product. The cost of each item is multiplied by the predetermined percentage and the
result is added to the cost to calculate the price. The formula for calculating the price
when markup on cost pricing is used is:
Price = Item Cost + (Item Cost × Markup Percentage)
Using this formula for 10,000 units, we have
Total Revenue = Total Cost + (Total Cost × Markup Percentage)
The markup amount is the amount of profit, i.e., Total Revenue minus Total Cost. The
markup percentage on cost is the amount of profit divided by the total cost. Therefore,
the total cost multiplied by the markup percentage on cost equals the amount of profit.
The total cost for 10,000 units of DMA is $3,000,000 ($300 × 10,000). A markup
percentage on cost of 42.9% will result in operating profit from the product of
$1,287,000 ($3,000,000 × 0.429).
$1,287,000 divided by $20,000,000 equals 0.0644. So the operating profit with a
markup on cost of 42.9% would be only 6.44% of the project's $20 million cost. That
would not achieve Leader's target rate of return of 20% on the investment.
Explanation for Choice D:
Hock P2 2020
Section C - Decision Analysis.
Answers
When markup on cost pricing is used, the company determines what its costs are and
then adds a standard markup percentage to the cost to arrive at the price for the
product. The cost of each item is multiplied by the predetermined percentage and the
result is added to the cost to calculate the price. The formula for calculating the price
when markup on cost pricing is used is:
Price = Item Cost + (Item Cost × Markup Percentage)
Using this formula for 10,000 units, we have
Total Revenue = Total Cost + (Total Cost × Markup Percentage)
The markup amount is the amount of profit, i.e., Total Revenue minus Total Cost. The
markup percentage on cost is the amount of profit divided by the total cost. Therefore,
the total cost multiplied by the markup percentage on cost equals the amount of profit.
The total cost for 10,000 units of DMA is $3,000,000 ($300 × 10,000). A markup
percentage on cost of 57.1% will result in operating profit from the product of
$1,713,000 ($3,000,000 × 0.571).
$1,713,000 divided by $20,000,000 equals 0.0857. So the operating profit with a
markup on cost of 57.1% would be only 8.57% of the project's $20 million cost. That
would not achieve Leader's target rate of return of 20% on the investment.
256. Question ID: ICMA 10.P2.272 (Topic: Pricing and Pricing Strategy)
If the demand for a good is elastic, then a(n)

 A. increase in price will increase total revenue.


 B. decrease in price will decrease total revenue.
 C. decrease in price will increase total revenue.correct
 D. increase in price will have no effect on total revenue.
Question was not answered
Correct Answer Explanation:
When a good is elastic, decreasing the price of the good will increase the total
revenues. This is for an elastic good any decrease in price will be more than offset by a
corresponding increase in quanity demanded that is larger than the decrease in price.
Explanation for Choice A:
For an elastic good, a decrease in price will lead to an increase in total revenue. An
increase in price will lead to a decrease in total revenue.
Explanation for Choice B:
For an elastic good, a decrease in price will lead to an increase in total revenue.
Explanation for Choice D:
Hock P2 2020
Section C - Decision Analysis.
Answers
For an elastic good, a decrease in price will lead to an increase in total revenue. An
increase in price will lead to a decrease in total revenue.
257. Question ID: ICMA 13.P2.050 (Topic: Pricing and Pricing Strategy)
An economic research firm performed extensive studies on the market for large screen
televisions (LSTs). Portions of the results are shown below.

Household Income LST Sales (units)


$50,000 20,000
60,000 28,000
72,000 39,200

Price of LSTs LST Sales (units)


$1,000 100,000
900 115,000
810 132,250
The price elasticity of demand for LSTs, using the midpoint method, is

 A. 1.15.
 B. 0.67.
 C. 2.00.
 D. 1.33.correct
Question was not answered
Correct Answer Explanation:
Using the midpoint method of calculating the price elasticity of demand, the formula is:

(Q2 − Q1) / [(Q2 + Q1) / 2]

Ed =

(P2 − P1) / [(P2 + P1) / 2]

Where: Q1 and 2 = First and second quantity points


Hock P2 2020
Section C - Decision Analysis.
Answers
P1 and 2 = First and second price points
The price elasticity of demand will be the same whether it is calculated using $1,000
and $900 as P1 and P2 and $100,000 and $115,000 as Q1 and Q2, or using $900 and
$810 as P1 and P2 and $115,000 and $132,250 as Q1 and Q2.

(115,000 − 100,000) / [(115,000 + 100,000) / 2]

Ed =

(900 − 1,000) / [(900 + 1,000) / 2]

15,000 /107,000

Ed = = 0.139535 / 0.105263 = 1.3255 or 1.33

100 / 950

Explanation for Choice A:


This is the number of units sold at a price of $900 divided by the number of units sold at
a price of $1,000.
Using the midpoint method, the correct formula for the price elasticity of demand is:

(Q2 − Q1) / [(Q2 + Q1) / 2]

Ed =

(P2 − P1) / [(P2 + P1) / 2]

Where: Q1 and 2 = First and second quantity points


P1 and 2 = First and second price points
Explanation for Choice B:
Hock P2 2020
Section C - Decision Analysis.
Answers
This is the percentage change in price divided by the percentage change in quantity.
The price elasticity of demand, calculated using the percentage method, is the
percentage change in quantity divided by the percentage change in price.
Using the midpoint method, the correct formula for the price elasticity of demand is:

(Q2 − Q1) / [(Q2 + Q1) / 2]

Ed =

(P2 − P1) / [(P2 + P1) / 2]

Where: Q1 and 2 = First and second quantity points


P1 and 2 = First and second price points
Explanation for Choice C:
This is the percentage method of calculating the income elasticity of demand. It is not
the price elasticity of demand.
Using the midpoint method, the correct formula for the price elasticity of demand is:

(Q2 − Q1) / [(Q2 + Q1) / 2]

Ed =

(P2 − P1) / [(P2 + P1) / 2]

Where: Q1 and 2 = First and second quantity points


P1 and 2 = First and second price points
258. Question ID: ICMA 10.P2.266 (Topic: Pricing and Pricing Strategy)
Which one of the following would cause the demand curve for bagels to shift to the left?

 A. An increase in the supply of bagels.


 B. A decrease in the cost of muffins.correct
 C. An increase in the population.
 D. A decrease in the price of bagels.
Hock P2 2020
Section C - Decision Analysis.
Answers
Question was not answered
Correct Answer Explanation:
Because muffins are a substitute product for bagels, a decrease in the price of muffins
will cause the demand curve for bagels to shift to the left. Because muffins are now
comparatively cheaper, fewer bagels will be demanded at every level of price.
Explanation for Choice A:
An increase in the supply of bagels will cause a change in the price of the bagels, which
will cause a movement ALONG the demand curve, not a shift of the entire demand
curve.
Explanation for Choice C:
An increase in the population would cause the demand curve for bagels (and all goods)
to shift to the right.
Explanation for Choice D:
A change in the price of the good in question will cause a movement ALONG the
demand curve, not a shift of the entire demand curve.
259. Question ID: CMA 1296 4.6 (Topic: Pricing and Pricing Strategy)
Several surveys point out that most managers use full product costs, including unit fixed
costs and unit variable costs, in developing cost-based pricing. Which one of the
following is least associated with cost-based pricing?

 A. Price justification.
 B. Fixed-cost recovery.
 C. Price stability.
 D. Target pricing.correct
Question was not answered
Correct Answer Explanation:
Target pricing starts with the expected price the product should be sold for in the
market, based on the company's knowledge of its customers and competitors. By
subtracting the per unit profit margin desired, the seller is able to estimate the unit target
cost. If the seller does not believe that it would be able to produce the product at or
below the target cost, the project would abandoned.
Explanation for Choice A:
Price justification is used in antitrust cases where the seller has to justify the price it is
charging to customers.
Explanation for Choice B:
Hock P2 2020
Section C - Decision Analysis.
Answers
Under fixed-cost recovery all costs in the long-term are relevant and have to be
recovered.
Explanation for Choice C:
Full product costing promotes price stability.
260. Question ID: ICMA 1603.P2.043 (Topic: Pricing and Pricing Strategy)
The management of a company is attempting to reduce the cost for Product X by
analyzing the trade-offs between different types of product features and total product
cost. What type of cost reduction strategy is the company using?

 A. Activity-based costing.
 B. Value engineering.correct
 C. Total quality management.
 D. Kaizen.
Question was not answered
Correct Answer Explanation:
Value engineering is a cost-reduction strategy that attempts to reduce the cost for a
product by analyzing the trade-offs between different types of product features and total
product cost. Value engineering involves evaluation of all the business functions in the
value chain with the objective of reducing costs while satisfying customer needs. This
evaluation may lead to design improvements, materials specification changes or
modifications in manufacturing methods.
When performing value engineering, management distinguishes between a value-added
cost and a non-value-added cost. If a value-added cost were eliminated, it would reduce
the product’s value, or usefulness, to customers. Since value-added costs cannot be
eliminated, value engineering seeks to reduce their costs by improving efficiency.
On the other hand, if a non-value-added cost were eliminated, it would not reduce the
value or utility of the product. A non-value-added cost is a cost the customer is not
willing to pay for. Examples of non-value-added costs are costs for expediting, re-work
and repair; and these are costs that can be reduced through improvements to the
manufacturing process.
Explanation for Choice A:
Activity-based costing is a method of allocating manufacturing overhead costs to
products based on cost drivers. ABC identifies the causal relationship between the
incurrence of cost and activities, determines the underlying driver of activities,
establishes cost pools related to individual drivers, develops costing rates, and applies
cost to product on the basis of resources consumed (drivers). It is not a cost reduction
strategy that attempts to reduce the cost for a product by analyzing the trade-offs
between different types of product features and total product cost.
Hock P2 2020
Section C - Decision Analysis.
Answers
Explanation for Choice C:
Total Quality Management describes an approach that is committed to customer
satisfaction and continuous improvement of products or services. The basic premise of
TQM is that quality improvement is a way of increasing revenues and decreasing costs,
but it is not a cost reduction strategy that attempts to reduce the cost for a product by
analyzing the trade-offs between different types of product features and total product
cost.
Explanation for Choice D:
The term kaizen is a Japanese word that means “improvement.” As used in business, it
implies “continuous improvement,” or slow but constant incremental improvements
being made in all areas of business operations. It is not a cost reduction strategy that
attempts to reduce the cost for a product by analyzing the trade-offs between different
types of product features and total product cost.
261. Question ID: ICMA 10.P2.274 (Topic: Pricing and Pricing Strategy)
Which one of the following situations best lends itself to a cost-based pricing approach?

 A. An industrial equipment fabricator negotiating pricing for one of its standard models
with a major steel manufacturer.
 B. A computer component manufacturer debating pricing with a new customer for a made
to order, state of the art application.correct
 C. A paper manufacturer negotiating the price for supplying copy paper to a new mass
merchandiser of office products.
 D. A computer component manufacturer debating pricing terms with a customer in a new
channel of distribution.
Question was not answered
Correct Answer Explanation:
Cost-based pricing focuses on what it costs to manufacture the product and the price
necessary to both recoup the company’s investment and achieve a desired return on its
investment. Cost-based pricing involves determining all the fixed and variable costs
associated with a product or service. After the total costs attributable to the product or
service have been determined, managers add a desired profit margin percentage to
each unit. The goal of the cost-oriented approach is to cover all costs incurred in
producing or delivering products or services and to achieve a targeted level of profit.
Cost-based pricing is used in a market where there is product differentiation, such as
automobile manufacturing and/or when a company is providing a unique product or
service. A made to order, state of the art application is a unique product.
Furthermore, determining the full cost of the custom order would not be difficult, since
the volume of the order would be known. There would be development costs for a
made-to-order, state-of-the-art application that the computer component manufacturer
Hock P2 2020
Section C - Decision Analysis.
Answers
would need to be sure of covering, because those are costs that would not exist if the
order is not accepted. So calculating the full cost and adding a desired profit margin
percentage to the order would be a reasonable way of approaching the pricing decision
to avoid having a loss on the order.
Explanation for Choice A:
Cost-based pricing focuses on what it costs to manufacture the product and the price
necessary to both recoup the company's investment and achieve a desired return on its
investment. Cost-based pricing involves determining all the fixed and variable costs
associated with a product or service. After the total costs attributable to the product or
service have been determined, managers add a desired profit margin percentage to
each unit. The goal of the cost-oriented approach is to cover all costs incurred in
producing or delivering products or services and to achieve a targeted level of profit.
Cost-based pricing is used in a market where there is product differentiation, such as
automobile manufacturing and/or when a company is providing a unique product or
service. This is not one of those situations though, because the equipment is a standard
model.
Explanation for Choice C:
Cost-based pricing focuses on what it costs to manufacture the product and the price
necessary to both recoup the company's investment and achieve a desired return on its
investment. Cost-based pricing involves determining all the fixed and variable costs
associated with a product or service. After the total costs attributable to the product or
service have been determined, managers add a desired profit margin percentage to
each unit. The goal of the cost-oriented approach is to cover all costs incurred in
producing or delivering products or services and to achieve a targeted level of profit.
Cost-based pricing is used in a market where there is product differentiation, such as
automobile manufacturing and/or when a company is providing a unique product or
service.
Cost-based pricing would not be the best method of setting a price in this situation.
There is nothing unique about copy paper. Furthermore, it can be very difficult to
determine a unit's cost before its price because unit costs may vary depending on
volume. Since the mass merchandiser is a new customer and its volume is unknown,
the manufacturer's full cost per carton of paper would also be unknown. Furthermore,
there are many other factors that should be considered. To win a new customer that
may turn out to be a very good account, pricing should be competitive.
Explanation for Choice D:
Cost-based pricing focuses on what it costs to manufacture the product and the price
necessary to both recoup the company's investment and achieve a desired return on its
investment. Cost-based pricing involves determining all the fixed and variable costs
associated with a product or service. After the total costs attributable to the product or
Hock P2 2020
Section C - Decision Analysis.
Answers
service have been determined, managers add a desired profit margin percentage to
each unit. The goal of the cost-oriented approach is to cover all costs incurred in
producing or delivering products or services and to achieve a targeted level of profit.
Cost-based pricing is used in a market where there is product differentiation, such as
automobile manufacturing and/or when a company is providing a unique product or
service. Computer components are standard, so this is not a situation where there is
product differentiation.
262. Question ID: ICMA 1603.P2.042 (Topic: Pricing and Pricing Strategy)
After completing a marketplace analysis of Product Z, a company’s accountant has
determined that a price change from $25 to $20 will result in a demand increase for
Product Z from 1,000 units to 1,500 units. Based on the information provided, what is
the price elasticity of demand for Product Z using the midpoint formula?

 A. 1.80.correct
 B. 1.33
 C. 2.50.
 D. 0.56.
Question was not answered
Correct Answer Explanation:
Using the midpoint method, the formula for the price elasticity of demand is:

(Q2 − Q1) / [(Q2 + Q1) / 2]

Ed =

(P2 − P1) / [(P2 + P1) / 2]

Where: Q1 and 2 = First and second quantity points


P1 and 2 = First and second price points
500 / 1,250 0.400

Ed = = = 1.80
Hock P2 2020
Section C - Decision Analysis.
Answers

$5 /$22.50 0.222

Explanation for Choice B:


This answer results from an attempt to calculate the price elasticity of demand using the
percentage method. It is incorrect for two reasons:
(1) The calculation is done incorrectly for the percentage method. For the percentage
method, the percentages of change in the quantity and the price should be calculated
by dividing the amount of change by the initial quantity and price, whereas the
percentages used in this answer were calculated by dividing the amount of change by
the changed quantity and price.
(2) The question says to use the midpoint method, not the percentage method, to
calculate the price elasticity of demand.
Using the midpoint method, the formula for the price elasticity of demand is:

(Q2 − Q1) / [(Q2 + Q1) / 2]

Ed =

(P2 − P1) / [(P2 + P1) / 2]

Where: Q1 and 2 = First and second quantity points


P1 and 2 = First and second price points
Explanation for Choice C:
This is the price elasticity of demand using the percentage method. The question says
to use the midpoint method, not the percentage method.
Using the midpoint method, the formula for the price elasticity of demand is:

(Q2 − Q1) / [(Q2 + Q1) / 2]

Ed =
Hock P2 2020
Section C - Decision Analysis.
Answers

(P2 − P1) / [(P2 + P1) / 2]

Where: Q1 and 2 = First and second quantity points


P1 and 2 = First and second price points
Explanation for Choice D:
Using the midpoint method, the formula for the price elasticity of demand is:

(Q2 − Q1) / [(Q2 + Q1) / 2]

Ed =

(P2 − P1) / [(P2 + P1) / 2]

Where: Q1 and 2 = First and second quantity points


P1 and 2 = First and second price points
This answer results from reversing the numerator and denominator in the formula, as
follows:

(P2 − P1) / [(P2 + P1) / 2]

Ed =

(Q2 − Q1) / [(Q2 + Q1) / 2]

263. Question ID: CIA 1185 IV.9 (Topic: Pricing and Pricing Strategy)
A manufacturer produces a product that sells for $10 per unit. Variable costs per unit
are $6 and total fixed costs are $12,000. At this selling price, the company earns a profit
equal to 10% of total dollar sales. By reducing its selling price to $9 per unit, the
manufacturer can increase its unit sales volume by 25%. Assume that there are no
taxes and that total fixed costs and variable costs per unit remain unchanged. If the
selling price were reduced to $9 per unit, the profit would be:

 A. $3,000correct
 B. $6,000
Hock P2 2020
Section C - Decision Analysis.
Answers
 C. $4,000
 D. $5,000
Question was not answered
Correct Answer Explanation:
The first step is to find the current sales volume using the facts given about the current
situation.
Letting X stand for sales volume, Total Revenue is 10X. Total Variable Cost is 6X. And
since Profit is 10% of Total Revenue, and Total Revenue is 10X, Profit is 0.10(10X),
which is equal to X. Therefore, our equation is:
10X − 6X − 12,000 = X
Solving for X:
4X − 12,000 = X
3X − 12,000 = 0
3X = 12,000
X = 4,000
We can prove this answer with a short income statement:

Revenue: $10 × 4,000 $40,000


Variable Cost: $6 × 4,000 24,000
Contribution margin $16,000
Fixed cost 12,000
Profit 4,000 which is 10% of total revenue of $40,000
Now that we know the current sales volume is 4,000 units, we can calculate the
increased sales volume when it increases by 25%: 4,000 × 1.25 = 5,000 units.
Now, we can create another income statement using a volume of 5,000 units and a
sales price of $9, to answer the question:

Revenue: $9 × 5,000 $45,000


Variable Cost: $6 × 5,000 30,000
Contribution margin $15,000
Fixed cost 12,000
Profit 3,000
Explanation for Choice B:
Hock P2 2020
Section C - Decision Analysis.
Answers
This is not the correct answer. Please see the correct answer for an explanation.
We have been unable to determine how to calculate this incorrect answer choice. If you
have calculated it, please let us know how you did it so we can create a full explanation
of why this answer choice is incorrect. Please send us an email at
support@hockinternational.com. Include the full Question ID number and the actual
incorrect answer choice -- not its letter, because that can change with every study
session created. The Question ID number appears at the top of the question. Thank you
in advance for helping us to make your HOCK study materials better.
Explanation for Choice C:
This is the profit at the current selling price and current volume.
Explanation for Choice D:
This is not the correct answer. Please see the correct answer for an explanation.
We have been unable to determine how to calculate this incorrect answer choice. If you
have calculated it, please let us know how you did it so we can create a full explanation
of why this answer choice is incorrect. Please send us an email at
support@hockinternational.com. Include the full Question ID number and the actual
incorrect answer choice -- not its letter, because that can change with every study
session created. The Question ID number appears at the top of the question. Thank you
in advance for helping us to make your HOCK study materials better.
264. Question ID: ICMA 10.P2.268 (Topic: Pricing and Pricing Strategy)
If the demand for a product is elastic, a price increase will result in

 A. an indeterminate change in revenue.


 B. an increase in total revenue.
 C. a decrease in total revenue.correct
 D. no change in total revenue.
Question was not answered
Correct Answer Explanation:
If a good is elastic, that means that a small increase in the price of the good will lead to
a larger decrease in the demand for that good. Therefore, for elastic goods an increase
in the price will lead to a decrease in the total revenue from that good. This is because
the (for example) 5% increase in price is more than offset by the (for example) 10%
decrease in the demand for the good.
Explanation for Choice A:
A change in the price of an elastic good will cause total revenue from that good to
decrease. See the correct answer for a full explanation.
Hock P2 2020
Section C - Decision Analysis.
Answers
Explanation for Choice B:
A change in the price of an elastic good will cause total revenue from that good to
decrease. See the correct answer for a full explanation.
Explanation for Choice D:
A change in the price of an elastic good will cause total revenue from that good to
decrease. See the correct answer for a full explanation.
265. Question ID: ICMA 1603.P2.002 (Topic: Pricing and Pricing Strategy)
A profit-maximizing company is considering a price increase on a particular product.
After extensive market research, the company has determined that demand for the
product is price inelastic. Assuming all other factors remain constant, determine what
course of action the company should take and the resulting impact on quantity
demanded.

 A. Increase price; minimal impact on quantity demanded.correct


 B. Do not increase price; minimal impact on quantity demanded.
 C. Do not increase price; quantity demanded will decrease significantly.
 D. Increase price; quantity demanded will increase significantly.
Question was not answered
Correct Answer Explanation:
Since demand for this product is inelastic, the company should increase the price
because a price increase will have minimal impact on the quantity demanded.
When the demand for a product is price inelastic, the quantity demanded changes by a
smaller percentage than the percentage of change in the product’s price. A price
increase for a product with inelastic demand will result in increased total revenue
because any decrease in demand and sales that result from the increased price will be
small enough that it will not completely offset the increase in revenue caused by the
price increase. At the same time, a price decrease will result in decreased total revenue
because the resulting increase in sales will not be enough to offset the lower price
received for each unit sold.
Explanation for Choice B:
When the demand for a product is price inelastic, the quantity demanded changes by a
smaller percentage than the percentage of change in the product’s price. Therefore, the
determination is correct that the price increase will have minimal impact on the quantity
demanded. However, the resulting decision with respect to the price increase is not
correct.
Explanation for Choice C:
Hock P2 2020
Section C - Decision Analysis.
Answers
When the demand for a product is price inelastic, the quantity demanded changes by a
smaller percentage than the percentage of change in the product’s price. Therefore, a
decrease in the quantity demanded should not be a concern in making this decision
because any decrease should be smaller, proportionately, than the increase in the
price.
Explanation for Choice D:
The quantity demanded may increase after a price increase, but the increase in demand
would be due to factors other than the price increase. A price increase does not cause
an increase in demand under any circumstances.
266. Question ID: ICMA 19.P2.016 (Topic: Pricing and Pricing Strategy)
Which one of the following pricing methods focuses on setting the price based on
recouping the manufacturing cost of the product and achieving a desired profit?

 A. Life-cycle based pricing.


 B. Cost-based pricing.correct
 C. Market-based pricing.
 D. Target pricing.
Question was not answered
Correct Answer Explanation:
A cost-based price is based on the costs of producing the product and then having
some amount of profit as well.
Explanation for Choice A:
Life-cycle based pricing is based on the costs of the product over the entire life cycle of
the product and is not based on the manufacturing costs of the product.
Explanation for Choice C:
A market-based price is not based on the manufacturing price, but is instead based on
other prices in the market.
Explanation for Choice D:
A target price is based on knowledge of customers’ perception of a product or service’s
value, the amount they are willing to pay, projected sales, and competitors’ responses.
It is not based on manufacturing costs.
267. Question ID: ICMA 10.P2.281 (Topic: Pricing and Pricing Strategy)
At the long-run profit maximizing equilibrium of a firm in a perfectly competitive market,
all of the following are correct except that

 A. marginal cost equals marginal revenue.


Hock P2 2020
Section C - Decision Analysis.
Answers
 B. economic profits are positive.correct
 C. price equals average total cost.
 D. price equals marginal cost.
Question was not answered
Correct Answer Explanation:
Economic profit is the amount by which total revenue exceeds the total economic costs
of the company. Total economic costs include all of the explicit (cash) costs that are
paid by the firm as well as the relevant implicit (opportunity) costs. Another way of
looking at economic profit is the excess of profit that a company has in its current
business over what it could make in its next best alternative business.
At the long-run profit maximizing equilibrium of a firm in a perfectly competitive market,
in the long run the company will not be able to maintain any economic profit. If there are
economic profits to be made in the market, other firms will enter the market. The entry
of the new firms will push the market price down until economic profit is zero.
Explanation for Choice A:
At the long-run profit maximizing equilibrium of a firm in a perfectly competitive market,
marginal cost will equal marginal revenue.
Explanation for Choice C:
At the long-run profit maximizing equilibrium of a firm in a perfectly competitive market,
the price does equal the average total cost. In economics, "total cost" includes a normal
profit. Since a normal profit is a part of the total cost, it is thus also a part of the average
total cost. In the long run, the most profit a firm in a perfectly competitive market can
earn is a normal profit.
For a small business that is operated by its owner, normal profit is the value of the
entrepreneurial skills of the owner of the business. It is the opportunity cost of the
wages that the individual gives up by not working at another job. For a large, publicly
held business, a normal profit is an amount of profit that is just enough to keep its
investors satisfied that they are earning the return they should be earning on their
investment. For a large business, normal profit is a normal return to capital.
So when we say that at the long-run profit maximizing equilibrium of a firm in a perfectly
competitive market, price equals average total cost, we are saying that in the long run,
all firms in a perfectly competitive market will earn a normal profit, but no firms will earn
an economic profit.
Explanation for Choice D:
For a firm in a perfectly competitive market, the price does equal the marginal cost. In a
perfectly competitive market, each seller can sell as many units as they wish at the
market price. In all other types of market structures, in order to sell more units, a seller
Hock P2 2020
Section C - Decision Analysis.
Answers
must decrease the price, so marginal revenue will not be the same as the price. But for
a perfectly competitive seller at the long-run profit maximizing equilibrium, marginal
revenue is equal to the price, and marginal revenue and price are both equal to
marginal cost.
268. Question ID: ICMA 10.P2.270 (Topic: Pricing and Pricing Strategy)
An economist determined the following market data for a commodity:

Quantity Quantity
Price Supplied Demanded
$25 250 750
$50 500 500
$75 750 250
$100 1,000 0
Based on this information, which one of the following statements is correct?

 A. In the long-run, if producers' costs per unit decline, then a reasonable market clearing
price could be $65.
 B. In the long-run, if producers' costs per unit increase, then a reasonable market clearing
price could be $70.correct
 C. In the short-term, there would be excess supply at a price of $40.
 D. In the short-term, there would be excess demand at a price of $70.
Question was not answered
Correct Answer Explanation:
The market clearing price is determined by the supply and demand curves. It is the
equilibrium price where the supply curve crosses the demand curve.
One of the factors that will impact the amount supplied to the market is the cost of
production. An increase in the cost of production will cause the supply curve to shift to
the left. When the supply curve shifts to the left, at every market price, suppliers will be
willing to supply a lower amount of the good. The result would be a higher equilibrium
price and a lower equilibrium quantity. Suppliers will require a higher price in order to
recover their costs — and therefore be willing to sell their goods.
Explanation for Choice A:
The market clearing price is determined by the supply and demand curve. While the
cost of production does impact the amount that the suppliers are willing to provide at a
specific price, a decrease in the cost of production would lead to a lower market clearing
price, not a higher market clearing price.
Explanation for Choice C:
Hock P2 2020
Section C - Decision Analysis.
Answers
At a price of $40, there will be excess demand. This is because at a price below the
equilibrium price, there will be more people who want to buy the product than sell at that
price.
Explanation for Choice D:
At a price higher than the market clearing price, there is excess supply, not excess
demand.
269. Question ID: CMA 1290 1.1 (Topic: Pricing and Pricing Strategy)
The existence of economic profit in pure monopoly will

 A. Lead to a decline in the number of firms in the industry.


 B. Have no influence on the number of firms in the industry.correct
 C. Lead to an increase in product supply.
 D. Lead to a decline in market prices for substitutes.
Question was not answered
Correct Answer Explanation:
The existence of economic profit is a characteristic of a pure monopoly. However,
another characteristic is barriers to entry which make it impossible or very costly for
another firm to enter the market. Therefore, a pure monopoly consists of only one firm.
As a result, in a pure monopoly market structure, the existence of an economic profit will
have no influence on the number of firms in the industry.
In a monopolistically competitive market, however, there are minimal barriers to entry of
new firms. The existence of economic profit will encourage new firms to enter the
market. As new firms enter the market, the existing firms will be forced to lower their
prices, and in the long run, all economic profit will be eliminated.
Explanation for Choice A:
A pure monopoly consists of only one firm due to the high barriers of entry. Therefore,
there cannot be a decline in the number of firms in the industry with only one firm and
with this firm earning economic profits.
Explanation for Choice C:
The existence of economic profit in a pure monopoly has nothing to do with the supply
side of production. The monopoly will continue to produce under the inefficient amount
that would be true in a competitive market.
Explanation for Choice D:
The presence of economic profits in a pure monopoly has no affect on the market prices
for substitutes.
270. Question ID: ICMA 10.P2.276 (Topic: Pricing and Pricing Strategy)
Hock P2 2020
Section C - Decision Analysis.
Answers
Companies that manufacture made-to-order industrial equipment typically use which
one of the following?

 A. Material-based pricing.
 B. Cost-based pricing.correct
 C. Market-based pricing.
 D. Price discrimination.
Question was not answered
Correct Answer Explanation:
Cost-based pricing focuses on what it costs to manufacture the product and the price
necessary to both recoup the company’s investment and achieve a desired return on its
investment. Cost-based pricing involves determining all the fixed and variable costs
associated with a product or service. After the total costs attributable to the product or
service have been determined, managers add a desired profit margin percentage to
each unit. The goal of the cost-oriented approach is to cover all costs incurred in
producing or delivering products or services and to achieve a targeted level of profit.
Companies that provide unique products and services usually use cost-plus pricing.
These companies include not only manufacturers of custom-made equipment but also
service professionals such as attorneys and public accountants.
Explanation for Choice A:
Material-based pricing is based upon the actual amount and type of material used in the
product. It is a variation on time-and-material pricing that would be used where the
amount of time required to produce an order is known but the cost of the material varies
depending on the customer's material specifications. Material-based pricing would not
be the best pricing for a made-to-order piece of industrial equipment, because the
material to be used would not be subject to customer specification, and furthermore
there would be no pre-determined time standards for a piece of equipment that is being
made to order.
Explanation for Choice C:
The market-based approach focuses on what the customers want and how competitors
will react to what the company does. Market-based pricing is used primarily when one
company’s products or services are very similar to another company’s, so an individual
company has no influence over the price to charge. Each company accepts the market
price. Companies operating in competitive markets, such as oil and gas, use this
approach. It would not typically be used by a company that manufactures made-to-order
industrial equipment.
Explanation for Choice D:
Hock P2 2020
Section C - Decision Analysis.
Answers
Price discrimination is the practice of charging different prices for the same product to
different customers. An example of this is in the airline industry, where a carrier will
charge a lower rate to some passengers than to others.
But when a product is unique, as a made-to-order piece of equipment is, price
discrimination is not possible because price discrimination requires the assumption that
the product or service provided is the same for all customers while the pricing is
different.
271. Question ID: ICMA 10.P2.271 (Topic: Pricing and Pricing Strategy)
If a product’s price elasticity of demand is greater than one, then a 1% price increase
will cause the quantity demanded to

 A. increase by more than 1%.


 B. decrease by less than 1%.
 C. decrease by more than 1%.correct
 D. increase by less than 1%.
Question was not answered
Correct Answer Explanation:
When price elasticity is more than one, this means that an X% increase in the price of
the good will result in a more than X% decrease in the demand for the good. Therefore,
a 1% increase in the price will cause the demand to decrease by more than 1%.
Explanation for Choice A:
When price elasticity is more than one, this means that an X% increase in the price of
the good will result in a more than X% decrease in the demand for the good. Therefore,
a 1% increase in the price will cause the demand to decrease by more than 1%.
Explanation for Choice B:
When price elasticity is more than one, this means that an X% increase in the price of
the good will result in a more than X% decrease in the demand for the good. Therefore,
a 1% increase in the price will cause the demand to decrease by more than 1%.
Explanation for Choice D:
When price elasticity is more than one, this means that an X% increase in the price of
the good will result in a more than X% decrease in the demand for the good. Therefore,
a 1% increase in the price will cause the demand to decrease by more than 1%.
272. Question ID: ICMA 10.P2.277 (Topic: Pricing and Pricing Strategy)
Which one of the following is not a characteristic of market-based pricing?

 A. It has a customer-driven external focus.


 B. It is used by companies facing stiff competition.
Hock P2 2020
Section C - Decision Analysis.
Answers
 C. It starts with a target selling price and target profit.
 D. It is used by companies facing minimal competition.correct
Question was not answered
Correct Answer Explanation:
This is not a true statement. Companies operating in competitive markets, such as oil
and gas, use market-based pricing in order to be competitive. Companies
facing minimal competition do not need to use market-based pricing. In fact, there
would probably not be any market prices in a market with little competition.
Explanation for Choice A:
Market-based pricing focuses on what the customers want, so this is a true statement.
Explanation for Choice B:
Companies operating in competitive markets, such as oil and gas, use market-based
pricing in order to be competitive. So this is a true statement.
Explanation for Choice C:
When market-based pricing is used, the company establishes a target price, which is a
price based on knowledge of customer perception of the value of the product or service
and what customers are willing to pay, as well as knowledge of competitors’ responses.
The target price then determines what the target cost per unit needs to be in order to
earn the target operating income per unit. The target cost per unit must then be the
target price minus the target operating income per unit, and the company has to figure
out how to provide the product or service at that cost. So this is a true statement.
273. Question ID: ICMA 10.P2.105 (Topic: Pricing and Pricing Strategy)
The concept of economic profit is best defined as total

 A. income minus the sum of total fixed and variable costs.


 B. revenue minus all accounting costs.
 C. revenue minus all explicit and implicit costs.correct
 D. revenue minus the sum of total fixed and variable costs.
Question was not answered
Correct Answer Explanation:
Economic profit is calculated as follows:
Revenue − Explicit Costs − Implicit Costs = Economic Profit
Revenue − Explicit Costs (only) = Accounting Profit. Accounting profit must be reduced
by implicit costs, such as opportunity costs, to arrive at economic profit. So economic
profit will always be lower than accounting profit.
Hock P2 2020
Section C - Decision Analysis.
Answers
Explanation for Choice A:
"Income" usually refers to "net income," which is revenue minus expenses. So
subtracting the sum of total fixed and variable costs from income would mean deducting
expenses twice. Furthermore, for a manufacturer, not all fixed and variable costs are
expensed in the period they are incurred. Any manufacturing fixed and variable costs for
unsold units remain in inventory on the balance sheet until the units they are attached to
are sold. Economic profit is not even equal to revenue minus fixed and variable costs for
sold units and other selling, general and administrative expenses. That is the definition
of accounting profit. Economic profit is lower than accounting profit because it includes
more costs than accounting profit includes.
Explanation for Choice B:
Revenue minus all accounting costs is accounting profit. Economic profit is lower than
accounting profit because it includes more costs than accounting profit includes.
Explanation for Choice D:
For a manufacturer, not all fixed and variable costs are expensed in the period they are
incurred. Any manufacturing fixed and variable costs for unsold units remain in
inventory on the balance sheet until the units they are attached to are sold. Economic
profit is not even equal to revenue minus fixed and variable costs for sold units and
other selling, general and administrative expenses. That is the definition of accounting
profit. Economic profit is lower than accounting profit because it includes more costs
than accounting profit includes.
274. Question ID: CIA 1195 IV.75 (Topic: Pricing and Pricing Strategy)
Which of the following price adjustment strategies is designed to stabilize production for
the selling firm?

 A. Cash discounts.
 B. Seasonal discounts.correct
 C. Quantity discounts.
 D. Functional discounts.
Question was not answered
Correct Answer Explanation:
Seasonal discounts are a good method used by manufactures to smooth or stabilize the
production cycle. The best example is a ski manufacturer offering discounts to retailers
during the spring and summer months. Out-of-season discounts encourage the retailers
to order early and can smooth out some of the seasonality of the ski manufacturer's
business.
Explanation for Choice A:
A cash discount strategy is used in order to get customers to make timely payments.
Hock P2 2020
Section C - Decision Analysis.
Answers
Explanation for Choice C:
Quantity discounts are used in order to get customers to make high volume purchases.
Explanation for Choice D:
Functional discounts are given to customers for performing certain functions, such as
selling, storing, etc. They are not methods to smooth the production cycle.
275. Question ID: ICMA 19.P2.020 (Topic: Pricing and Pricing Strategy)
Product X was launched ten years ago as an innovative product. The initial price was
relatively high, which yielded a high profit margin. The market for Product X is now
becoming very competitive, and demand for the product is slowing. What pricing
strategy should the company follow for Product X based on the current market
conditions?

 A. Because of the increased competition, Product X’s price should be decreased.correct


 B. Prices should be held steady with only inflation adjustments.
 C. Pricing for Product X should be at its highest level to recoup its innovation R&D.
 D. The company should sell Product X below cost so that it can force competitors out of
the market.
Question was not answered
Correct Answer Explanation:
This is a good marketing strategy in this case. Because of increased competition, the
price will need to be reduced, or customers will be lost.
Explanation for Choice B:
At a time of increased competition, keeping the price steady would probably not be a
good pricing strategy.
Explanation for Choice C:
At a time of increased competition, raising the price would not be a good pricing
strategy.
Explanation for Choice D:
Selling a product below the cost of production is rarely a good pricing strategy.
276. Question ID: ICMA 10.QA.P2.032 (Topic: Pricing and Pricing Strategy)
Highfield Corporation expects to sell 10,000 units of its product at a target price of $50
per unit. The current full cost of the product is $60 per unit. If Highland wants to earn an
operating profit margin of 20%, the target cost per unit is

 A. $40.correct
 B. $12.
Hock P2 2020
Section C - Decision Analysis.
Answers
 C. $10.
 D. $38.
Question was not answered
Correct Answer Explanation:
The target price minus the target cost equals the target operating profit. The target
operating profit is 20% of the target price.
Thus, the target cost per unit equals the target price minus 20% of the target price. The
target price is $50 per unit. Thus, the target cost is calculated using the following
formula:
TC = 50 − (0.20 × 50)
TC = 50 − 10
TC = 40
Explanation for Choice B:
$12 is 20% of the current full cost of $60, not the target cost needed to achieve a 20%
operating profit margin.
Explanation for Choice C:
$10 is 20% of the target price of $50. This is the target operating profit per unit, not the
target cost per unit needed to achieve a target operating profit of 20% per unit.
Explanation for Choice D:
$38 is the target price of $50 minus 20% of the current full cost. This is not the target
cost needed to achieve a 20% operating profit margin.
277. Question ID: CIA 1183 IV.23 (Topic: Pricing and Pricing Strategy)
A company produced the following data (rounded) on its product:

Unit Cost Marginal


Units Produced Fixed Variable Total Cost Revenue
1 100 85 185 85 90
2 50 70 120 55 90
3 33 65 98 55 90
4 25 67 92 73 90
5 20 75 95 107 90
How many units should be produced?
Hock P2 2020
Section C - Decision Analysis.
Answers
 A. 5
 B. 2
 C. 4correct
 D. 3
Question was not answered
Correct Answer Explanation:
This company cannot operate profitably at any production level given. However, its
lowest loss occurs at the production level of 4 units. Total revenue at that level is $90 ×
4, or $360 and total cost is $92 × 4, or $368, for a loss of $8. This is also the level where
MC is closest to MR without going over MR, with the exception of 1 unit produced.
However, at a production level of 1 unit, total revenue would be $90 and total cost would
be $185, for a loss of $95.
In this case, marginal revenue is greater than its marginal cost at the production level of
4 units, but at 5 units the marginal cost is greater than its marginal revenue.
Explanation for Choice A:
This is not the most optimal production level. Production should be increased to the
level where MC is closest to MR without going over MR. At a level of 5 units MC > MR,
so the company's total costs would increase by more than its total revenue by producing
one additional unit and the company would lose income.
Explanation for Choice B:
This is not the most optimal production level. Production should be increased to the
level where MC is closest to MR without going over MR.
Explanation for Choice D:
This is not the most optimal production level. Production should be increased to the
level where MC is closest to MR without going over MR.
278. Question ID: CMA 693 4.3 (Topic: Pricing and Pricing Strategy)
Delphi Company has developed a new project that will be marketed for the first time
during the next fiscal year. Although the Marketing Department estimates that 35,000
units could be sold at $36 per unit, Delphi's management has allocated only enough
manufacturing capacity to produce a maximum of 25,000 units of the new product
annually. The fixed costs associated with the new product are budgeted at $450,000 for
the year, which includes $60,000 for depreciation on new manufacturing equipment.
Data associated with each unit of product are presented as follows. Delphi is subject to
a 40% income tax rate.

Variable
Costs
Hock P2 2020
Section C - Decision Analysis.
Answers
Direct material $ 7.00
Direct labor 3.50
Manufacturing overhead 4.00
Total variable manufacturing cost $14.50
Selling expenses 1.50
Total variable cost $16.00
Delphi Company's management has stipulated that it will not approve the continued
manufacture of the new product after the next fiscal year unless the after-tax profit is at
least $75,000 the first year. The unit selling price to achieve this target profit must be at
least

 A. $34.60.
 B. $37.00.
 C. $36.60.
 D. $39.00.correct
Question was not answered
Correct Answer Explanation:
If the management of Delphi has required that this project have an after-tax profit of
$75,000, we will need to include this pre-tax required income as a fixed cost. The pre-
tax income that is required is $125,000 ($75,000 ÷ 0.6). Adding this to the fixed costs of
$450,000, the company now needs to cover $575,000 of costs with the sale of the
25,000 units that can be sold. This means that each unit must provide $23 of
contribution. Adding this $23 to the variable costs of $16 per unit, we get a selling price
of $39 per unit. If they sell each of the 25,000 units for $39, they will have an after-tax
profit of $75,000.
Explanation for Choice A:
This answer results from two errors:
1) Fixed costs have been reduced by the amount of the depreciation. The depreciation
is a fixed cost and should not be excluded.
2) The $75,000 required after-tax profit has not been converted to its equivalent before-
tax profit.
Explanation for Choice B:
This answer results from treating the $75,000 after-tax profit requirement as a before-
tax profit requirement. The after-tax profit requirement needs to be converted to a
Hock P2 2020
Section C - Decision Analysis.
Answers
before-tax requirement by dividing it by (1 - the tax rate) before using it to calculate the
selling price needed.
Explanation for Choice C:
This answer results from reducing fixed costs by the amount of the depreciation. The
depreciation is a fixed cost and should not be excluded.
279. Question ID: ICMA 19.P2.019 (Topic: Pricing and Pricing Strategy)
All of the following statements about product life-cycle pricing are correct except that
price

 A. may be set high to skim the market when the product is first introduced.
 B. is maintained by introducing new features to differentiate the product in the later
stages.
 C. is set to cover costs incurred from the time when the product is manufactured.correct
 D. may be set low to penetrate the market when the product is first introduced.
Question was not answered
Correct Answer Explanation:
The price that is set also needs to cover the costs that are incurred in the product life
cycle before production begins.
Explanation for Choice A:
This is a true statement about life-cycle pricing.
Explanation for Choice B:
This is a true statement about life-cycle pricing.
Explanation for Choice D:
This is a true statement about life-cycle pricing.
280. Question ID: CMA 687 4.10 (Topic: Pricing and Pricing Strategy)
Donnelly Corporation manufactures and sells T-shirts imprinted with college names and
slogans. Last year, the shirts sold for $7.50 each, and the variable cost to manufacture
them was $2.25 per unit. The company needed to sell 20,000 shirts to break even. The
net income last year was $5,040. Donnelly's expectations for the coming year include
the following:

 The sales price of the T-shirts will be $9


 Variable cost to manufacture will increase by one-third
 Fixed costs will increase by 10%
 The income tax rate of 40% will be unchanged
Hock P2 2020
Section C - Decision Analysis.
Answers
The selling price that would maintain the same contribution margin rate as last year is

 A. Some amount other than those given.


 B. $10.00correct
 C. $9.00
 D. $8.25
Question was not answered
Correct Answer Explanation:
Last year's contribution margin rate is 70%. The contribution margin is $5.25, calculated
as $7.50 − $2.25. The contribution margin ratio is $5.25 ÷ $7.50, or 0.70.
The variable cost will increase by 1/3 in the coming year. 1/3 of $2.25 is $0.75, so the
new variable cost will be $2.25 plus $0.75, or $3.00. To find what selling price would
maintain the same contribution margin as last year, we can use the following formula:
X − 3 = 0.7X
Solving for X:
Subtract 0.7X from both sides of the equation:
0.3X − 3 = 0
Add 3 to both sides of the equation:
0.3X = 3
Divide both sides of the equation by 0.3:
X = 10
A sellilng price of $10 would create a contribution margin of $10 − $3, or $7. The
contribution margin ratio would be $7 ÷ $10, or 0.70, the same as last year's
contribution margin.
Explanation for Choice A:
The correct answer is one of the answer choices.
Explanation for Choice C:
$9 is the new planned sales price of the T-shirts. However, this selling price would not
maintain the same contribution margin ratio as the company had last year.
Explanation for Choice D:
This selling price would maintain the same unit contribution margin as last year, but it
would not maintain the same contribution margin ratio as last year. Since the selling
price will be different next year, to maintain the same contribution margin ratio, the unit
contribution margin must change, too.
Hock P2 2020
Section C - Decision Analysis.
Answers
281. Question ID: ICMA 10.P2.279 (Topic: Pricing and Pricing Strategy)
Fennel Products is using cost-based pricing to determine the selling price for its new
product based on the following information.

Annual volume 25,000 units


Fixed costs $700,000 per year
Variable costs $200 per unit
Plant investment $3,000,000
Working capital $1,000,000
Effective tax rate 40%
The target price that Fennel needs to set for the new product to achieve a 15% after-tax
return on investment (ROI) would be

 A. $258.
 B. $238.
 C. $228.
 D. $268.correct
Question was not answered
Correct Answer Explanation:
The company requires a 15% after-tax return on investment. Investment is $4,000,000,
including $3,000,000 in the plant and $1,000,000 in working capital. So the after-tax
profit required is $4,000,000 × 0.15, which is $600,000.
To convert an after-tax profit to a before-tax profit, we divide it by 1 − the tax rate.
$600,000 ÷ (1 − 0.40) = $1,000,000 and $1,000,000 is the required before-tax profit.
The contribution margin needs to be enough to cover fixed cost of $700,000 plus the
profit requirement of $1,000,000. Therefore, the contribution margin needs to be
$1,700,000 to achieve an after-tax return on investment of 15%.
With sales of 25,000 units expected, this means the contribution margin per unit needs
to be $1,700,000 ÷ 25,000, or $68 per unit.
The variable cost per unit is $200. The price per unit minus the variable cost per unit
equals the contribution margin per unit. Therefore, the formula to use to calculate the
required price is
X − 200 = 68
Where X = the price per unit
Solving for X, we get X = $268.
Hock P2 2020
Section C - Decision Analysis.
Answers
Explanation for Choice A:
This answer results from using the $3,000,000 invested in the plant as total investment
to calculate the required ROI. Total investment includes the $3,000,000 invested in the
plant plus the $1,000,000 invested in working capital.
Explanation for Choice B:
This answer results from using the $1,000,000 invested in working capital as total
investment to calculate the required ROI. Total investment includes the $1,000,000
invested in working capital plus the $3,000,000 invested in the plant.
Explanation for Choice C:
Using $228 as a sales price, the firm would break even. Since the firm wants to achieve
a 15% after-tax return on investment, this price is not correct.
282. Question ID: ICMA 13.P2.051 (Topic: Pricing and Pricing Strategy)
Which one of the following statements best describes characteristics of the growth
phase of the product life cycle?

 A. There is limited competition and prices are high.


 B. Competition increases and prices are falling.correct
 C. There is limited competition and prices are falling.
 D. Competition increases and prices are high.
Question was not answered
Correct Answer Explanation:
During the growth phase of the product life cycle, sales increase rapidly and competition
increases. Prices are usually decreased to remain competitive because the marketing
objective at this stage is to maximize market share.
Explanation for Choice A:
During the growth phase of the product life cycle, competition is not limited and prices
are not high.
Explanation for Choice C:
During the growth phase of the product life cycle, competition is not limited.
Explanation for Choice D:
During the growth phase of the product life cycle, prices are not high.
283. Question ID: ICMA 10.P2.275 (Topic: Pricing and Pricing Strategy)
Basic Computer Company (BCC) sells its microcomputers using bid pricing. It develops
its bids on a full cost basis. Full cost includes estimated material, labor, variable
overheads, fixed manufacturing overheads, and reasonable incremental computer
Hock P2 2020
Section C - Decision Analysis.
Answers
assembly administrative costs, plus a 10% return on full cost. BCC believes bids in
excess of $1,050 per computer are not likely to be considered.
BCC's current cost structure, based on its normal production levels, is $500 for
materials per computer and $20 per labor hour. Assembly and testing of each computer
requires 17 labor hours. BCC expects to incur variable manufacturing overhead of $2
per labor hour, fixed manufacturing overhead of $3 per labor hour, and incremental
administrative costs of $8 per computer assembled.
BCC has received a request from a school board for 200 computers. Using the full-cost
criteria and desired level of return, which one of the following prices should be
recommended to BCC's management for bidding purposes?

 A. $1,026.30.correct
 B. $874.00.
 C. $961.40.
 D. $882.00.
Question was not answered
Correct Answer Explanation:
The question says to use the full-cost criteria and the desired return on full cost (10%) in
developing this bid. It further says that bids in excess of $1,050 per computer are not
likely to be considered. Therefore, the bid needs to be less than $1,050 per computer
(but all of the answer choices are less than $1,050 per computer, so that information is
not very helpful in selecting from among the answer choices).
The full costs per computer are:

Materials $ 500.00
Labor - 17 direct labor hours @ $20/hour 340.00
Variable overhead - 17 direct labor hours × $2 34.00
Incremental administrative costs 8.00
Fixed overhead - 17 direct labor hours × $3 51.00
Total cost $ 933.00
Plus profit - $933 × 0.10 93.30
Price $1,026.30
Fixed costs are included in this bid for two reasons. One, the problem says that BCC
expects to incur fixed manufacturing overhead of $3 per labor hour. That implies that
the fixed costs would not be incurred if the company does not produce this order. That
Hock P2 2020
Section C - Decision Analysis.
Answers
makes the fixed costs a relevant cost that needs to be covered. And two, the problem
tells us to use the full-cost criteria, and fixed costs are included in the full cost.
Explanation for Choice B:
This answer omits the fixed manufacturing overhead, the incremental administrative
costs, and the 10% return on full cost.
Explanation for Choice C:
This answer omits the fixed manufacturing overhead and the incremental administrative
costs.
Explanation for Choice D:
This answer omits the fixed manufacturing overhead as well as the 10% return on full
cost.
284. Question ID: ICMA 10.P2.278 (Topic: Pricing and Pricing Strategy)
Almelo Manpower Inc. provides contracted bookkeeping services. Almelo has annual
fixed costs of $100,000 and variable costs of $6 per hour. This year the company
budgeted 50,000 hours of bookkeeping services. Almelo prices its services at full cost
and uses a cost-plus pricing approach. The company developed a billing price of $9 per
hour. The company’s mark-up level would be

 A. 50.0%.
 B. 66.6%.
 C. 33.3%.
 D. 12.5%.correct
Question was not answered
Correct Answer Explanation:
The full cost per hour is $6 variable costs plus $2 fixed costs ($100,000 annual fixed
costs ÷ 50,000 budgeted hours of service). The full cost is therefore $8 per hour. The
billing price is $9 per hour, and the difference is $1 per hour. $1 per hour divided by the
full cost of $8 per hour = a markup percentage of 0.125 or 12.5%.
Explanation for Choice A:
This would be the markup on the variable costs only. The problem says that Almelo
prices its services at full cost and uses a cost-plus pricing approach. Therefore, the full
cost including fixed costs should be used in determining the markup percentage.
Explanation for Choice B:
This is the variable cost per hour divided by the billing price per hour. This is not the
way the markup percentage is calculated. The markup percentage is the percentage of
the full cost that is added to the full cost to arrive at the billing price.
Hock P2 2020
Section C - Decision Analysis.
Answers
Explanation for Choice C:
This is the billing price per hour minus the variable cost per hour, the difference divided
by the billing price per hour. This is not the way the markup percentage is calculated.
The markup percentage is the percentage of the full cost that is added to the full cost to
arrive at the billing price.
285. Question ID: CIA 1193 IV.11 (Topic: Pricing and Pricing Strategy)
A retail company determines its selling price by marking up variable costs 60%. In
addition, the company uses frequent selling price markdowns to stimulate sales. If the
markdowns average 10%, what is the company's contribution margin ratio?

 A. 43.75%
 B. 30.6%correct
 C. 27.5%
 D. 37.5%
Question was not answered
Correct Answer Explanation:
The contribution margin is equal to selling price minus variable costs (Sales − VC =
CM). If variable costs are $100 per unit, then the sales price is $160 per unit ($100 ×
160%). If there is a markdown of 10%, then the new sales price will $144 ($160 × 90%)
and the new contribution margin will be $44 ($144 − $100). The new contribution margin
ratio will be 30.6% ($44 ÷ $144).
Explanation for Choice A:
This contribution margin percentage was calculated by taking 10% off variable costs
instead of discounting the sales price.
Explanation for Choice C:
This answer results from dividing the unit contribution margin (calculated correctly using
the selling price reduced by the average 10% markdown) by the selling
price unreduced by the 10% markdown, using either representative currency amounts
or just the decimal equivalents of the percentages given. However, since markdowns
average 10%, the contribution margin should be divided by the reduced selling price,
which is 90% of the unreduced selling price.
Explanation for Choice D:
This contribution margin percentage calculation did not include the 10% sales discount.
286. Question ID: ICMA 10.P2.269 (Topic: Pricing and Pricing Strategy)
The advantages of incorporating full product costs in pricing decisions include all of the
following except
Hock P2 2020
Section C - Decision Analysis.
Answers
 A. ease in identifying unit fixed costs with individual products.correct
 B. a pricing formula that meets the cost-benefit test; i.e., simplicity.
 C. the promotion of price stability.
 D. full product cost recovery.
Question was not answered
Correct Answer Explanation:
Using all of the costs to set prices does not make it easier to identify the fixed costs by
product. It only makes sure the fixed costs are covered by the prices set.
Explanation for Choice B:
The total cost approach allows the company to recover all product costs by using a
simple markup over costs to determine price. This is an advantage because of its
simplicity.
Explanation for Choice C:
Because all costs, fixed and variable, are considered in setting the price, the price will
be more stable than under a short run, contribution margin focused, variable cost pricing
plan. This is an advantage.
Explanation for Choice D:
This would be an advantage as it ensures that all product costs are covered by the set
price.
287. Question ID: ICMA 10.P2.280 (Topic: Pricing and Pricing Strategy)
A monopoly will maximize profits if it produces an output where marginal cost is

 A. greater than marginal revenue.


 B. less than marginal revenue.
 C. equal to marginal revenue.correct
 D. equal to price.
Question was not answered
Correct Answer Explanation:
As is the case in all market structures, a monopoly firm will maximize profit if it produces
the level of output where marginal cost (the cost to produce one more unit) is equal to
the marginal revenue (the revenue received from producing and selling one more unit).
Explanation for Choice A:
As with all market structures, a monopoly will maximize profit if it produces the level of
output where marginal cost (the cost to produce one more unit) is equal to the marginal
revenue (the revenue received from producing and selling one more unit). If they
Hock P2 2020
Section C - Decision Analysis.
Answers
produce at a level where marginal cost is greater than marginal revenue, they are
reducing profit by producing units that cost more to produce than they receive from
selling that unit.
Explanation for Choice B:
As with all market structures, a monopoly will maximize profit if it produces the level of
output where marginal cost (the cost to produce one more unit) is equal to the marginal
revenue (the revenue received from producing and selling one more unit). If they
produce at a level where marginal cost is less than marginal revenue, they are losing
profit by not producing units that would be profitable.
Explanation for Choice D:
As with all market structures, a monopoly will maximize profit if it produces the level of
output where marginal cost (the cost to produce one more unit) is equal to the marginal
revenue (the revenue received from producing and selling one more unit).
The only market structure where marginal revenue is equal to the price is a perfectly
competitive market; and perfectly competitive markets exist only in theory. In a perfectly
competitive market, each seller can sell as many units as they wish at the market price.
But in all other types of market structures, in order to sell more units, a seller must
decrease the price. Therefore, the marginal revenue received from producing and
selling one more unit will not be equal to the price charged per unit.
It is the marginal revenue, not the price, that should be equal to the marginal cost in
order to maximize profits.
288. Question ID: ICMA 19.P2.018 (Topic: Pricing and Pricing Strategy)
When a product has elastic demand, the percentage change in price

 A. results in no change in quantity demanded.


 B. results in an equal change in quantity demanded.
 C. is greater than the percentage change in quantity demanded.
 D. is less than the percentage change in quantity demanded.correct
Question was not answered
Correct Answer Explanation:
When a product has an elastic demand, any percentage change in price results in
a larger percentage change
in the quantity demanded. For example, a 12% decrease in price will cause the quantity
demanded to increase by more than 12%.
Explanation for Choice A:
If there is no change in the quantity demanded, the product is perfectly inelastic.
Hock P2 2020
Section C - Decision Analysis.
Answers
Explanation for Choice B:
When the change in demand is equal to the change in price, the product has unitary
elasticity.
Explanation for Choice C:
When the change in demand is less than the change in price, the product is inelastic.
289. Question ID: ICMA 13.P2.049 (Topic: Pricing and Pricing Strategy)
Adams Corporation’s goal is for operating income to equal 6% of sales. Adams
estimates that the highest selling price the market will bear is $115 per unit. Adams
expects to sell 100,000 units, incur fixed costs of $3,500,000, and has an effective
income tax rate of 40%. To achieve these plans, the target variable cost per unit must
be

 A. $73.10.correct
 B. $108.10.
 C. $68.50.
 D. $62.75.
Question was not answered
Correct Answer Explanation:
Total revenue is $115 × 100,000, or $11,500,000. The company’s desired operating
income is thus 6% of $11,500,000, or $690,000. Letting X represent the variable cost
per unit (and 100,000X the total variable cost for 100,000 units) in the formula for
operating profit, we get the following:
Revenues – Variable costs – Fixed costs = Operating profit
11,500,000 – 100,000X – 3,500,000 = 690,000
Solving for X:
8,000,000 – 100,000X = 690,000
7,310,000 = 100,000X
X = $73.10
Explanation for Choice B:
This answer does not take into consideration the fixed costs that also reduce operating
income. If the variable costs per unit were $108.10, the company would lose money.
Explanation for Choice C:
If variable costs were $68.50, the company would be profitable but would not achieve
the desired amount of operating profit. The company's goal for operating profit is
$690,000. This variable cost would provide an after-tax profit of $690,000. Operating
profit is calculated before taxes.
Hock P2 2020
Section C - Decision Analysis.
Answers
Explanation for Choice D:
If variable costs were $62.75, the company would be profitable but would not achieve
the desired amount of operating profit.
290. Question ID: ICMA 10.P2.267 (Topic: Pricing and Pricing Strategy)
Which one of the following would cause the demand curve for prepared meals sold in
supermarkets to shift to the right?

 A. An increase in consumer income.correct


 B. An increase in the price of prepared meals.
 C. An increase in the supply of prepared meals.
 D. A decrease in the price of restaurant meals.
Question was not answered
Correct Answer Explanation:
If consumer income increases, the demand curve for all normal goods will shift to the
right.
Explanation for Choice B:
A change in the price of the good under consideration cause a movement ALONG the
demand curve, not a shift of the entire demand curve.
Explanation for Choice C:
A change in the supply of a good will cause a change in the price of the good. This is a
movement ALONG the demand curve instead of a movement of the curve itself.
Explanation for Choice D:
A decrease of the price of a substitute good would cause the demand curve of the good
in consideration to shift to the left. Because restaurant meals are a substitute for
preprepared meals, the decrease in the price of restaurant meals will cause the demand
curve for prepared meals to shift to the left.
291. Question ID: CIA 1195 IV.76 (Topic: Pricing and Pricing Strategy)
In which product-mix pricing strategy is it appropriate for the seller to accept any price
that exceeds the storage and delivery costs for the product?

 A. By-product pricing.correct
 B. Captive-product pricing.
 C. Product-bundle pricing.
 D. Optional-product pricing.
Question was not answered
Hock P2 2020
Section C - Decision Analysis.
Answers
Correct Answer Explanation:
A by-product is a product of little value that was produced during the production
process. Any amount received above and beyond storage and delivery costs will be
accepted because it will improve profit.
Explanation for Choice B:
An example of captive-product pricing would be a printer and printer ink. The ink is
called captive because it must be used along with the printer. It is often the case that
the seller will make more money on the captive product (the ink) than on the main
product (the printer). Therefore, the captive product would have to be priced above the
costs of storage and delivery.
Explanation for Choice C:
An example of product-bundle pricing would be season tickets to the theater. The price
of the season ticket is lower than the price would be to purchase tickets to each
production separately. This is done to promote the sale of tickets. Although the price of
the season ticket is lower than if the tickets were bought separately, it is high enough so
the seller still can make a profit on the sale. Therefore, the price would have to above
the costs of storage and delivery.
Explanation for Choice D:
Optional-product pricing optional products, features or services that are offered along
with the main product as upgrades or options. The optional product or products have
costs, so they must be priced above their storage and delivery costs.
292. Question ID: ICMA 19.P2.090 (Topic: Responsibility Centers and Reporting
Segments)
A component part is manufactured in one country and transferred to the company’s
production facilities in a second country for completion. The exporting country’s effective
income tax rate is 30% while the importing country’s effective income tax rate is 25%.
The transfer price should be

 A. the highest allowable amount to minimize income taxes.


 B. fairly set to evenly divide profits between the two divisions.
 C. the lowest allowable amount to minimize income taxes.correct
 D. negotiated between the two divisions to allow participative management.
Question was not answered
Correct Answer Explanation:
One of the reasons to choose a specific transfer price is to minimize total income taxes.
The lowest transfer price that does this is the transfer price to use.
Explanation for Choice A:
Hock P2 2020
Section C - Decision Analysis.
Answers
While the goal is to minimize total income taxes, usually the lowest price that does this
is selected, and not the highest price.
Explanation for Choice B:
With the different tax rates in different countries, the transfer price should be the lowest
price that will minimize the total income taxes for the company.
Explanation for Choice D:
With the different tax rates in different countries, the transfer price should be the lowest
price that will minimize the total income taxes for the company.

You might also like